You are on page 1of 222

The NCLEX tests are required to receive pro- NCLEX Test Information

fessional licensure in the field of nursing and


are created by the National Council of State You should answer NCLEX questions using
Boards of Nursing (NCSBN). The NCLEX “book” knowledge and not practical experience.
means National Council Licensure Exami- On the NCLEX hospitals operate on massive
nation. Both the NCLEX-RN and NCLEX-PN budgets and no expense is spared to provide
cover a lot of overlapping material; however, proper care. NCLEX test writers are covering
the scoring and number of questions vary all their bases and focus on patient care that
between the exams. is sometimes unrealistic in today’s healthcare
world. Generally, The correct answer is the an-
The NCLEX tests are designed to be one swer that identifies the safest approach. It may
of the final hurdles in your nursing career. not be the fastest or the best, but it is the safest.
Consequently, the questions focus on your The old medical slogan of “Do No Harm” applies
ability to make decisions in various patient to NCLEX test takers. NCLEX test writers are
care scenarios under critical conditions. The trying to make sure that you are competent and
NCLEX test requires that you understand the recognize that safety is the key.
basic principles of nursing and apply this to
different elements of patient safety manage- NCLEX Test Information
ment.
Another key point on reviewing for the NCLEX,
NCLEX Introduction is know your normal laboratory data ranges. Lab
test results on the NCLEX will not be flagged with
Many stare at limited funding and the over- an asterisk if the number is outside of normal
whelming task of studying to pass the NCLEX. ranges. NCLEX test takers must memorize the
This website has been created to help students basic lab values. Focus on the blood gas values.
overcome the challenge of the NCLEX test. The These values in particular can be complicated.
key NCLEX testing tips are stated as follows: If you do not remember the normal ranges you
will have no chance of answering lab value ques-
1. Assess, Assess, Assess: In almost all cases tions on the NCLEX. Many times these abnormal
something can be done before contacting the values will require further assessment.
MD.
2. Prioritize: Delegate to the appropriate sup- It is also important to note that notifying the phy-
port personal and prioritize your tasks. sician or contacting other health care workers
3. Review Medical Terminology: Understand is not The correct answer in many cases on
the definition of all medical abbreviations and the NCLEX. Remember the guidelines: Assess,
terminology used in the NCLEX questions. Assess, Assess. Choose answers that require
further assessment before contacting someone
The hours of studying and the class work are fi- else on the NCLEX questions. Basically, collect
nally worth it when you are free to practice nurs- more data and factual information before calling
ing after passing the NCLEX, and you have high- in other healthcare professionals.
ly sought after job skills.
Finally, think safety with all types of patient care
Please take your time to review all of the course on the NCLEX. If equipment breaks down and
notes and links put together on this site about the the patient is in trouble, work on solving the pa-
NCLEX test and the pitfalls that some students tient’s problems before getting someone else to
fall into with the NCLEX test. Hopefully you can fix the equipment. Let maintenance deal with the
avoid mistakes others have made when prepar- equipment mess and focus on getting the patient
ing for the NCLEX and will find the following in- in a safe environment. The NCLEX is attempting
formation to be helpful and informative on deal- to determine competencies related to all of the
ing with the NCLEX test. above situations.

Collected by :DeepaRajesh [ 1 ]
rajesh.ks21@gmail.com
Kuwait
NCLEX Format
Question3: What is the maximum
Note: The NCLEX Exam is offered by Pearsonamount of time that I could spend on
VUE. The National Council of State Boards of
each Questionon the NCLEX?
Nursing has partnered with Pearson VUE to de-
Answer: If you take the maximum
liver the NCLEX exam. Pearson VUE offers both
the NCLEX-RN and NCLEX-PN exams. number of questions a safe time would
be around 1 minute per Questionon
The NCLEX CAT testing format stands the NCLEX.
for a computer adaptive testing format.
The computer during the NCLEX test Question4: What study aides have you
will give you harder questions if you found that help you understand the
answer a Questioncorrectly or easier format better?
questions if you answer a Question- Answer: The link on the right of this page offers
valuable help with the NCLEX format.
incorrectly. The first Questionon the
NCLEX will be below the baseline re- Question5: What if the CAT format of-
quired passing score. Consequently, fers questions beyond the minimum
a graph could be constructed using number required to pass?
questions as points on the graph. The Answer: Keep taking the test and don’t get nerv-
points above and below the passing ous. You still have the opportunity to do well and
baseline contribute to your overall test- pass.
ing score. The NCLEX test attempts to
match you with questions that are at Learn How to Quickly Solve Difficult
your level of nursing knowledge and NCLEX Test Questions
NCLEX Flashcard Study System Free Online
understanding. Sample Practice Test Questions

The NCLEX does not time each Ques- Click here to see free sample flashcards.
tionpresented in the CAT format. You
are allowed to answer each Question- Dear Friend,
without time constraints. However, the
Here’s a little “secret” about the NCLEX Exami-
NCLEX does have a test taking time of
nation: the NCLEX is what we in the test prepa-
5 hours. ration field call a “content driven” test.

Question1: What is the maximum While some tests are looking to see what you are
amount of time allowed to take the ABLE to learn, the purpose of the National Coun-
NCLEX? cil Licensure Examination for Registered Nurses
Answer: 5 hours (NCLEX-RN) and National Council Licensure
Examination for Practical Nurses (NCLEX-PN),
Question2: How do I prepare for the offered by the National Council of State Boards
of Nursing (NCSBN), is to test your understand-
CAT format on the NCLEX?
ing of what you have already learned. The goal
Answer: Allow each Questiona reason- of the NCLEX is to make sure you have a mini-
able amount of time and thought. Treat mum competency level to protect the integrity of
each Questionwith the same level of the testing process.
difficulty. Don’t be scared if questions
are getting “easier,” and you think that In other words, it’s more about what you know
than your ability to solve clever puzzles. This is
you are falling below the passing base-
good news for those who are serious about be-
line of difficulty. ing prepared, because it boils down to a very
Collected by :DeepaRajesh [ 2 ]
rajesh.ks21@gmail.com
Kuwait
simple strategy: questions, and the flipping action gets you ac-
tively involved in the learning process
You can succeed on the NCLEX and become a * Our cards are printed on heavy, bright white
Registered Nurse (RN) or Practical Nurse (PN) 67 lb. cover stock, and are laser printed at
by learning critical concepts on the test so that 1200 dpi on our industrial printers- these are
you are prepared for as many questions as pos- professional-quality cards that will not smear
sible. or wear out with heavy usage
* We cover all of the major categories of the
Repetition and thorough preparation is a process NCLEX test (see the list below)
that rewards those who are serious about being * Our flashcards include an edge that is micro-
prepared, which means that succeeding on the perforated, which means that you are much
NCLEX is within the reach of virtually anyone in- less likely to have a painful papercut on your
terested in learning the material. fingers when moving quickly through the
cards
This is great news! It means that if you’ve been * Our cards are portable, making it easy for
worried about your upcoming NCLEX, you can you to grab a few and study while waiting for
rest easy IF you have a good strategy for know- the bus or the doctor, or anywhere where you
ing what to study and how to effectively use rep- have a spare moment that would otherwise be
etition to your advantage. wasted
* Our cards are written in an easy to under-
But it also creates another set of problems. stand, straightforward style - we don’t include
any more technical jargon than what you need
If you tried to memorize every single possible to pass the test
thing you can for the NCLEX, the field of pos- * The cards are a generous size- 3.67 x 4.25
sible things to review would be so huge that you inches- they fit perfectly in your hands and
could not hope to cover everything in a reason- they aren’t so small that you have to use a
able time. magnifying glass to read tiny type- all ques-
tions and answers are in a normal-size print
That’s why I created the NCLEX Flashcard Study for easy studying
System: I have taken all of the possible topics * Our cards include in-depth explanations-
and reduced them down to the hundreds of con- you won’t see any “one word” answers on our
cepts you must know and provided an easy-to- cards that require you to go get a textbook to
use learning method to guarantee success on understand why your answer was wrong- all of
the NCLEX. our cards include generous, thorough expla-
nations so you not only get it right or wrong-
I wanted this system to be simple, effective, and but you also know why!
fast so that you can succeed on your NCLEX * We use a font created by Microsoft to make
with a minimum amount of time spent preparing reading easier- this will enable you to absorb
for it. more information painlessly during late night
study sessions
Note: This product will also work for the HESI * Our system enables you to study in small,
exit exam and help you graduate from your nurs- digestible bits of information- unlike using bor-
ing program. ing textbooks, flashcards turn learning into a
Here Are Some of the Features of Our NCLEX “game” you can play until you’ve mastered the
Flashcard Study System material
* It’s easy for a friend to help you study- they
* Study after study has shown that repetition is don’t even have to know anything about the
the most effective form of learning- and noth- NCLEX- if they can read, then they can quiz
ing beats flashcards when it comes to making you with our flashcards!
repetitive learning fun and fast
* Flashcards engage more of your senses Now, let me explain what the NCLEX Flashcard
in the learning process- you “compete” with Study System is not. It is not a comprehensive
yourself to see if you know the answers to the review of your education. There’s no way we
Collected by :DeepaRajesh [ 3 ]
rajesh.ks21@gmail.com
Kuwait
could fit that onto flashcards- if we claimed to, it * Assessing Fetal Lung Maturity
would be an insult to what you know. * Pathology of Eclampsia
* PMS and Menopause
Don’t get us wrong: we’re not saying that memo- * Attributes of Battered Women
rization alone will automatically result in a pass- * Apgar Scores
ing NCLEX score- you have to have the ability to * Types of Cardiomyopathies
apply it as well. However, without the foundation * Opportunistic Infections
of the core concepts, you cannot possibly hope * Classifications of Cancer
to apply the information. After all, you can’t apply * Medical Nutritional Therapy
what you don’t know. * Staging of Pressure Ulcers
* Disease Pathology
NCLEX Flashcard Study System is a compilation * Types of Shock
of the 615+ critical concepts you must under- * Lipid Profile Labs
stand to pass the NCLEX. Nothing more, noth- * Coagulation Studies
ing less. * CBC Components
Here’s Exactly What You Get With the NCLEX * Acne Treatment Medications
Flashcard Study System * Phases of Adolescence
* Three Types of Jaundice
When you order the NCLEX Flashcard Study * Pain Assessment
System, you’ll get our set of over 615 flashcards * Lymphoma Characteristics
specially selected to give you the most NCLEX * Sexually Transmitted Diseases
performance improvement for the least time. This * Tanner Staging
is just a small sampling of the topics covered: * Vaccinations and Immunizations
* Symptoms of Child Abuse
NCLEX Exam Topics: * Performing Newborn Assessments
* Types of Nosocomial Infections * Motor Development
* Principles of Surgical Asepsis * Development of Language
* Medical Testing and Labs * Pharmacology
* TURP Procedure * Types of Adrenergic Receptors
* Romberg’s Test * Properties of Decongestants
* Lithotripsy Procedure * Classifications of Drugs
* Levels of Consciousness * Antipsychotic Classifications
* Mental Exam Basics * Drug Interactions
* Grading of Deep Tendon Reflexes * Major Injection Sites
* Glascow Coma Scale * Calcium Channel Blockers
* Normative Values * Phases of Burn Management
* Methods of Oxygen Delivery * Types of Burns
* Dementia and Delirium * Wound Healing Phases
* Types of Injections
* Ethical Duties of Nurses Click here to see 3 free NCLEX Flashcard Study
* Patient Rights System sample cards.
* Bioethical Principles
* Changes Associated with Aging Remember, this is just a small sampling of the
* Drip Rate Calculations topics covered in our system. Overall, you get
* Barriers to Communication over 615 premium-quality flashcards covering
* Nutrition and TPN everything you’ll need to succeed on the NCLEX.
* Attributes of Nutrients The price for this package is only $39.95.
* Methods of Absorption Receive the Following Bonus
* Metabolism and Nutrition
* Medical Nutrition Therapy Since I know it’s 100% to your benefit to use our
* Cultural Aspects of Diets flashcards, I want to sweeten the pot and give
* Placenta Previa you every possible reason to say YES! With your
* Stages of Labor order, you’ll also receive the following:
Collected by :DeepaRajesh [ 4 ]
rajesh.ks21@gmail.com
Kuwait
antee. So you have nothing to lose and every-
Special Report- The “Leitner Method” for Maxi- thing to gain.
mizing Flashcard Learning- in the 1970’s a Ger-
man psychologist developed a learning system My belief is simple: either this product helps you
that turned the humble flashcard into an ad- or you don’t pay. Period. No gimmicks, no aster-
vanced learning technology. His method teach- isks.
es you to learn faster by playing a simple game
with your flashcards, with the help of a few items
you probably have around your home. Simple NCLEX Medical Terminology Review
to learn, but incredibly effective- our free report
shows you exactly how to use his method in Understanding the medical terminology used on
plain, easy-to-understand language. the NCLEX should be a top priority when pre-
paring for the NCLEX. Medical terms can some-
Note: we cannot guarantee that this free report times be confusing due to the use of medical ab-
will be available indefinitely, so act now to lock in breviations.
your copy.
If you are unable to understand the medical ter-
By the way, this bonus is yours to keep even in minology used on the NCLEX then you will have
the unlikely event you decide to take advantage poor chance of picking The correct answer.
of our ironclad money-back guarantee: Depending on your clinical rotations you may
You Cannot Lose With My No-Questions-Asked also be more familiar with certain medical terms
1-Year Money-Back Guarantee in a specific area of nursing. Generally, nurses
that have the broadest experience with medical
We stand behind our offer with a no-questions- terminology will have a better understanding to
asked 1-year guarantee on our products. answer questions that contain complex medical
terminology on the NCLEX test.
So go ahead and order your copy of the NCLEX
Flashcard Study System today. Read them, study Take time to review the following abbreviations
them, and profit from them. If you don’t think they on the NCLEX test as well as a more thorough
are helping you prepare for your NCLEX exam list as found in the NCLEX study guide linked to
- you can return them for an immediate 100% the right hand side of this page.
refund of your purchase price, no questions
asked. ADH antidieuretic hormone
AML acute myelogenous leukemia
I think that speaks volumes about our confidence APC atrial premature contraction
in our products. We are also members of the Bet- ASD atrial septal defect
ter Business Bureau of Southeast Texas. BPH benign prostatic hypertrophy
BUN blood, urea, nitrogen
If you think there’s even the smallest chance that Ca calcium
these flashcards will help you, you owe it to your- CA cancer
self to try them out. Don’t let fear or doubt stand CAPD continuous ambulatory peritoneal dialysis
in the way of what could be your best opportunity CC chief complaint
to achieve the test score you need to fulfill the CPK creatine phosphokinase
dream you deserve. CRP C-reactive protein
DIFF differential blood count
What I’m saying is, don’t decide now if these DOE dyspnea on exertion
flashcards are for you. Just get them and try D/W dextrose in water
them out. If they don’t do everything I say and ECT electroconvulsive therapy
more, if you don’t save money, time and frustra- ESRD end stage renal disease
tion, if they aren’t what you thought they were, FUO fever of undetermined origin
if they don’t work for you, you have nothing to GH growth hormone
worry about because you can get every dime of GSC glascow coma scale
your money back under our no-loopholes guar- Hg mercury
Collected by :DeepaRajesh [ 5 ]
rajesh.ks21@gmail.com
Kuwait
HLA human leukocyte antigen NCLEX-RN and NCLEX-PN is the different
Hz hertz number of questions. Please review the follow-
ICS intercostal space ing chart:
IPG impedance plethysmogram
JRA juvenile rheumatoid arthritis NCLEX-RN NCLEX-PN
Minimum Number of Questions 75 85
Practicing nurses have the luxury of being able Maximum Number of Questions 265 205
to look up medial abbreviations and definitions
before making patient care decisions. However, Both of these tests require the same ba-
the NCLEX test does not allow that option. If you
sic understanding of nursing practice
are confused by the medical terminology on the
NCLEX, you will not be able to use a medical and knowledge. The NCLEX-RN ques-
dictionary for reference purposes. tions and the NCLEX-PN questions are
presented with four multiple choice
answer scenarios. In some cases, the
NCLEX Preparation NCLEX is using a more difficult Ques-
tionformat that requires multiple right
The most important thing that you can do pre-
answers to be selected. However, the
paring for the NCLEX is not stress out. A score
in the 90th percentile is not required to pass the material that is covered is the same.
NCLEX test. You only have to show a minimum
level of competency in the field of nursing. NCLEX-RN vs. NCLEX-PN

1. Begin your preparation by sending in your On the NCLEX-RN nursing students are
application to the board of licensure. required to concentrate for a longer pe-
2. Then schedule with the Chauncey Group for riod of time due to the higher number
the exam. Finally, you will be sent authorization
of questions. If the computer doesn’t
to test (ATT).
3. Next set-up a time that works for you and turn off at the minimum number of
show up with all the required documents at the questions, continue to answer each
testing center for the NCLEX. If you are recently Questionin a reasonable amount of
married with a name change, bring your mar- time. Do not begin to rush through the
riage license. You may not need it, but if you did questions, because you may have to
it could cause you to miss your testing time.
answer the maximum number of ques-
**The testing center will require at least 2 forms tions on the exam. Anticipate going the
of identification to allow you to take the NCLEX distance and concentrating on each
test. question.

Don’t make the mistake of altering hair color or Obviously, some of the questions related to del-
facial hair prior to the exam. Your picture has to egation of responsibility are different between
match the application picture. You will also have these exams. Registered nurses will be asked
to be thumb printed to take the NCLEX. In addi- to assign tasks to practical nurses and nursing
tion, bring a drink and some snack food for your assistants while prioritizing their patients. Like-
testing break and wear layered clothing. Stu- wise practical nurses will be asked questions
dents that take the NCLEX in shorts and a tee that require assigning tasks to nursing assistants
shirt may find the testing center unbearable cold and requesting more assistance from registered
and be unable to concentrate. nurses.

Many of the study guides on the market have


NCLEX-RN vs. NCLEX-PN questions that help understand the delegation of
responsibility task with nursing.
One of the primary differences between the
Collected by :DeepaRajesh [ 6 ]
rajesh.ks21@gmail.com
Kuwait
NCLEX Testing Recommendations look discolored. If you believe this change is due
to medication, which of the following patient’s
1. Watch out for the words: except, always and medication does not cause urine discoloration?
not in all NCLEX questions.
2. Answer each question. You are not penal- A: Sulfasalazine
ized for taking an educated guess. B: Levodopa
3. Get a good night’s sleep before the C: Phenolphthalein
NCLEX. D: Aspirin
4. Wear layered clothing to the exam.
5. Practice with a watch and bring a watch to 4. You are responsible for reviewing the nursing
the test. unit’s refrigerator. If you found the following drug
6. Study for each material section of the NCLEX in the refrigerator it should be removed from the
individually. refrigerator’s contents?
7. If you extremely weak in one area of content
focus on that area. A: Corgard
8. Don’t cram for the NCLEX. Read over a B: Humulin (injection)
good practice study guide at least one week C: Urokinase
in advance. D: Epogen (injection)
9. Stay away from negative talk about the
NCLEX with other students. 5. A 34 year old female has recently been diag-
10. Know the route to the testing center, nosed with an autoimmune disease. She has
11. Double check that you have the appropriate also recently discovered that she is pregnant.
ID prior to the NCLEX test. Which of the following is the only immunoglobu-
12. Work through several practice tests prior to lin that will provide protection to the fetus in the
the exam. womb?

1. A nurse is reviewing a patient’s medication dur- A: IgA


ing shift change. Which of the following medica- B: IgD
tion would be contraindicated if the patient were C: IgE
pregnant? Note: More than one answer may be D: IgG
correct.
6. A second year nursing student has just suf-
A: Coumadin fered a needlestick while working with a patient
B: Finasteride that is positive for AIDS. Which of the following
C: Celebrex is the most important action that nursing student
D: Catapress should take?
E: Habitrol
F: Clofazimine A: Immediately see a social worker
B: Start prophylactic AZT treatment
2. A nurse is reviewing a patient’s PMH. The his- C: Start prophylactic Pentamide treatment
tory indicates photosensitive reactions to medi- D: Seek counseling
cations. Which of the following drugs has not
been associated with photosensitive reactions? 7. A thirty five year old male has been an insulin-
Note: More than one answer may be correct. dependent diabetic for five years and now is un-
able to urinate. Which of the following would you
A: Cipro most likely suspect?
B: Sulfonamide
C: Noroxin A: Atherosclerosis
D: Bactrim B: Diabetic nephropathy
E: Accutane C: Autonomic neuropathy
F: Nitrodur D: Somatic neuropathy

3. A patient tells you that her urine is starting to 8. You are taking the history of a 14 year old girl
Collected by :DeepaRajesh [ 7 ]
rajesh.ks21@gmail.com
Kuwait
who has a (BMI) of 18. The girl reports inability to
eat, induced vomiting and severe constipation. 11. A new mother has some questions about
Which of the following would you most likely sus- (PKU). Which of the following statements made
pect? by a nurse is not correct regarding PKU?

A: Multiple sclerosis A: A Guthrie test can check the necessary lab


B: Anorexia nervosa values.
C: Bulimia B: The urine has a high concentration of phe-
D: Systemic sclerosis nylpyruvic acid
C: Mental deficits are often present with PKU.
9. A 24 year old female is admitted to the ER for D: The effects of PKU are reversible.
confusion. This patient has a history of a mye-
loma diagnosis, constipation, intense abdominal 12. A patient has taken an overdose of aspi-
pain, and polyuria. Which of the following would rin. Which of the following should a nurse most
you most likely suspect? closely monitor for during acute management of
this patient?
A: Diverticulosis
B: Hypercalcaemia A: Onset of pulmonary edema
C: Hypocalcaemia B: Metabolic alkalosis
D: Irritable bowel syndrome C: Respiratory alkalosis
D: Parkinson’s disease type symptoms
10. Rho gam is most often used to treat____
mothers that have a ____ infant. 13. A fifty-year-old blind and deaf patient has
been admitted to your floor. As the charge nurse
A: RH positive, RH positive your primary responsibility for this patient is?
B: RH positive, RH negative
C: RH negative, RH positive A: Let others know about the patient’s deficits.
D: RH negative, RH negative B: Communicate with your supervisor your pa-
tient safety concerns.
Answer Key C: Continuously update the patient on the social
environment.
1. (A) and (B) are both contraindicated with preg- D: Provide a secure environment for the patient.
nancy.
2. (F) All of the others have can cause photosen- 14. A patient is getting discharged from a SNF fa-
sitivity reactions. cility. The patient has a history of severe COPD
3. (D) All of the others can cause urine discolora- and PVD. The patient is primarily concerned
tion. about their ability to breath easily. Which of the
4. (A) Corgard could be removed from the refig- following would be the best instruction for this
erator. patient?
5. (D) IgG is the only immunoglobulin that can
cross the placental barrier. A: Deep breathing techniques to increase O2
6. (B) AZT treatment is the most critical innerven- levels.
tion. B: Cough regularly and deeply to clear airway
7. (C) Autonomic neuropathy can cause inability passages.
to urinate. C: Cough following bronchodilator utilization
8. (B) All of the clinical signs and systems point D: Decrease CO2 levels by increase oxygen take
to a condition of anorexia nervosa. output during meals.
9. (B) Hypercalcaemia can cause polyuria, se-
vere abdominal pain, and confusion. 15. A nurse is caring for an infant that has recent-
10. (C) Rho gam prevents the production of anti- ly been diagnosed with a congenital heart defect.
RH antibodies in the mother that has a Rh posi- Which of the following clinical signs would most
tive fetus. likely be present?

Collected by :DeepaRajesh [ 8 ]
rajesh.ks21@gmail.com
Kuwait
A: Slow pulse rate D: The life span of RBC is 120 days.
B: Weight gain
C: Decreased systolic pressure Answer Key 11-20.
D: Irregular WBC lab values
11. (D) The effects of PKU stay with the infant
16. A mother has recently been informed that her throughout their life.
child has Down’s syndrome. You will be assigned 12. (D) Aspirin overdose can lead to metabolic
to care for the child at shift change. Which of the acidosis and cause pulmonary edema develop-
following characteristics is not associated with ment.
Down’s syndrome? 13. (D) This patient’s safety is your primary con-
cern.
A: Simian crease 14. (C) The bronchodilator will allow a more pro-
B: Brachycephaly ductive cough.
C: Oily skin 15. (B) Weight gain is associated with CHF and
D: Hypotonicity congenital heart deficits.
16. (C) The skin would be dry and not oily.
17. A patient has recently experienced a (MI) 17. (A) Streptokinase is a clot busting drug and
within the last 4 hours. Which of the following the best choice in this situation.
medications would most like be administered? 18. (A) Green vegetables and liver are a great
source of folic acid.
A: Streptokinase 19. (D) Cl. difficile has not been linked to men-
B: Atropine ingitis.
C: Acetaminophen 20. (D) RBC’s last for 120 days in the body.
D: Coumadin

18. A patient asks a nurse, “My doctor recom- 21. A 65 year old man has been admitted to the
mended I increase my intake of folic acid. What hospital for spinal stenosis surgery. When does
type of foods contain the highest concentration the discharge training and planning begin for this
of folic acids?” patient?

A: Green vegetables and liver A: Following surgery


B: Yellow vegetables and red meat B: Upon admit
C: Carrots C: Within 48 hours of discharge
D: Milk D: Preoperative discussion

19. A nurse is putting together a presentation on 22. A child is 5 years old and has been recently
meningitis. Which of the following microorgan- admitted into the hospital. According to Erickson
isms has noted been linked to meningitis in hu- which of the following stages is the child in?
mans?
A: Trust vs. mistrust
A: S. pneumonia B: Initiative vs. guilt
B: H. influenza C: Autonomy vs. shame
C: N. meningitis D: Intimacy vs. isolation
D: Cl. difficile
23. A toddler is 16 months old and has been re-
20. A nurse is administering blood to a patient cently admitted into the hospital. According to
who has a low hemoglobin count. The patient Erickson which of the following stages is the tod-
asks how long to RBC’s last in my body? The dler in?
correct response is.
A: Trust vs. mistrust
A: The life span of RBC is 45 days. B: Initiative vs. guilt
B: The life span of RBC is 60 days. C: Autonomy vs. shame
C: The life span of RBC is 90 days. D: Intimacy vs. isolation
Collected by :DeepaRajesh [ 9 ]
rajesh.ks21@gmail.com
Kuwait
toacidosis. Which of the following would you not
24. A young adult is 20 years old and has been expect to see with this patient if this condition
recently admitted into the hospital. According were acute?
to Erickson which of the following stages is the
adult in? A: Vomiting
B: Extreme Thirst
A: Trust vs. mistrust C: Weight gain
B: Initiative vs. guilt D: Acetone breath smell
C: Autonomy vs. shame
D: Intimacy vs. isolation 30. A patient’s chart indicates a history of men-
ingitis. Which of the following would you not ex-
25. A nurse is making rounds taking vital signs. pect to see with this patient if this condition were
Which of the following vital signs is abnormal? acute?

A: 11 year old male – 90 b.p.m, 22 resp/min. , A: Increased appetite


100/70 mm Hg B: Vomiting
B: 13 year old female – 105 b.p.m., 22 resp/min., C: Fever
105/60 mm Hg D: Poor tolerance of light
C: 5 year old male- 102 b.p.m, 24 resp/min.,
90/65 mm Hg Answer Key 21-30.
D: 6 year old female- 100 b.p.m., 26 resp/min.,
90/70mm Hg 21. (B) Discharge education begins upon admit.
22. (B) Initiative vs. guilt- 3-6 years old
26. When you are taking a patient’s history, she 23. (A) Trust vs. Mistrust- 12-18 months old
tells you she has been depressed and is dealing 24. (D) Intimacy vs. isolation- 18-35 years old
with an anxiety disorder. Which of the following 25. (B) HR and Respirations are slightly in-
medications would the patient most likely be tak- creased. BP is down.
ing? 26. (A) Elavil is a tricyclic antidepressant.
27. (D) Erythromycin is used to treat conditions
A: Elavil A-C.
B: Calcitonin 28. (D) Answer choices A-C were symptoms of
C: Pergolide acute hyperkalemia.
D: Verapamil 29. (C) Weight loss would be expected.
30. (A) Loss of appetite would be expected.
27. Which of the following conditions would a
nurse not administer erythromycin? 31. A nurse if reviewing a patient’s chart and no-
tices that the patient suffers from conjunctivitis.
A: Campylobacterial infection Which of the following microorganisms is related
B: Legionnaire’s disease to this condition?
C: Pneumonia
D: Multiple Sclerosis A: Yersinia pestis
B: Helicobacter pyroli
28. A patient’s chart indicates a history of hyper- C: Vibrio cholera
kalemia. Which of the following would you not D: Hemophilus aegyptius
expect to see with this patient if this condition
were acute? 32. A nurse if reviewing a patient’s chart and no-
tices that the patient suffers from Lyme disease.
A: Decreased HR Which of the following microorganisms is related
B: Paresthesias to this condition?
C: Muscle weakness of the extremities
D: Migranes A: Borrelia burgdorferi
B: Streptococcus pyrogens
29. A patient’s chart indicates a history of ke- C: Bacilus anthracis
Collected by :DeepaRajesh [ 10 ]
rajesh.ks21@gmail.com
Kuwait
D: Enterococcus faecalis D: Frequent attempts with positive reinforce-
ment.
33. A fragile 87 year-old female has recently been
admitted to the hospital with increased confusion 38. A parent calls the pediatric clinic and is frantic
and falls over last 2 weeks. She is also noted to about the bottle of cleaning fluid her child drank
have a mild left hemiparesis. Which of the follow- 20 minutes. Which of the following is the most
ing tests is most likely to be performed? important instruction the nurse can give the par-
ent?
A: FBC (full blood count)
B: ECG (electrocardiogram) A: This too shall pass.
C: Thyroid function tests B: Take the child immediately to the ER
D: CT scan C: Contact the Poison Control Center quickly
D: Give the child syrup of ipecac
34. A 84 year-old male has been loosing mobility
and gaining weight over the last 2 months. The 39. A nurse is administering a shot of Vitamin K
patient also has the heater running in his house to a 30 day-old infant. Which of the following tar-
24 hours a day, even on warm days. Which of the get areas is the most appropriate?
following tests is most likely to be performed?
A: Gluteus maximus
A: FBC (full blood count) B: Gluteus minimus
B: ECG (electrocardiogram) C: Vastus lateralis
C: Thyroid function tests D: Vastus medialis
D: CT scan
40. A nurse has just started her rounds delivering
35. A 20 year-old female attending college is medication. A new patient on her rounds is a 4
found unconscious in her dorm room. She has a year-old boy who is non-verbal. This child does
fever and a noticeable rash. She has just been not have on any identification. What should the
admitted to the hospital. Which of the following nurse do?
tests is most likely to be performed first?
A: Contact the provider
A: Blood sugar check B: Ask the child to write their name on paper.
B: CT scan C: Ask a co-worker about the identification of the
C: Blood cultures child.
D: Arterial blood gases D: Ask the father who is in the room the child’s
name.
36. A 28 year old male has been found wander-
ing around in a confused pattern. The male is Answer Key 31-40.
sweaty and pale. Which of the following tests is
most likely to be performed first? 31. (D) Choice A is linked to Plague, Choice B
is linked to peptic ulcers, Choice C is linked to
A: Blood sugar check Cholera.
B: CT scan 32. (A) Choice B is linked to Rheumatic fever,
C: Blood cultures Choice C is linked to Anthrax, Choice D is linked
D: Arterial blood gases to Endocarditis.
33. (D) A CT scan would be performed for further
37. A mother is inquiring about her child’s ability investigation of the hemiparesis.
to potty train. Which of the following factors is the 34. (C) Weight gain and poor temperature toler-
most important aspect of toilet training? ance indicate something may be wrong with the
thyroid function.
A: The age of the child 35. (C) Blood cultures would be performed to in-
B: The child ability to understand instruction. vestigate the fever and rash symptoms.
C: The overall mental and physical abilities of the 36. (A) With a history of diabetes, the first re-
child. sponse should be to check blood sugar levels.
Collected by :DeepaRajesh [ 11 ]
rajesh.ks21@gmail.com
Kuwait
37. (C) Age is not the greatest factor in potty
training. The overall mental and physical abilities 2. Nervous Tissue: Nervous tissue is com-
of the child is the most important factor. posed of cells (neurons) that respond to exter-
38. (C) The poison control center will have an nal and internal stimuli and have the capability to
exact plan of action for this child. transmit a message (impulse) from one area of
39. (C) Vastus lateralis is the most appropriate the body to another. This tissue thus induces a
location. response of distant muscles or glands, as well as
40. (D) In this case you are able to determine the regulating body processes such as respiration,
name of the child by the father’s statement. You circulation, and digestion.
should not withhold the medication from the child 3. Epithelial Tissue: Epithelial tissue covers the
following identification. external surfaces of the body and lines the inter-
nal tubes and cavities. It also forms the glands of
the body. Characteristics of epithelial tissue (epi-
NCLEX Skeletal Muscle Review thelium) are that it

In order for the human being to carry out the many 1. has compactly aggregated cells;
intricate movements that must be performed, 2. has limited intercellular spaces and sub-
approximately 650 skeletal muscles of various stance;
lengths, shapes, and strength play a part. Each 3. is avascular (no blood vessels);
muscle consists of many muscle cells or fibers 4. lies on a connective tissue layer—the
held together and surrounded by connective tis- basal lamina;
sue that gives functional integrity to the system. 5. has cells that form sheets and are polar-
Three definite units are commonly referred to: ized;
6. is derived from all three germ layers.
1. endomysium—connective tissue layer en-
veloping a single fiber; Microvilli—fingerlike projections of plasma
2. perimysium—connective tissue layer envel- membranes.
oping a bundle of fibers;
3. epimysium—connective tissue layer envel- Cilia—motile organelles extending into the
oping the entire muscle luman consisting of specifically arranged micro-
tubules.
Muscle Attachment and Function
Flagella—similar to cilia. Primary examples
For coordinated movement to take place, the are human spermatozoa.
muscle must attach to either bone or cartilage
or, as in the case of the muscles of facial expres- Stereocilia—are actually very elongated Mi-
sion, to skin. The portion of a muscle attaching crovilli.
to bone is the tendon. A muscle has two extremi- 4. Connective Tissue: Connective tissue is the
ties, its origin and its insertion. packing and supporting material of the body tis-
sues and organs. It develops from mesoderm
(mesenchyme). All connective tissues consist of
NCLEX Four Basic Tissues
three distinct components: ground substance,
cells and fibers.
1. Muscle Tissue: Muscle tissue is contractile in
nature and functions to move the skeletal system
* Ground substance. Ground substance
and body viscera.
is located between the cells and fibers, both of
which are embedded in it. It forms an amorphous
Type Characteristics Location
intercellar material. In the fresh state, it appears
Skeletal Striated, voluntary Skeletal mus-
as a transparent and homogenous gel. It acts as
cles of the body
a route for the passage of nutrients and wastes
Smooth Non-striated, involuntary W a l l s
to and from the cells within or adjacent to the
of digestive tract and blood vessels, uterus, uri-
connective tissue.
nary bladder
* Fibers. The fiber components of con-
Cardiac Striated, involuntary Heart
Collected by :DeepaRajesh [ 12 ]
rajesh.ks21@gmail.com
Kuwait
nective tissue add support and strength. Three Thalamus Contralateral thalamus pain, con-
types of fibers are present: collagenous, elastic tralateral hemisensory loss
and reticular. Pineal gland Early hydrocephalus, papillary ab-
normalities, Parinaud’s syndrome
NCLEX Cardiac Review Internal capsule Hemisensory loss, homony-
mous hemianopsia, contralateral hemiplegia
The heart is a highly specialized blood vessel Basal ganglia Contralateral dystonia, Con-
which pumps 72 times per minute and propels tralateral choreoathetosis
about 4,000 gallons (about 15,000 liters) of blood Pons Diplopia, internal strabismus, VI and VII
daily to the tissues. It is composed of: involvement, contralateral hemisensory and
hemiparesis loss, issilateral cerebellar ataxia
Endocardium (lining coat; epithelium) Broca’s area Motor dysphasia
Precentral gyrus Jacksonian seizures, gen-
Myocardium (middle coat; cardiac muscle) eralized seizures, hemiparesis
Superficial parietal lobe Receptive dysphasia
Epicardium (external coat or visceral layer of
pericardium; epithelium and mostly connective
tissue)
NCLEX Tumor Review
Impulse conducting system
Primary Tumors
Cardiac Nerves: Modification of the intrinsic
rhythmicity of the heart muscle is produced by * Neuromas- 80-90% of brain tumors, named
cardiac nerves of the sympathetic and parasym- for what part of nerve cell affected.
pathetic nervous system. Stimulation of the sym- * Meningiomas- outside of arachnoidal tissue,
pathetic system increases the rate and force of usually benign and slow growing
the heartbeat and dilates the coronary arteries. * Glioblastoma Multiform-50% of all primary
Stimulation of the parasympathetic (vagus nerve) tumors, linked to specific genetic mutations
reduces the rate and force of the heartbeat and
constricts the coronary circulation. Visceral affer- Secondary Tumors
ent (sensory) fibers from the heart end almost
wholly in the first four segments of the thoracic * Metastatic carcinomas
spinal cord.
Scale –degree of anaplasia: differentiation of
Cardiac Cycle: Alternating contraction and relax- mature (good) vs. immature cells (bad)
ation is repeated about 75 times per minute; the
duration of one cycle is about 0.8 second. Three Grade I: up to 25% anaplasia
phases succeed one another during the cycle:
Grade II: 26-50% anaplasia
a) atrial systole: 0.1 second,
Grade III: 51-75% anaplasia
b) ventricular systole: 0.3 second,
Grade IV: 76-100% anaplasia
c) diastole: 0.4 second
Primary Tumor Effect:
The actual period of rest for each chamber is 0.7
second for the atria and 0.5 second for the ven- 1. Headaches
tricles, so in spite of its activity, the heart is at rest 2. Vomiting
longer than at work.
Secondary Tumor Effect:
NCLEX Lesion Review
Occipital Lobe Homonymous hemianopsia, 1. Direct compression/necrosis
partial seizures with limited visual phenomena 2. Herniation of brain tissue
Collected by :DeepaRajesh [ 13 ]
rajesh.ks21@gmail.com
Kuwait
3. Increase ICP Opposition is one of the most critical movements
in humans; it allows us to have pulp-to-pulp op-
Noteworthy Tumor Markers position, which gives us the great dexterity of
our hands. In this movement the thumb pad is
1. AFP brought to a finger pad. A median nerve injury
2. Alkaline phosphatase negates this action.
3. b-hCG
4. CA-125 NCLEX Cell Structure Review
5. PSA
Endoplasmic Reticulum ( ER)
NCLEX Movement Terms
This cellular organelle was first described using
Flexion is bending, most often ventrally to de- phase microscopy by Porter, Claude and Fallam
crease the angle between two parts of the body; in 1945. It is an extensive network of intercon-
it is usually an action at an articulation or joint. necting channels. The endoplasmic reticular
membranes are unit membranes (triminar). When
Extension is straightening, or increasing the an- ribosomes line the outer surface it is designated
gle between two parts of the body; a stretching as rough endoplasmic reticulum ( RER). The pri-
out or making the flexed part straight. mary form of this organelle is the rough variety.
The smooth is derived from the rough due to loss
Abduction is a movement away from the midsag- of ribosomes. The amount of each depends on
ittal plane (midline); to adduct is to move medi- the cell type and the cellular activity.
ally and bring a part back to the mid-axis.
The RER is the synthetic machinery of the cell. It
Circumduction is a circular movement at a ball is mainly concerned with protein synthesis.
and socket (shoulder or hip) joint, utilizing the
movements of flexion, extension, abduction, and The Golgi Complex
adduction.
This structure was discovered by Camillo Golgi
Rotation is a movement of a part of the body in 1898. All eukaryotic cells, except for the red
around its long axis. blood cell, possess a Golgi apparatus. Generally
speaking the Golgi complex is prominent in glan-
Supination refers only to the movement of the dular cells and is thought to function in the pro-
radius around the ulna. In supination the palm of duction, concentration packaging, and transpor-
the hand is oriented anteriorly; turning the palm tation of secretory material. IN summary one can
dorsally puts it into pronation. The body on its link the Golgi complex to: secretion, membrane
back is in the supine position. biogenesis, lysosome formation, membrane re-
cycling, hormone modulation.
Pronation refers to the palm of the hand being
oriented posteriorly. The body on its belly is the Lysosome
prone position.
Lysosomes are described as containing proteo-
Inversion refers only to the lower extremity, spe- lytic enzymes (hydrolases).Lysosomes contain
cifically the ankle joint. When the foot (plantar acid phosphatase and other hydrolytic enzymes..
surface) is turned inward, so that the sole is These enzymes are enclosed by a membrane
pointing and directed toward the midline of the and are released when needed into the cell or
body and is parallel with the median plane, we into phagocytic vesicles.
speak of inversion. Its opposite is eversion.
Lysosomal enzymes have the capacity to hydro-
Eversion refers to the foot (plantar surface) be- lyze all classes of macromolecules.
ing turned outward so that the sole is pointing
laterally. A generalized list of substrates acted upon by re-
spective enzymes is given below:
Collected by :DeepaRajesh [ 14 ]
rajesh.ks21@gmail.com
Kuwait
Lipids by lipases and phospholipases;
NCLEX Cranial Nerve Review
Proteins by proteases or peptidases;
Polysaccharides by glycosidases; I-Olfactory-Smell

Nucleic acids by nucleases; II-Optic-Vision acuity

Phosphates ( organic-linked) by phosphatases; III-Oculomotor – Eye function

Sulphates (organic-linked) by sulfatases. IV-Trochlear – Eye function

NCLEX Definitions V-Trigeminal – Sensory of the face, chewing

Pachyderma-increased thickness of the skin VI-Abducens – Eye function

Paroxysm-sudden attack VII-Facial – Facial expression, wrinkle forehead,


taste anterior tongue
Pathogenic-disease causing
VIII-Vestibulocochlear – Auditory acuity, balance
Pathologist-individual who studies pathology and postural responses

Pediculosis-condition of lice IX-Glossopharyngeal – Taste on posterior 33%


of the scale
Percutaneous- penetrating the skin
X-Vagus – Cardiac, respiratory reflexes
Pleuritis-inflammation of the pleura
XI-Spinal Accessory - Strength of trapezius and
Pneumonia-disease of the lung related to infec- Sternocleidomastoid muscles
tion
XII-Hypoglossal – Motor function of the tongue
Pneumoconiosis-dust in the lung’s condition
NCLEX Cholinomimetrics
Pneumothorax-air in the chest resulting in the
collapse of a lung 1. Muscarinic Agonists

Pneumatocele-hernia associated with the lung A. Bethanecol (URECHOLINE) – increase GI


motility
Posterior-related to the rear/back position B. Carbachol (ISOPTO, MIOSTAT, CARBA-
CHOL) – various types of glaucoma
Prognosis-opinion of an individual about out- C. Methacholine (PROVOCHOLINE) – test
comes hyperactivity of airways
D. Pilocarpine – used for glaucoma
Pruritus-uncontrollable itching 2. Anticholinesterases

Pyelolithotomy-incision to remove a stone from A. Pysostigmine (ANTILIRIUM) – treat glau-


the renal pelvis coma, crosses BBB, reverse anticholinergic tox-
icity.
Pyeloplasty-repair of the renal pelvis B. Neostigmine (PROSTIGMIN) – synthetic
form of Pysostigmine
Pyosalpinx-pus in the fallopian tube
(Anticholinesterases) – used for Myasthenia
Pyuria-pus in the urine gravis, glaucoma, and to increase tone in blad-
Collected by :DeepaRajesh [ 15 ]
rajesh.ks21@gmail.com
Kuwait
der the water by “bulk flow”
2. Usually limited to movement through gap
Symptoms of Anticholinesterase tox- junctions because size too large for pores.
icity:
Active Transport Review:
1. Miosis
2. Rhinitis 1. Requires energy and requires a transport
3. Bradycardia protein
4. GI spasms 2. Drugs must be similar to some endogenous
5. brochoconstriction substance.
6. involuntary voiding of urine 3. Can carry substances against a gradient
4. Some drugs may exert their effect by in-
creasing or decreasing transport proteins.
NCLEX Drug Distribution Review

Bioavailability dependant on several things: Facilitated Diffusion Review:

1. Route of administration 1. Requires transport protein


2. The drug’s ability to cross membranes 2. Does not require energy
3. The drug’s binding to plasma proteins and 3. Very few drugs move this way
intracellular component
Endocytosis:
Membrane Review:
1. Drug gets engulfed by cell via invagination
1. Membranes separate the body in compo- 2. Very few drugs move this way and only in
nents certain cells.
2. The ability of membranes to act as barriers
is related to its structure Regulation of distribution determined by:
3. Lipid Soluable compounds (many drugs)
pass through by becoming dissolved in the lipid 1. Lipid permeability
bylayer. 2. Blood flow
4. Glucose, H20, electrolytes can’t pass on 3. Binding to plasma proteins
their own. They use pores. 4. Binding to subcellular components
5. In excitable tissues, the pores open and
close. Volume of Distribution (Vd) - is a calculation of
where the drug is distributed.
1. Movement occurs by:
2. passive diffusion Vd = amount of drug given (mg)
3. active transport
4. facilitated diffusion concentration in plasma (mg/ml)
5. endocytosis
Calculate the Vd and compare to the total amount
of body H20 in a person.
Passive Diffusion Review:
-if Vd = total amount of body (approx. 42) is uni-
1. No energy expended.
formly distributed
2. Weak acids and bases need to be in non-
ionized form (no net charge).
-if Vd is less than 42 – retained in plasma and
3. Drugs can also move between cell junctions.
probably bound to plasma proteins
BBB is exception.
4. Must be lipid soluable to pass through
-if Vd is more than 42 – concentrated in tissues
pores.
5. Osmosis is a special case of diffusion
This is not a “real value” but tells you where the
drug is being distributed.
1. A drug dissolved in H2O will move with
Collected by :DeepaRajesh [ 16 ]
rajesh.ks21@gmail.com
Kuwait
Placental Transfer of Drugs
NCLEX Study Guide Recommendation
1. Some drugs cause congenital anomalies
2. Cross placenta by simple diffusion The amount of effort required to pass the NCLEX
3. Must be polar or lipid-insoluable Not to En- varies between students. There is no magic for-
ter mula that students can get plugged into to pass
4. Must assume the fetus is subjected to all the NCLEX test . Consequently, some students
drugs taken by the mother to some extent. may be able pass without reviewing any refer-
ence materials, and other will buy 5-7 reference
guides and attend NCLEX review courses that
NCLEX Score Review are extremely expensive. Obviously, it is foolish
to take the NCLEX without any review of poten-
Usually, NCLEX scores are sent out 2-6 weeks tial questions and NCLEX content. The applica-
after the NCLEX test. Each state board is dif- tion fee and a failing score simply add up to frus-
ferent with score reporting. Don’t call the state tration.
boards for a score report.
NCLEX Study Guide Recommendation
NCLEX Score Review
The NCLEX test takers should be aware of
If you do not pass the NCLEX don’t lose hope. the fact that sometimes too much information
Thousands of good nurses have missed a pass- on the NCLEX can lead to “overkill”. The right
ing score by a few questions. Any given day a study guide offers brevity, precision, and no fluff.
good student can make a few mistakes. You will NCLEX test takers should be aware of the many
be required to wait 3 months to retest for the pitfalls of NCLEX preparation. The NCLEX will
NCLEX. be the most important test nursing students take
in the process of becoming a nurse.
If you don’t pass the NCLEX ask yourself the fol-
lowing questions. If you memorize facts piled upon facts about pa-
tient care in a cram course before the NCLEX,
1. Did I know the material on the NCLEX? you will most likely be distracted by the details
2. Did I feel comfortable with testing format? and not focus on the concepts. A concise review
3. Was there something else going on in my life of the NCLEX that helps you recall details with-
that was a distracter? out giving you all the minute details will work the
4. Did I feel like I was guessing on the NCLEX best, because you will be able to think on test
test? day and not be attempting to regurgitate data.
5. Did I give each Questionmy best
effort?
6. Did I focus on the practical and not book
knowledge regarding patient care?
7. Did I prepare for the minimum number of
questions and not the maximum on the NCLEX
test?

Use the above questions to improve your focus


for your NCLEX preparation. Hundreds if not
thousands of nursing students have never gone
back to take the NCLEX test the second time.
The NCLEX is just another hurdle in a series of
qualifying requirements to become a nurse. Don’t
lose hope and become a better nurse by pursing
knowledge that will help you pass the NCLEX.

Collected by :DeepaRajesh [ 17 ]
rajesh.ks21@gmail.com
Kuwait
Free NCLEX-RN
Sample Test Questions

Collected by :DeepaRajesh [ 18 ]
rajesh.ks21@gmail.com
Kuwait
1. The correct answer is B.
Question: What are the needs of the patient with Needed Info: Mask, eye protection, face shield
acute lymphocytic leukemia and thrombocytope- protect mucous membrane exposure; used if ac-
nia? tivities are likely to generate splash or sprays.
Gowns used if activities are likely to generate
Needed Info: Lymphocytic leukemia, disease splashes or sprays.
characterized by proliferation of immature WBCs.
Immature cells unable to fight infection as com- (A) Gloves, gown, goggles, and surgical cap —
petently as mature white cells. Treatment: chem- surgical caps offer protection to hair but aren>t
otherapy, antibiotics, blood transfusions, bone required.
marrow transplantation. Nursing responsibilities: (B) Sterile gloves, mask, plastic bags, and gown
private room, no raw fruits or vegs, small fre- — plastic bags provide no direct protection and
quent meals, O2, good skin care. aren>t part of universal precautions
(C) Gloves, gown, mask, and goggles — COR-
(A) to a private room so she will not infect other RECT: must use universal precautions on ALL
patients and health care workers — poses little patients; prevent skin and mucous membrane
or no threat exposure when contact with blood or other body
(B) to a private room so she will not be infected by fluids is anticipated
other patients and health care workers — COR- (D) Double gloves, goggles, mask, and surgical
RECT: protects patient from exogenous bacteria, cap — surgical cap not required; unnecessary to
risk for developing infection from others due to double glove
depressed WBC count, alters ability to fight in- 4. The correct answer is B.
fection Question: What is the best position after tonsillec-
(C) to a semiprivate room so she will have stim- tomy to help with drainage of oral secretions?
ulation during her hospitalization — should be
placed in a room alone Strategy: Picture the patient as described.
(D) to a semiprivate room so she will have the
opportunity to express her feelings about her ill- (A) Sims> — on side with top knee flexed and
ness — ensure that patient is provided with op- thigh drawn up to chest and lower knee less
portunities to express feelings about illness sharply flexed: used for vaginal or rectal exami-
nation
2. The correct answer is A. (B) Side-lying — CORRECT: most effective to
Question: What is the BEST way to prevent ac- facilitate drainage of secretions from the mouth
cidental poisoning in children? and pharynx; reduces possibility of airway ob-
struction.
Strategy: Picture toddlers at play. (C) Supine — increased risk for aspiration, would
not facilitate drainage of oral secretions
(A) Lock all medications in a cabinet — COR- (D) Prone — risk for airway obstruction and as-
RECT: improper storage most common cause of piration, unable to observe the child for signs of
poisoning; highest incidence in two-year-olds bleeding such as increased swallowing
(B) Child proof all the caps to medication bottles 5. The correct answer is A.
— children can open Question: Which patient is an appropriate as-
(C) Store medications on the highest shelf in a signment for the LPN/LVN?
cupboard — toddlers climb
(D) Place medications in different containers — Strategy: Think about the skill level involved in
keep in original container each patient>s care.

3. The correct answer is C. Needed Info: LPN/LVN: assists with implementa-


Question: What is the correct universal precau- tion of care; performs procedures; differentiates
tion? normal from abnormal; cares for stable patients
with predictable conditions; has knowledge of
Strategy: Think about each answer choice. How asepsis and dressing changes; administers med-
is each measure protecting the nurse? ications (varies with educational background and
Collected by :DeepaRajesh [ 19 ]
rajesh.ks21@gmail.com
Kuwait
state nurse practice act). (C) Above the umbilicus, on the mother>s left
side — found in breech presentation
(A) A 72-year-old patient with diabetes who re- (D) Above the umbilicus, on the mother>s right
quires a dressing change for a stasis ulcer — side — found in breech presentation
CORRECT: stable patient with an expected out- 2. The correct answer is B.
come Question: What is a contraindication to receiving
(B) A 42-year-old patient with cancer of the bone flu vaccine?
complaining of pain — requires assessment; RN
is the appropriate caregiver Strategy: Think about what each answer choice
(C) A 55-year-old patient with terminal cancer means.
being transferred to hospice home care — re-
quires nursing judgement; RN is the appropriate Needed Info: Influenza vaccine: given yearly,
caregiver preferably Oct.-Nov.; recommended for people
(D) A 23-year-old patient with a fracture of the age 65 or older; people under 65 with heart dis-
right leg who asks to use the urinal — standard ease, lung disease, diabetes, immuno-suppres-
unchanging procedure; assign to the nursing as- sion, chronic care facility residents.
sistant
(A) A 45-year-old male who is allergic to shellfish
— allergy to eggs is a contraindication
Health Promotion and Mainte- (B) A 60-year-old female who says she has a
sore throat — CORRECT: vaccine deferred in
nance presence of acute respiratory disease
1. The answer is B. (C) A 66-year-old female who lives in a group
Question: The fetus is ROA. Where should the home — vaccine deferred only if patient has an
nurse listen for the FHT? active immunization
(D) A 70-year-old female with congestive heart
Strategy: Picture the situation described. It may failure — no contraindication
be helpful for you to draw this out so that you can 3. The correct answer is D.
imagine where the heartbeat would be found. Question: What is the treatment for thrush?

Needed Info: Describing fetal position: practice Strategy: Determine the outcome of each an-
of defining position of baby relative to mother>s swer choice.
pelvis. The point of maximum intensity (PMI) of
the fetus: point on the mother>s abdomen where Needed Info: Thrush (oral candidiasis): white
the FHT is the loudest, usually over the fetal back. plaque on oral mucous membranes, gums, or
Divide the mother>s pelvis into 4 parts or quad- tongue; treatment includes good handwashing,
rants: right and left anterior, which is the front, nystatin (Mycostatin).
and right and left posterior, which is the back. Ab-
breviated: R and L for right and left, and A and P
(A) Determine the baby>s blood glucose level —
for anterior and posterior. The head, particularly
thrush in newborns caused by poor handwashing
the occiput, is the most common presenting part,or exposure to an infected vagina during birth
and is abbreviated O. LOA is most common fetal (B) Suggest that the newborn>s formula be
position and FHT heard on left side. In a vertexchanged — not related to thrush
presentation, FHT is heard below the umbilicus. (C) Remind the caretaker not to let the infant
In a breech presentation, FHT is heard above sleep with the bottle — not related to thrush
the umbilicus. (D) Explain that the newborn will need to receive
some medication — CORRECT: thrush most of-
(A) Below the umbilicus, on the mother>s left ten treated with nystatin (Mycostatin)
side — found on right not left side 4. The correct answer is C.
(B) Below the umbilicus, on the mother>s right Question: What will you see with congenital hip
side — CORRECT: occiput and back are press- dislocation?
ing against right side of mother>s abdomen; FHT
would be heard below umbilicus on right side Strategy: Form a mental image of the deformity.
Collected by :DeepaRajesh [ 20 ]
rajesh.ks21@gmail.com
Kuwait
2. The correct answer is B.
Needed Info: Subluxation: most common type Question: What is your responsibility concerning
of congenital hip dislocation. Head of femur re- informed consent?
mains in contact with acetabulum but is partially
displaced. Diagnosed in infant less than 4 weeks Needed Info: Physician>s responsibility to obtain
old S/S: unlevel gluteal folds, limited abduction informed consent.
of hip, shortened femur affected side, Ortolani>s
sign (click). Treatment: abduction splint, hip spi- (A) The nurse should explain the procedure to
ca cast, Bryant>s traction, open reduction. the patient and ask her to sign the consent form
— Physician should get patient to sign consent
(A) lengthening of the limb on the affected side (B) The nurse should verify that the consent form
— inaccurate has been signed by the patient and that it is at-
(B) deformities of the foot and ankle — inaccu- tached to her chart — CORRECT
rate (C) The nurse should tell the physician that the
(C) asymmetry of the gluteal and thigh folds — patient agrees to have the examination — Physi-
CORRECT: restricted movement on affected cian should explain procedure and get consent
side form signed
(D) plantar flexion of the foot — seen with club- (D) The nurse should verify that the patient or a
foot family member has signed the consent form —
5. The correct answer is D. must be signed by patient unless unable to do
Question: How do you determine the frequency 3. The correct answer is C.
of uterine contractions? Question: What should you do to communicate
with a person with a moderate hearing loss?
Needed Info: There must be at least 3 contrac-
tions to establish frequency. Needed Info: Presbycusis: age-related hearing
loss due to inner ear changes. Decreased ability
(A) from the beginning of one contraction to the to hear high sounds.
end of the next contraction — not accurate
(B) from the beginning of one contraction to the(A) Raise your voice until the patient is able to
end of the same contraction — defines duration hear you — would result in high tones patient
(C) by the strength of the contraction at its peak
unable to hear
— describes intensity (B) Face the patient and speak quickly using a
(D) by the number of contractions that occur high voice — usually unable to hear high tones
within a given period of time — CORRECT (C) Face the patient and speak slowly using a
slightly lowered voice — CORRECT: also de-
1. The correct answer is C. crease background noise; speak at a slow pace,
use nonverbal cues
Question: What is the goal of family therapy? (D) Use facial expressions and speak as you
would normally — nonverbal cues help, but need
Needed Info: Symptoms of depression: a low low tones
self-esteem, obsessive thoughts, regressive be- 4. The correct answer is C.
havior, unkempt appearance, a lack of energy,
weight loss, decreased concentration, withdrawn Question: What is the reason for the wife>s be-
behavior. havior?

(A) trust the nurse who will solve his problem — Needed Info: Stages of grief: 1) shock and dis-
not realistic belief, 2) awareness of pain and loss, 3) restitu-
(B) learn to live with anxiety and tension — mini- tion. Acute period: 4-8 weeks, usual resolution:
mizes concerns 1 year.
(C) accept responsibility for his actions and
choices — CORRECT (A) She has already moved through the stages
(D) use the members of the therapeutic milieu to of the grieving process — takes one year
solve his problems — must do it himself (B) She is repressing anger related to her
Collected by :DeepaRajesh [ 21 ]
rajesh.ks21@gmail.com
Kuwait
husband>s death — not accurate; second stage: acute lymphocytic leukemia and thromocytope-
crying, regression nia?
(C) She is experiencing shock and disbelief re-
lated to her husband>s death — CORRECT: de- Needed Info: Thromocytopenia: decreased
nial first stage; inability to comprehend reality of platelet count increases the patient>s risk for in-
situation jury, normal count: 200,000-400,000 per mm3.
(D) She is demonstrating resolution of her Leukemia: group of malignant disorders involv-
husband>s death — too soon ing overproduction of immature leukocytes in
5. The correct answer is C. bone marrow. This shuts down normal bone mar-
row production of erythrocytes, platelets, normal
Question: Is the depression normal, or some- leukocytes. Causes anemia, leukopenia, and
thing to be concerned about? thrombocytopenia leading to infection and hem-
orrhage. Symptoms: pallor of nail beds and con-
(A) The treatment plan is not effective; the patient junctiva, petechiae (small hemorrhagic spot on
requires a larger dose of lithium — not accurate skin), tachycardia, dyspnea, weight loss, fatigue.
(B) This is a normal response to lithium therapy; Treatment: chemotherapy, antibiotics, blood
the patient should continue with the current treat- transfusions, bone marrow transplantation. Nurs-
ment plan — does not address safety needs ing responsibilities: private room, no raw fruits or
(C) This is a normal response to lithium therapy; vegs, small frequent meals, O2, good skin care.
the patient should be monitored for suicidal be-
havior — CORRECT: delay of 1-3 weeks before (A) Potential for injury — CORRECT: low plate-
med benefits seen let increases risk of bleeding from even minor
(D) The treatment plan is not effective; the patient injuries. Safety measures: shave with an elec-
requires an antidepressant — normal response tric razor, use soft tooth brush, avoid SQ or IM
meds and invasive procedures (urinary drainage
catheter or a nasogastric tube), side-rails up, re-
move sharp objects, frequently assess for signs
Physiological Integrity of bleeding, bruising, hemorrhage.
1. The correct answer is B. (B) Self-care deficit — may feel weak, doesn>t
Question: Which lab values should you monitor address condition
for a patient receiving Gentamicin? (C) Potential for self-harm — implies risk for pur-
poseful self-injury, not given any info, assump-
Needed Info: Gentamicin: broad spectrum an- tion
tibiotic. Side effects: neuromuscular blockage, (D) Alteration in comfort — patient is not com-
ototoxic to eighth cranial nerve (tinnitus, vertigo, fortable, and comfort measures would address
ataxia, nystagmus, hearing loss), nephrotoxic. problem
Nursing responsibilities: monitor renal function,
3. The correct answer is C.
force fluids, monitor hearing acuity. Draw blood
Question: What is the best site for nitroglycerine
for peak levels 1 hr. after IM and 30 min - 1 hr. af-
ointment?
ter IV infusion, draw blood for trough just before
next dose.
Strategy: Think about each site.
(A) Hemoglobin and hematocrit — can cause
Needed Info: Nitroglycerine: used in treatment
anemia; less common
of angina pectoris to reduce ischemia and re-
(B) BUN and creatinine — CORRECT: nephro-
lieve pain by decreasing myocardial oxygen
toxic; will see proteinuria, oliguria, hematuria,
consumption; dilates veins and arteries. Side
thirst, increased BUN, decreased creatine clear-
effects: throbbing headache, flushing, hypoten-
ance
sion, tachycardia. Nursing responsibilities: teach
(C) Platelet count and clotting time — do not
appropriate administration, storage, expected
usually change
pain relief, side effects. Ointment applied to skin;
(D) Sodium and potassium — hypokalemia infre-
sites rotated to avoid skin irritaion. Prolonged ef-
quent problem
fect up to 24 hours.
2. The correct answer is A.
Question: What nursing diagnosis is seen with
Collected by :DeepaRajesh [ 22 ]
rajesh.ks21@gmail.com
Kuwait
(A) muscular — not most important
(B) near the heart — not most important Review Information: The correct answer is D:
(C) non-hairy — CORRECT: skin site free of hair Improve venous return. Elevating the leg both
will increase absorption; avoid distal part of ex- improves venous return and reduces swelling.
tremities due to less than maximal absorption Client comfort will be improved as well.
(D) over a bony prominence — most important is
that the site be non-hairy Question2
4. The correct answer is B. The nurse is reviewing with a client how to col-
Question: Why is a patient defibrillated? lect a clean catch urine specimen. What is the
appropriate sequence to teach the client?
Strategy: Think about each answer choice.
A) Clean the meatus, begin voiding, then catch
Needed Info: Defibrillation: produces asystole urine stream
of heart to provide opportunity for natural pace- B) Void a little, clean the meatus, then collect
maker (SA node) to resume as pacer of heart specimen
activity. C) Clean the meatus, then urinate into container
D) Void continuously and catch some of the
(A) increase cardiac contractility and cardiac out- urine
put — inaccurate
(B) cause asystole so the normal pacemaker Review Information: The correct answer is
can recapture — CORRECT: allows SA node to A: Clean the meatus, begin voiding, then catch
resume as pacer of heart activity urine stream. A clean catch urine is difficult to
(C) reduce cardiac ischemia and acidosis — in- obtain and requires clear directions. Instructing
accurate the client to carefully clean the meatus, then void
(D) provide energy for depleted myocardial cells naturally with a steady stream prevents surface
— inaccurate bacteria from contaminating the urine specimen.
5. The correct answer is C. As starting and stopping flow can be difficult,
Question: How should you regulate the IV flow once the client begins voiding it>>s best to just
rate? slip the container into the stream. Other respons-
es do not reflect correct technique.
Strategy: Use formula and avoid making math
errors. Question3
Following change-of-shift report on an orthoped-
Needed Info: total volume x the drop factor di- ic unit, which client should the nurse see first?
vided by the total time in minutes. A) 16 year-old who had an open reduction of a
fractured wrist 10 hours ago
(A) 21 — inaccurate B) 20 year-old in skeletal traction for 2 weeks
(B) 28 — inaccurate since a motor cycle accident
(C) 31 — CORRECT: 3,000 x 15 divided by 24 C) 72 year-old recovering from surgery after a
x 60 hip replacement 2 hours ago
(D) 42 — inaccurate D) 75 year-old who is in skin traction prior to
planned hip pinning surgery.

Review Information: The correct answer is C:


Question1 72 year-old recovering from surgery after a hip
A client has been hospitalized after an automo- replacement 2 hours ago. Look for the client who
bile accident. A full leg cast was applied in the has the most imminent risks and acute vulnerabil-
emergency room. The most important reason for ity. The client who returned from surgery 2 hours
the nurse to elevate the casted leg is to ago is at risk for life threatening hemorrhage and
A) Promote the client>s comfort should be seen first. The 16 year-old should be
B) Reduce the drying time seen next because it is still the first post-op day.
C) Decrease irritation to the skin The 75 year-old is potentially vulnerable to age-
D) Improve venous return related physical and cognitive consequences in
Collected by :DeepaRajesh [ 23 ]
rajesh.ks21@gmail.com
Kuwait
skin traction should be seen next. The client who D) Repeat the peak flow reading in 30 minutes
can safely be seen last is the 20 year-old who is
2 weeks post-injury. Review Information: The correct answer is
Question4 B: Administer the prn dose of albuterol. Peak
A client with Guillain Barre is in a nonresponsive flow monitoring during exacerbations of asthma
state, yet vital signs are stable and breathing is is recommended for clients with moderate-to-
independent. What should the nurse document severe persistent asthma to determine the se-
to most accurately describe the client>s condi- verity of the exacerbation and to guide the treat-
tion? ment. A peak flow reading of less than 50% of
A) Comatose, breathing unlabored the client>>s baseline reading is a medical alert
B) Glascow Coma Scale 8, respirations regular condition and a short-acting beta-agonist must
C) Appears to be sleeping, vital signs stable be taken immediately.
D) Glascow Coma Scale 13, no ventilator re- Question7
quired A client had 20 mg of Lasix (furosemide) PO at
10 AM. Which would be essential for the nurse to
Review Information: The correct answer is include at the change of shift report?
B: Glascow Coma Scale 8, respirations regular. A) The client lost 2 pounds in 24 hours
The Glascow Coma Scale provides a standard B) The client’s potassium level is 4 mEq/liter.
reference for assessing or monitoring level of C) The client’s urine output was 1500 cc in 5
consciousness. Any score less than 13 indicates hours
a neurological impairment. Using the term coma- D) The client is to receive another dose of Lasix
tose provides too much room for interpretation at 10 PM
and is not very precise.
Question5 Review Information: The correct answer is C:
When caring for a client receiving warfarin so- The client’s urine output was 1500 cc in 5 hours.
dium (Coumadin), which lab test would the nurse Although all of these may be correct information
monitor to determine therapeutic response to the to include in report, the essential piece would be
drug? the urine output.
A) Bleeding time Question8
B) Coagulation time A client has been tentatively diagnosed with
C) Prothrombin time Graves> disease (hyperthyroidism). Which of
D) Partial thromboplastin time these findings noted on the initial nursing assess-
ment requires quick intervention by the nurse?
Review Information: The correct answer is A) a report of 10 pounds weight loss in the last
C: Prothrombin time. Coumadin is ordered daily, month
based on the client>>s prothrombin time (PT). B) a comment by the client «I just can>t sit
This test evaluates the adequacy of the extrinsic still.»
system and common pathway in the clotting cas- C) the appearance of eyeballs that appear to
cade; Coumadin affects the Vitamin K depend- «pop» out of the client>s eye sockets
ent clotting factors. D) a report of the sudden onset of irritability in
the past 2 weeks

Question6 Review Information: The correct answer is C:


A client with moderate persistent asthma is ad- the appearance of eyeballs that appear to «pop»
mitted for a minor surgical procedure. On ad- out of the client>>s eye sockets. Exophthalmos
mission the peak flow meter is measured at 480 or protruding eyeballs is a distinctive characteris-
liters/minute. Post-operatively the client is com- tic of Graves>> Disease. It can result in corneal
plaining of chest tightness. The peak flow has abrasions with severe eye pain or damage when
dropped to 200 liters/minute. What should the the eyelid is unable to blink down over the pro-
nurse do first? truding eyeball. Eye drops or ointment may be
A) Notify both the surgeon and provider needed.
B) Administer the prn dose of albuterol Question9
C) Apply oxygen at 2 liters per nasal cannula The nurse has performed the initial assessments
Collected by :DeepaRajesh [ 24 ]
rajesh.ks21@gmail.com
Kuwait
of 4 clients admitted with an acute episode of B) Incontinence
asthma. Which assessment finding would cause C) Muscle weakness
the nurse to call the provider immediately? D) Rapid pulse
A) prolonged inspiration with each breath
B) expiratory wheezes that are suddenly absent Review Information: The correct answer is A:
in 1 lobe Slurred speech. Changes in speech patterns and
C) expectoration of large amounts of purulent level of conscious can be indicators of continued
mucous intracranial bleeding or extension of the stroke.
D) appearance of the use of abdominal muscles Further diagnostic testing may be indicated.
for breathing Question12
A school-aged child has had a long leg (hip to
Review Information: The correct answer is B: ankle) synthetic cast applied 4 hours ago. Which
expiratory wheezes that are suddenly absent in 1 statement from the parent indicates that teach-
lobe. Acute asthma is characterized by expiratory ing has been inadequate?
wheezes caused by obstruction of the airways. A) «I will keep the cast uncovered for the next
Wheezes are a high pitched musical sounds pro- day to prevent burning of the skin.»
duced by air moving through narrowed airways. B) «I can apply an ice pack over the area to re-
Clients often associate wheezes with the feeling lieve itching inside the cast.»
of tightness in the chest. However, sudden ces- C) «The cast should be propped on at least 2 pil-
sation of wheezing is an ominous or bad sign lows when my child is lying down.»
that indicates an emergency -- the small airways D) «I think I remember that my child should not
are now collapsed. stand until after 72 hours.»
Question10
During the initial home visit, a nurse is discuss- Review Information: The correct answer is D:
ing the care of a client newly diagnosed with «I think I remember that my child should not stand
Alzheimer>s disease with family members. until after 72 hours.». Synthetic casts will typi-
Which of these interventions would be most cally set up in 30 minutes and dry in a few hours.
helpful at this time? Thus, the client may stand within the initial 24
A) leave a book about relaxation techniques hours. With plaster casts, the set up and drying
B) write out a daily exercise routine for them to time, especially in a long leg cast which is thicker
assist the client to do than an arm cast, can take up to 72 hours. Both
C) list actions to improve the client>s daily nutri- types of casts give off a lot of heat when drying
tional intake and it is preferable to keep the cast uncovered
D) suggest communication strategies for the first 24 hours. Clients may complain of a
chill from the wet cast and therefore can simply
Review Information: The correct answer is D: be covered lightly with a sheet or blanket. Apply-
suggest communication strategies. Alzheimer>>s ing ice is a safe method of relieving the itching.
disease, a progressive chronic illness, greatly
challenges caregivers. The nurse can be of
greatest assistance in helping the family to use Question13
communication strategies to enhance their ability Which blood serum finding in a client with dia-
to relate to the client. By use of select verbal and betic ketoacidosis alerts the nurse that immedi-
nonverbal communication strategies the family ate action is required?
can best support the client’s strengths and cope A) pH below 7.3
with any aberrant behavior. B) Potassium of 5.0
Question11 C) HCT of 60
An 80 year-old client admitted with a diagnosis D) Pa O2 of 79%
of possible cerebral vascular accident has had a
blood pressure from 160/100 to 180/110 over the Review Information: The correct answer is C:
past 2 hours. The nurse has also noted increased HCT of 60. This high hematocrit is indicative of
lethargy. Which assessment finding should the severe dehydration which requires priority atten-
nurse report immediately to the provider? tion in diabetic ketoacidosis. Without sufficient
A) Slurred speech hydration, all systems of the body are at risk
Collected by :DeepaRajesh [ 25 ]
rajesh.ks21@gmail.com
Kuwait
for hypoxia from a lack of or sluggish circula- D) synthetic skin.»
tion. In the absence of insulin, which facilitates
the transport of glucose into the cell, the body Review Information: The correct answer is C:
breaks down fats and proteins to supply energy my thigh.». Autografts are done with tissue trans-
ketones, a by-product of fat metabolism. These planted from the client>>s own skin.
accumulate causing metabolic acidosis (pH < Question17
7.3), which would be the second concern for this A client is admitted to the emergency room fol-
client. The potassium and PaO2 levels are near lowing an acute asthma attack. Which of the fol-
normal. lowing assessments would be expected by the
Question14 nurse?
The nurse is preparing a client with a deep vein
thrombosis (DVT) for a Venous Doppler evalua- A) Diffuse expiratory wheezing
tion. Which of the following would be necessary B) Loose, productive cough
for preparing the client for this test? C) No relief from inhalant
A) Client should be NPO after midnight D) Fever and chills
B) Client should receive a sedative medication
prior to the test Review Information: The correct answer is A:
C) Discontinue anti-coagulant therapy prior to Diffuse expiratory wheezing. In asthma, the air-
the test ways are narrowed, creating difficulty getting air
D) No special preparation is necessary in. A wheezing sound results.
Question18
Review Information: The correct answer is A client has been admitted with a fractured femur
D: No special preparation is necessary. This is and has been placed in skeletal traction. Which
a non-invasive procedure and does not require of the following nursing interventions should re-
preparation other than client education. ceive priority?
Question15 A) Maintaining proper body alignment
A client is admitted with infective endocarditis B) Frequent neurovascular assessments of the
(IE). Which finding would alert the nurse to a affected leg
complication of this condition? C) Inspection of pin sites for evidence of drain-
A) dyspnea age or inflammation
B) heart murmur D) Applying an over-bed trapeze to assist the cli-
C) macular rash ent with movement in bed
D) hemorrhage
Review Information: The correct answer is
Review Information: The correct answer is B: Frequent neurovascular assessments of the
B: heart murmur. Large, soft, rapidly developing affected leg. The most important activity for the
vegetations attach to the heart valves. They have nurse is to assess neurovascular status. Com-
a tendency to break off, causing emboli and leav- partment syndrome is a serious complication of
ing ulcerations on the valve leaflets. These em- fractures. Prompt recognition of this neurovascu-
boli produce findings of cardiac murmur, fever, lar problem and early intervention may prevent
anorexia, malaise and neurologic sequelae of permanent limb damage.
emboli. Furthermore, the vegetations may travel Question19
to various organs such as spleen, kidney, coro- The nurse is assigned to care for a client who
nary artery, brain and lungs, and obstruct blood had a myocardial infarction (MI) 2 days ago. The
flow. client has many questions about this condition.
Question16 What area is a priority for the nurse to discuss at
The nurse explains an autograft to a client sched- this time?
uled for excision of a skin tumor. The nurse knows A) Daily needs and concerns
the client understands the procedure when the B) The overview cardiac rehabilitation
client says, «I will receive tissue from C) Medication and diet guideline
A) a tissue bank.» D) Activity and rest guidelines
B) a pig.»
C) my thigh.» Review Information: The correct answer is A:
Collected by :DeepaRajesh [ 26 ]
rajesh.ks21@gmail.com
Kuwait
Daily needs and concerns. At 2 days post-MI, the es, milk
client’s education should be focused on the im-
mediate needs and concerns for the day. Review Information: The correct answer is B:
Question20 Ground beef patty, lima beans, wheat roll, raisins,
A 3 year-old child is brought to the clinic by his milk. Iron rich foods include red meat, fish, egg
grandmother to be seen for «scratching his bot- yolks, green leafy vegetables, legumes, whole
tom and wetting the bed at night.» Based on grains, and dried fruits such as raisins. This din-
these complaints, the nurse would initially as- ner is the best choice: It is high in iron and is ap-
sess for which problem? propriate for a toddler.
A) allergies Question23
B) scabies The nurse admitting a 5 month-old who vomited
C) regression 9 times in the past 6 hours should observe for
D) pinworms signs of which overall imbalance?
A) Metabolic acidosis
Review Information: The correct answer is B) Metabolic alkalosis
D: pinworms. Signs of pinworm infection include C) Some increase in the serum hemoglobin
intense perianal itching, poor sleep patterns, D) A little decrease in the serum potassium
general irritability, restlessness, bed-wetting,
distractibility and short attention span. Scabies Review Information: The correct answer is B:
is an itchy skin condition caused by a tiny, eight- Metabolic alkalosis. Vomiting causes loss of acid
legged burrowing mite called Sarcoptes scabiei . from the stomach. Prolonged vomiting can re-
The presence of the mite leads to intense itching sult in excess loss of acid and lead to metabolic
in the area of its burrows. alkalosis. Findings include irritability, increased
Question21 activity, hyperactive reflexes, muscle twitching
The nurse is caring for a newborn with tra- and elevated pulse. Options C and D are correct
cheoesophageal fistula. Which nursing diagno- answers but not the best answers since they are
sis is a priority? too general.
A) Risk for dehydration Question24
B) Ineffective airway clearance A two year-old child is brought to the provider>s
C) Altered nutrition office with a chief complaint of mild diarrhea for
D) Risk for injury two days. Nutritional counseling by the nurse
should include which statement?
Review Information: The correct answer is B: A) Place the child on clear liquids and gelatin for
Ineffective airway clearance. The most common 24 hours
form of TEF is one in which the proximal esopha- B) Continue with the regular diet and include oral
geal segment terminates in a blind pouch and rehydration fluids
the distal segment is connected to the trachea C) Give bananas, apples, rice and toast as toler-
or primary bronchus by a short fistula at or near ated
the bifurcation. Thus, a priority is maintaining an D) Place NPO for 24 hours, then rehydrate with
open airway, preventing aspiration. Other nurs- milk and water
ing diagnoses are then addressed.
Question22 Review Information: The correct answer is B:
The nurse is developing a meal plan that would Continue with the regular diet and include oral
provide the maximum possible amount of iron for rehydration fluids. Current recommendations for
a child with anemia. Which dinner menu would mild to moderate diarrhea are to maintain a nor-
be best? mal diet with fluids to rehydrate.
A) Fish sticks, french fries, banana, cookies, Question25
milk The nurse is teaching parents about the appro-
B) Ground beef patty, lima beans, wheat roll, rai- priate diet for a 4 month-old infant with gastro-
sins, milk enteritis and mild dehydration. In addition to oral
C) Chicken nuggets, macaroni, peas, canta- rehydration fluids, the diet should include
loupe, milk
D) Peanut butter and jelly sandwich, apple slic- A) formula or breast milk
Collected by :DeepaRajesh [ 27 ]
rajesh.ks21@gmail.com
Kuwait
B) broth and tea ginning with cereal
C) rice cereal and apple juice B) Finely ground meat should be started early to
D) gelatin and ginger ale provide iron
C) Egg white is added early to increase protein
Review Information: The correct answer is A: intake
formula or breast milk. The usual diet for a young D) Solid foods should be mixed with formula in
infant should be followed. a bottle
Question26
A child is injured on the school playground and Review Information: The correct answer is A:
appears to have a fractured leg. The first action Solid foods are introduced one at a time begin-
the school nurse should take is ning with cereal. Solid foods should be added
one at a time between 4-6 months. If the infant is
A) call for emergency transport to the hospital able to tolerate the food, another may be added
B) immobilize the limb and joints above and be- in a week. Iron fortified cereal is the recommend-
low the injury ed first food.
C) assess the child and the extent of the injury Question29
D) apply cold compresses to the injured area The nurse planning care for a 12 year-old child
with sickle cell disease in a vaso-occlusive crisis
Review Information: The correct answer is of the elbow should include which one of the fol-
C: assess the child and the extent of the injury. lowing as a priority?
When applying the nursing process, assessment
is the first step in providing care. The «5 Ps» A) Limit fluids
of vascular impairment can be used as a guide B) Client controlled analgesia
(pain, pulse, pallor, paresthesia, paralysis). C) Cold compresses to elbow
Question27 D) Passive range of motion exercise
The mother of a 3 month-old infant tells the nurse
that she wants to change from formula to whole Review Information: The correct answer is B:
milk and add cereal and meats to the diet. What Client controlled analgesia. Management of a
should be emphasized as the nurse teaches sickle cell crisis is directed towards supportive
about infant nutrition? and symptomatic treatment. The priority of care
is pain relief. In a 12 year-old child, client control-
A) Solid foods should be introduced at 3-4 led analgesia promotes maximum comfort.
months Question30
B) Whole milk is difficult for a young infant to di- The nurse is performing a physical assessment
gest on a toddler. Which of the following actions
C) Fluoridated tap water should be used to dilute should be the first?
milk
D) Supplemental apple juice can be used be- A) Perform traumatic procedures
tween feedings B) Use minimal physical contact
C) Proceed from head to toe
Review Information: The correct answer is B: D) Explain the exam in detail
Whole milk is difficult for a young infant to digest.
Cow>>s milk is not given to infants younger than Review Information: The correct answer
1 year because the tough, hard curd is difficult to is B: Use minimal physical contact. The nurse
digest. In addition, it contains little iron and cre- should approach the toddler slowly and use min-
ates a high renal solute load. imal physical contact initially so as to gain the
Question28 toddler>>s cooperation. Be flexible in the se-
The nurse is preparing a handout on infant feed- quence of the exam, and give only brief simple
ing to be distributed to families visiting the clinic. explanations just prior to the action.
Which notation should be included in the teach- Question31
ing materials? What finding signifies that children have attained
the stage of concrete operations (Piaget)?
A) Solid foods are introduced one at a time be-
Collected by :DeepaRajesh [ 28 ]
rajesh.ks21@gmail.com
Kuwait
A) Explores the environment with the use of sight
and movement
B) Thinks in mental images or word pictures Question34
C) Makes the moral judgment that «stealing is While teaching the family of a child who will take
wrong» phenytoin (Dilantin) regularly for seizure control,
D) Reasons that homework is time-consuming it is most important for the nurse to teach them
yet necessary about which of the following actions?

Review Information: The correct answer is A) Maintain good oral hygiene and dental care
C: Makes the moral judgment that «stealing is B) Omit medication if the child is seizure free
wrong». The stage of concrete operations is de- C) Administer acetaminophen to promote sleep
picted by logical thinking and moral judgments. D) Serve a diet that is high in iron
Question32
The mother of a child with a neural tube defect Review Information: The correct answer is
asks the nurse what she can do to decrease the A: Maintain good oral hygiene and dental care.
chances of having another baby with a neural Swollen and tender gums occur often with use of
tube defect. What is the best response by the phenytoin. Good oral hygiene and regular visits
nurse? to the dentist should be emphasized.

A) «Folic acid should be taken before and after


conception.» Question35
B) «Multivitamin supplements are recommended The nurse is offering safety instructions to a par-
during pregnancy.» ent with a four month-old infant and a four year-
C) «A well balanced diet promotes normal fetal old child. Which statement by the parent indi-
development.» cates understanding of appropriate precautions
D) «Increased dietary iron improves the health of to take with the children?
mother and fetus.»
A) «I strap the infant car seat on the front seat to
Review Information: The correct answer is face backwards.»
A: «Folic acid should be taken before and after B) «I place my infant in the middle of the living
conception.». The American Academy of Pedi- room floor on a blanket to play with my four year-
atrics recommends that all childbearing women old while I make supper in the kitchen.»
increase folic acid from dietary sources and/or C) «My sleeping baby lies so cute in the crib with
supplements. There is evidence that increased the little buttocks stuck up in the air while the four
amounts of folic acid prevents neural tube de- year-old naps on the sofa.»
fects. D) «I have the four year-old hold and help feed
the four month-old a bottle in the kitchen while I
make supper.»
Question33
The provider orders Lanoxin (digoxin) 0.125 mg Review Information: The correct answer is
PO and furosemide 40 mg every day. Which of D: «I have the four year-old hold and help feed
these foods would the nurse reinforce for the cli- the four month-old a bottle in the kitchen while I
ent to eat at least daily? make supper.». The infant seat is to be placed
on the rear seat. Small children and infants are
A) Spaghetti not to be left unsupervised. Infants are
B) Watermelon
C) Chicken
D) Tomatoes Question36
The nurse admits a 7 year-old to the emergency
Review Information: The correct answer is B: room after a leg injury. The x-rays show a femur
Watermelon. Watermelon is high in potassium fracture near the epiphysis. The parents ask what
and will replace potassium lost by the diuretic. will be the outcome of this injury. The appropriate
The other foods are not high in potassium. response by the nurse should be which of these
Collected by :DeepaRajesh [ 29 ]
rajesh.ks21@gmail.com
Kuwait
statements?

A) «The injury is expected to heal quickly be- Question39


cause of thin periosteum.» A nurse is providing a parenting class to individu-
B) «In some instances the result is a retarded als living in a community of older homes. In dis-
bone growth.» cussing formula preparation, which of the follow-
C) «Bone growth is stimulated in the affected ing is most important to prevent lead poisoning?
leg.»
D) «This type of injury shows more rapid union A) Use ready-to-feed commercial infant formula
than that of younger children.» B) Boil the tap water for 10 minutes prior to pre-
paring the formula
Review Information: The correct answer is B: C) Let tap water run for 2 minutes before adding
«In some instances the result is a retarded bone to concentrate
growth.». An epiphyseal (growth) plate fracture D) Buy bottled water labeled «lead free» to mix
in a 7 year-old often results in retarded bone the formula
growth. The leg often will be different in length
than the uninjured leg. Review Information: The correct answer is C:
Let tap water run for 2 minutes before adding to
concentrate. Use of lead-contaminated water to
Question37 prepare formula is a major source of poisoning
The parents of a 4 year-old hospitalized child tell in infants. Drinking water may be contaminated
the nurse, “We are leaving now and will be back by lead from old lead pipes or lead solder used
at 6 PM.” A few hours later the child asks the in sealing water pipes. Letting tap water run for
nurse when the parents will come again. What is several minutes will diminish the lead contami-
the best response by the nurse? nation.

A) «They will be back right after supper.»


B) «In about 2 hours, you will see them.» Question40
C) «After you play awhile, they will be here.» Which of the following manifestations observed
D) «When the clock hands are on 6 and 12.» by the school nurse confirms the presence of pe-
diculosis capitis in students?
Review Information: The correct answer is A:
«They will be back right after supper.». Time is A) Scratching the head more than usual
not completely understood by a 4 year-old. Pre- B) Flakes evident on a student>s shoulders
schoolers interpret time with their own frame of C) Oval pattern occipital hair loss
reference. Thus, it is best to explain time in rela- D) Whitish oval specks sticking to the hair
tionship to a known, common event.
Review Information: The correct answer is D:
Whitish oval specks sticking to the hair. Diagno-
Question38 sis of pediculosis capitis is made by observation
The nurse is giving instructions to the parents of the white eggs (nits) firmly attached to the
of a child with cystic fibrosis. The nurse would hair shafts. Treatment can include application of
emphasize that pancreatic enzymes should be a medicated shampoo with lindane for children
taken over 2 years of age, and meticulous combing
A) once each day and removal of all nits.
B) 3 times daily after meals
C) with each meal or snack
D) each time carbohydrates are eaten Question41
When interviewing the parents of a child with
Review Information: The correct answer is C: asthma, it is most important to assess the child>s
with each meal or snack. Pancreatic enzymes environment for what factor?
should be taken with each meal and every snack
to allow for digestion of all foods that are eaten. A) Household pets
Collected by :DeepaRajesh [ 30 ]
rajesh.ks21@gmail.com
Kuwait
B) New furniture «Keep in mind that for the age this is a normal
C) Lead based paint response to being in the hospital.». The protest
D) Plants such as cactus phase of separation anxiety is a normal response
for a child this age. In toddlers, ages 1 to 3, sepa-
Review Information: The correct answer is A: ration anxiety is at its peak
Household pets. Animal dander is a very com-
mon allergen affecting persons with asthma.
Other triggers may include pollens, carpeting Question44
and household dust. A couple experienced the loss of a 7 month-old
fetus. In planning for discharge, what should the
nurse emphasize?
Question42
The mother of a 2 month-old baby calls the nurse A) To discuss feelings with each other and use
2 days after the first DTaP, IPV, Hepatitis B and support persons
HIB immunizations. She reports that the baby B) To focus on the other healthy children and
feels very warm, cries inconsolably for as long as move through the loss
3 hours, and has had several shaking spells. In C) To seek causes for the fetal death and come
addition to referring her to the emergency room, to some safe conclusion
the nurse should document the reaction on the D) To plan for another pregnancy within 2 years
baby>s record and expect which immunization and maintain physical health
to be most associated with the findings the infant
is displaying? Review Information: The correct answer is A:
To discuss feelings with each other and use sup-
A) DTaP port persons. To communicate in a therapeutic
B) Hepatitis B manner, the nurse>>s goal is to help the couple
C) Polio begin the grief process by suggesting they talk
D) H. Influenza to each other, seek family, friends and support
groups to listen to their feelings.
Review Information: The correct answer is A:
DTaP. The majority of reactions occur with the
administration of the DTaP vaccination. Contra- Question45
dictions to giving repeat DTaP immunizations in- The nurse is performing a pre-kindergarten phys-
clude the occurrence of severe side effects after ical on a 5 year-old. The last series of vaccines
a previous dose as well as signs of encephalop- will be administered. What is the preferred site
athy within 7 days of the immunization. for injection by the nurse?

A) vastus intermedius
Question43 B) gluteus maximus
The mother of a 2 year-old hospitalized child asks C) vastus lateralis
the nurse>s advice about the child>s screaming D) dorsogluteaI
every time the mother gets ready to leave the
hospital room. What is the best response by the Review Information: The correct answer is C:
nurse? vastus lateralis. Vastus lateralis, a large and well
developed muscle, is the preferred site, since it is
A) «I think you or your partner needs to stay with removed from major nerves and blood vessels.
the child while in the hospital.»
B) «Oh, that behavior will stop in a few days.»
C) «Keep in mind that for the age this is a normal Question46
response to being in the hospital.» A 7 month pregnant woman is admitted with com-
D) «You might want to «sneak out» of the room plaints of painless vaginal bleeding over several
once the child falls asleep.» hours. The nurse should prepare the client for an
immediate
Review Information: The correct answer is C:
Collected by :DeepaRajesh [ 31 ]
rajesh.ks21@gmail.com
Kuwait
A) Non stress test parents say. Parental caretakers are often quite
B) Abdominal ultrasound sensitive to variations in their children>>s condi-
C) Pelvic exam tion that may not be immediately evident to oth-
D) X-ray of abdomen ers.

Review Information: The correct answer is B:


Abdominal ultrasound. The standard for diagno- Question49
sis of placenta previa, which is suggested in the The nurse is caring for a client who was success-
client>>s history of painless bleeding, is abdomi- fully resuscitated from a pulseless dysrhythmia.
nal ultrasound. Which of the following assessments is critical for
the nurse to include in the plan of care?

Question47 A) hourly urine output


A nurse entering the room of a postpartum moth- B) white blood count
er observes the baby lying at the edge of the C) blood glucose every 4 hours
bed while the woman sits in a chair. The mother D) temperature every 2 hours
states «This is not my baby, and I do not want it.»
After repositioning the child safely, the nurse>s Review Information: The correct answer is A:
best response is hourly urine output. Clients who have had an epi-
sode of decreased glomerular perfusion are at
A) «This is a common occurrence after birth, but risk for pre-renal failure. This is caused by any
you will come to accept the baby.» abnormal decline in kidney perfusion that reduc-
B) «Many women have postpartum blues and es glomerular perfusion. Pre-renal failure occurs
need some time to love the baby.» when the effective arterial blood volume falls.
C) «What a beautiful baby! Her eyes are just like Examples of this phenomena include a drop in
yours.» circulating blood volume as in a cardiac arrest
D) «You seem upset; tell me what the pregnancy state or in low cardiac perfusion states such as
and birth were like for you.» congestive heart failure associated with a cardi-
omyopathy. Close observation of hourly urinary
Review Information: The correct answer is D: output is necessary for early detection of this
«You seem upset; tell me what the pregnancy condition.
and birth were like for you.». A non-judgmental,
open ended response facilitates dialogue be-
tween the client and nurse. Question50
A client is admitted to the rehabilitation unit fol-
lowing a cerebral vascular accident (CVA) and
Question48 mild dysphagia. The most appropriate interven-
The nurse notes that a 2 year-old child recovering tion for this client is to
from a tonsillectomy has an temperature of 98.2
degrees Fahrenheit at 8:00 AM. At 10:00 AM the A) position client in upright position while eating
child>s parent reports that the child «feels very B) place client on a clear liquid diet
warm» to touch. The first action by the nurse C) tilt head back to facilitate swallowing reflex
should be to D) offer finger foods such as crackers or pret-
zels
A) reassure the parent that this is normal
B) offer the child cold oral fluids Review Information: The correct answer is A:
C) reassess the child>s temperature position client in upright position while eating. An
D) administer the prescribed acetaminophen upright position facilitates proper chewing and
swallowing.
Review Information: The correct answer is C:
reassess the child>>s temperature. A child>>s
temperature may have rapid fluctuations. The Question51
nurse should listen to and show respect for what A 72 year-old client with osteomyelitis requires a
Collected by :DeepaRajesh [ 32 ]
rajesh.ks21@gmail.com
Kuwait
6 week course of intravenous antibiotics. In plan- medication
ning for home care, what is the most important A) retards pepsin production
action by the nurse? B) stimulates hydrochloric acid production
C) slows stomach emptying time
A) Investigating the client>s insurance coverage D) decreases production of hydrochloric acid
for home IV antibiotic therapy
B) Determining if there are adequate hand wash- Review Information: The correct answer is
ing facilities in the home B: stimulates hydrochloric acid production. Dec-
C) Assessing the client>s ability to participate in adron increases the production of hydrochloric
self care and/or the reliability of a caregiver acid, which may cause gastrointestinal ulcers.
D) Selecting the appropriate venous access de-
vice
Question54
Review Information: The correct answer is A client receiving chlorpromazine HCL (Thora-
C: Assessing the client>>s ability to participate zine) is in psychiatric home care. During a home
in self care and/or the reliability of a caregiver. visit the nurse observes the client smacking her
The cognitive ability of the client as well as the lips alternately with grinding her teeth. The nurse
availability and reliability of a caregiver must be recognizes this assessment finding as what?
assessed to determine if home care is a feasible A) Dystonia
option. B) Akathisia
C) Brady dyskinesia
D) Tardive dyskinesia
Question52
A nurse administers the influenza vaccine to a Review Information: The correct answer is D:
client in a clinic. Within 15 minutes after the im- Tardive dyskinesia. Signs of tardive dyskinesia
munization was given, the client complains of include smacking lips, grinding of teeth and «fly
itchy and watery eyes, increased anxiety, and catching» tongue movements. These findings
difficulty breathing. The nurse expects that the are often described as Parkinsonian.
first action in the sequence of care for this client Question55
will be to Which of the following findings contraindicate the
use of haloperidol (Haldol) and warrant withhold-
A) Maintain the airway ing the dose?
B) Administer epinephrine 1:1000 as ordered A) Drowsiness, lethargy, and inactivity
C) Monitor for hypotension with shock B) Dry mouth, nasal congestion, and blurred vi-
D) Administer diphenhydramine as ordered sion
C) Rash, blood dyscrasias, severe depression
Review Information: The correct answer is B: D) Hyperglycemia, weight gain, and edema
Administer epinephrine 1:1000 as ordered. All
the answers are correct given the circumstances, Review Information: The correct answer is
but the priority is to administer the epinephrine, C: Rash, blood dyscrasias, severe depression.
then maintain the airway. In the early stages of Rash and blood dyscrasias are side effects of
anaphylaxis, when the patient has not lost con- anti-psychotic drugs. A history of severe depres-
sciousness and is normotensive, administering sion is a contraindication to the use of neurolep-
the epinephrine is first, and applying the oxygen, tics.
and watching for hypotension and shock, are lat-
er responses. The prevention of a severe crisis
is maintained by using diphenhydramine. Question56
The nurse is reinforcing teaching to a 24 year-old
woman receiving acyclovir (Zovirax) for a Herpes
Question53 Simplex Virus type 2 infection. Which of these in-
The nurse instructs the client taking dexametha- structions should the nurse give the client?
sone (Decadron) to take it with food or milk. The
physiological basis for this instruction is that the A) Complete the entire course of the medication
Collected by :DeepaRajesh [ 33 ]
rajesh.ks21@gmail.com
Kuwait
for an effective cure Question59
B) Begin treatment with acyclovir at the onset of A 42 year-old male client refuses to take pro-
symptoms of recurrence pranolol hydrochloride (Inderal) as prescribed.
C) Stop treatment if she thinks she may be preg- Which client statement from the assessment
nant to prevent birth defects data is likely to explain his noncompliance?
D) Continue to take prophylactic doses for at
least 5 years after the diagnosis A) «I have problems with diarrhea.»
B) «I have difficulty falling asleep.»
Review Information: The correct answer is C) «I have diminished sexual function.»
B: Begin treatment with acyclovir at the onset D) «I often feel jittery.»
of symptoms of recurrence. When the client is
aware of early symptoms, such as pain, itching Review Information: The correct answer is C:
or tingling, treatment is very effective. Medica- «I have diminished sexual function.». Inderal, a
tions for herpes simplex do not cure the disease; beta-blocking agent used in hypertension, pro-
they simply decrease the level of symptoms. hibits the release of epinephrine into the cells;
this may result in hypotension which results in
decreased libido and impotence.
Question57
A 14 month-old child ingested half a bottle of
aspirin tablets. Which of the following would the Question60
nurse expect to see in the child? The nurse caring for a 9 year-old child with a
fractured femur is told that a medication error
A) Hypothermia occurred. The child received twice the ordered
B) Edema dose of morphine an hour ago. Which nursing
C) Dyspnea diagnosis is a priority at this time?
D) Epistaxis
A) Risk for fluid volume deficit related to mor-
Review Information: The correct answer is phine overdose
D: Epistaxis. A large dose of aspirin inhibits pro- B) Decreased gastrointestinal mobility related to
thrombin formation and lowers platelet levels. mucosal irritation
With an overdose, clotting time is prolonged. C) Ineffective breathing patterns related to cen-
tral nervous system depression
D) Altered nutrition related to inability to control
Question58 nausea and vomiting
An 80 year-old client on digitalis (Lanoxin) re-
ports nausea, vomiting, abdominal cramps and Review Information: The correct answer is C:
halo vision. Which of the following laboratory re- Ineffective breathing patterns related to central
sults should the nurse analyze first? nervous system depression. Respiratory depres-
sion is a life-threatening risk in this overdose.
A) Potassium levels
B) Blood pH
C) Magnesium levels Question61
D) Blood urea nitrogen Lactulose (Chronulac) has been prescribed for a
client with advanced liver disease. Which of the
Review Information: The correct answer is A: following assessments would the nurse use to
Potassium levels. The most common cause of evaluate the effectiveness of this treatment?
digitalis toxicity is a low potassium level. Clients
must be taught that it is important to have ad- A) An increase in appetite
equate potassium intake especially if taking diu- B) A decrease in fluid retention
retics that enhance the loss of potassium while C) A decrease in lethargy
they are taking digitalis. D) A reduction in jaundice

Review Information: The correct answer is


Collected by :DeepaRajesh [ 34 ]
rajesh.ks21@gmail.com
Kuwait
C: A decrease in lethargy. Lactulose produces his warnings must be heeded to prevent nuclear
an acid environment in the bowel and traps am- war. What is the most therapeutic approach by
monia in the gut; the laxative effect then aids in the nurse?
removing the ammonia from the body. This de-
creases the effects of hepatic encephalopathy, A) Listen quietly without comment
including lethargy and confusion. B) Ask for further information on the spies
C) Confront the client’s delusion
D) Contact the government agency
Question62
The nurse is teaching a class on HIV prevention. Review Information: The correct answer is A:
Which of the following should be emphasized as Listen quietly without comment. The client>>s
increasing risk? comments demonstrate grandiose ideas. The
most therapeutic response is to listen but avoid
A) Donating blood being incorporated into the client’s delusional
B) Using public bathrooms system.
C) Unprotected sex
D) Touching a person with AIDS Question65
The nurse is assessing a 17 year-old female cli-
Review Information: The correct answer is C: ent with bulimia. Which of the following labora-
Unprotected sex. Because HIV is spread through tory reports would the nurse anticipate?
exposure to bodily fluids, unprotected intercourse
and shared drug paraphernalia remain the high- A) Increased serum glucose
est risks for infection. B) Decreased albumin
C) Decreased potassium
D) Increased sodium retention
Question63
While interviewing a new admission, the nurse Review Information: The correct answer is C:
notices that the client is shifting positions, wring- Decreased potassium. In bulimia, loss of elec-
ing her hands, and avoiding eye contact. It is im- trolytes can occur in addition to other findings of
portant for the nurse to starvation and dehydration.
A) ask the client what she is feeling
B) assess the client for auditory hallucinations
C) recognize the behavior as a side effect of Question66
medication A client, recovering from alcoholism, asks the
D) re-focus the discussion on a less anxiety pro- nurse, «What can I do when I start recognizing
voking topic relapse triggers within myself?» How might the
nurse best respond?
Review Information: The correct answer is A:
ask the client what she is feeling. The initial step A) «When you have the impulse to stop in a bar,
in anxiety intervention is observing, identifying, contact a sober friend and talk with him.»
and assessing anxiety. The nurse should seek B) «Go to an AA meeting when you feel the urge
client validation of the accuracy of nursing as- to drink.»
sessments and avoid drawing conclusions based C) «It is important to exercise daily and get in-
on limited data. In the situation above, the client volved in activities that will cause you not to think
may simply need to use the restroom but be re- about drug use.»
luctant to communicate her need! D) «Let’s talk about possible options you have
when you recognize relapse triggers in your-
self.»
Question64
A young adult seeks treatment in an outpatient Review Information: The correct answer is D:
mental health center. The client tells the nurse «Let’s talk about possible options you have when
he is a government official being followed by you recognize relapse triggers in yourself.». This
spies. On further questioning, he reveals that option encourages the process of self evalua-
Collected by :DeepaRajesh [ 35 ]
rajesh.ks21@gmail.com
Kuwait
tion and problem solving, while avoiding telling A) provide a businesslike atmosphere where cli-
the client what to do. Encouraging the client to ents can work on individual goals
brainstorm about response options validates the B) provide a group forum in which clients decide
nurse’s belief in the client’s personal competen- on unit rules, regulations, and policies
cy and reinforces a coping strategy that will be C) provide a testing ground for new patterns of
needed when the nurse may not be available to behavior while the client takes responsibility for
offer solutions. his or her own actions
D) discourage expressions of anger because
Question67 they can be disruptive to other clients
Therapeutic nurse-client interaction occurs when
the nurse Review Information: The correct answer is C:
A) assists the client to clarify the meaning of what provide a testing ground for new patterns of be-
the client has said havior while the client takes responsibility for his
B) interprets the client’s covert communication or her own actions. A therapeutic milieu is pur-
C) praises the client for appropriate feelings and poseful and planned to provide safety and a test-
behavior ing ground for new patterns of behavior.
D) advises the client on ways to resolve prob-
lems
Question70
Review Information: The correct answer is A: A client with paranoid delusions stares at the
assists the client to clarify the meaning of what nurse over a period of several days. The client
the client has said. Clarification is a facilitating/ suddenly walks up to the nurse and shouts «You
therapeutic communication strategy. Interpreta- think you’re so perfect and pure and good.» An
tion, changing the focus/subject, giving approv- appropriate response for the nurse is
al, and advising are non-therapeutic/barriers to
communication. A) «Is that why you’ve been staring at me?»
B) «You seem to be in a really bad mood.»
Question68 C) «Perfect? I don’t quite understand.»
Which nursing intervention will be most effective D) «You seem angry right now.»
in helping a withdrawn client to develop relation-
ship skills? Review Information: The correct answer is D:
A) Offer the client frequent opportunities to inter- «You seem angry right now.». The nurse recog-
act with 1 person nizes the underlying emotion with a matter of fact
B) Provide the client with frequent opportunities attitude, but avoids telling the clients how they
to interact with other clients feel.
C) Assist the client to analyze the meaning of the
withdrawn behavior
D) Discuss with the client the focus that other Question71
clients have similar problems A client who is a former actress enters the day
room wearing a sheer nightgown, high heels, nu-
Review Information: The correct answer is A: merous bracelets, bright red lipstick and heavily
Offer the client frequent opportunities to interact rouged cheeks. Which nursing action is the best
with 1 person. The withdrawn client is uncomfort- in response to the client’s attire?
able in social interaction. The nurse-client rela-
tionship is a corrective relationship in which the A) Gently remind her that she is no longer on
client learns both tolerance and skills for relation- stage
ships. B) Directly assist client to her room for appropri-
ate apparel
C) Quietly point out to her the dress of other cli-
Question69 ents on the unit
An important goal in the development of a thera- D) Tactfully explain appropriate clothing for the
peutic inpatient milieu is to hospital

Collected by :DeepaRajesh [ 36 ]
rajesh.ks21@gmail.com
Kuwait
Review Information: The correct answer is B: B) handshaking keeps the gesture on a profes-
Directly assist client to her room for appropriate sional level
apparel. It assists the client to maintain self-es- C) refusal to touch a client denotes lack of con-
teem while modifying behavior. cern
D) inappropriate touch often results in charges of
assault and battery
Question72
When teaching suicide prevention to the parents Review Information: The correct answer is A:
of a 15 year-old who recently attempted suicide, some clients misconstrue hugs as an invitation
the nurse describes the following behavioral cue to sexual advances. Touch denotes positive feel-
as indicating a need for intervention. ings for another person. The client may interpret
hugging and holding hands as sexual advanc-
A) Angry outbursts at significant others es.
B) Fear of being left alone
C) Giving away valued personal items
D) Experiencing the loss of a boyfriend Question75
A client with anorexia is hospitalized on a medi-
Review Information: The correct answer is C: cal unit due to electrolyte imbalance and cardiac
Giving away valued personal items. Eighty per- dysrhythmias. Additional assessment findings
cent of all potential suicide victims give some type that the nurse would expect to observe are
of indication that self-destructiveness should be
addressed. These clues might lead one to sus- A) brittle hair, lanugo, amenorrhea
pect that a client is having suicidal thoughts or is B) diarrhea, nausea, vomiting, dental erosion
developing a plan. C) hyperthermia, tachycardia, increased meta-
bolic rate
D) excessive anxiety about symptoms
Question73
Which statement made by a client indicates to Review Information: The correct answer is A:
the nurse that the client may have a thought dis- brittle hair, lanugo, amenorrhea. Physical find-
order? ings associated with anorexia also include re-
A) «I>m so angry about this. Wait until my part- duced metabolic rate and lower vital signs.
ner hears about this.»
B) «I>m a little confused. What time is it?»
C) «I can>t find my <mesmer> shoes. Have you
seen them?» NCLEX Study Tips
D) «I>m fine. It>s my daughter who has the prob- Jul31,
lem.»
If you are going to prepare for taking the NCLEX
Review Information: The correct answer is C: exam and still don>t know what to do i would like
«I can>>t find my <>mesmer>> shoes. Have you to share some effective advices for you.
seen them?». A neologism is a new word self in-
vented by a person and not readily understood Picking review courses:
by another. Using neologisms is often associ-
ated with a thought disorder. the best choice for review courses is Kaplan or
NCSBN (National Council State Boards of Nurs-
ing). Kaplan teaches effective techniques on
Question74 how to answer exam questions with ease and
In a psychiatric setting, the nurse limits touch teaches you to land with The correct answer.
or contact used with clients to handshaking be-
cause NCSBN sure is another top choice for review
course because the contents are very close to
A) some clients misconstrue hugs as an invita- the actual exam itself.
tion to sexual advances
Collected by :DeepaRajesh [ 37 ]
rajesh.ks21@gmail.com
Kuwait
Never Cram The current fee to register with NCLEX is $200
and you must indicate at the time of application
Cramming is never effective in preparing for the which Board you>ve chosen.Processing times
NCLEX. Give yourself at least 3 months to study vary from state to state from (4-16 weeks)
for the exam.
After you have met the requirements, been ap-
References proved by the Board and applied for NCLEX you
will be issued an ATT (Authorization to Test) and
Lipincott is known to be the best review book for can schedule your NCLEX exam at your convin-
preparing yourself for the exam. ience.
Some naysayers say that the NCLEX structured
questions are based on lipincott.
You must have an ATT before you can to take
the exam.
Do alot of practice testing and never sleep late
before the exam day. Just relax you will do fine. The NCLEX exam can be scheduled anywhere
NCLEX could be re-taken after 91 days from tak- in the US or it>s territories, and other countries
ing the first exam. like the Philippines and Hong Kong and is of-
fered year-round.
0 comments
You do not have to take the NCLEX exam in the
Labels: nclex review, nclex study tips State where you applied.

How To Apply For NCLEX Testing


At some testing centers the appointments go
Here are the guidelines on how to apply for very quickly so plan to schedule your appoint-
NCLEX Testing in the US: ment early.

In order to receive a US nursing license, you 0 comments


must pick a state, complete the application and
meet their requirements and if they find you eli- Labels: how to apply for nclex-rn, nclex testing
gible, you will have to undergo testing also com- guidelines
monly known as NCLEX, before you can work as
an RN in the USA. US Boards Of Nursing For NCLEX Application

Here are the list of US Nursing Boards state by


Each State Nursing Board has its own fee sched- state. Click on them and it will link you to each
ule and specific requirements (e.g. CGFNS, state board webpage and check the guidelines
CES, TOEFL, TSE) on how you could apply for examination (NCLEX)
The time it takes to approve your credentials or reciprocity to practice as a registered nurse in
and process your application varies from state your choice of state.
to state.

Alabama Board of Nursing


Alaska Board of Nursing
Arizona State Board of Nursing
All nurses must pass NCLEX .
Arkansas State Board of Nursing
California Board of Registered Nursing
*California Board of Vocational Nursing and Psy-
chiatric Technicians
The Board will supply you with an NCLEX appli-
Colorado Board of Nursing
cation which could be downlowded in their web-
Connecticut Board of Examiners for Nursing
pages.
Delaware Board of Nursing
Collected by :DeepaRajesh [ 38 ]
rajesh.ks21@gmail.com
Kuwait
District of Columbia Board of Nursing ing
Florida Board of Nursing Wyoming State Board of Nursing
Georgia Board of Nursing
*Georgia State Board of Licensed Practical *Some states have separate Web sites for boards
Nurses of licensed practical or vocational nursing (LPN/
Hawaii Board of Nursing LVN)
Idaho Board of Nursing
Illinois Division of Professional Regulation
Indiana State Board of Nursing
Iowa Board of Nursing Free NCLEX-RN Sample Test Ques-
Kansas State Board of Nursing tions For Nursing Review (Pharmacol-
Kentucky Board of Nursing
ogy Set 2)
Louisiana State Board of Nursing
*Louisiana State Board of Practical Nurse Exam- Jul31,
iners
Maine State Board of Nursing A nurse is assigned to perform well-child assess-
Maryland Board of Nursing ments at a day care center. A staff member in-
Massachusetts Board of Registration in Nursing terrupts the examinations to ask for assistance.
Michigan CIS/Bureau of Health Professions They find a crying 3 year-old child on the floor
Minnesota Board of Nursing with mouth wide open and gums bleeding. Two
Mississippi Board of Nursing unlabeled open bottles lie nearby. The nurse>s
Missouri Division of Professional Registration first action should be
Montana State Board of Nursing A) call the poison control center, then 911
Nebraska Department of Health and Human B) administer syrup of Ipecac to induce vomiting
Services Regulation and Licensure, Nursing and C) give the child milk to coat her stomach
Nursing Support D) ask the staff about the contents of the bottles
Nevada State Board of Nursing
New Hampshire Board of Nursing
New Jersey Board of Nursing Review Information: The correct answer is D:
New Mexico Board of Nursing ask the staff about the contents of the bottles
New York State Board of Nursing The nurse needs to assess what the child in-
North Carolina Board of Nursing gested before determining the next action. Once
North Dakota Board of Nursing the substance is identified, the poison control
Ohio Board of Nursing center and emergency response team should be
Oklahoma Board of Nursing called.
Oregon State Board of Nursing
Pennsylvania State Board of Nursing Question2
Rhode Island Board of Nurse Registration and A client with atrial fibrillation is receiving digoxin
Nursing Education (Lanoxin). Which of these assessments is most
South Carolina Board of Nursing important for the nurse to perform?
South Dakota Board of Nursing A) Monitor blood pressure every 4 hours
Tennessee State Board of Nursing B) Measure apical pulse prior to administration
Texas Board of Nurse Examiners C) Maintain accurate intake and output records
Utah State Board of Nursing D) Record an EKG strip after administration
Vermont State Board of Nursing
Virginia Board of Nursing
Washington State Nursing Care Quality Assur- Review Information: The correct answer is B:
ance Commission Measure apical pulse prior to administration
West Virginia Board of Examiners for Registered Digitoxin decreases conduction velocity through
Professional Nurses the AV node and prolongs the refractory period. If
*West Virginia State Board of Examiners for Li- the apical heart rate is less than 60 beats/minute,
censed Practical Nurses withhold the drug. The apical pulse should be
Wisconsin Department of Regulation and Licens- taken with a stethoscope so that there will be no
mistake about what the heart rate actually is.
Collected by :DeepaRajesh [ 39 ]
rajesh.ks21@gmail.com
Kuwait
Question3
The nurse is administering an intravenous vesi- Review Information: The correct answer is A:
cant chemotherapeutic agent to a client. Which Buffalo hump
assessment would require the nurse>s immedi- With high doses of glucocorticoid, iatrogenic
ate action? Cushing>>s syndrome develops. The exagger-
A) Stomatitis lesion in the mouth ated physiological action causes abnormal fat
B) Severe nausea and vomiting distribution which results in a moon-shaped face,
C) Complaints of pain at site of infusion a intrascapular pad on the neck (buffalo hump)
D) A rash on the client>s extremities and truncal obesity with slender limbs.

Review Information: The correct answer is C: Question6


Complaints of pain at site of infusion The health care provider has written «Morphine
A vesicant is a chemotherapeutic agent capa- sulfate 2 mgs IV every 3-4 hours prn for pain» on
ble of causing blistering of tissues and possible the chart of a child weighing 22 lb. (10 kg). What
tissue necrosis if there is extravasation. These is the nurse>s initial action?
agents are irritants which cause pain along the A) Check with the pharmacist
vein wall, with or without inflammation. B) Hold the medication and contact the provider
C) Administer the prescribed dose as ordered
Question4 D) Give the dose every 6-8 hours
The nurse practicing in a long term care facility
recognizes that elderly clients are at greater risk
for drug toxicity than younger adults because of Review Information: The correct answer is B:
which of the following physiological changes of Hold the medication and contact the provider
advancing age? The usual pediatric dose of morphine is 0.1 mg/
A) Drugs are absorbed more readily from the GI kg every 3 to 4 hours. At 10 kg, this child typically
tract should receive 1.0 mg every 3 to 4 hours.
B) Elders have less body water and more fat
C) The elderly have more rapid hepatic metabo- Question7
lism A client is ordered atropine to be administered
D) Older people are often malnourished and preoperatively. Which physiological effect should
anemic the nurse monitor for?
A) Elevate blood pressure
B) Drying up of secretions
Review Information: The correct answer is B: C) Reduce heart rate
Elders have less body water and more fat D) Enhance sedation
Because elderly persons have decreased lean
body tissue/water in which to distribute medica-
tions, more drug remains in the circulatory system Review Information: The correct answer is B:
with potential for drug toxicity. Increased body fat Drying up of secretions
results in greater amounts of fat-soluble drugs Atropine dries secretions which may get in the
being absorbed, leaving less in circulation, thus way during the operative procedure.
increasing the duration of action of the drug
Question8
Question5 A client is receiving digitalis. The nurse should
The nurse is assessing a client who is on long instruct the client to report which of the following
term glucocorticoid therapy. Which of the follow- side effects?
ing findings would the nurse expect? A) Nausea, vomiting, fatigue
A) Buffalo hump B) Rash, dyspnea, edema
B) Increased muscle mass C) Polyuria, thirst, dry skin
C) Peripheral edema D) Hunger, dizziness, diaphoresis
D) Jaundice
Collected by :DeepaRajesh [ 40 ]
rajesh.ks21@gmail.com
Kuwait
the parents, which principle must the nurse un-
Review Information: The correct answer is A: derstand?
Nausea, vomiting, fatigue A) The problem occurs in stages with recovery
Side effects of digitalis toxicity include fatigue, within 12-24 hours
nausea, vomiting, anorexia, and bradycardia. B) Hepatic problems may occur and may be life-
Digitalis inhibits the sodium potassium ATPase, threatening
which makes more calcium available for contrac- C) Full and rapid recovery can be expected in
tile proteins, resulting in increased cardiac out- most children
put. D) This poisoning is usually fatal, as no antidote
is available
Question9
A client is receiving dexamethasone (Decadron)
therapy. What should the nurse plan to monitor Review Information: The correct answer is B:
in this client? Hepatic problems may occur and may be life-
A) Urine output every 4 hours threatening
B) Blood glucose levels every 12 hours Clinical manifestations associated with acetami-
C) Neurological signs every 2 hours nophen poisoning occurs in 4 stages. The third
D) Oxygen saturation every 8 hours stage is hepatic involvement which may last up
to 7 days and be permanent. Clients who do not
Review Information: The correct answer is B: die in the hepatic stage gradually recover.
Blood glucose levels every 12 hours
The drug Decadron increases glycogenesis. Question12
This may lead to hyperglycemia. Therefore the A client has been receiving dexamethasone
blood sugar level and acetone production must (Decadron) for control of cerebral edema. Which
be monitored. of the following assessments would indicate that
the treatment is effective?
Question10 A) A positive Babinski>s reflex
The nurse is caring for a client with schizophrenia B) Increased response to motor stimuli
who has been treated with quetiapine (Seroquel) C) A widening pulse pressure
for 1 month. Today the client is increasingly agi- D) Temperature of 37 degrees Celsius
tated and complains of muscle stiffness. Which
of these findings should be reported to the health Review Information: The correct answer is B:
care provider? Increased response to motor stimuli
A) Elevated temperature and sweating. Decadron is a corticosteroid that acts on the cell
B) Decreased pulse and blood pressure. membrane to decrease inflammatory responses
C) Mental confusion and general weakness. as well as stabilize the blood-brain barrier. Once
D) Muscle spasms and seizures. Decadron reaches a therapeutic level, there
should be a decrease in symptomology with im-
provement in motor skills.
Review Information: The correct answer is A:
Elevated temperature and sweating. Question13
Neuroleptic malignant syndrome (NMS) is a rare The provider has ordered transdermal nitroglyc-
disorder that can occur as a side effect of antipsy- erin patches for a client. Which of these instruc-
chotic medications. It is characterized by muscu- tions should be included when teaching a client
lar rigidity, tachycardia, hyperthermia, sweating, about how to use the patches?
altered consciousness, autonomic dysfunction, A) Remove the patch when swimming or bath-
and increase in CPK. This is a life-threatening ing
complication. B) Apply the patch to any non-hairy area of the
body
Question11 C) Apply a second patch with chest pain
A child presents to the Emergency Department D) Remove the patch if ankle edema occurs
with documented acetaminophen poisoning. In
order to provide counseling and education for
Collected by :DeepaRajesh [ 41 ]
rajesh.ks21@gmail.com
Kuwait
Review Information: The correct answer is with which substance?
B: Apply the patch to any non-hairy area of the A) Acetaminophen
body B) Orange juice
The patch application sites should be rotated. C) Low fat milk
D) An antacid
Question14
A newly admitted client has a diagnosis of de- Review Information: The correct answer is B:
pression. She complains of “twitching muscles” Orange juice
and a “racing heart”, and states she stopped Ascorbic acid enhances the absorption of iron.
taking Zoloft a few days ago because it was not
helping her depression. Instead, she began to Question17
take her partner>s Parnate. The nurse should A client with an aplastic sickle cell crisis is receiv-
immediately assess for which of these adverse ing a blood transfusion and begins to complain
reactions? of «feeling hot.» Almost immediately, the client
A) Pulmonary edema begins to wheeze. What is the nurse>s first ac-
B) Atrial fibrillation tion?
C) Mental status changes A) Stop the blood infusion
D) Muscle weakness B) Notify the health care provider
C) Take/record vital signs
Review Information: The correct answer is C: D) Send blood samples to lab
Mental status changes
Use of serotonergic agents may result in Se- Review Information: The correct answer is A:
rotonin Syndrome with confusion, nausea, pal- Stop the blood infusion
pitations, increased muscle tone with twitching If a reaction of any type is suspected during ad-
muscles, and agitation. Serotonin syndrome is ministration of blood products, stop the infusion
most often reported in patients taking 2 or more immediately, keep the line open with saline, no-
medications that increase CNS serotonin levels tify the health care provider, monitor vital signs
by different mechanisms. The most common and other changes, and then send a blood sam-
drug combinations associated with serotonin ple to the lab.
syndrome involve the MAOIs, SSRIs, and the tri-
cyclic antidepressants. Question18
A client confides in the RN that a friend has told
Question15 her the medication she takes for depression,
A client with bi-polar disorder is taking lithium Wellbutrin, was taken off the market because
(Lithane). What should the nurse emphasize it caused seizures. What is an appropriate re-
when teaching about this medication? sponse by the nurse?
A) Take the medication before meals A) «Ask your friend about the source of this in-
B) Maintain adequate daily salt intake formation.»
C) Reduce fluid intake to minimize diuresis B) «Omit the next doses until you talk with the
D) Use antacids to prevent heartburn doctor.»
C) «There were problems, but the recommended
dose is changed.»
Review Information: The correct answer is B: D) «Your health care provider knows the best
Maintain adequate daily salt intake drug for your condition.»
Salt intake affects fluid volume, which can affect
lithium (Lithane) levels; therefore, maintaining
adequate salt intake is advised. Review Information: The correct answer is C:
«There were problems, but the recommended
Question16 dose is changed.»
A client with anemia has a new prescription for Wellbutrin was introduced in the U.S. in 1985
ferrous sulfate. In teaching the client about diet and then withdrawn because of the occurrence
and iron supplements, the nurse should empha- of seizures in some patients taking the drug. The
size that absorption of iron is enhanced if taken drug was reintroduced in 1989 with specific rec-
Collected by :DeepaRajesh [ 42 ]
rajesh.ks21@gmail.com
Kuwait
ommendations regarding dose ranges to limit tests.»
the occurrence of seizures. The risk of seizure
appears to be strongly associated with dose.
Review Information: The correct answer is C:
Question19 «Continue to take your medications even when
When providing discharge teaching to a client you are feeling fine.»
with asthma, the nurse will warn against the use The most important piece of information the tu-
of which of the following over-the-counter medi- berculosis client needs is to understand the im-
cations? portance of medication compliance, even if no
A) Cortisone ointments for skin rashes longer experiencing symptoms. Clients are most
B) Aspirin products for pain relief infectious early in the course of therapy. The
C) Cough medications containing guaifenesin numbers of acid-fast bacilli are greatly reduced
D) Histamine blockers for gastric distress as early as 2 weeks after therapy begins.

Question22
Review Information: The correct answer is B: The nurse is applying silver sulfadiazine (Sil-
Aspirin products for pain relief vadene) to a child with severe burns to arms
Aspirin is known to induce asthma attacks. As- and legs. Which side effect should the nurse be
pirin can also cause nasal polyps and rhinitis. monitoring for?
Warn individuals with asthma about signs and A) Skin discoloration
symptoms resulting from complications due to B) Hardened eschar
aspirin ingestion. C) Increased neutrophils
D) Urine sulfa crystals
Question20
The nurse is caring for a client who is receiving
procainamide (Pronestyl) intravenously. It is im- Review Information: The correct answer is D:
portant for the nurse to monitor which of the fol- Urine sulfa crystals
lowing parameters? Silver sulfadiazine is a broad spectrum anti-
A) Hourly urinary output microbial, especially effective against pseu-
B) Serum potassium levels domonas. When applied to extensive areas,
* C) Continuous EKG readings however, it may cause a transient neutropenia,
D) Neurological signs as well as renal function changes with sulfa crys-
tals production and kernicterus.

Review Information: The correct answer is C: Question23


Continuous EKG readings The nurse is monitoring a client receiving a
Procainamide (Pronestyl) is used to suppress thrombolytic agent, alteplase (Activase tissue
cardiac arrhythmias. When administered intra- plasminogen activator), for treatment of a myo-
venously, it must be accompanied by continuous cardial infarction. What outcome indicates the
cardiac monitoring by ECG. client is receiving adequate therapy within the
first hours of treatment?
Question21 A) Absence of a dysrhythmia (or arrhythmia)
The nurse is providing education for a client with B) Blood pressure reduction
newly diagnosed tuberculosis. Which statement C) Cardiac enzymes are within normal limits
should be included in the information that is giv- D) Return of ST segment to baseline on ECG
en to the client?
A) «Isolate yourself from others until you are fin-
ished taking your medication.» Review Information: The correct answer is D:
B) «Follow up with your primary care provider in Return of ST segment to baseline on ECG
3 months.» Improved perfusion should result from this medi-
C) «Continue to take your medications even cation, along with the reduction of ST segment
when you are feeling fine.» elevation.
D) «Continue to get yearly tuberculin skin
Collected by :DeepaRajesh [ 43 ]
rajesh.ks21@gmail.com
Kuwait
Question24 A) Vertigo
The provider has ordered daily high doses of B) Drowsiness
aspirin for a client with rheumatoid arthritis. The C) Gingival hyperplasia
nurse instructs the client to discontinue the medi- D) Vomiting
cation and contact the provider if which of the
following symptoms occur?
A) Infection of the gums
B) Diarrhea for more than one day Review Information: The correct answer is C:
C) Numbness in the lower extremities Gingival hyperplasia
D) Ringing in the ears Swollen and tender gums occur often with use of
phenytoin. Good oral hygiene and regular visits
to the dentist should be emphasized.
Review Information: The correct answer is D:
Ringing in the ears Question27
Aspirin stimulates the central nervous system The use of atropine for treatment of symptomatic
which may result in ringing in the ears. bradycardia is contraindicated for a client with
Deglin, J.D. and Vallerand, A.H. (2001). Davis’ which of the following conditions?
drug guide for nurses. (7th edition). Philadelphia: A) Urinary incontinence
F.A. Davis Company. B) Glaucoma
Key, J.L. and Hayes, E.R. (2003). Pharmacol- C) Increased intracranial pressure
ogy, a nursing process approach. (4th edition). D) Right sided heart failure
Philadelphia: Saunders.

Question25 Review Information: The correct answer is B:


A nurse is caring for a client who is receiving Glaucoma
methyldopa hydrochloride (Aldomet) intrave- Atropine is contraindicated in clients with angle-
nously. Which of the following assessment find- closure glaucoma because it can cause pupillary
ings would indicate to the nurse that the client dilation with an increase in aqueous humor, lead-
may be having an adverse reaction to the medi- ing to a resultant increase in optic pressure.
cation?
A) Headache Question28
B) Mood changes A pregnant woman is hospitalized for treatment
C) Hyperkalemia of pregnancy induced hypertension (PIH) in the
D) Palpitations third trimester. She is receiving magnesium sul-
fate intravenously. The nurse understands that
this medication is used mainly for what pur-
Review Information: The correct answer is B: pose?
Mood changes A) Maintain normal blood pressure
The nurse should assess the client for alterations B) Prevent convulsive seizures
in mental status such as mood changes. These C) Decrease the respiratory rate
symptoms should be reported promptly. D) Increase uterine blood flow
Deglin, J.D. and Vallerand, A.H. (2001). Davis’
drug guide for nurses. (7th edition). Philadelphia:
F.A. Davis Company. Review Information: The correct answer is B:
Wilson, B.A., Shannon, M.T., and Stang, C.L. Prevent convulsive seizures
(2004). Nurse’s drug guide. Upper Saddle River, Magnesium sulfate is a central nervous system
New Jersey: Pearson Prentice Hall. depressant. While it has many systemic effects,
it is used in the client with pregnancy induced
Question26 hypertension (PIH) to prevent seizures.
The nurse is teaching a child and the family about
the medication phenytoin (Dilantin) prescribed
for seizure control. Which of the following side Question29
effects is most likely to occur? The nurse is teaching a group of women in a com-
Collected by :DeepaRajesh [ 44 ]
rajesh.ks21@gmail.com
Kuwait
munity clinic about prevention of osteoporosis. bination?
Which of the following over-the-counter medica- A) Enhanced pain relief
tions should the nurse recognize as having the B) Minimized side effects
most elemental calcium per tablet? C) Prevention of drug tolerance
A) Calcium chloride D) Increased onset of action
B) Calcium citrate
C) Calcium gluconate
D) Calcium carbonate Review Information: The correct answer is A:
Enhanced pain relief
Combination of analgesics with different mecha-
Review Information: The correct answer is D: nisms of action can afford greater pain relief.
Calcium carbonate
Calcium carbonate contains 400mg of elemental
calcium in 1 gram of calcium carbonate. Question33
A client is receiving erythromycin 500mg IV eve-
Question30 ry 6 hours to treat a pneumonia. Which of the
The nurse is administering diltiazem (Cardizem) following is the most common side effect of the
to a client. Prior to administration, it is important medication?
for the nurse to assess which parameter? A) Blurred vision
A) Temperature B) Nausea and vomiting
B) Blood pressure C) Severe headache
C) Vision D) Insomnia
D) Bowel sounds

Review Information: The correct answer is B:


Review Information: The correct answer is B: Nausea and vomiting
Blood pressure Nausea is a common side-effect of erythromycin
Diltiazem (Cardizem) is a calcium channel block- in both oral and intravenous forms.
er that causes systemic vasodilation resulting in
decreased blood pressure. Question34
The health care provider orders an IV aminophyl-
Question31 line infusion at 30 mg/hr. The pharmacy sends a
The nurse is instructing a client with moderate 1,000 ml bag of D5W containing 500 mg of ami-
persistent asthma on the proper method for us- nophylline. In order to administer 30 mg per hour,
ing MDIs (multi-dose inhalers). Which medica- the RN will set the infusion rate at:
tion should be administered first? A) 20 ml per hour
A) Steroid B) 30 ml per hour
B) Anticholinergic C) 50 ml per hour
C) Mast cell stabilizer D) 60 ml per hour
D) Beta agonist

Review Information: The correct answer is D:


Review Information: The correct answer is D: 60 ml per hour
Beta agonist Using the ratio method to calculate infusion rate:
The beta-agonist drugs help to relieve bronchos- mg to be given (30) : ml to be infused (X) :: mg
pasm by relaxing the smooth muscle of the air- available (500) : ml of solution (1,000). Solve for
way. These drugs should be taken first so that X by cross-multiplying: 30 x 1,000 = 500 x X (or
other medications can reach the lungs. cancel), 30,000 = 500 X, X = 30,000/500, X =
60ml per hour.
Question32
A post-operative client has a prescription for Question35
acetaminophen with codeine. What should the The nurse is assessing a 7 year-old after several
nurse recognizes as a primary effect of this com- days of treatment for a documented strep throat.
Collected by :DeepaRajesh [ 45 ]
rajesh.ks21@gmail.com
Kuwait
Which of the following statements suggests that induced seizures.
further teaching is needed?
A) «Sometimes I take my medicine with fruit Question38
juice.» A 5 year-old has been rushed to the emergency
B) «My mother makes me take my medicine right room several hours after acetaminophen poison-
after school.» ing. Which laboratory result should receive atten-
C) «Sometimes I take the pills in the morning tion by the nurse?
and other times at night.» A) Sedimentation rate
D) «I am feeling much better than I did last B) Profile 2
week.» C) Bilirubin
D) Neutrophils

Review Information: The correct answer is C:


«Sometimes I take the pills in the morning and Review Information: The correct answer is C:
other times at night.» Bilirubin
Inconsistency in taking the prescribed medica- Bilirubin, along with liver enzymes ALT and AST,
tion indicates more teaching is needed. may rise in the second stage (1-3 days) after a
significant overdose, indicating cellular necrosis
Question36 and liver dysfunction. A prolonged prothrombin
The nurse is caring for a 10 year-old client who time may also be found.
will be placed on heparin therapy. Which assess-
ment is critical for the nurse to make before initi- Question39
ating therapy An elderly client is on an anticholinergic metered
A) Vital signs dose inhaler (MDI) for chronic obstructive pulmo-
B) Weight nary disease. The nurse would suggest a spacer
C) Lung sounds to
D) Skin turgor A) enhance the administration of the medication
B) increase client compliance
C) improve aerosol delivery in clients who are
Review Information: The correct answer is B: not able to coordinate the MDI
Weight D) prevent exacerbation of COPD
Check the client>>s weight because dosage is
calculated on the basis of weight.
Review Information: The correct answer is C:
Question37 improve aerosol delivery in clients who are not
In providing care for a client with pain from a sick- able to coordinate the MDI
le cell crisis, which one of the following medica- Spacers improve the medication delivery in cli-
tion orders for pain control should be questioned ents who are unable to coordinate the move-
by the nurse? ments of administering a dose with an MDI.
A) Demerol
B) Morphine
C) Methadone Question40
D) Codeine The nurse is teaching a parent how to adminis-
ter oral iron supplements to a 2 year-old child.
Review Information: The correct answer is A: Which of the following interventions should be
Demerol included in the teaching?
Meperidine is not recommended in clients with A) Stop the medication if the stools become tarry
sickle cell disease. Normeperidine, a metabolite green
of meperidine, is a central nervous system stim- B) Give the medicine with orange juice and
ulant that produces anxiety, tremors, myoclonus, through a straw
and generalized seizures when it accumulates C) Add the medicine to a bottle of formula
with repetitive dosing. Clients with sickle cell dis- D) Administer the iron with your child>s meals
ease are particularly at risk for normeperidine-
Collected by :DeepaRajesh [ 46 ]
rajesh.ks21@gmail.com
Kuwait
Review Information: The correct answer is B:
Give the medicine with orange juice and through Question3
a straw A male client is admitted with a spinal cord injury
Absorption of iron is facilitated in an environment at level C4. The client asks the nurse how the
rich in Vitamin C. Since liquid iron preparation injury is going to affect his sexual function. The
will stain teeth, a straw is preferred. nurse would respond
A) «Normal sexual function is not possible.»
B) «Sexual functioning will not be impaired at
Free NCLEX-RN Sample Test Ques- all.»
tions For Nursing Review (Pharmacol- C) «Erections will be possible.»
ogy Set 1) D) «Ejaculation will be normal.»
Jul31,
Review Information: The correct answer is C:
Question1 «Erections will be possible.»
A client has an order for antibiotic therapy after Because they are a reflex reaction, erections can
hospital treatment of a staph infection. Which of be stimulated by stroking the genitalia.
the following should the nurse emphasize?
A) Scheduling follow-up blood cultures
Question4
B) Completing the full course of medications
An 82 year-old client complains of chronic con-
C) Visiting the provider in a few weeks
stipation. To improve bowel function, the nurse
D) Monitoring for signs of recurrent infection
should first suggest
A) Increasing fiber intake to 20-30 grams daily
B) Daily use of laxatives
Review Information: The correct answer is B:
C) Avoidance of binding foods such as cheese
Completing the full course of medications
and chocolate
In order for antibiotic therapy to be effective in
D) Monitoring a balance between activity and
eradicating an infection, the client must compete
rest
the entire course of prescribed therapy. When
findings subside, stopping the medication early
may lead to recurrence or subsequent drug re-
Review Information: The correct answer is A:
sistance.
Increasing fiber intake to 20-30 grams daily
The incorporation of high fiber into the diet is an
effective way to promote bowel elimination in the
Question2 elderly.
A 72 year-old client is admitted for possible de-
hydration. The nurse knows that older adults are
particularly at risk for dehydration because they
Question5
have
A 4 year-old child is admitted with burns on his
A) an increased need for extravascular fluid
legs and lower abdomen. When assessing the
B) a decreased sensation of thirst
child’s hydration status, which of the following in-
C) an increase in diaphoresis
dicates a less than adequate fluid replacement?
D) higher metabolic demands
A) Decreasing hematocrit and increasing urine
volume
B) Rising hematocrit and decreasing urine vol-
Review Information: The correct answer is B:
ume
a decreased sensation of thirst
C) Falling hematocrit and decreasing urine vol-
The elderly have a reduction in thirst sensation
ume
causing them to consume less fluid. Other risk
D) Stable hematocrit and increasing urine vol-
factors may include fear of incontinence, inability
ume
to drink fluids independently and lack of motiva-
tion.
Collected by :DeepaRajesh [ 47 ]
rajesh.ks21@gmail.com
Kuwait
Review Information: The correct answer is B: D) 20-50 beats/minute
Rising hematocrit and decreasing urine volume
A rising hematocrit indicates a decreased total
blood volume, a finding consistent with dehydra- Review Information: The correct answer is C:
tion. 40-70 beats/minute
The intrinsic rate of the AV node is within the
range of 40-70 beats per minute.
Question6
A client receiving chemotherapy has developed
sores in his mouth. He asks the nurse why this Question9
happened. What is the nurse’s best response? A client is to receive 3 doses of potassium chlo-
A) «It is a sign that the medication is working.» ride 10 mEq in 100cc normal saline to infuse
B) «You need to have better oral hygiene.» over 30 minutes each. Which of the following is a
C) «The cells in the mouth are sensitive to the priority assessment to perform before giving this
chemotherapy.» medication?
D) «This always happens with chemotherapy.» A) Oral fluid intake
B) Bowel sounds
C) Grip strength
Review Information: The correct answer is D) Urine output
C: «The cells in the mouth are sensitive to the
chemotherapy.»
The epithelial cells in the mouth are very sensi- Review Information: The correct answer is D:
tive to chemotherapy due to their high rate of cell Urine output
turnover. Potassium chloride should only be administered
after adequate urine output (>20cc/hour for 2
consecutive hours) has been established. Im-
Question7 paired ability to excrete potassium via the kid-
You are caring for a client with deep vein throm- neys can result in hyperkalemia.
bosis who is on Heparin IV. The latest APTT is 50
seconds. If the laboratory normal range is 16-24
seconds, you would anticipate Question10
A) maintaining the current heparin dose The unlicensed assistive personnel (UAP) re-
B) increasing the heparin as it does not appear ports to the nurse that a client with cirrhosis who
therapeutic. had a paracentesis yesterday has become more
C) giving protamine sulfate as an antidote. lethargic and has musty smelling breath. A criti-
D) repeating the blood test 1 hour after giving cal assessment for increasing encephalopathy
heparin. is
A) monitor the client>s clotting status
B) assess upper abdomen for bruits
Review Information: The correct answer is A: C) assess for flap-like tremors of the hands
maintaining the current heparin dose D) measure abdominal girth changes
The range for a therapeutic APTT is 1.5-2 times
the control. Therefore the client is receiving a
therapeutic dose of Heparin. Review Information: The correct answer is C:
assess for flap-like tremors of the hands
A client with cirrhosis of the liver who develops
Question8 subtle changes in mental status and has a musty
A client is admitted with a diagnosis of nodal odor to the breath is at risk for developing more
bigeminy. The nurse knows that the atrioven- advanced signs of encephalopathy.
tricular (AV) node has an intrinsic rate of
A) 60-100 beats/minute
B) 10-30 beats/minute Question11
C) 40-70 beats/minute A client is scheduled for an intravenous pyelo-
Collected by :DeepaRajesh [ 48 ]
rajesh.ks21@gmail.com
Kuwait
gram (IVP). After the contrast material is inject- A woman with a 28 week pregnancy is on the
ed, which of the following client reactions should way to the emergency department by ambulance
be reported immediately? with a tentative diagnosis of abruptio placenta.
A) Feeling warm Which should the nurse do first when the woman
B) Face flushing arrives?
C) Salty taste A) administer oxygen by mask at 100%
D) Hives B) start a second IV with an 18 gauge cannula
C) check fetal heart rate every 15 minutes
D) insert urethral catheter with hourly urine out-
Review Information: The correct answer is D: puts
Hives
This is a sign of anaphylaxis and should be re-
ported immediately. The other reactions are con- Review Information: The correct answer is A:
sidered normal and the client should be informed administer oxygen by mask at 100%
that they may occur. Administering oxygen in this situation would in-
. crease the circulating oxygen in the mother’s cir-
culation to the fetus’s circulation. This action will
Question12 minimize complications.
A client is prescribed an inhaler. How should the
nurse instruct the client to breathe in the medica-
tion? Question15
A) As quickly as possible A client in respiratory distress is admitted with
B) As slowly as possible arterial blood gas results of: PH 7.30; PO2 58,
C) Deeply for 3-4 seconds PCO2 34; and HCO3 19. The nurse determines
D) Until hearing whistling by the spacer that the client is in
A) metabolic acidosis
B) metabolic alkalosis
Review Information: The correct answer is C: C) respiratory acidosis
Deeply for 3-4 seconds D) respiratory alkalosis
The client should be instructed to breath in the
medication for 3-4 seconds in order to receive Review Information: The correct answer is A:
the correct dosage of medication. metabolic acidosis
These lab values indicate metabolic acidosis:
the PH is low, PCO2 is normal, and bicarbonate
Question13 level is low.
The nurse is caring for clients over the age of 70.
The nurse knows that due to age-related chang-
es, the elderly clients tolerate diets that are Question16
A) high protein A client is diagnosed with gastroesophageal re-
B) high carbohydrates flux disease (GERD). The nurse>s instruction to
C) low fat the client regarding diet should be to
D) high calories A) avoid all raw fruits and vegetables
B) increase intake of milk products
C) decrease intake of fatty foods
Review Information: The correct answer is C: D) focus on 3 average size meals a day
low fat
Due to age related changes, the diet of the eld-
erly should include a lower quantity and higher Review Information: The correct answer is C:
quality of food. Fewer carbohydrates and fats decrease intake of fatty foods
are required in their diets. GERD may be aggravated by a fatty diet. A diet
low in fat would decrease the symptoms of GERD.
Other agents which should also be decreased or
Question14 avoided are: cigarette smoking, caffeine, alco-
Collected by :DeepaRajesh [ 49 ]
rajesh.ks21@gmail.com
Kuwait
hol, chocolate, and meperidine (Demerol). months
D) The client>s risk for cardiac complications

Question17
After surgery, a client with a nasogastric tube Review Information: The correct answer is
complains of nausea. What action would the C: The average blood glucose for the past 2-3
nurse take? months
A) Call the health care provider By testing the portion of the hemoglobin that ab-
B) Administer an antiemetic sorbs glucose, it is possible to determine the av-
C) Put the bed in Fowler’s position erage blood glucose over the life span of the red
D) Check the patency of the tube cell, 120 days.

Review Information: The correct answer is D: Question20


Check the patency of the tube A client is admitted for a possible pacemaker in-
An indication that the nasogastric tube is ob- sertion. What is the intrinsic rate of the heart>s
structed is a client’s complaint of nausea. Na- own pacemaker?
sogastric tubes may become obstructed with A) 30-50 beats/minute
mucus or sediment. B) 60-100 beats/minute
C) 20-60 beats/minute
D) 90-100 beats/minute
Question18
A client with testicular cancer has had an orchiec-
tomy. Prior to discharge the client expresses his Review Information: The correct answer is B:
fears related to his prognosis. Which principle 60-100 beats/minute
should the nurse base the response on? This is the intrinsic rate of the SA node.
A) Testicular cancer has a cure rate of 90% with
early diagnosis
B) Testicular cancer has a cure rate of 50% with Question21
early diagnosis The nurse discusses nutrition with a pregnant
C) Intensive chemotherapy is the treatment of woman who is iron deficient and follows a veg-
choice etarian diet. The selection of which foods indi-
D) Testicular cancer is usually fatal cates the woman has learned sources of iron?
A) Cereal and dried fruits
B) Whole grains and yellow vegetables
Review Information: The correct answer is C) Leafy green vegetables and oranges
A: Testicular cancer has a cure rate of 90% with D) Fish and dairy products
early diagnosis
With aggressive treatment and early detection/
diagnosis the cure rate is 90%. Review Information: The correct answer is A:
Cereal and dried fruits
Both of these foods would be a good source of
Question19 iron.
A client newly diagnosed with Type I Diabetes Question22
Mellitus asks the purpose of the test measur- Prior to administering Alteplase (TPA) to a client
ing glycosylated hemoglobin. The nurse should admitted for a cerebral vascular accident (CVA),
explain that the purpose of this test is to deter- it is critical that the nurse assess:
mine: A) Neuro signs
A) The presence of anemia often associated with B) Mental status
Diabetes C) Blood pressure
B) The oxygen carrying capacity of the client>s D) PT/PTT
red cells
C) The average blood glucose for the past 2-3 Review Information: The correct answer is D:
Collected by :DeepaRajesh [ 50 ]
rajesh.ks21@gmail.com
Kuwait
PT/PTT of your decision.»
TPA is a potent thrombolytic enzyme. Because
bleeding is the most common side effect, it is Review Information: The correct answer is C:
most essential to evaluate clotting studies in- «You have the right to change your mind. You
cluding PT, PTT, APTT, platelets, and hematocrit seem anxious. Can we talk about it?»
before beginning therapy. This response indicates acknowledgment of the
Question23 client’s rights and the opportunity for the client
The nurse enters the room of a client diagnosed to clarify and ventilate concerns. After this, if the
with COPD. The client’s skin is pink, and respi- client continues to refuse, the provider should be
rations are 8 per minute. The client’s oxygen is notified.
running at 6 liters per minute. What should be
the nurse’s first action? Question26
A) Call the health care provider A nurse who has been named in a lawsuit can
B) Put the client in Fowler’s position use which of these factors for the best protection
C) Lower the oxygen rate in a court of law?
D) Take the vital signs A) Clinical specialty certification in the associ-
ated area of practice
Review Information: The correct answer is C: B) Documentation on the specific client record
Lower the oxygen rate with a focus on the nursing process
In client’s diagnosed with COPD, the drive to C) Yearly evaluations and proficiency reports
breathe is hypoxia. If oxygen is delivered at too prepared by nurse’s manager
high of a concentration, this drive will be elimi- D) Verification of provider>s orders for the plan
nated and the client’s depth and rate of respi- of care with identification of outcomes
rations will decrease. Therefore the first action
should be to lower the oxygen rate. Review Information: The correct answer is B:
Question24 Documentation on the specific client record with
The client with goiter is treated with potassium a focus on the nursing process
iodide preoperatively. What should the nurse Documentation is the key to protect nurses when
recognize as the purpose of this medication? a lawsuit is filed. The thorough documentation
A) Reduce vascularity of the thyroid should include all steps of the nursing process –
B) Correct chronic hyperthyroidism assessment, analysis, plan, intervention, evalu-
C) Destroy the thyroid gland function ation. In addition, it should include pertinent data
D) Balance enzymes and electrolytes such as times, dosages and sites of actions,
assessment data, the nurse’s response to a
Review Information: The correct answer is A: change in the client’s condition, specific actions
Reduce vascularity of the thyroid taken, if and when the notification occurred to
Potassium iodide solution, or Lugol>>s solution the provider or other health care team members,
may be used preoperatively to reduce the size and what was prescribed along with the client’s
and vascularity of the thyroid gland. outcomes.
Question27
Question25 The nurse is caring for clients over the age of 70.
One hour before the first treatment is scheduled, The nurse is aware that when giving medications
the client becomes anxious and states he does to older clients, it is best to
not wish to go through with electroconvulsive A) start low, go slow
therapy. Which response by the nurse is most B) avoid stopping a medication entirely
appropriate? C) avoid drugs with side effects that impact cog-
A) «I’ll go with you and will be there with you dur- nition
ing the treatment.» D) review the drug regimen yearly
B) «You’ll be asleep and won’t remember any-
thing.» Review Information: The correct answer is A:
C) «You have the right to change your mind. You start low, go slow
seem anxious. Can we talk about it?» Due to physiological changes in the elderly, as
D) «I’ll call the health care provider to notify them well as conditions such as dehydration, hyper-
Collected by :DeepaRajesh [ 51 ]
rajesh.ks21@gmail.com
Kuwait
thermia, immobility and liver disease, the effec- Question31
tive metabolism of drugs may decrease. As a Which of these clients should the charge nurse
result, drugs can accumulate to toxic levels and assign to the registered nurse (RN)?
cause serious adverse reactions. A) A 56 year-old with atrial fibrillation receiving
Question28 digoxin
You are caring for a hypertensive client with a B) A 60 year-old client with COPD on oxygen at
new order for captopril (Capoten). Which infor- 2 L/min
mation should the nurse include in client teach- C) A 24 year-old post-op client with type 1 diabe-
ing? tes in the process of discharge
A) Avoid green leafy vegetables D) An 80 year-old client recovering 24 hours post
B) Restrict fluids to 1000cc/day right hip replacement
C) Avoid the use of salt substitutes
D) Take the medication with meals Review Information: The correct answer is C:
A 24 year-old post-op client with type 1 diabetes
Review Information: The correct answer is C: in the process of discharge
Avoid the use of salt substitutes Discharge teaching must be done by an RN.
Captopril can cause an accumulation of potas- Practical nurses (PNs) or unlicensed assistive
sium or hyperkalemia. Clients should avoid the personnel (UAPs) can reinforce education after
use of salt substitutes, which are generally po- the RN does the initial teaching.
tassium-based. Question32
Question29 A hypertensive client is started on atenolol (Ten-
A client has bilateral knee pain from osteoarthri- ormin). The nurse instructs the client to immedi-
tis. In addition to taking the prescribed non-ster- ately report which of these findings?
oidal anti-inflammatory drug (NSAID), the nurse A) Rapid breathing
should instruct the client to B) Slow, bounding pulse
A) start a regular exercise program C) Jaundiced sclera
B) rest the knees as much as possible to de- D) Weight gain
crease inflammation
C) avoid foods high in citric acid Review Information: The correct answer is B:
D) keep the legs elevated when sitting Slow, bounding pulse
Atenolol (Tenormin) is a beta-blocker that can
Review Information: The correct answer is A: cause side effects including bradycardia and hy-
start a regular exercise program potension.
A regular exercise program is beneficial in treat- Question33
ing osteoarthritis. It can restore self-esteem and An 80 year-old client is admitted with a diagnosis
improve physical functioning. of malnutrition. In addition to physical assess-
Question30 ments, which of the following lab tests should be
An arterial blood gases test (ABG) is ordered closely monitored?
for a confused client. The respiratory therapist A) Urine protein
draws the blood and then asks the nurse to apply B) Urine creatinine
pressure to the area so the therapist can take the C) Serum calcium
specimen to the lab. How long should the nurse D) Serum albumin
apply pressure to the area?
A) 3 minutes Review Information: The correct answer is D:
B) 5 minutes Serum albumin
C) 8 minutes Serum albumin is a valuable indicator of protein
D) 10 minutes deficiency and, later, nutritional status in adults.
A normal reading for an elder’s serum albumin is
Review Information: The correct answer is B: between 3.0-5.0 g/dl.
5 minutes Question34
It is necessary to apply pressure to the area for Upon admission to an intensive care unit, a client
5 minutes to prevent bleeding and the formation diagnosed with an acute myocardial infarction is
of hematomas. ordered oxygen. The nurse knows that the major
Collected by :DeepaRajesh [ 52 ]
rajesh.ks21@gmail.com
Kuwait
reason that oxygen is administered in this situa- During nursing rounds which of these assess-
tion is to ments would require immediate corrective action
A) saturate the red blood cells and further instruction to the practical nurse (PN)
B) relieve dyspnea about proper care?
C) decrease cyanosis A) The weights of the skin traction of a client are
D) increase oxygen level in the myocardium hanging about 2 inches from the floor
B) A client with a hip prosthesis 1 day post op-
Review Information: The correct answer is D: eratively is lying in bed with internal rotation and
increase oxygen level in the myocardium adduction of the affected leg
Anoxia of the myocardium occurs in myocardial C) The nurse observes that the PN moves the
infarction. Oxygen administration will help re- extremity of a client with an external fixation de-
lieve dyspnea and cyanosis associated with the vice by picking up the frame
condition but the major purpose is to increase D) A client with skeletal traction states «The other
the oxygen concentration in the damaged myo- nurse said that the clear, yellow and crusty drain-
cardial tissue. age around the pin site is a good sign»
Question35
The nurse is teaching a client with chronic re- Review Information: The correct answer is B:
nal failure (CRF) about medications. The client A client with a hip prosthesis 1 day post opera-
questions the purpose of aluminum hydroxide tively is lying in bed with internal rotation and ad-
(Amphojel) in her medication regimen. What is duction of the affected leg
the best explanation for the nurse to give the cli- This position should be prevented in order to
ent about the therapeutic effects of this medica- prevent dislodgment of the hip prosthesis, es-
tion? pecially in the first 48 to 72 hours post-op. The
A) It decreases serum phosphate other assessments are not of concern.
B) It will reduce serum calcium Question38
C) Amphojel increases urine output A client diagnosed with gouty arthritis is admit-
D) The drug is taken to control gastric acid se- ted with severe pain and edema in the right foot.
cretion When the nurse develops a plan of care, which
Review Information: The correct answer is A: intervention should be included?
It decreases serum phosphate A) high protein diet
Aluminum binds phosphates that tend to accu- B) salicylates
mulate in the patient with chronic renal failure C) hot compresses to affected joints
due to decreased filtration capacity of the kidney. D) intake of at least 3000cc/day
Antacids such as Amphojel are commonly used
to accomplish this. Review Information: The correct answer is D:
Question36 intake of at least 3000cc/day
A 66 year-old client is admitted for mitral valve Fluid intake should be increased to prevent pre-
replacement surgery. The client has a history cipitation of urate in the kidneys.
of mitral valve regurgitation and mitral stenosis Question39
since her teenage years. During the admission A 55 year-old woman is taking Prednisone and
assessment, the nurse should ask the client if as aspirin (ASA) as part of her treatment for rheu-
a child she had matoid arthritis. Which of the following would be
A) measles an appropriate intervention for the nurse?
B) rheumatic fever A) Assess the pulse rate q 4 hours
C) hay fever B) Monitor her level of consciousness q shift
D) encephalitis C) Test her stools for occult blood
D) Discuss fiber in the diet to prevent constipa-
Review Information: The correct answer is B: tion
rheumatic fever
Clients that present with mitral stenosis often Review Information: The correct answer is C:
have a history of rheumatic fever or bacterial en- Test her stools for occult blood
docarditis. Both Prednisone and ASA can lead to GI bleed-
Question37 ing, therefore monitoring for occult blood would
Collected by :DeepaRajesh [ 53 ]
rajesh.ks21@gmail.com
Kuwait
be appropriate. getting out of bed. In the event that this restricted
Question40 movement could cause more harm, such as as-
A client with testicular cancer is scheduled for a piration, then a sitter should be requested. These
right orchiectomy. The nurse knows that an or- are to be provided by the facility in the event the
chiectomy is the family cannot do so. This client needs to cough
A) surgical removal of the entire scrotum and be watched rather than restricted. Suction-
B) surgical removal of a testicle ing will not prevent aspiration in this situation.
C) dissection of related lymph nodes Cough suppressants should be avoided for this
D) partial surgical removal of the penis client.
Question3
Review Information: The correct answer is B: A couple trying to conceive asks the nurse when
surgical removal of a testicle ovulation occurs. The woman reports a regular
The affected testicle is surgically removed along 32 day cycle. Which response by the nurse is
with its tunica and spermatic cord. correct?
A) Days 7-10
Free NCLEX-RN Sample Test Ques- B) Days 10-13
tions For Nursing Review (Part 5) C) Days 14-16
Jul31, D) Days 17-19
Review Information: The correct answer is D:
Days 17-19
Question1
Ovulation occurs 14 days prior to menses. Con-
A client complains of some discomfort after a
sidering that the woman>>s cycle is 32 days,
below the knee amputation. Which action by the
subtracting 14 from 32 suggests ovulation is at
nurse is most appropriate initially?
about the 18th day.
A) Conduct guided imagery or distraction
B) Ensure that the stump is elevated the first day Question4
post-op A newborn is having difficulty maintaining a tem-
C) Wrap the stump snugly in an elastic bandage perature above 98 degrees Fahrenheit and has
D) Administer opioid narcotics as ordered been placed in an incubator. Which action is a
nursing priority?
Review Information: The correct answer is B: A) Protect the eyes of the neonate from the heat
Ensure that the stump is elevated the first day lamp
post-op B) Monitor the neonate’s temperature
This priority intervention prevents pressure C) Warm all medications and liquids before giv-
caused by pooling of blood, thus minimizing the ing
pain. Without this measure, a firm elastic band- D) Avoid touching the neonate with cold hands
age, opioid narcotics, or guided imagery will
have little effect. Opioid narcotics are given for Review Information: The correct answer is B:
severe pain. Monitor the neonate’s temperature
When using a warming device the neonate’s
Question2
temperature should be continuously monitored
A 78 year-old client with pneumonia has a pro-
for undesired elevations. The use of heat lamps
ductive cough, but is confused. Safety protective
is not safe as there is no way to regulate their
devices (restraints) have been ordered for this
temperature. Warming medications and fluids is
client. How can the nurse prevent aspiration?
not indicated. While touching with cold hands can
A) Suction the client frequently while restrained
startle the infant it does not pose a safety risk.
B) Secure all 4 restraints to 1 side of bed
C) Obtain a sitter for the client while restrained
D) Request an order for a cough suppressant Question5
Which oxygen delivery system would the nurse
Review Information: The correct answer is C: apply that would provide the highest concentra-
Obtain a sitter for the client while restrained tions of oxygen to the client?
The plan to use safety devices (restraints) should A) Venturi mask
be rethought. Restraints are used to protect the B) Partial rebreather mask
client from harm caused by removing tubes or C) Non-rebreather mask
Collected by :DeepaRajesh [ 54 ]
rajesh.ks21@gmail.com
Kuwait
D) Simple face mask expected with this type of surgery. The bleed-
ing should also be documented in the nurse’s
Review Information: The correct answer is C: notes.
Non-rebreather mask Question8
The non-rebreather mask has a one-way valve The nurse is caring for a 1 year-old child who
that prevents exhales air from entering the res- has 6 teeth. What is the best way for the nurse to
ervoir bag and one or more valves covering the give mouth care to this child?
air holes on the face mask itself to prevent inha- A) Using a moist soft brush or cloth to clean teeth
lation of room air but to allow exhalation of air. and gums
When a tight seal is achieved around the mask B) Swabbing teeth and gums with flavored
up to 100% of the oxygen is available. mouthwash
Question6 C) Offering a bottle of water for the child to drink
At a senior citizens meeting a nurse talks with a D) Brushing with toothpaste and flossing each
client who has Type 1 diabetes mellitus. Which tooth
statement by the client during the conversation
is most predictive of a potential for impaired skin Review Information: The correct answer is A:
integrity? Using a moist soft brush or cloth to clean teeth
A) «I give my insulin to myself in my thighs.» and gums
B) «Sometimes when I put my shoes on I don>t The nurse should use a soft cloth or soft brush to
know where my toes are.» do mouth care so that the child can adjust to the
C) «Here are my up and down glucose readings routine of cleaning the mouth and teeth.
that I wrote on my calendar.»
D) «If I bathe more than once a week my skin Question9
feels too dry.» In addition to standard precautions, a nurse
should implement contact precautions for which
Review Information: The correct answer is B: client?
«Sometimes when I put my shoes on I don>>t A) 60 year-old with herpes simplex
know where my toes are.» B) 6 year-old with mononucleosis
Peripheral neuropathy can lead to lack of sensa- C) 45 year-old with pneumonia
tion in the lower extremities. Clients who do not D) 3 year-old with scarlet fever
feel pressure and/or pain are at high risk for skin
impairment. Review Information: The correct answer is A:
60 year-old with herpes simplex
Question7 Clients who have herpes simplex infections must
A client returns from surgery after an open reduc- have contact precautions in addition to standard
tion of a femur fracture. There is a small blood- precautions because of the associated, potential-
stain on the cast. Four hours later, the nurse ob- ly weeping, skin lesions. Contact precautions are
serves that the stain has doubled in size. What is used for clients who are infected by microorgan-
the best action for the nurse to take? isms that are transmitted by direct contact with
A) Call the health care provider the client, including hand or skin-to-skin contact.
B) Access the site by cutting a window in the
cast Question10
C) Simply record the findings in the nurse>s Which of the following situations is most likely to
notes only produce sepsis in the neonate?
D) Outline the spot with a pencil and note the A) Maternal diabetes
time and date on the cast B) Prolonged rupture of membranes
C) Cesarean delivery
Review Information: The correct answer is D: D) Precipitous vaginal birth
Outline the spot with a pencil and note the time
and date on the cast Review Information: The correct answer is B:
This is a good way to assess the amount of bleed- Prolonged rupture of membranes
ing over a period of time. The bleeding does not Premature rupture of the membranes (PROM) is
appear to be excessive and some bleeding is a leading cause of newborn sepsis. After 12-24
Collected by :DeepaRajesh [ 55 ]
rajesh.ks21@gmail.com
Kuwait
hours of leaking fluid, measures are taken to re- Provide water feedings at least every 2 hours
duce the risk to mother and the fetus/newborn. Protecting the eyes of the neonates is very im-
portant to prevent damage when under the ultra-
Question11 violet lights, but since the blanket is used, extra
The nurse is teaching a parent about side effects protection of the eyes is unnecessary. It is rec-
of routine immunizations. Which of the following ommended that the neonate remain under the
must be reported immediately? lights for extended periods. The neonate’s skin is
A) Irritability exposed to the light and the temperature is moni-
B) Slight edema at site tored, but a heater may not be necessary. There
C) Local tenderness is no reason to withhold feedings. Frequent wa-
D) Seizure activity ter or feedings are given to help with the excre-
tion of the bilirubin in the stool.
Review Information: The correct answer is D: Question14
Seizure activity A nurse is performing the routine daily cleaning
Other reactions that should be reported include of a tracheostomy. During the procedure, the cli-
crying for >3 hours, temperature over 104.8 de- ent coughs and displaces the tracheostomy tube.
grees Fahrenheit following DPT immunization, This negative outcome could have avoided by
and tender, swollen, reddened areas. A) placing an obturator at the client’s bedside
Question12 B) having another nurse assist with the proce-
The nurse is at the community center speaking dure
with retired people about glaucoma. Which com- C) fastening clean tracheostomy ties before re-
ment by one of the retirees would the nurse sup- moving old ties
port to reinforce correct information? D) placing the client in a flat, supine position
A) «I usually avoid driving at night since lights
sometimes seem to make things blur.» Review Information: The correct answer is C:
B) «I take half of the usual dose for my sinuses fastening clean tracheostomy ties before remov-
to maintain my blood pressure.» ing old ties
C) «I have to sit at the side of the pool with the Fastening clean tracheostomy ties before remov-
grandchildren since I can>t swim with this eye ing old ones will ensure that the tracheostomy
problem.» is secured during the entire cleaning procedure.
D) «I take extra fiber and drink lots of water to The obturator is useful to keep the airway open
avoid getting constipated.» only after the tracheostomy outer tube is coughed
out. A second nurse is not needed. Changing the
Review Information: The correct answer is D: position may not prevent a dislodged tracheos-
«I take extra fiber and drink lots of water to avoid tomy.
getting constipated.» Question15
Any activity that involves straining should be A 4 year-old hospitalized child begins to have a
avoided in clients with glaucoma. Such activitiesseizure while playing with hard plastic toys in the
would increase intraocular pressure. hallway. Of the following nursing actions, which
Question13 one should the nurse do first?
A newborn has hyperbilirubinemia and is un- A) Place the child in the nearest bed
dergoing phototherapy with a fiberoptic blanket. B) Administer IV medication to slow down the
Which safety measure is most important during seizure
this process? C) Place a padded tongue blade in the child>s
A) Regulate the neonate’s temperature using a mouth
radiant heater D) Remove the child>s toys from the immediate
B) Withhold feedings while under the photother- area
apy
C) Provide water feedings at least every 2 hours Review Information: The correct answer is D:
D) Protect the eyes of neonate from the photo- Remove the child>>s toys from the immediate
therapy lights area
Nursing care for a child having a seizure includes,
Review Information: The correct answer is C: maintaining airway patency, ensuring safety, ad-
Collected by :DeepaRajesh [ 56 ]
rajesh.ks21@gmail.com
Kuwait
ministering medications, and providing emotion- D) Peripheral glucose stick
al support. Since the seizure has already started,
nothing should be forced into the child>>s mouth Review Information: The correct answer is C:
and the child should not be moved. Of the choic- Pulse oximetry
es given, the first priority would be to provide a A sudden change in mental status in any post-
safe environment. op client should trigger a nursing intervention
Question16 directed toward respiratory evaluation. Pulse
The nurse is teaching home care to the parents oximetry would be the initial assessment. If avail-
of a child with acute spasmodic croup. The most able, arterial blood gases would be better. Acute
important aspects of this care is/are respiratory failure is the sudden inability of the
A) sedation as needed to prevent exhaustion respiratory system to maintain adequate gas
B) antibiotic therapy for 10 to 14 days exchange which may result in hypercapnia and/
C) humidified air and increased oral fluids or hypoxemia. Clinical findings of hypoxemia in-
D) antihistamines to decrease allergic response clude these finding which are listed in order of
initial to later findings: restlessness, irritability,
Review Information: The correct answer is C: agitation, dyspnea, disorientation, confusion, de-
humidified air and increased oral fluids lirium, hallucinations, and loss of consciousness.
The most important aspects of home care for a While there may be other factors influencing the
child with acute spasmodic croup are humidified client>>s behavior, the first nursing action should
air and increased oral fluids. Moisture soothes be directed toward maintaining oxygenation.
inflamed membranes. Adequate systemic hydra- Once respiratory or oxygenation issues are ruled
tion aids is mucociliary clearance and keeps se- out then significant changes in glucose would be
cretions thin, white, watery, and easily removed evaluated.
with minimal coughing. Question19
Question17 A newborn delivered at home without a birth at-
The nurse is assigned to care for a client who tendant is admitted to the hospital for observa-
has a leaking intracranial aneurysm. To minimize tion. The initial temperature is 95 degrees Fahr-
the risk of rebleeding, the nurse should plan to enheit (35 degrees Celsius) axillary. The nurse
A) restrict visitors to immediate family recognizes that cold stress may lead to what
B) avoid arousal of the client except for family complication?
visits A) Lowered BMR
C) keep client>s hips flexed at no less than 90 B) Reduced PaO2
degrees C) Lethargy
D) apply a warming blanket for temperatures of D) Metabolic alkalosis
98 degrees Fahrenheit or less
Review Information: The correct answer is B:
Review Information: The correct answer is A: Reduced PaO2
restrict visitors to immediate family Cold stress causes increased risk for respira-
Maintaining a quiet environment will assist in tory distress. The baby delivered in such circum-
minimizing cerebral rebleeding. When family vis- stances needs careful monitoring. In this situa-
it, the client should not be disturbed. If the client tion, the newborn must be warmed immediately
is awake, topics of a general nature are better to increase its temperature to at least 97 degrees
choices for discussion than topics that result in Fahrenheit (36 degrees Celsius).
emotional or physiological stimulation. Question20
Question18 Which contraindication should the nurse assess
A client who is 12 hour post-op becomes con- for prior to giving a child immunizations?
fused and says: “Giant sharks are swimming A) Mild cold symptoms
across the ceiling.” Which assessment is nec- B) Chronic asthma
essary to adequately identify the source of this C) Depressed immune system
client>s behavior? D) Allergy to eggs
A) Cardiac rhythm strip
B) Pupillary response Review Information: The correct answer is C:
C) Pulse oximetry Depressed immune system
Collected by :DeepaRajesh [ 57 ]
rajesh.ks21@gmail.com
Kuwait
Children who have a depressed immune system tis. This side effect is age-related and can be de-
related to HIV or chemotherapy should not be tected with regular assessment of liver enzymes,
given routine immunizations. which are released into the blood from damaged
liver cells.
Question21 Question24
The nurse is caring for a client with a myocardial A woman in her third trimester complains of se-
infarction. Which finding requires the nurse>s vere heartburn. What is appropriate teaching
immediate action? by the nurse to help the woman alleviate these
A) Periorbital edema symptoms?
B) Dizzy spells A) Drink small amounts of liquids frequently
C) Lethargy B) Eat the evening meal just before retiring
D) Shortness of breath C) Take sodium bicarbonate after each meal
D) Sleep with head propped on several pillows
Review Information: The correct answer is B:
Dizzy spells Review Information: The correct answer is D:
Cardiac dysrhythmias may cause a transient Sleep with head propped on several pillows
drop in cardiac output and decreased blood flow Heartburn is a burning sensation caused by re-
to the brain. Near syncope refers to lightheart- gurgitation of gastric contents. It is best relieved
edness, dizziness, temporary confusion. Such by sleeping position, eating small meals, and not
«spells» may indicate runs of ventricular tachy- eating before bedtime.
cardia or periods of asystole and should be re- Question25
ported immediately. A 16 year-old boy is admitted for Ewing>s sar-
coma of the tibia. In discussing his care with the
Question22 parents, the nurse understands that the initial
Decentralized scheduling is used on a nursing treatment most often includes
unit. A chief advantage of this management strat- A) amputation just above the tumor
egy is that it: B) surgical excision of the mass
A) considers client and staff needs C) bone marrow graft in the affected leg
B) conserves time spent on planning D) radiation and chemotherapy
C) frees the nurse manager to handle other pri-
orities Review Information: The correct answer is D:
D) allows requests for special privileges radiation and chemotherapy
The initial treatment of choice for Ewing>>s sar-
Review Information: The correct answer is A: coma is a combination of radiation and chemo-
considers client and staff needs therapy.
Decentralized staffing takes into consideration
specific client needs and staff interests and abili- Question26
ties. A new nurse manager is responsible for inter-
viewing applicants for a staff nurse position.
Question23 Which interview strategy would be the best ap-
Included in teaching the client with tuberculo- proach?
sis taking isoniazid (INH) about follow-up home A) Vary the interview style for each candidate to
care, the nurse should emphasize that a labora- learn different techniques
tory appointment for which of the following lab B) Use simple questions requiring «yes» and
tests is critical? «no» answers to gain definitive information
A) Liver function C) Obtain an interview guide from human re-
B) Kidney function sources for consistency in interviewing each
C) Blood sugar candidate
D) Cardiac enzymes D) Ask personal information of each applicant to
assure he/she can meet job demands
Review Information: The correct answer is A:
Liver function Review Information: The correct answer is C:
INH can cause hepatocellular injury and hepati- Obtain an interview guide from human resources
Collected by :DeepaRajesh [ 58 ]
rajesh.ks21@gmail.com
Kuwait
for consistency in interviewing each candidate C) oliguria
An interview guide used for each candidate ena- D) neck veins are distended
bles the nurse manager to be more objective in
the decision making. The nurse should use re- Review Information: The correct answer is C:
sources available in the agency before attempts oliguria
to develop one from scratch. Certain personal Kidneys maintain fluid volume through adjust-
questions are prohibited, and HR can identify ments in urine volume.
these for novice managers. Question30
Question27 A 70 year-old woman is evaluated in the emer-
What is the best way that parents of pre-school- gency department for a wrist fracture of unknown
ers can begin teaching their child about injury causes. During the process of taking client his-
prevention? tory, which of these items should the nurse iden-
A) Set good examples themselves tify as related to the client’s greatest risk factors
B) Protect their child from outside influences for osteoporosis?
C) Make sure their child understands all the A) History of menopause at age 50
safety rules B) Taking high doses of steroids for arthritis for
D) Discuss the consequences of not wearing many years
protective devices C) Maintaining an inactive lifestyle for the past
10 years
Review Information: The correct answer is A: D) Drinking 2 glasses of red wine each day for
Set good examples themselves the past 30 years
The preschool years are the time for parents to
begin emphasizing safety principles as well as Review Information: The correct answer is
providing protection. Setting a good example B: Taking high doses of steroids for arthritis for
themselves is crucial because of the imitative many years
behaviors of pre-schoolers; they are quick to no- The use of steroids, especially at high doses over
tice discrepancies between what they see and time, increases the risk for osteoporosis. The
what they are told. other options also predispose to osteoporosis,
as do low bone mass, poor calcium absorption
Question28 and moderate to high alcohol ingestion. Long-
A nurse assessing the newborn of a mother with term steroid treatment is the most significant risk
diabetes understands that hypoglycemia is re- factor, however.
lated to what pathophysiological process? Question31
A) Disruption of fetal glucose supply The nurse is caring for a 2 year-old who is being
B) Pancreatic insufficiency treated with chelation therapy, calcium disodium
C) Maternal insulin dependency edetate, for lead poisoning. The nurse should be
D) Reduced glycogen reserves alert for which of the following side effects?
A) Neurotoxicity
Review Information: The correct answer is A: B) Hepatomegaly
Disruption of fetal glucose supply C) Nephrotoxicity
After delivery, the high glucose levels which D) Ototoxicity
crossed the placenta to the fetus are suddenly
stopped. The newborn continues to secrete insu- Review Information: The correct answer is C:
lin in anticipation of glucose. When oral feedings Nephrotoxicity
begin, the newborn will adjust insulin production Nephrotoxicity is a common side effect of calci-
within a day or two. um disodium edetate, in addition to lead poison-
ing in general.
Question29 Question32
The nurse is caring for a client with extracellular The parents of a toddler ask the nurse how long
fluid volume deficit. Which of the following as- their child will have to sit in a car seat while in the
sessments would the nurse anticipate finding? automobile. What is the nurse’s best response to
A) bounding pulse the parents?
B) rapid respirations A) «Your child must use a care seat until he
Collected by :DeepaRajesh [ 59 ]
rajesh.ks21@gmail.com
Kuwait
weighs at least 40 pounds.» Review Information: The correct answer is D:
B) «The child must be 5 years of age to use a remove a mucus plug
regular seat belt.» While no longer recommended for routine suc-
C) «Your child must reach a height of 50 inches tioning, saline may thin and loosen viscous se-
to sit in a seat belt.» cretions that are very difficult to move, perhaps
D) «The child can use a regular seat belt when making them easier to suction.
he can sit still.» Question36
The nurse is performing a gestational age as-
Review Information: The correct answer is A: sessment on a newborn delivered 2 hours ago.
«Your child must use a care seat until he weighs When coming to a conclusion using the Bal-
at least 40 pounds.» lard scale, which of these factors may affect the
Children should use car seats until they weigh score?
40 pounds. A) Birth weight
Question33 B) Racial differences
A client asks the nurse to explain the basic ideas C) Fetal distress in labor
of homeopathic medicine. The response that best D) Birth trauma
explains this approach is that such remedies
A) destroy organisms causing disease Review Information: The correct answer is C:
B) maintain fluid balance Fetal distress in labor
C) boost the immune system The effects of earlier distress may alter the find-
D) increase bodily energy ings of reflex responses as measured on the Bal-
lard tool. Other physical characteristics that esti-
Review Information: The correct answer is C: mate gestational age, such as amount of lanugo,
boost the immune system sole creases and ear cartilage are unaffected by
The practitioner treats with minute doses of plant, the other factors.
mineral or animal substances which provide a Question37
gentle stimulus to the body>>s own defenses. A nurse is caring for a client who had a closed
Question34 reduction of a fractured right wrist followed by
A client with a fractured femur has been in Rus- the application of a fiberglass cast 12 hours ago.
sell’s traction for 24 hours. Which nursing action Which finding requires the nurse’s immediate at-
is associated with this therapy? tention?
A) Check the skin on the sacrum for breakdown A) Capillary refill of fingers on right hand is 3 sec-
B) Inspect the pin site for signs of infection onds
C) Auscultate the lungs for atelectasis B) Skin warm to touch and normally colored
D) Perform a neurovascular check for circula- C) Client reports prickling sensation in the right
tion hand
D) Slight swelling of fingers of right hand
Information: The correct answer is D: Perform
a neurovascular check for circulation Review Information: The correct answer is
While each of these is an important assessment, C: Client reports prickling sensation in the right
the neurovascular integrity check is most associ- hand
ated with this type of traction. Russell’s traction A prickling sensation is an indication of compart-
is Buck’s traction with a sling under the knee. ment syndrome and requires immediate action
by the nurse. The other findings are normal for a
Question35 client in this situation.
When suctioning a client>s tracheostomy, the Question38
nurse should instill saline in order to A client is admitted with the diagnosis of pulmo-
A) decrease the client>s discomfort nary embolism. While taking a history, the client
B) reduce viscosity of secretions tells the nurse he was admitted for the same thing
C) prevent client aspiration twice before, the last time just 3 months ago. The
D) remove a mucus plug nurse would anticipate the provider ordering
A) pulmonary embolectomy
B) vena caval interruption
Collected by :DeepaRajesh [ 60 ]
rajesh.ks21@gmail.com
Kuwait
C) increasing the Coumadin therapy to an INR
of 3-4 Free NCLEX-RN Sample Test Ques-
D) thrombolytic therapy tions For Nursing Review (Part 4)
Jul31,
Review Information: The correct answer is B:
vena caval interruption
Clients with contraindications to Heparin, re- Question1
current PE or those with complications related The clinic nurse is counseling a substance-abus-
to the medical therapy may require vena caval ing post partum client on the risks of continued
interruption by the placement of a filter device cocaine use. In order to provide continuity of
in the inferior vena cava. A filter can be placed care, which nursing diagnosis is a priority?
transvenously to trap clots before they travel to A) Social isolation
the pulmonary circulation. B) Ineffective coping
C) Altered parenting
D) Sexual dysfunction
Question39
Which client is at highest risk for developing a
Review Information: The correct answer is C:
pressure ulcer?
Altered parenting
A) 23 year-old in traction for fractured femur
The cocaine abusing mother puts her newborn
B) 72 year-old with peripheral vascular disease,
and other children at risk for neglect and abuse.
who is unable to walk without assistance
Continuing to use drugs has the potential to im-
C) 75 year-old with left sided paresthesia who is
pact parenting behaviors. Social service referrals
incontinent of urine and stool
are indicated.
D) 30 year-old who is comatose following a rup-
tured aneurysm
Question2
Review Information: The correct answer is C: The nurse is teaching about nonsteroidal anti-
75 year-old with left sided paresthesia who is in- inflammatory drugs (NSAIDs) to a group of ar-
continent of urine and stool thritic clients. To minimize the side effects, the
Risk factors for pressure ulcers include: immo- nurse should emphasize which of the following
bility, absence of sensation, decreased LOC, actions?
poor nutrition and hydration, skin moisture, in- A) Reporting joint stiffness in the morning
continence, increased age, decreased immune B) Taking the medication 1 hour before or 2 hours
response. This client has the greatest number of after meals
risk factors. C) Using alcohol in moderation unless driving
D) Continuing to take aspirin for short term relief
Question40
The nurse is teaching the mother of a 5 month-
Review Information: The correct answer is B:
old about nutrition for her baby. Which state-
Taking the medication 1 hour before or 2 hours
ment by the mother indicates the need for further
after meals
teaching?
Taking the medication 1 hour before or 2 hours
A) «I>m going to try feeding my baby some rice
after meals will result in a more rapid effect.
cereal.»
B) «When he wakes at night for a bottle, I feed Question3
him.» The nurse is preparing to administer a tube feed-
C) «I dip his pacifier in honey so he>ll take it.» ing to a postoperative client. To accurately as-
D) «I keep formula in the refrigerator for 24 sess for a gastrostomy tube placement, the pri-
hours.» ority is to
A) auscultate the abdomen while instilling 10 cc
Review Information: The correct answer is C: of air into the tube
«I dip his pacifier in honey so he>>ll take it.» B) place the end of the tube in water to check for
Honey has been associated with infant botu- air bubbles
lism and should be avoided. Older children and C) retract the tube several inches to check for
adults have digestive enzymes that kill the botu- resistance
lism spores. D) measure the length of tubing from nose to epi-
Collected by :DeepaRajesh [ 61 ]
rajesh.ks21@gmail.com
Kuwait
gastrium known to cross the placenta and is therefore re-
ported to be teratogenic.

Review Information: The correct answer is A: Question7


auscultate the abdomen while instilling 10 cc of The nurse is caring for a client with Hodgkin>s
air into the tube disease who will be receiving radiation therapy.
If a swoosh of air is heard over the abdominal The nurse recognizes that, as a result of the ra-
cavity while instilling air into the gastric tube, this diation therapy, the client is most likely to experi-
indicates that it is accurately placed in the stom- ence
ach. The feeding can begin after further assess- A) high fever
ing the client for bowel sounds. B) nausea
Question4 C) face and neck edema
While assessing the vital signs in children, the D) night sweats
nurse should know that the apical heart rate is
preferred until the radial pulse can be accurately Review Information: The correct answer is B:
assessed at about what age? nausea
A) 1 year of age Because the client with Hodgkin>>s disease is
B) 2 years of age usually healthy when therapy begins, the nausea
C) 3 years of age is especially troubling.
D) 4 years of age Question8
A client is brought to the emergency room fol-
Review Information: The correct answer is B: lowing a motor vehicle accident. When assess-
2 years of age ing the client one-half hour after admission, the
A child should be at least 2 years of age to use nurse notes several physical changes. Which
the radial pulse to assess heart rate. finding would require the nurse>s immediate at-
Question5 tention?
A client is receiving Total Parenteral Nutrition A) increased restlessness
(TPN) via a Hickman catheter. The catheter acci- B) tachycardia
dentally becomes dislodged from the site. Which C) tracheal deviation
action by the nurse should take priority? D) tachypnea
A) Check that the catheter tip is intact
B) Apply a pressure dressing to the site Review Information: The correct answer is C:
C) Monitor respiratory status tracheal deviation
D) Assess for mental status changes The deviated trachea is a sign that a mediastinal
shift has occurred. This is a medical emergency.
Question6
A pregnant client who is at 34 weeks gestation Question9
is diagnosed with a pulmonary embolism (PE). An 18 month-old child is on peritoneal dialysis in
Which of these medications would the nurse an-preparation for a renal transplant in the near fu-
ticipate the provider ordering? ture. When the nurse obtains the child>s health
A) Oral Coumadin therapy history, the mother indicates that the child has
B) Heparin 5000 units subcutaneously B.I.D. not had the first measles, mumps, rubella (MMR)
immunization. The nurse understands that which
C) Heparin infusion to maintain the PTT at 1.5-
2.5 times the control value of the following is true in regards to giving immu-
D) Heparin by subcutaneous injection to main- nizations to this child?
tain the PTT at 1.5 times the control value A) Live vaccines are withheld in children with re-
nal chronic illness
Review Information: The correct answer is D: B) The MMR vaccine should be given now, prior
Heparin by subcutaneous injection to maintain to the transplant
the PTT at 1.5 times the control value C) An inactivated form of the vaccine can be giv-
Several studies have been conducted in preg- en at any time
nant women where oral anticoagulation agents D) The risk of vaccine side effects precludes giv-
are contraindicated. Warfarin (Coumadin) is ing the vaccine
Collected by :DeepaRajesh [ 62 ]
rajesh.ks21@gmail.com
Kuwait
are to receive electroconvulsive therapy?
Review Information: The correct answer is B: A) Benzodiazepines
The MMR vaccine should be given now, prior to B) Chlorpromazine (Thorazine)
the transplant C) Succinylcholine (Anectine)
MMR is a live virus vaccine, and should be given D) Thiopental sodium (Pentothal Sodium)
at this time. Post-transplant, immunosuppressive
drugs will be given and the administration of the Review Information: The correct answer is C:
live vaccine at that time would be contraindicated Succinylcholine (Anectine)
because of the compromised immune system. Succinylcholine is given intravenously to pro-
Question10 mote skeletal muscle relaxation.
The nurse is preparing to take a toddler>s blood Question13
pressure for the first time. Which of the following Which approach is a priority for the nurse who
actions should the nurse perform first? works with clients from many different cultures?
A) Explain that the procedure will help him to get A) Speak at least 2 other languages of clients in
well the neighborhood
B) Show a cartoon character with a blood pres- B) Learn about the cultures of clients who are
sure cuff most often encountered
C) Explain that the blood pressure checks the C) Have a list of persons for referral when inter-
heart pump action with these clients occur
D) Permit handling the equipment before putting D) Recognize personal attitudes about cultural
the cuff in place differences and real or expected biases

Review Information: The correct answer is D: Review Information: The correct answer is D:
Permit handling the equipment before putting the Recognize personal attitudes about cultural dif-
cuff in place ferences and real or expected biases
The best way to gain the toddler>>s cooperation The nurse must discover personal attitudes, prej-
is to encourage handling the equipment. Detailed udices and biases specific to different cultures.
explanations are not helpful. Awareness of these will prevent negative conse-
Question11 quences for interactions with clients and families
Which statement made by a nurse about the across cultures.
goal of total quality management or continuous Question14
quality improvement in a health care setting is A client with chronic obstructive pulmonary dis-
correct? ease (COPD) and a history of coronary artery
A) It is to observe reactive service and product disease is receiving aminophylline, 25mg/hour.
problem solving Which one of the following findings by the nurse
B) Improvement of the processes in a proactive, would require immediate intervention?
preventive mode is paramount A) Decreased blood pressure and respirations
C) A chart audits to finds common errors in prac- B) Flushing and headache
tice and outcomes associated with goals C) Restlessness and palpitations
D) A flow chart to organize daily tasks is critical D) Increased heart rate and blood pressure
to the initial stages
Review Information: The correct answer is C:
Restlessness and palpitations
Review Information: The correct answer is Side effects of Aminophylline include restless-
B: Improvement of the processes in a proactive, ness and palpitations.
preventive mode is paramount Question15
Total quality management and continuous qual- A client has gastroesophageal reflux. Which rec-
ity improvement have a major goal of identifying ommendation made by the nurse would be most
ways to do the right thing at the right time in the helpful to the client?
right way by proactive problem-solving. A) Avoid liquids unless a thickening agent is
Question12 used
Which of the following drugs should the nurse B) Sit upright for at least 1 hour after eating
anticipate administering to a client before they C) Maintain a diet of soft foods and cooked veg-
Collected by :DeepaRajesh [ 63 ]
rajesh.ks21@gmail.com
Kuwait
etables descent is diagnosed with ovarian cancer. The
D) Avoid eating 2 hours before going to sleep client states, “I refuse both radiation and chemo-
therapy because they are <hot.>” The next ac-
Review Information: The correct answer is D: tion for the nurse to take is to
Avoid eating 2 hours before going to sleep A) document the situation in the notes
Eating before sleeping enhances the regurgita- B) report the situation to the health care provid-
tion of stomach contents, which have increased er
acidity, into the esophagus. An upright posture C) talk with the client>s family about the situa-
should be maintained for about 2 hours after eat- tion
ing to allow for the stomach emptying. Options A D) ask the client to talk about concerns regarding
and C are interventions for clients with swallow- «hot» treatments
ing difficulties.
Question16 Review Information: The correct answer is D:
A client with a panic disorder has a new prescrip- ask the client to talk about concerns regarding
tion for Xanax (alprazolam). In teaching the client «hot» treatments
about the drug>s actions and side effects, which The «hot-cold» system is found among Mexican-
of the following should the nurse emphasize? Americans, Puerto Ricans, and other Hispanic-
A) Short-term relief can be expected Latinos. Most foods, beverages, herbs, and
B) The medication acts as a stimulant medicines are categorized as hot or cold, which
C) Dosage will be increased as tolerated are symbolic designations and do not necessar-
D) Initial side effects often continue ily indicate temperature or spiciness. Care and
treatment regimens can be negotiated with cli-
Review Information: The correct answer is A: ents within this framework.
Short-term relief can be expected Question19
Xanax is a short-acting benzodiazepine useful in A 72 year-old client is scheduled to have a cardi-
controlling panic symptoms quickly. oversion. A nurse reviews the client’s medication
Question17 administration record. The nurse should notify the
A client being discharged from the cardiac step- health care provider if the client received which
down unit following a myocardial infarction (MI), medication during the preceding 24 hours?
is given a prescription for a beta-blocking drug. A A) Digoxin (Lanoxin)
nursing student asks the charge nurse why this B) Diltiazem (Cardizem)
drug would be used by a client who is not hyper- C) Nitroglycerine ointment
tensive. What is an appropriate response by the D) Metoprolol (Toprol XL)
charge nurse?
A) «Most people develop hypertension following Review Information: The correct answer is A:
an MI.» Digoxin (Lanoxin)
B) «A beta-Blocker will prevent orthostatic hypo- Digoxin increases ventricular irritability and in-
tension.» creases the risk of ventricular fibrillation follow-
C) «This drug will decrease the workload on his ing cardioversion. The other medications do not
heart.» increase ventricular irritability.
D) «Beta-blockers increase the strength of heart Question20
contractions.» Which of these clients, all of whom have the find-
ings of a board-like abdomen, would the nurse
Review Information: The correct answer is suggest that the provider examine first?
C: «This drug will decrease the workload on his A) An elderly client who stated, «My awful pain
heart.» in my right side suddenly stopped about 3 hours
One action of beta-blockers is to decrease sys- ago.»
temic vascular resistance by dilating arterioles. B) A pregnant woman of 8 weeks newly diag-
This is useful for the client with coronary artery nosed with an ectopic pregnancy
disease, and will reduce the risk of another MI or C) A middle-aged client admitted with diverticu-
sudden death. litis who has taken only clear liquids for the past
Question18 week
A 35-year-old client of Puerto Rican-American D) A teenager with a history of falling off a bicycle
Collected by :DeepaRajesh [ 64 ]
rajesh.ks21@gmail.com
Kuwait
without hitting the handle bars The nurse is assessing a comatose client receiv-
ing gastric tube feedings. Which of the following
Review Information: The correct answer is A: assessments requires an immediate response
An elderly client who stated, «My awful pain in from the nurse?
my right side suddenly stopped about 3 hours A) Decreased breath sounds in right lower lobe
ago.» B) Aspiration of a residual of 100cc of formula
This client has the highest risk for hypovolemic C) Decrease in bowel sounds
and septic shock since the appendix has most D) Urine output of 250 cc in past 8 hours
likely ruptured, based on the history of the pain
suddenly stopping over three hours ago. Elderly Review Information: The correct answer is A:
clients have less functional reserve for the body Decreased breath sounds in right lower lobe
to cope with shock and infection over long peri- The most common problem associated with en-
ods. The others are at risk for shock also, how- teral feedings is atelectasis. Maintain client at 30
ever given that they fall in younger age groups, degrees of head elevation during feedings and
they would more likely be able to tolerate an im- monitor for signs of aspiration. Check for tube
balance in circulation. A common complication of placement prior to each feeding or every 4 to 8
falling off a bicycle is hitting the handle bars in hours if the client is receiving continuous feed-
the upper abdomen often on the left, resulting in ing.
a ruptured spleen. Question24
Question21 A client is prescribed warfarin sodium (Couma-
The nurse is teaching parents of a 7 month-old din) to be continued at home. Which focus is
about adding table foods. Which of the following critical to be included in the nurse’s discharge
is an appropriate finger food? instruction?
A) Hot dog pieces A) Maintain a consistent intake of green leafy
B) Sliced bananas foods
C) Whole grapes B) Report any nose or gum bleeds
D) Popcorn C) Take Tylenol for minor pains
D) Use a soft toothbrush
Review Information: The correct answer is B:
Sliced bananas Review Information: The correct answer is B:
Finger foods should be bite-size pieces of soft Report any nose or gum bleeds
food such as bananas. Hot dogs and grapes can The client should notify the health care provider
accidentally be swallowed whole and can occlude if blood is noted in stools or urine, or any other
the airway. Popcorn is too difficult to chew at this signs of bleeding occur.
age and can irritate the airway if swallowed.
Question25
Question22 When teaching a client about the side effects of
To prevent drug resistance from developing, the fluoxetine (Prozac), which of the following will
nurse is aware that which of the following is a the nurse include?
characteristic of the typical treatment plan to A) Tachycardia blurred vision, hypotension, ano-
eliminate the tuberculosis bacilli? rexia
A) An anti-inflammatory agent B) Orthostatic hypotension, vertigo, reactions to
B) High doses of B complex vitamins tyramine-rich foods
C) Aminoglycoside antibiotics C) Diarrhea, dry mouth, weight loss, reduced li-
D) Administering two anti-tuberculosis drugs bido
D) Photosensitivity, seizures, edema, hypergly-
Review Information: The correct answer is D: cemia
Administering two anti-tuberculosis drugs
Resistance of the tubercle bacilli often occurs to Review Information: The correct answer is C:
a single antimicrobial agent. Therefore, therapy Diarrhea, dry mouth, weight loss, reduced libido
with multiple drugs over a long period of time Commonly reported side effects for fluoxetine
helps to ensure eradication of the organism. (Prozac) are diarrhea, dry mouth, weight loss
Question23 and reduced libido.
Collected by :DeepaRajesh [ 65 ]
rajesh.ks21@gmail.com
Kuwait
Question29
Question26 The nurse is planning care for an 8 year-old child.
A newborn weighed 7 pounds 2 ounces at birth. Which of the following should be included in the
The nurse assesses the newborn at home 2 plan of care?
days later and finds the weight to be 6 pounds 7 A) Encourage child to engage in activities in the
ounces. What should the nurse tell the parents playroom
about this weight loss? B) Promote independence in activities of daily
A) The newborn needs additional assessments living
B) The mother should breast feed more often C) Talk with the child and allow him to express
C) A change to formula is indicated his opinions
D) The loss is within normal limits D) Provide frequent reassurance and cuddling

Review Information: The correct answer is D: Review Information: The correct answer is
The loss is within normal limits A: Encourage child to engage in activities in the
A newborn is expected to lose 5-10% of the birth playroom
weight in the first few days post-partum because According to Erikson, the school age child is in
of changes in elimination and feeding. the stage of industry versus inferiority. To help
Question27 them achieve industry, the nurse should encour-
The nurse manager informs the nursing staff at age them to carry out tasks and activities in their
morning report that the clinical nurse specialist room or in the playroom.
will be conducting a research study on staff at- Question30
titudes toward client care. All staff are invited to The nurse is assigned to care for 4 clients. Which
participate in the study if they wish. This affirms of the following should be assessed immediately
the ethical principle of after hearing the report?
A) Anonymity A) The client with asthma who is now ready for
B) Beneficence discharge
C) Justice B) The client with a peptic ulcer who has been
D) Autonomy vomiting all night
C) The client with chronic renal failure returning
Review Information: The correct answer is D: from dialysis
Autonomy D) The client with pancreatitis who was admitted
Individuals must be free to make independent yesterday
decisions about participation in research without
coercion from others. Review Information: The correct answer is B:
Question28 The client with a peptic ulcer who has been vom-
The nurse is talking with the family of an 18 iting all night
months-old newly diagnosed with retinoblasto- A perforated peptic ulcer could cause nausea,
ma. A priority in communicating with the parents vomiting and abdominal distention, and may be
is a life threatening situation. The client should be
A) Discuss the need for genetic counseling assessed immediately and findings reported to
B) Inform them that combined therapy is seldom the provider.
effective Question31
C) Prepare for the child>s permanent disfigure- During a routine check-up, an insulin-depend-
ment ent diabetic has his glycosylated hemoglobin
D) Suggest that total blindness may follow sur- checked. The results indicate a level of 11%.
gery Based on this result, what teaching should the
nurse emphasize?
Review Information: The correct answer is A: A) Rotation of injection sties
Discuss the need for genetic counseling B) Insulin mixing and preparation
The hereditary aspects of this disease are well C) Daily blood sugar monitoring
documented. While the parents focus on the D) Regular high protein diet
needs of this child, they should be aware that
the risk is high for future offspring. Review Information: The correct answer is C:
Collected by :DeepaRajesh [ 66 ]
rajesh.ks21@gmail.com
Kuwait
Daily blood sugar monitoring gency room (ER). The client, diagnosed with a
Normal hemoglobin A1C (glycosylated hemo- myocardial infarction, is complaining of subster-
globin) level is 7 to 9%. Elevation indicates el- nal chest pain, diaphoresis and nausea. The first
evated glucose levels over time. action by the nurse should be to
Question32 A) order an EKG
A client taking isoniazid (INH) for tuberculosis B) administer morphine sulfate
asks the nurse about side effects of the medica- C) start an IV
tion. The client should be instructed to immedi- D) measure vital signs
ately report which of these?
A) Double vision and visual halos Review Information: The correct answer is B:
B) Extremity tingling and numbness administer morphine sulfate
C) Confusion and lightheadedness Decreasing the clients pain is the most important
D) Sensitivity of sunlight priority at this time. As long as pain is present
there is danger in extending the infarcted area.
Review Information: The correct answer is B: Morphine will decrease the oxygen demands of
Extremity tingling and numbness the heart and act as a mild diuretic as well. It is
Peripheral neuropathy is the most common side probable that an EKG and IV insertion were per-
effect of INH and should be reported to the pro- formed in the ER.
vider. It can be reversed. Question36
Question33 The nurse admits a 2 year-old child who has had
Which of these questions is priority when as- a seizure. Which of the following statement by
sessing a client with hypertension? the child>s parent would be important in deter-
A) «What over-the-counter medications do you mining the etiology of the seizure?
take?» A) «He has been taking long naps for a week.»
B) «Describe your usual exercise and activity B) «He has had an ear infection for the past 2
patterns.» days.»
C) «Tell me about your usual diet.» C) «He has been eating more red meat lately.»
D) «Describe your family>s cardiovascular his- D) «He seems to be going to the bathroom more
tory.» frequently.»
Review Information: The correct answer is
B: «He has had an ear infection for the past 2
Review Information: The correct answer is days.»
A: «What over-the-counter medications do you Contributing factors to seizures in children in-
take?» clude those such as age (more common in first
Over-the-counter medications, especially those 2 years), infections (late infancy and early child-
that contain cold preparations can increase the hood), fatigue, not eating properly and excessive
blood pressure to the point of hypertension. fluid intake or fluid retention.
Question34 Question37
The nurse is performing an assessment of the Which of these clients would the nurse monitor
motor function in a client with a head injury. The for the complication of C. difficile diarrhea?
best technique is A) An adolescent taking medications for acne
A) touching the trapezius muscle or arm firmly B) An elderly client living in a retirement center
B) pinching any body part taking prednisone
C) shaking a limb vigorously C) A young adult at home taking a prescribed
D) rubbing the sternum aminoglycoside
D) A hospitalized middle aged client receiving
Review Information: The correct answer is D: clindamycin
rubbing the sternum
The purpose is to assess the non-responsive cli- Review Information: The correct answer is D:
A hospitalized middle aged client receiving clin-
ent’s reaction to a painful stimulus after less nox-
ious methods have been tried. damycin
Question35 Hospitalized patients, especially those receiving
A nurse admits a client transferred from the emer- antibiotic therapy, are primary targets for C. diffi-
Collected by :DeepaRajesh [ 67 ]
rajesh.ks21@gmail.com
Kuwait
cile. Of clients receiving antibiotics, 5-38% expe- Pain is a complex phenomenon that is perceived
rience antibiotic-associated diarrhea; C. difficile differently by each individual. Pain is whatever
causes 15 to 20% of the cases. Several antibiot- the client says it is. The other statements are cor-
ic agents have been associated with C. difficile. rect but not the most important considerations.
Broad-spectrum agents, such as clindamycin, Question40
ampicillin, amoxicillin, and cephalosporins, are As a part of a 9 pound full-term newborn>s as-
the most frequent sources of C. difficile. Also, sessment, the nurse performs a dextro-stick at 1
C. difficile infection has been caused by the ad- hour post birth. The serum glucose reading is 45
ministration of agents containing beta-lactamase mg/dl. What action by the nurse is appropriate at
inhibitors (i.e., clavulanic acid, sulbactam, tazo- this time?
bactam) and intravenous agents that achieve A) Give oral glucose water
substantial colonic intraluminal concentrations B) Notify the pediatrician
(i.e., ceftriaxone, nafcillin, oxacillin). Fluoroqui- C) Repeat the test in 2 hours
nolones, aminoglycosides, vancomycin, and tri- D) Check the pulse oximetry reading
methoprim are seldom associated with C. difficile
infection or pseudomembranous colitis. Review Information: The correct answer is C:
Question38 Repeat the test in 2 hours
The nurse is performing an assessment on a cli- This blood sugar is within the normal range for a
ent who is cachectic and has developed an en- full-term newborn. Normal values are: Premature
terocutaneous fistula following surgery to relieve infant: 20-60 mg/dl or 1.1-3.3 mmol/L, Neonate:
a small bowel obstruction. The client>s total pro- 30-60 mg/dl or 1.7-3.3 mmol/L, Infant: 40-90 mg/
tein level is reported as 4.5 g/dl. Which of the dl or 2.2-5.0 mmol/L. Critical values are: Infant:
following would the nurse anticipate? <40 mg/dl and in a Newborn: <30 and >300 mg/
A) Additional potassium will be given IV dl. Because of the increased birth weight which
B) Blood for coagulation studies will be drawn can be associated with diabetes mellitus, repeat-
C) Total parenteral nutrition (TPN) will be start- ed blood sugars will be drawn
ed
D) Serum lipase levels will be evaluated
Free NCLEX-RN Sample Test Ques-
Review Information: The correct answer is C: tions For Nursing Review (Part 3)
Total parenteral nutrition (TPN) will be started Jul31,
The client is not absorbing nutrients adequately
Question1
as evidenced by the cachexia and low protein
A client diagnosed with chronic depression is
levels. (A normal total serum protein level is 6.0-
maintained on tranylcypromine (Parnate). An im-
8.0 g/dl.) TPN will promote a positive nitrogen
portant nursing intervention is to teach the client
balance in this client who is unable to digest and
to avoid which of the following foods?
absorb nutrients adequately.
A) Wine, beer, cheese, liver and chocolate
Question39 B) Wine, citrus fruits, yogurt and broccoli
During a situation of pain management, which C) Beer, cheese, beef and carrots
statement is a priority to consider for the ethical D) Wine, apples, sour cream and beef steak
guidelines of the nurse?
A) The client>s self-report is the most important Review Information: The correct answer is A:
consideration Wine, beer, cheese, liver and chocolate
B) Cultural sensitivity is fundamental to pain These foods are tyramine rich and ingestion of
management these foods while taking monoamine oxidase in-
C) Clients have the right to have their pain re- hibitors (MAOIs) can precipitate a life-threaten-
lieved ing hypertensive crisis.
D) Nurses should not prejudge a client>s pain
Question2
using their own values
The nurse is working in a high risk antepartum
clinic. A 40 year-old woman in the first trimes-
Review Information: The correct answer is A:
ter gives a thorough health history. Which infor-
The client>>s self-report is the most important
mation should receive priority attention by the
consideration
nurse?
Collected by :DeepaRajesh [ 68 ]
rajesh.ks21@gmail.com
Kuwait
A) Her father and brother are insulin dependent B) The blood alcohol level of the client
diabetics C) The blood pressure level of the client
B) She has taken 800 mcg of folic acid daily for D) The blood glucose level of the client
the past year
C) Her husband was treated for tuberculosis as Review Information: The correct answer is B:
a child The blood alcohol level of the client
D) She reports recent use of over-the counter si- Blood alcohol levels are generally obtained to
nus remedies determine the level of intoxication. The amount
of alcohol consumed determines how much
Review Information: The correct answer is D: medication the client needs for detoxification and
She reports recent use of over-the counter sinus treatment. Reports of alcohol consumption are
remedies notoriously inaccurate.
Over-the-counter drugs are a possible danger in Question6
early pregnancy. A report by the client that she Which clinical finding would the nurse expect to
has taken medications should be followed up im- assess first in a newborn with spastic cerebral
mediately. palsy?
Question3 A) cognitive impairment
What must be the priority consideration for nurs- B) hypotonic muscular activity
es when communicating with children? C) seizures
A) Present environment D) criss-crossing leg movement
B) Physical condition
C) Nonverbal cues Review Information: The correct answer is D:
D) Developmental level criss-crossing leg movement
Cerebral palsy is a neuromuscular impairment
Review Information: The correct answer is D: resulting in muscular and reflexive hypertonicity
Developmental level and the criss-crossing, or scissoring leg move-
While each of the factors affect communication, ments.
the nurse recognizes that developmental differ- Question7
ences have implications for processing and un- Which medication is more helpful in treating
derstanding information. Consequently, a child’s bulimia than anorexia?
developmental level must be considered when A) Amphetamines
selecting communication approaches. B) Sedatives
Question4 C) Anticholinergics
The nurse is assessing a client>s home in prep- D) Narcotics
aration for discharge. Which of the following
should be given priority consideration? Review Information: The correct answer is C:
A) Family understanding of client needs Anticholinergics
B) Financial status In contrast to anorexics, individuals with bulimia
C) Location of bathrooms are troubled by their behavioral characteristics
D) Proximity to emergency services and become depressed. The person feels com-
pelled to binge, purge and fast. Feeling helpless
Review Information: The correct answer is A: to stop the behavior, feelings of self-disgust oc-
Family understanding of client needs cur.
Functional communication patterns between Question8
family members are fundamental to meeting the The nurse is assessing a woman in early labor.
needs of the client and family. While positioning for a vaginal exam, she com-
Question5 plains of dizziness and nausea and appears
As a general guide for emergency management pale. Her blood pressure has dropped slightly.
of acute alcohol intoxication, it is important for What should be the initial nursing action?
the nurse initially to obtain data regarding which A) Call the health care provider
of the following? B) Encourage deep breathing
A) What and how much the client drinks, accord- C) Elevate the foot of the bed
ing to family and friends D) Turn her to her left side
Collected by :DeepaRajesh [ 69 ]
rajesh.ks21@gmail.com
Kuwait
A) Monitor respiratory rate
Review Information: The correct answer is D: B) Monitor intake and output every hour
Turn her to her left side C) Assist the client to breathe into a paper bag
The weight of the uterus can put pressure on the D) Prepare to administer oxygen by mask
vena cava and aorta when a pregnant woman
is flat on her back causing supine hypotension. Review Information: The correct answer is C:
Action is needed to relieve the pressure on the Assist the client to breathe into a paper bag
vena cava and aorta. Turning the woman to the Side effects of aspirin toxicity include hyperventi-
side reduces this pressure and relieves postural lation, which can result in respiratory alkalosis in
hypotension. the initial stages. Breathing into a paper bag will
Question9 prevent further reduction in PaCO2.
A client has been started on a long term corticos-
teroid therapy. Which of the following comments Question12
by the client indicate the need for further teach- After assessing a 70 year-old male client>s labo-
ing? ratory results during a routine clinic visit, which
A) «I will keep a weekly weight record.» one of the following findings would indicate an
B) «I will take medication with food.» area in which teaching is needed:
C) «I will stop taking the medication for 1 week A) Serum albumin 2.5 g/dl
every month.» B) LDL Cholesterol 140 mg/dl
D) «I will eat foods high in potassium.» C) Serum glucose 90 mg/dl
D) RBC 5.0 million/mm3
Review Information: The correct answer is C:
«I will stop taking the medication for 1week every
month.» Review Information: The correct answer is A:
Emphatically warn against discontinuing steroid Serum albumin 2.5 g/dl
dosage abruptly because that may produce a fa- Serum albumin level is low (normal 3.0 – 5.0 g/dl
tal adrenal crisis. in elders), indicating nutritional counseling to in-
Question10 crease dietary protein is needed. Socioeconomic
A male client calls for a nurse because of chest factors may need to be addressed to help the
pain. Which statement by the client would require client comply with the recommendation.
the most immediate action by the nurse?
A) «When I take in a deep breath, it stabs like a Question13
knife.» When teaching a client with a new prescription
B) «The pain came on after dinner. That soup for lithium (Lithane) for treatment of a bi-polar
seemed very spicy.» disorder which of these should the nurse empha-
C) «When I turn in bed to reach the remote for size?
the TV, my chest hurts.» A) Maintaining a salt restricted diet
D) «I feel pressure in the middle of my chest, like B) Reporting vomiting or diarrhea
an elephant is sitting on my chest.» C) Taking other medication as usual
D) Substituting generic form if desired
Review Information: The correct answer is D:
«I feel pressure in the middle of my chest, like an Review Information: The correct answer is B:
elephant is sitting on my chest.» Reporting vomiting or diarrhea
This is a classic description of chest pain in men If dehydration results from vomiting, diarrhea
caused by myocardial ischemia. Women experi- or excessive perspiration, tolerance to the drug
ence vague feelings of fatigue and back and jaw may be altered and symptoms may return.
pain.
Question11 Question14
A nurse is caring for a client who has just been A client is discharged on warfarin sulfate (Cou-
admitted with an overdose of aspirin. The fol- madin). Which statement by the client indicated
lowing lab data is available: PaO2 95, PaCO2 a need for further teaching?
30, pH 7.5, K 3.2 mEq/l. Which should be the A) «I know I must avoid crowds.»
nurse>s first action? B) «I will keep all laboratory appointments.»
Collected by :DeepaRajesh [ 70 ]
rajesh.ks21@gmail.com
Kuwait
C) «I plan to use an electric razor for shaving.» B) heart rate
D) «I will report any bruises for bleeding.» C) peripheral pulses
D) lung sounds
Review Information: The correct answer is A:
«I know I must avoid crowds.»
There are no specific reasons for the client on Review Information: The correct answer is D:
Coumadin to avoid crowds. General instructions lung sounds
for any cardiac surgical client include limiting ex- Lung sounds are critical assessments at this
posure to infection. point. The nurse should be alert to crackles or a
pleural friction rub, highly suggestive of a pulmo-
Question15 nary embolism.
A client is taking tranylcypromine (Parnate) and
has received dietary instruction. Which of the fol- Question18
lowing food selections would be contraindicated The nurse is administering lidocaine (Xylocaine)
for this client? to a client with a myocardial infarction. Which of
A) Fresh juice, carrots, vanilla pudding the following assessment findings requires the
B) Apple juice, ham salad, fresh pineapple nurse>s immediate action?
C) Hamburger, fries, strawberry shake A) Central venous pressure reading of 11
D) Red wine, fava beans, aged cheese B) Respiratory rate of 22
C) Pulse rate of 48 BPM
D) Blood pressure of 144/92
Review Information: The correct answer is D:
Red wine, fava beans, aged cheese
Red wine and cheese contain tyramine (as do Review Information: The correct answer is C:
chicken liver and ripe bananas) and so are con- Pulse rate of 48 BPM
traindicated when taking MAOIs. Fava beans One of the side effects of lidocaine is bradycardia,
contain other vasopressors that can interact with heart block, cardiovascular collapse and cardiac
MAOIs also causing malignant hypertension. arrest (this drug should never be administered
without continuous EKG monitoring).
Question16
A client is admitted with severe injuries from an Question19
auto accident. The client>s vital signs are BP The nurse is teaching a group of college students
120/50, pulse rate 110, and respiratory rate of about breast self-examination. A woman asks for
28. The initial nursing intervention would be to the best time to perform the monthly exam. What
A) begin intravenous therapy is the best reply by the nurse?
B) initiate continuous blood pressure monitoring A) «The first of every month, because it is easi-
C) administer oxygen therapy est to remember»
D) institute cardiac monitoring B) «Right after the period, when your breasts are
less tender»
Review Information: The correct answer is C: C) «Do the exam at the same time every
administer oxygen therapy month»
Early findings of shock reveal hypoxia with rapid D) «Ovulation, or mid-cycle is the best time to
heart rate and rapid respirations, and oxygen is detect changes»
the most critical initial intervention. The other in-
terventions are secondary to oxygen therapy.
Review Information: The correct answer is B:
Question17 «Right after the period, when your breasts are
A client is admitted to the hospital with a diag- less tender»
nosis of deep vein thrombosis. During the ini- The best time for a breast self exam (BSE) is a
tial assessment, the client complains of sudden week after a menstrual cycle, when the breasts
shortness of breath. The SaO2 is 87. The priority are no longer swollen and tender due to hormone
nursing assessment at this time is elevation.
A) bowel sounds
Collected by :DeepaRajesh [ 71 ]
rajesh.ks21@gmail.com
Kuwait
Question20 T.I.D. to treat bipolar disorder. Which of these in-
The nurse is caring for a post-operative client dicate early signs of toxicity?
who develops a wound evisceration. The first A) Ataxia and course hand tremors
nursing intervention should be to B) Vomiting, diarrhea and lethargy
A) medicate the client for pain C) Pruritus, rash and photosensitivity
B) call the provider D) Electrolyte imbalance and cardiac arrhyth-
C) cover the wound with sterile saline dressing mias
D) place the bed in a flat position
Review Information: The correct answer is B:
Vomiting, diarrhea and lethargy
Review Information: The correct answer is C: These are early signs of lithium toxicity.
cover the wound with sterile saline dressing
When evisceration occurs, the wound should first Question24
be quickly covered by sterile dressings soaked in The nurse can best ensure the safety of a client
sterile saline. This prevents tissue damage until suffering from dementia who wanders from the
a repair can be effected. room by which action?
A) Repeatedly remind the client of the time and
Question21 location
The spouse of a client with Alzheimer>s disease B) Explain the risks of walking with no purpose
expresses concern about the burden of caregiv- C) Use protective devices to keep the client in
ing. Which of the following actions by the nurse the bed or chair in the room
should be a priority? D) Attach a wander-guard sensor band to the
A) Link the caregiver with a support group client>s wrist
B) Ask friends to visit regularly
C) Schedule a home visit each week
D) Request anti-anxiety prescriptions Review Information: The correct answer is
D: Attach a wander-guard sensor band to the
Review Information: The correct answer is A: client>>s wrist
Link the caregiver with a support group This type of identification band easily tracks the
Assisting caregivers to locate and join support client>>s movements and ensures safety while
groups is most helpful. Families share feelings the client wanders on the unit. Restriction of ac-
and learn about services such as respite care. tivity is inappropriate for any client unless they
Health education is also available through local are potentially harmful to themselves or others.
and national Alzheimer>>s chapters.
Question25
Question22 The nurse is teaching a client about the difference
Clients taking lithium must be particularly sure to between tardive dyskinesia (TD) and neuroleptic
maintain adequate intake of which of these ele- malignant syndrome (NMS). Which statement is
ments? true with regards to tardive dyskinesia?
A) Potassium A) TD develops within hours or years of contin-
B) Sodium ued antipsychotic drug use in people under 20
C) Chloride and over 30
D) Calcium B) It can occur in clients taking antipsychotic
drugs longer than 2 years
Review Information: The correct answer is B: C) Tardive dyskinesia occurs within minutes of
Sodium the first dose of antipsychotic drugs and is re-
Clients taking lithium need to maintain an ade- versible
quate intake of sodium. Serum lithium concen- D) TD can easily be treated with anticholinergic
trations may increase in the presence of condi- drugs
tions that cause sodium loss.

Question23 Review Information: The correct answer is B:


A client is receiving lithium carbonate 600 mg It can occur in clients taking antipsychotic drugs
Collected by :DeepaRajesh [ 72 ]
rajesh.ks21@gmail.com
Kuwait
longer than 2 years Review Information: The correct answer is A:
Tardive dyskinesia is a extrapyramidal side ef- Provide small feedings every 3 hours
fect that appears after prolonged treatment with Infants with congenital heart defects are at in-
antipsychotic medication. Early symptoms of tar- creased risk for developing congestive heart fail-
dive dyskinesia are fasciculations of the tongue ure. Infants with congestive heart failure have an
or constant smacking of the lips. increased metabolic rate and require additional
calories to grow. At the same time, however, rest
Question26 and conservation of energy for eating is impor-
The nurse is aware that the effect of antihyper- tant. Feedings should be smaller and every 3
tensive drug therapy may be affected by a 75 hours rather than the usual 4 hour schedule.
year-old client>s
A) poor nutritional status Question29
B) decreased gastrointestinal motility The nurse is caring for a client receiving intrave-
C) increased splanchnic blood flow nous nitroglycerin for acute angina. What is the
D) altered peripheral resistance most important assessment during treatment?
A) Heart rate
Review Information: The correct answer is B: B) Neurologic status
decreased gastrointestinal motility C) Urine output
Together with shrinkage of the gastric mucosa, D) Blood pressure
and changes in the levels of hydrochloric acid,
this will decrease absorption of medications and Review Information: The correct answer is D:
interfere with their actions. Blood pressure
The vasodilatation that occurs as a result of this
Question27 medication can cause profound hypotension.
In response to a call for assistance by a client in The client>>s blood pressure must be evaluated
labor, the nurse notes that a loop on the umbili- every 15 minutes until stable and then every 30
cal cord protrudes from the vagina. What is the minutes to every hour.
priority nursing action?
A) call the health care provider Question30
B) check fetal heart beat A client telephones the clinic to ask about a home
C) put the client in knee-chest position pregnancy test she used this morning. The nurse
D) turn the client to the side understands that the presence of which hormone
strongly suggests a woman is pregnant?
Review Information: The correct answer is C: A) Estrogen
put the client in knee-chest position B) HCG
Immediate action is needed to relieve pressure C) Alpha-fetoprotein
on the cord, which puts the fetus at risk due to D) Progesterone
hypoxia. The Trendelenburg position accom-
plishes this. The exposed cord is covered with Review Information: The correct answer is B:
saline soaked gauze, not reinserted. The fe- HCG
tal heart rate also should be checked, and the Human chorionic gonadotropin (HCG) is the
provider called. A prolapsed umbilical cord is a biologic marker on which pregnancy tests are
medical emergency. based. Reliability is about 98%, but the test does
not conclusively confirm pregnancy.
Question28
The nurse is caring for a 2 month-old infant with Question31
a congenital heart defect. Which of the following A client, admitted to the unit because of severe
is a priority nursing action? depression and suicidal threats, is placed on sui-
A) Provide small feedings every 3 hours cidal precautions. The nurse should be aware
B) Maintain intravenous fluids that the danger of the client committing suicide
C) Add strained cereal to the diet is greatest
D) Change to reduced calorie formula A) during the night shift when staffing is limited
B) when the client’s mood improves with an in-
Collected by :DeepaRajesh [ 73 ]
rajesh.ks21@gmail.com
Kuwait
crease in energy level exchange.
C) at the time of the client>s greatest despair
D) after a visit from the client>s estranged part- Question34
ner The nurse is assessing a client with chronic ob-
structive pulmonary disease receiving oxygen for
Review Information: The correct answer is low PaO2 levels. Which assessment is a nursing
B: when the client’s mood improves with an in- priority?
crease in energy level A) Evaluating SaO2 levels frequently
Suicide potential is often increased when there B) Observing skin color changes
is an improvement in mood and energy level. At C) Assessing for clubbing fingers
this time ambivalence is often decreased and a D) Identifying tactile fremitus
decision is made to commit suicide.
Review Information: The correct answer is A:
Question32 Evaluating SaO2 levels frequently
After 4 electroconvulsive treatments over 2 The best method to evaluate a client>>s oxygen-
weeks, a client is very upset and states “I am so ation is to evaluate the SaO2. This is just as ef-
confused. I lose my money. I just can’t remem- fective as an arterial blood gas reading to evalu-
ber telephone numbers.” The most therapeutic ate oxygenation status, and is less traumatic and
response for the nurse to make is expensive.
A) «You were seriously ill and needed the treat-
ments.» Question35
B) «Don>t get upset. The confusion will clear up The visiting nurse makes a postpartum visit to
in a day or two.» a married female client. Upon arrival, the nurse
C) «It is to be expected since most clients have observes that the client has a black eye and nu-
the same results.» merous bruises on her arms and legs. The initial
D) «I can hear your concern and that your confu- nursing intervention would be to
sion is upsetting to you.» A) call the police to report indications of domestic
violence
Review Information: The correct answer is D: B) confront the husband about abusing his wife
«I can hear your concern and that your confusion C) leave the home because of the unsafe envi-
is upsetting to you.» ronment
Communicating caring and empathy with the ac- D) interview the client alone to determine the ori-
knowledgement of feelings is the initial response. gin of the injuries
Afterwards, teaching about the expected short
term effects would be discussed.
Review Information: The correct answer is D:
Question33 interview the client alone to determine the origin
A woman in labor calls the nurse to assist her in of the injuries
the bathroom. The nurse notices a large amount It would be wrong to assume domestic violence
of clear fluid on the bed linens. The nurse knows without further assessment. Separate the sus-
that fetal monitoring must now assess for what pected victim from the partner until battering has
complication? been ruled out.
A) Early decelerations
B) Late accelerations
C) Variable decelerations Question36
D) Periodic accelerations When teaching a client about an oral hypoglyc-
emic medication, the nurse should place primary
Review Information: The correct answer is C: emphasis on
Variable decelerations A) recognizing findings of toxicity
When the membranes rupture, there is increased B) taking the medication at specified times
risk initially of cord prolapse. Fetal heart rate pat- C) increasing the dosage based on blood glu-
terns may show variable decelerations, which cose
require immediate nursing action to promote gas D) distinguishing hypoglycemia from hyperglyc-
Collected by :DeepaRajesh [ 74 ]
rajesh.ks21@gmail.com
Kuwait
emia D) Assess the client>s knowledge about his
health problems
Review Information: The correct answer is B:
taking the medication at specified times
A regular interval between doses should be Review Information: The correct answer is D:
maintained since oral hypoglycemics stimulate Assess the client>>s knowledge about his health
the islets of Langerhans to produce insulin. problems
The nursing process is continuous and cyclical in
Question37 nature. When a client expresses a specific con-
Initial postoperative nursing care for an infant cern, the nurse performs a focused assessment
who has had a pyloromyotomy would initially in- to gather additional data prior to planning and
clude implementing nursing interventions.
A) bland diet appropriate for age
B) intravenous fluids for 3-4 days Question40
C) NPO then glucose and electrolyte solutions The client asks the nurse how the health care
D) formula or breast milk as tolerated provider could tell she was pregnant “just by
looking inside.” What is the best explanation by
Review Information: The correct answer is C: the nurse?
NPO then glucose and electrolyte solutions A) Bluish coloration of the cervix and vaginal
Post-operatively, the initial feedings are clear liq- walls
uids in small quantities to provide calories and B) Pronounced softening of the cervix
electrolytes. C) Clot of very thick mucous that obstructs the
cervical canal
Question38 D) Slight rotation of the uterus to the right
A client is treated in the emergency room for
diabetic ketoacidosis and a glucose level of
650mg.D/L. In assessing the client, the nurse>s Review Information: The correct answer is A:
review of which of the following tests suggests Bluish coloration of the cervix and vaginal walls
an understanding of this health problem? Chadwick>>s sign is a bluish-purple coloration
A) Serum calcium of the cervix and vaginal walls, occurring at 4
B) Serum magnesium weeks of pregnancy, that is caused by vasocon-
C) Serum creatinine gestion.
D) Serum potassium
Free NCLEX-RN Sample Test Ques-
Review Information: The correct answer is D: tions For Nursing Review (Part 2)
Serum potassium Jul31,
Potassium is lost in diabetic ketoacidosis during
rehydration and insulin administration. Review of Question1
this lab finding suggests the nurse has knowl- The feeling of trust can best be established by
edge of this problem. the nurse during the process of the development
of a nurse-client relationship by which of these
Question39 characteristics?
A male client is preparing for discharge follow- A) Reliability and kindness
ing an acute myocardial infarction. He asks the B) Demeanor and sincerity
nurse about his sexual activity once he is home. C) Honesty and consistency
What would be the nurse>s initial response? D) Sympathy and appreciativeness
A) Give him written material from the American
Heart Association about sexual activity with heart Review Information: The correct answer is C:
disease Honesty and consistency
B) Answer his questions accurately in a private Characteristics of a trusting relationship include
environment respect, honesty, consistency, faith and caring.
C) Schedule a private, uninterrupted teaching
session with both the client and his wife Question2
Collected by :DeepaRajesh [ 75 ]
rajesh.ks21@gmail.com
Kuwait
A nurse has administered several blood trans- of clozapine (Clozaril) therapy?
fusions over 3 days to a 12 year-old client with A) Dry mouth
Thalassemia. What lab value should the nurse B) Rhinitis
monitor closely during this therapy? C) Dry skin
A) Hemoglobin D) Extreme salivation
B) Red Blood Cell Indices
C) Platelet count Review Information: The correct answer is D:
D) Neutrophil percent Extreme salivation
A significant number of clients receiving Clozap-
Review Information: The correct answer is A: ine (Clozaril) therapy experience extreme saliva-
Hemoglobin tion.
Hemoglobin should be in a therapeutic range of
approximately 10 g/dl (100gL). «This level is low Question6
enough to foster the patient>>s own erythropoi- A client has had a positive reaction to purified
esis without enlarging the spleen.» protein derivative (PPD). The client asks the
nurse what this means. The nurse should indi-
Question3 cate that the client has
The nurse is providing care to a newly a hospital- A) active tuberculosis
ized adolescent. What is the major threat experi- B) been exposed to mycobacterium tuberculo-
enced by the hospitalized adolescent? sis
A) Pain management C) never had tuberculosis
B) Restricted physical activity D) never been infected with mycobacterium tu-
C) Altered body image berculosis
D) Separation from family
Review Information: The correct answer is B:
Review Information: The correct answer is C: been exposed to mycobacterium tuberculosis
Altered body image The PPD skin test is used to determine the pres-
The hospitalized adolescent may see each of ence of tuberculosis antibodies and a positive re-
these as a threat, but the major threat that they sult indicates that the person has been exposed
feel when hospitalized is the fear of altered body to mycobacterium tuberculosis. Additional tests
image, because of the emphasis on physical ap- are needed to determine if active tuberculosis is
pearance during this developmental stage. present.

Question4
A 12 year-old child is admitted with a broken arm Question7
and is told surgery is required. The nurse finds A client is receiving and IV antibiotic infusion and
him crying and unwilling to talk. What is the most is scheduled to have blood drawn at 1:00 pm
appropriate response by the nurse? for a «peak» antibiotic level measurement. The
A) Give him privacy nurse notes that the IV infusion is running behind
B) Tell him he will get through the surgery with schedule and will not be competed by 1:00. The
no problem nurse should:
C) Try to distract him A) Notify the client>s health care provider
D) Make arrangements for his friends to visit B) Stop the infusion at 1:00 pm
C) Reschedule the laboratory test
D) Increase the infusion rate
Review Information: The correct answer is A:
Give him privacy
A 12 year-old child needs the opportunity to ex- Review Information: The correct answer is C:
press his emotions privately. Reschedule the laboratory test
If the antibiotic infusion will not be completed at
Question5 the time the peak blood level is due to be drawn,
In discharge teaching, the nurse should empha- the nurse should ask that the blood sampling
size that which of these is a common side effect time be adjusted
Collected by :DeepaRajesh [ 76 ]
rajesh.ks21@gmail.com
Kuwait
Question8 Review Information: The correct answer is C:
The nurse is caring for a client with a new order A genetic predisposition
for bupropion (Wellbutrin) for treatment of de- Malignant hyperthermia is a rare, potentially fatal
pression. The order reads “Wellbutrin 175 mg. adverse reaction to inhaled anesthetics. There is
BID x 4 days.” What is the appropriate action? a genetic predisposition to this disorder.
A) Give the medication as ordered
B) Questionthis medication dose Question11
C) Observe the client for mood swings A 9 year-old is taken to the emergency room with
D) Monitor neuro signs frequently right lower quadrant pain and vomiting. When
preparing the child for an emergency appen-
Review Information: The correct an- dectomy, what must the nurse expect to be the
child>s greatest fear?
swer is B: Questionthis medication
A) Change in body image
dose B) An unfamiliar environment
Bupropion (Wellbutrin) should be started at C) Perceived loss of control
100mg BID for three days then increased to D) Guilt over being hospitalized
150mg BID. When used for depression, it may
take up to four weeks for results. Common side Review Information: The correct answer is C:
effects are dry mouth, headache, and agitation. Perceived loss of control
Doses should be administered in equally spaced For school age children, major fears are loss of
time increments throughout the day to minimize control and separation from friends/peers.
the risk of seizures.
Question12
Question9 A client is to begin taking Fosamax. The nurse
The clinic nurse is discussing health promotion must emphasize which of these instructions to
with a group of parents. A mother is concerned the client when taking this medication? «Take
about Reye>s Syndrome, and asks about pre- Fosamax
vention. Which of these demonstrates appropri- A) on an empty stomach.»
ate teaching? B) after meals.»
A) «Immunize your child against this disease.» C) with calcium.»
B) «Seek medical attention for serious injuries.» D) with milk 2 hours after meals.»
C) «Report exposure to this illness.»
D) «Avoid use of aspirin for viral infections.»
Review Information: The correct answer is A:
on an empty stomach.»
Review Information: The correct answer is D: Fosamax should be taken first thing in the morn-
«Avoid use of aspirin for viral infections.» ing with 6-8 ounces of plain water at least 30
The link between aspirin use and Reye>>s Syn- minutes before other medication or food. Food
drome has not been confirmed, but evidence and fluids (other than water) greatly decrease
suggests that the risk is sufficiently grave to in- the absorption of Fosamax. The client must also
clude the warning on aspirin products. be instructed to remain in the upright position for
30 minutes following the dose to facilitate pas-
Question10 sage into the stomach and minimize irritation of
A post-operative client is admitted to the post-an- the esophagus.
esthesia recovery room (PACU). The anesthet-
ist reports that malignant hyperthermia occurred Question13
during surgery. The nurse recognizes that this An older adult client is to receive and antibiotic,
complication is related to what factor? gentamicin. What diagnostic finding indicates
A) Allergy to general anesthesia the client may have difficult excreting the medi-
B) Pre-existing bacterial infection cation?
C) A genetic predisposition A) High gastric pH
D) Selected surgical procedures B) High serum creatinine
Collected by :DeepaRajesh [ 77 ]
rajesh.ks21@gmail.com
Kuwait
C) Low serum albumin D) Review the specific procedures unique to the
D) Low serum blood urea nitrogen assignment

Review Information: The correct answer is C:


Review Information: The correct answer is B: Ask about prior experience with similar clients.
High serum creatinine The first step in delegation is to determine the
An elevated serum creatinine indicates reduced qualifications of the person to whom one is del-
renal function. Reduced renal function will delay egating. By asking about the PN>>s prior ex-
the excretion of many mediations. perience with similar clients/tasks, the RN can
determine whether the PN has the requisite ex-
Question14 perience to care for the assigned clients.
A nurse is assigned to care for a comatose dia-
betic on IV insulin therapy. Which task would be Question17
most appropriate to delegate to an unlicensed The mother of a 4 month-old infant asks the
assistive personnel (UAP)? nurse about the dangers of sunburn while they
A) Check the client>s level of consciousness are on vacation at the beach. Which of the fol-
B) Obtain the regular blood glucose readings lowing is the best advice about sun protection for
C) Determine if special skin care is needed this child?
D) Answer questions from the client>s spouse A) «Use a sunscreen with a minimum sun pro-
about the plan of care tective factor of 15.»
B) «Applications of sunscreen should be repeat-
Review Information: The correct answer is B: ed every few hours.»
Obtain the regular blood glucose readings C) «An infant should be protected by the maxi-
The UAP can safely obtain blood glucose read- mum strength sunscreen.»
ings, which are routine tasks. D) «Sunscreens are not recommended in chil-
dren younger than 6 months.»
Question15
Which of the following laboratory results would
suggest to the emergency room nurse that a cli- Review Information: The correct answer is D:
ent admitted after a severe motor vehicle crash «Sunscreens are not recommended in children
is in acidosis? younger than 6 months.»
A) Hemoglobin 15 gm/dl Infants under 6 months of age should be kept out
B) Chloride 100 mEq/L of the sun or shielded from it. Even on a cloudy
C) Sodium 130 mEq/L day, the infant can be sunburned while near wa-
D) Carbon dioxide 20 mEq/L ter. A hat and light protective clothing should be
worn.
Review Information: The correct answer is D:
Carbon dioxide 20 mEq/L Question18
Serum carbon dioxide is an indicator of acid-base The nurse administers cimetidine (Tagamet) to a
status. This finding would indicate acidosis. 79 year-old male with a gastric ulcer. Which pa-
rameter may be affected by this drug, and should
Question16 be closely monitored by the nurse?
The nurse has just received report on a group A) Blood pressure
of clients and plans to delegate care of several B) Liver function
of the clients to a practical nurse (PN). The first C) Mental status
thing the RN should do before the delegation of D) Hemoglobin
care is
A) Provide a time-frame for the completion of the Review Information: The correct answer is C:
client care Mental status
B) Assure the PN that the RN will be available for The elderly are at risk for developing confusion
assistance when taking cimetidine, a drug that interacts with
C) Ask about prior experience with similar cli- many other medications.
ents
Collected by :DeepaRajesh [ 78 ]
rajesh.ks21@gmail.com
Kuwait
Question19 Review Information: The correct answer is B:
The nurse assesses the use of coping mecha- High risk for infection
nisms by an adolescent 1 week after the client Cyclosporin (Neoral) inhibits normal immune re-
had a motor vehicle accident resulting in multiple sponses. Clients receiving cyclosporin are at risk
serious injuries. Which of these characteristics for infection.
are most likely to be displayed?
A) Ambivalence, dependence, demanding Question22
B) Denial, projection, regression A client on telemetry begins having premature
C) Intellectualization, rationalization, repression ventricular beats (PVBs) at 12 per minute. In re-
D) Identification, assimilation, withdrawal viewing the most recent laboratory results, which
would require immediate action by the nurse?
Review Information: The correct answer is B: A) Calcium 9 mg/dl
Denial, projection, regression B) Magnesium 2.5 mg/dl
Helplessness and hopelessness may contribute C) Potassium 2.5 mEq/L
to regressive, dependent behavior which often D) PTT 70 seconds
occurs at any age with hospitalization. Deny-
ing or minimizing the seriousness of the illness Review Information: The correct answer is C:
is used to avoid facing the worst situation. Re- Potassium 2.5 mEq/L
call that denial is the initial step in the process of The patient is at risk for ventricular dysrhythmias
working through any loss. when the potassium level is low.
Daniels, R. (2003).
Question20
A 52 year-old post menopausal woman asks the Question23
nurse how frequently she should have a mam- The nurse is caring for a client who is 4 days
mogram. What is the nurse>s best response? post-op for a transverse colostomy. The client is
A) «Your doctor will advise you about your ready for discharge and asks the nurse to empty
risks.» his colostomy pouch. What is the best response
B) «Unless you had previous problems, every 2 by the nurse?
years is best.» A) «You should be emptying the pouch your-
C) «Once a woman reaches 50, she should have self.»
a mammogram yearly.» B) «Let me demonstrate to you how to empty the
D) «Yearly mammograms are advised for all pouch.»
women over 35.» C) «What have you learned about emptying your
pouch?»
Review Information: The correct answer is C: D) «Show me what you have learned about emp-
«Once a woman reaches 50, she should have a tying your pouch.»
mammogram yearly.»
The American Cancer Society recommends a
screening mammogram by age 40, every 1 - 2 Review Information: The correct answer is D:
years for women 40-49, and every year from age «Show me what you have learned about empty-
50. If there are family or personal health risks, ing your pouch.»
other assessments may be recommended. Most adult learners obtain skills by participating
in the activities. Anxiety about discharge can be
Question21 causing the client to forget that they have mas-
The nurse is planning care for a client who is tak- tered the skill of emptying the pouch. The client
ing cyclosporin (Neoral). What would be an ap- should show the nurse how the pouch is emp-
propriate nursing diagnosis for this client? tied.
A) Alteration in body image
B) High risk for infection Question24
C) Altered growth and development A 3 year-old child has tympanostomy tubes in
D) Impaired physical mobility place. The child>s parent asks the nurse if he
can swim in the family pool. The best response
from the nurse is
Collected by :DeepaRajesh [ 79 ]
rajesh.ks21@gmail.com
Kuwait
A) «Your child should not swim at all while the B) The family must observe the child for dehy-
tubes are in place.» dration
B) «Your child may swim in your own pool but not C) Parents should administer the daily intramus-
in a lake or ocean.» cular injections
C) «Your child may swim if he wears ear plugs.» D) The client needs to take daily injections in the
D) «Your child may swim anywhere.» short-term

Review Information: The correct answer is C: Review Information: The correct answer is A:
«Your child may swim if he wears ear plugs.» The child should carry a nasal spray for emer-
gency use
Water should not enter the ears. Children should Diabetes insipidus results from reduced secre-
use ear plugs when bathing or swimming and tion of the antidiuretic hormone, vasopressin.
should not put their heads under the water. The child will need to administer daily injections
of vasopressin, and should have the nasal spray
Question25 form of the medication readily available. A medi-
The nurse is caring for a client with asthma who cal alert tag should be worn.
has developed gastroesophageal reflux disease
(GERD). Which of the following medications pre- Question28
scribed for the client may aggravate GERD? A client diagnosed with cirrhosis is started on
A) Anticholinergics lactulose (Cephulac). The main purpose of the
B) Corticosteroids drug for this client is to
C) Histamine blocker A) add dietary fiber
D) Antibiotics B) reduce ammonia levels
C) stimulate peristalsis
Review Information: The correct answer is A: D) control portal hypertension
Anticholinergics
An anticholinergic medication will decrease gas- Review Information: The correct answer is B:
tric emptying and the pressure on the lower es- reduce ammonia levels
ophageal sphincter. Lactulose blocks the absorption of ammonia from
the GI tract and secondarily stimulates bowel
Question26 elimination.
A client is receiving a nitroglycerin infusion for Question29
unstable angina. What assessment would be a The nurse is explaining the effects of cocaine
priority when monitoring the effects of this medi- abuse to a pregnant client. Which of the follow-
cation? ing must the nurse understand as a basis for
A) Blood pressure teaching?
B) Cardiac enzymes A) Cocaine use can cause fetal growth retarda-
C) ECG analysis tion
D) Respiratory rate B) The drug has been linked to neural tube de-
fects
Review Information: The correct answer is A: C) Newborn withdrawal generally occurs imme-
Blood pressure diately after birth
Since an effect of this drug is vasodilation, the D) Breast feeding promotes positive parenting
client must be monitored for hypotension. behaviors

Question27
The nurse is caring for a 10 year-old child who Review Information: The correct answer is A:
has just been diagnosed with diabetes insipidus. Cocaine use can cause fetal growth retardation
The parents ask about the treatment prescribed, Cocaine is vasoconstrictive, and this effect in
vasopressin. A What is priority in teaching the the placental vessels causes fetal hypoxia and
child and family about this drug? diminished growth. Other risks of continued co-
A) The child should carry a nasal spray for emer- caine use during pregnancy include preterm la-
gency use bor, congenital abnormalities, altered brain de-
Collected by :DeepaRajesh [ 80 ]
rajesh.ks21@gmail.com
Kuwait
velopment and subsequent behavioral problems C) «Let>s see if your partner could bring food
in the infant. from home.»
D) «If you don>t eat, I will have to suggest for
Question30 you to be tube fed.»
A client has just been diagnosed with breast
cancer. The nurse enters the room and the cli-
ent tells the nurse that she is stupid. What is the Review Information: The correct answer is C:
most therapeutic response by the nurse? «Let>>s see if your partner could bring food from
A) Explore what is going on with the client home.»
B) Accept the client’s statement without com- Reassurance is ineffective when a client is ac-
ment tively delusional. This option avoids both arguing
C) Tell the client that the comment is inappropri- with the client and agreeing with the delusional
ate premise. Option D offers a logical response to
D) Leave the client>s room a primarily affective concern. When the client’s
condition has improved, gentle negation of the
Review Information: The correct answer is A: delusional premise can be employed.
Explore what is going on with the client
Exploring feelings with the verbally aggressive Question33
client helps to put angry feelings into words and A client with tuberculosis is started on Rifampin.
then to engage in problem solving. Which one of the following statements by the
nurse would be appropriate to include in teach-
Question31 ing? «You may notice:
A client has many delusions. As the nurse helps A) an orange-red color to your urine.»
the client prepare for breakfast the client com- B) your appetite may increase for the first
ments «Don’t waste good food on me. I’m dying week.”
from this disease I have.» The appropriate re- C) it is common to experience occasional sleep
sponse would be disturbances.»
A) «You need some nutritious food to help you D) if you take the medication with food, you may
regain your weight.» have nausea.»
B) «None of the laboratory reports show that you
have any physical disease.»
C) «Try to eat a little bit, breakfast is the most Review Information: The correct answer is A:
important meal of the day.» an orange-red color to your urine.»
D) «I know you believe that you have an incur- Discoloration of the urine and other body fluids
able disease.» may occur. It is a harmless response to the drug,
but the patient needs to be aware it may hap-
Review Information: The correct answer is D: pen.
«I know you believe that you have an incurable
disease.» Question34
This response does not challenge the client’s A client tells the RN she has decided to stop tak-
delusional system and thus forms an alliance by ing sertraline (Zoloft) because she doesn’t like
providing reassurance of desire to help the cli- the nightmares, sex dreams, and obsessions
ent. she’s experiencing since starting on the medi-
cation. What is an appropriate response by the
Question32 nurse?
A client with paranoid thoughts refuses to eat be- A) «It is unsafe to abruptly stop taking any pre-
cause of the belief that the food is poisoned. The scribed medication.»
appropriate statement at this time for the nurse B) «Side effects and benefits should be dis-
to say is cussed with your health care provider.»
A) «Here, I will pour a little of the juice in a medi- C) «This medication should be continued despite
cine cup to drink it to show you that it is OK.» unpleasant symptoms.»
B) «The food has been prepared in our kitchen D) «Many medications have potential side ef-
and is not poisoned.» fects.»
Collected by :DeepaRajesh [ 81 ]
rajesh.ks21@gmail.com
Kuwait
Question37
Review Information: The correct answer is The nurse is teaching a school-aged child and
A: «It is unsafe to abruptly stop taking any pre- family about the use of inhalers prescribed for
scribed medication.» asthma. What is the best way to evaluate effec-
Abrupt withdrawal may occasionally cause sero- tiveness of the treatments?
tonin syndrome, consisting of lethargy, nausea,
headache, fever, sweating and chills. A slow with- A) Rely on child>s self-report
drawal may be prescribed with sertraline to avoid B) Use a peak-flow meter
dizziness, nausea, vomiting, and diarrhea. C) Note skin color changes
D) Monitor pulse rate
Question35
A client is admitted to the hospital with findings of
liver failure with ascites. The health care provider Review Information: The correct answer is B:
orders spironolactone (Aldactone). What is the Use a peak-flow meter
pharmacological effect of this medication? The peak flowmeter, if used correctly, shows ef-
A) Promotes sodium and chloride excretion fectiveness of inhalants.
B) Increases aldosterone levels
C) Depletes potassium reserves Question38
D) Combines safely with antihypertensives The nurse is teaching a client about the toxicity
of digoxin. Which one of the following statements
Review Information: The correct answer is A: made by the client to the nurse indicates more
Promotes sodium and chloride excretion teaching is needed?
Spironolactone promotes sodium and chloride A) «I may experience a loss of appetite.»
excretion while sparing potassium and decreas- B) «I can expect occasional double vision.»
ing aldosterone levels. It had no effect on am- C) «Nausea and vomiting may last a few days.»
monia levels. D) «I must report a bounding pulse of 62 imme-
diately.»
Question36
A client was admitted to the psychiatric unit for Review Information: The correct answer is D:
severe depression. After several days, the cli- «I must report a bounding pulse of 62 immedi-
ent continues to withdraw from the other clients. ately.»
Which of these statements by the nurse would Slow heart rate is related to increased cardiac
be the most appropriate to promote interaction output and an intended effect of digoxin. The ide-
with other clients? al heart rate is above 60 BPM with digoxin. The
A) «Your team here thinks it>s good for you to client needs further teaching.
spend time with others.»
B) «It is important for you to participate in group Question39
activities.» Which of the following assessments by the nurse
C) «Come with me so you can paint a picture to would indicate that the client is having a possible
help you feel better.» adverse response to the isoniazid (INH)?
D) «Come play Chinese Checkers with Gloria A) Severe headache
and me.» B) Appearance of jaundice
C) Tachycardia
Review Information: The correct answer is D: D) Decreased hearing
«Come play Chinese Checkers with Gloria and
me.» Review Information: The correct answer is B:
This gradually engages the client in interactions Appearance of jaundice
with others in small groups rather than large Clients receiving INH therapy are at risk for de-
groups. In addition, focusing on an activity is less veloping drug induced hepatitis. The appearance
anxiety-provoking than unstructured discussion. of jaundice may indicate that the client has liver
The statement is an example of a positive be- damage.
havioral expectation.
Question40
Collected by :DeepaRajesh [ 82 ]
rajesh.ks21@gmail.com
Kuwait
The nurse is beginning nutritional counseling/ B) Void a little, clean the meatus, then collect
teaching with a pregnant woman. What is the ini- specimen
tial step in this interaction? C) Clean the meatus, then urinate into container
D) Void continuously and catch some of the
A) Teach her how to meet the needs of self and urine
her family
B) Explain the changes in diet necessary for Review Information: The correct answer is
pregnant women A: Clean the meatus, begin voiding, then catch
C) Questionher understanding and use urine stream. A clean catch urine is difficult to
of the food pyramid obtain and requires clear directions. Instructing
D) Conduct a diet history to determine her nor- the client to carefully clean the meatus, then void
mal eating routines naturally with a steady stream prevents surface
bacteria from contaminating the urine specimen.
Review Information: The correct answer is D: As starting and stopping flow can be difficult,
Conduct a diet history to determine her normal once the client begins voiding it>>s best to just
eating routines. slip the container into the stream. Other respons-
Assessment is always the first step in planning es do not reflect correct technique.
teaching for any client. A thorough and accurate
history is essential for gathering the needed in- Question3
formation. Following change-of-shift report on an orthoped-
ic unit, which client should the nurse see first?
Free NCLEX-RN Sample Test Ques- A) 16 year-old who had an open reduction of a
fractured wrist 10 hours ago
tions For Nursing Review (Part 1)
B) 20 year-old in skeletal traction for 2 weeks
Jul31,
since a motor cycle accident
C) 72 year-old recovering from surgery after a
These are sample nursing review questions and
hip replacement 2 hours ago
not actual test questions made for educational
D) 75 year-old who is in skin traction prior to
and practice test purposes only. 75 questions
planned hip pinning surgery.
have been posted here with answer keys.
Review Information: The correct answer is C:
Question1 72 year-old recovering from surgery after a hip
A client has been hospitalized after an automo- replacement 2 hours ago. Look for the client who
bile accident. A full leg cast was applied in the has the most imminent risks and acute vulnerabil-
emergency room. The most important reason for ity. The client who returned from surgery 2 hours
the nurse to elevate the casted leg is to ago is at risk for life threatening hemorrhage and
A) Promote the client>s comfort should be seen first. The 16 year-old should be
B) Reduce the drying time seen next because it is still the first post-op day.
C) Decrease irritation to the skin The 75 year-old is potentially vulnerable to age-
D) Improve venous return related physical and cognitive consequences in
skin traction should be seen next. The client who
Review Information: The correct answer is D: can safely be seen last is the 20 year-old who is
Improve venous return. Elevating the leg both 2 weeks post-injury.
improves venous return and reduces swelling.
Client comfort will be improved as well.
Question4
A client with Guillain Barre is in a nonresponsive
Question2 state, yet vital signs are stable and breathing is
The nurse is reviewing with a client how to col- independent. What should the nurse document
lect a clean catch urine specimen. What is the to most accurately describe the client>s condi-
appropriate sequence to teach the client? tion?
A) Comatose, breathing unlabored
A) Clean the meatus, begin voiding, then catch B) Glascow Coma Scale 8, respirations regular
urine stream C) Appears to be sleeping, vital signs stable
Collected by :DeepaRajesh [ 83 ]
rajesh.ks21@gmail.com
Kuwait
D) Glascow Coma Scale 13, no ventilator re-
quired Question7
A client had 20 mg of Lasix (furosemide) PO at
Review Information: The correct answer is 10 AM. Which would be essential for the nurse to
B: Glascow Coma Scale 8, respirations regular. include at the change of shift report?
The Glascow Coma Scale provides a standard A) The client lost 2 pounds in 24 hours
reference for assessing or monitoring level of B) The client’s potassium level is 4 mEq/liter.
consciousness. Any score less than 13 indicates C) The client’s urine output was 1500 cc in 5
a neurological impairment. Using the term coma- hours
tose provides too much room for interpretation D) The client is to receive another dose of Lasix
and is not very precise. at 10 PM

Question5 Review Information: The correct answer is C:


When caring for a client receiving warfarin so- The client’s urine output was 1500 cc in 5 hours.
dium (Coumadin), which lab test would the nurse Although all of these may be correct information
monitor to determine therapeutic response to the to include in report, the essential piece would be
drug? the urine output.
A) Bleeding time
B) Coagulation time Question8
C) Prothrombin time A client has been tentatively diagnosed with
D) Partial thromboplastin time Graves> disease (hyperthyroidism). Which of
these findings noted on the initial nursing assess-
Review Information: The correct answer is ment requires quick intervention by the nurse?
C: Prothrombin time. Coumadin is ordered daily, A) a report of 10 pounds weight loss in the last
based on the client>>s prothrombin time (PT). month
This test evaluates the adequacy of the extrinsic B) a comment by the client «I just can>t sit
system and common pathway in the clotting cas- still.»
cade; Coumadin affects the Vitamin K depend- C) the appearance of eyeballs that appear to
ent clotting factors. «pop» out of the client>s eye sockets
D) a report of the sudden onset of irritability in
Question6 the past 2 weeks
A client with moderate persistent asthma is ad-
mitted for a minor surgical procedure. On ad- Review Information: The correct answer is C:
mission the peak flow meter is measured at 480 the appearance of eyeballs that appear to «pop»
liters/minute. Post-operatively the client is com- out of the client>>s eye sockets. Exophthalmos
plaining of chest tightness. The peak flow has or protruding eyeballs is a distinctive characteris-
dropped to 200 liters/minute. What should the tic of Graves>> Disease. It can result in corneal
nurse do first? abrasions with severe eye pain or damage when
A) Notify both the surgeon and provider the eyelid is unable to blink down over the pro-
B) Administer the prn dose of albuterol truding eyeball. Eye drops or ointment may be
C) Apply oxygen at 2 liters per nasal cannula needed.
D) Repeat the peak flow reading in 30 minutes
Question9
Review Information: The correct answer is The nurse has performed the initial assessments
B: Administer the prn dose of albuterol. Peak of 4 clients admitted with an acute episode of
flow monitoring during exacerbations of asthma asthma. Which assessment finding would cause
is recommended for clients with moderate-to- the nurse to call the provider immediately?
severe persistent asthma to determine the se- A) prolonged inspiration with each breath
verity of the exacerbation and to guide the treat- B) expiratory wheezes that are suddenly absent
ment. A peak flow reading of less than 50% of in 1 lobe
the client>>s baseline reading is a medical alert C) expectoration of large amounts of purulent
condition and a short-acting beta-agonist must mucous
be taken immediately. D) appearance of the use of abdominal muscles
Collected by :DeepaRajesh [ 84 ]
rajesh.ks21@gmail.com
Kuwait
for breathing intracranial bleeding or extension of the stroke.
Further diagnostic testing may be indicated.
Review Information: The correct answer is B:
expiratory wheezes that are suddenly absent in 1 Question12
lobe. Acute asthma is characterized by expiratory A school-aged child has had a long leg (hip to
wheezes caused by obstruction of the airways. ankle) synthetic cast applied 4 hours ago. Which
Wheezes are a high pitched musical sounds pro- statement from the parent indicates that teach-
duced by air moving through narrowed airways. ing has been inadequate?
Clients often associate wheezes with the feeling A) «I will keep the cast uncovered for the next
of tightness in the chest. However, sudden ces- day to prevent burning of the skin.»
sation of wheezing is an ominous or bad sign B) «I can apply an ice pack over the area to re-
that indicates an emergency -- the small airways lieve itching inside the cast.»
are now collapsed. C) «The cast should be propped on at least 2 pil-
lows when my child is lying down.»
Question10 D) «I think I remember that my child should not
During the initial home visit, a nurse is discuss- stand until after 72 hours.»
ing the care of a client newly diagnosed with
Alzheimer>s disease with family members. Review Information: The correct answer is D:
Which of these interventions would be most «I think I remember that my child should not stand
helpful at this time? until after 72 hours.». Synthetic casts will typi-
A) leave a book about relaxation techniques cally set up in 30 minutes and dry in a few hours.
B) write out a daily exercise routine for them to Thus, the client may stand within the initial 24
assist the client to do hours. With plaster casts, the set up and drying
C) list actions to improve the client>s daily nutri- time, especially in a long leg cast which is thicker
tional intake than an arm cast, can take up to 72 hours. Both
D) suggest communication strategies types of casts give off a lot of heat when drying
and it is preferable to keep the cast uncovered
Review Information: The correct answer is D: for the first 24 hours. Clients may complain of a
suggest communication strategies. Alzheimer>>s chill from the wet cast and therefore can simply
disease, a progressive chronic illness, greatly be covered lightly with a sheet or blanket. Apply-
challenges caregivers. The nurse can be of ing ice is a safe method of relieving the itching.
greatest assistance in helping the family to use
communication strategies to enhance their ability Question13
to relate to the client. By use of select verbal and Which blood serum finding in a client with dia-
nonverbal communication strategies the family betic ketoacidosis alerts the nurse that immedi-
can best support the client’s strengths and cope ate action is required?
with any aberrant behavior. A) pH below 7.3
B) Potassium of 5.0
Question11 C) HCT of 60
An 80 year-old client admitted with a diagnosis D) Pa O2 of 79%
of possible cerebral vascular accident has had a
blood pressure from 160/100 to 180/110 over the Review Information: The correct answer is C:
past 2 hours. The nurse has also noted increased HCT of 60. This high hematocrit is indicative of
lethargy. Which assessment finding should the severe dehydration which requires priority atten-
nurse report immediately to the provider? tion in diabetic ketoacidosis. Without sufficient
A) Slurred speech hydration, all systems of the body are at risk
B) Incontinence for hypoxia from a lack of or sluggish circula-
C) Muscle weakness tion. In the absence of insulin, which facilitates
D) Rapid pulse the transport of glucose into the cell, the body
breaks down fats and proteins to supply energy
Review Information: The correct answer is A: ketones, a by-product of fat metabolism. These
Slurred speech. Changes in speech patterns and accumulate causing metabolic acidosis (pH <
level of conscious can be indicators of continued 7.3), which would be the second concern for this
Collected by :DeepaRajesh [ 85 ]
rajesh.ks21@gmail.com
Kuwait
client. The potassium and PaO2 levels are near planted from the client>>s own skin.
normal.
Question17
Question14 A client is admitted to the emergency room fol-
The nurse is preparing a client with a deep vein lowing an acute asthma attack. Which of the fol-
thrombosis (DVT) for a Venous Doppler evalua- lowing assessments would be expected by the
tion. Which of the following would be necessary nurse?
for preparing the client for this test?
A) Client should be NPO after midnight A) Diffuse expiratory wheezing
B) Client should receive a sedative medication B) Loose, productive cough
prior to the test C) No relief from inhalant
C) Discontinue anti-coagulant therapy prior to D) Fever and chills
the test
D) No special preparation is necessary Review Information: The correct answer is A:
Diffuse expiratory wheezing. In asthma, the air-
Review Information: The correct answer is ways are narrowed, creating difficulty getting air
D: No special preparation is necessary. This is in. A wheezing sound results.
a non-invasive procedure and does not require
preparation other than client education. Question18
A client has been admitted with a fractured femur
Question15 and has been placed in skeletal traction. Which
A client is admitted with infective endocarditis of the following nursing interventions should re-
(IE). Which finding would alert the nurse to a ceive priority?
complication of this condition? A) Maintaining proper body alignment
A) dyspnea B) Frequent neurovascular assessments of the
B) heart murmur affected leg
C) macular rash C) Inspection of pin sites for evidence of drain-
D) hemorrhage age or inflammation
D) Applying an over-bed trapeze to assist the cli-
Review Information: The correct answer is ent with movement in bed
B: heart murmur. Large, soft, rapidly developing
vegetations attach to the heart valves. They have Review Information: The correct answer is
a tendency to break off, causing emboli and leav- B: Frequent neurovascular assessments of the
ing ulcerations on the valve leaflets. These em- affected leg. The most important activity for the
boli produce findings of cardiac murmur, fever, nurse is to assess neurovascular status. Com-
anorexia, malaise and neurologic sequelae of partment syndrome is a serious complication of
emboli. Furthermore, the vegetations may travel fractures. Prompt recognition of this neurovascu-
to various organs such as spleen, kidney, coro- lar problem and early intervention may prevent
nary artery, brain and lungs, and obstruct blood permanent limb damage.
flow.
Question19
Question16 The nurse is assigned to care for a client who
The nurse explains an autograft to a client sched- had a myocardial infarction (MI) 2 days ago. The
uled for excision of a skin tumor. The nurse knows client has many questions about this condition.
the client understands the procedure when the What area is a priority for the nurse to discuss at
client says, «I will receive tissue from this time?
A) a tissue bank.» A) Daily needs and concerns
B) a pig.» B) The overview cardiac rehabilitation
C) my thigh.» C) Medication and diet guideline
D) synthetic skin.» D) Activity and rest guidelines

Review Information: The correct answer is C: Review Information: The correct answer is A:
my thigh.». Autografts are done with tissue trans- Daily needs and concerns. At 2 days post-MI, the
Collected by :DeepaRajesh [ 86 ]
rajesh.ks21@gmail.com
Kuwait
client’s education should be focused on the im- loupe, milk
mediate needs and concerns for the day. D) Peanut butter and jelly sandwich, apple slic-
es, milk
Question20
A 3 year-old child is brought to the clinic by his Review Information: The correct answer is B:
grandmother to be seen for «scratching his bot- Ground beef patty, lima beans, wheat roll, raisins,
tom and wetting the bed at night.» Based on milk. Iron rich foods include red meat, fish, egg
these complaints, the nurse would initially as- yolks, green leafy vegetables, legumes, whole
sess for which problem? grains, and dried fruits such as raisins. This din-
A) allergies ner is the best choice: It is high in iron and is ap-
B) scabies propriate for a toddler.
C) regression
D) pinworms Question23
The nurse admitting a 5 month-old who vomited
Review Information: The correct answer is 9 times in the past 6 hours should observe for
D: pinworms. Signs of pinworm infection include signs of which overall imbalance?
intense perianal itching, poor sleep patterns, A) Metabolic acidosis
general irritability, restlessness, bed-wetting, B) Metabolic alkalosis
distractibility and short attention span. Scabies C) Some increase in the serum hemoglobin
is an itchy skin condition caused by a tiny, eight- D) A little decrease in the serum potassium
legged burrowing mite called Sarcoptes scabiei .
The presence of the mite leads to intense itching Review Information: The correct answer is B:
in the area of its burrows. Metabolic alkalosis. Vomiting causes loss of acid
from the stomach. Prolonged vomiting can re-
Question21 sult in excess loss of acid and lead to metabolic
The nurse is caring for a newborn with tra- alkalosis. Findings include irritability, increased
cheoesophageal fistula. Which nursing diagno- activity, hyperactive reflexes, muscle twitching
sis is a priority? and elevated pulse. Options C and D are correct
A) Risk for dehydration answers but not the best answers since they are
B) Ineffective airway clearance too general.
C) Altered nutrition
D) Risk for injury Question24
A two year-old child is brought to the provider>s
Review Information: The correct answer is B: office with a chief complaint of mild diarrhea for
Ineffective airway clearance. The most common two days. Nutritional counseling by the nurse
form of TEF is one in which the proximal esopha- should include which statement?
geal segment terminates in a blind pouch and A) Place the child on clear liquids and gelatin for
the distal segment is connected to the trachea 24 hours
or primary bronchus by a short fistula at or near B) Continue with the regular diet and include oral
the bifurcation. Thus, a priority is maintaining an rehydration fluids
open airway, preventing aspiration. Other nurs- C) Give bananas, apples, rice and toast as toler-
ing diagnoses are then addressed. ated
D) Place NPO for 24 hours, then rehydrate with
Question22 milk and water
The nurse is developing a meal plan that would
provide the maximum possible amount of iron for Review Information: The correct answer is B:
a child with anemia. Which dinner menu would Continue with the regular diet and include oral
be best? rehydration fluids. Current recommendations for
A) Fish sticks, french fries, banana, cookies, mild to moderate diarrhea are to maintain a nor-
milk mal diet with fluids to rehydrate.
B) Ground beef patty, lima beans, wheat roll, rai-
sins, milk
C) Chicken nuggets, macaroni, peas, canta- Question25
Collected by :DeepaRajesh [ 87 ]
rajesh.ks21@gmail.com
Kuwait
The nurse is teaching parents about the appro- ates a high renal solute load.
priate diet for a 4 month-old infant with gastro-
enteritis and mild dehydration. In addition to oral Question28
rehydration fluids, the diet should include The nurse is preparing a handout on infant feed-
ing to be distributed to families visiting the clinic.
A) formula or breast milk Which notation should be included in the teach-
B) broth and tea ing materials?
C) rice cereal and apple juice
D) gelatin and ginger ale A) Solid foods are introduced one at a time be-
ginning with cereal
Review Information: The correct answer is A: B) Finely ground meat should be started early to
formula or breast milk. The usual diet for a young provide iron
infant should be followed. C) Egg white is added early to increase protein
intake
Question26 D) Solid foods should be mixed with formula in
A child is injured on the school playground and a bottle
appears to have a fractured leg. The first action
the school nurse should take is Review Information: The correct answer is A:
Solid foods are introduced one at a time begin-
A) call for emergency transport to the hospital ning with cereal. Solid foods should be added
B) immobilize the limb and joints above and be- one at a time between 4-6 months. If the infant is
low the injury able to tolerate the food, another may be added
C) assess the child and the extent of the injury in a week. Iron fortified cereal is the recommend-
D) apply cold compresses to the injured area ed first food.

Review Information: The correct answer is Question29


C: assess the child and the extent of the injury. The nurse planning care for a 12 year-old child
When applying the nursing process, assessment with sickle cell disease in a vaso-occlusive crisis
is the first step in providing care. The «5 Ps» of the elbow should include which one of the fol-
of vascular impairment can be used as a guide lowing as a priority?
(pain, pulse, pallor, paresthesia, paralysis).
A) Limit fluids
Question27 B) Client controlled analgesia
The mother of a 3 month-old infant tells the nurse C) Cold compresses to elbow
that she wants to change from formula to whole D) Passive range of motion exercise
milk and add cereal and meats to the diet. What
should be emphasized as the nurse teaches Review Information: The correct answer is B:
about infant nutrition? Client controlled analgesia. Management of a
sickle cell crisis is directed towards supportive
A) Solid foods should be introduced at 3-4 and symptomatic treatment. The priority of care
months is pain relief. In a 12 year-old child, client control-
B) Whole milk is difficult for a young infant to di- led analgesia promotes maximum comfort.
gest
C) Fluoridated tap water should be used to dilute Question30
milk The nurse is performing a physical assessment
D) Supplemental apple juice can be used be- on a toddler. Which of the following actions
tween feedings should be the first?

Review Information: The correct answer is B: A) Perform traumatic procedures


Whole milk is difficult for a young infant to digest. B) Use minimal physical contact
Cow>>s milk is not given to infants younger than C) Proceed from head to toe
1 year because the tough, hard curd is difficult to D) Explain the exam in detail
digest. In addition, it contains little iron and cre-
Collected by :DeepaRajesh [ 88 ]
rajesh.ks21@gmail.com
Kuwait
Review Information: The correct answer these foods would the nurse reinforce for the cli-
is B: Use minimal physical contact. The nurse ent to eat at least daily?
should approach the toddler slowly and use min-
imal physical contact initially so as to gain the A) Spaghetti
toddler>>s cooperation. Be flexible in the se- B) Watermelon
quence of the exam, and give only brief simple C) Chicken
explanations just prior to the action. D) Tomatoes

Question31 Review Information: The correct answer is B:


What finding signifies that children have attained Watermelon. Watermelon is high in potassium
the stage of concrete operations (Piaget)? and will replace potassium lost by the diuretic.
The other foods are not high in potassium.
A) Explores the environment with the use of sight
and movement Question34
B) Thinks in mental images or word pictures While teaching the family of a child who will take
C) Makes the moral judgment that «stealing is phenytoin (Dilantin) regularly for seizure control,
wrong» it is most important for the nurse to teach them
D) Reasons that homework is time-consuming about which of the following actions?
yet necessary
A) Maintain good oral hygiene and dental care
Review Information: The correct answer is B) Omit medication if the child is seizure free
C: Makes the moral judgment that «stealing is C) Administer acetaminophen to promote sleep
wrong». The stage of concrete operations is de- D) Serve a diet that is high in iron
picted by logical thinking and moral judgments.
Review Information: The correct answer is
Question32 A: Maintain good oral hygiene and dental care.
The mother of a child with a neural tube defect Swollen and tender gums occur often with use of
asks the nurse what she can do to decrease the phenytoin. Good oral hygiene and regular visits
chances of having another baby with a neural to the dentist should be emphasized.
tube defect. What is the best response by the
nurse? Question35
The nurse is offering safety instructions to a par-
A) «Folic acid should be taken before and after ent with a four month-old infant and a four year-
conception.» old child. Which statement by the parent indi-
B) «Multivitamin supplements are recommended cates understanding of appropriate precautions
during pregnancy.» to take with the children?
C) «A well balanced diet promotes normal fetal
development.» A) «I strap the infant car seat on the front seat to
D) «Increased dietary iron improves the health of face backwards.»
mother and fetus.» B) «I place my infant in the middle of the living
room floor on a blanket to play with my four year-
Review Information: The correct answer is old while I make supper in the kitchen.»
A: «Folic acid should be taken before and after C) «My sleeping baby lies so cute in the crib with
conception.». The American Academy of Pedi- the little buttocks stuck up in the air while the four
atrics recommends that all childbearing women year-old naps on the sofa.»
increase folic acid from dietary sources and/or D) «I have the four year-old hold and help feed
supplements. There is evidence that increased the four month-old a bottle in the kitchen while I
amounts of folic acid prevents neural tube de- make supper.»
fects.
Review Information: The correct answer is
Question33 D: «I have the four year-old hold and help feed
The provider orders Lanoxin (digoxin) 0.125 mg the four month-old a bottle in the kitchen while I
PO and furosemide 40 mg every day. Which of make supper.». The infant seat is to be placed
Collected by :DeepaRajesh [ 89 ]
rajesh.ks21@gmail.com
Kuwait
on the rear seat. Small children and infants are C) with each meal or snack
not to be left unsupervised. Infants are D) each time carbohydrates are eaten

Question36 Review Information: The correct answer is C:


The nurse admits a 7 year-old to the emergency with each meal or snack. Pancreatic enzymes
room after a leg injury. The x-rays show a femur should be taken with each meal and every snack
fracture near the epiphysis. The parents ask what to allow for digestion of all foods that are eaten.
will be the outcome of this injury. The appropriate
response by the nurse should be which of these
statements? Question39
A nurse is providing a parenting class to individu-
A) «The injury is expected to heal quickly be- als living in a community of older homes. In dis-
cause of thin periosteum.» cussing formula preparation, which of the follow-
B) «In some instances the result is a retarded ing is most important to prevent lead poisoning?
bone growth.»
C) «Bone growth is stimulated in the affected A) Use ready-to-feed commercial infant formula
leg.» B) Boil the tap water for 10 minutes prior to pre-
D) «This type of injury shows more rapid union paring the formula
than that of younger children.» C) Let tap water run for 2 minutes before adding
to concentrate
Review Information: The correct answer is B: D) Buy bottled water labeled «lead free» to mix
«In some instances the result is a retarded bone the formula
growth.». An epiphyseal (growth) plate fracture
in a 7 year-old often results in retarded bone Review Information: The correct answer is C:
growth. The leg often will be different in length Let tap water run for 2 minutes before adding to
than the uninjured leg. concentrate. Use of lead-contaminated water to
prepare formula is a major source of poisoning
Question37 in infants. Drinking water may be contaminated
The parents of a 4 year-old hospitalized child tell by lead from old lead pipes or lead solder used
the nurse, “We are leaving now and will be back in sealing water pipes. Letting tap water run for
at 6 PM.” A few hours later the child asks the several minutes will diminish the lead contami-
nurse when the parents will come again. What is nation.
the best response by the nurse?
Question40
A) «They will be back right after supper.» Which of the following manifestations observed
B) «In about 2 hours, you will see them.» by the school nurse confirms the presence of pe-
C) «After you play awhile, they will be here.» diculosis capitis in students?
D) «When the clock hands are on 6 and 12.»
A) Scratching the head more than usual
Review Information: The correct answer is A: B) Flakes evident on a student>s shoulders
«They will be back right after supper.». Time is C) Oval pattern occipital hair loss
not completely understood by a 4 year-old. Pre- D) Whitish oval specks sticking to the hair
schoolers interpret time with their own frame of
reference. Thus, it is best to explain time in rela- Review Information: The correct answer is D:
tionship to a known, common event. Whitish oval specks sticking to the hair. Diagno-
sis of pediculosis capitis is made by observation
Question38 of the white eggs (nits) firmly attached to the
The nurse is giving instructions to the parents hair shafts. Treatment can include application of
of a child with cystic fibrosis. The nurse would a medicated shampoo with lindane for children
emphasize that pancreatic enzymes should be over 2 years of age, and meticulous combing
taken and removal of all nits.
A) once each day
B) 3 times daily after meals
Collected by :DeepaRajesh [ 90 ]
rajesh.ks21@gmail.com
Kuwait
Question41 response to being in the hospital.»
When interviewing the parents of a child with D) «You might want to «sneak out» of the room
asthma, it is most important to assess the child>s once the child falls asleep.»
environment for what factor?
Review Information: The correct answer is C:
A) Household pets «Keep in mind that for the age this is a normal
B) New furniture response to being in the hospital.». The protest
C) Lead based paint phase of separation anxiety is a normal response
D) Plants such as cactus for a child this age. In toddlers, ages 1 to 3, sepa-
ration anxiety is at its peak
Review Information: The correct answer is A:
Household pets. Animal dander is a very com- Question44
mon allergen affecting persons with asthma. A couple experienced the loss of a 7 month-old
Other triggers may include pollens, carpeting fetus. In planning for discharge, what should the
and household dust. nurse emphasize?

Question42 A) To discuss feelings with each other and use


The mother of a 2 month-old baby calls the nurse support persons
2 days after the first DTaP, IPV, Hepatitis B and B) To focus on the other healthy children and
HIB immunizations. She reports that the baby move through the loss
feels very warm, cries inconsolably for as long as C) To seek causes for the fetal death and come
3 hours, and has had several shaking spells. In to some safe conclusion
addition to referring her to the emergency room, D) To plan for another pregnancy within 2 years
the nurse should document the reaction on the and maintain physical health
baby>s record and expect which immunization
to be most associated with the findings the infant Review Information: The correct answer is A:
is displaying? To discuss feelings with each other and use sup-
port persons. To communicate in a therapeutic
A) DTaP manner, the nurse>>s goal is to help the couple
B) Hepatitis B begin the grief process by suggesting they talk
C) Polio to each other, seek family, friends and support
D) H. Influenza groups to listen to their feelings.

Review Information: The correct answer is A:


DTaP. The majority of reactions occur with the Question45
administration of the DTaP vaccination. Contra- The nurse is performing a pre-kindergarten phys-
dictions to giving repeat DTaP immunizations in- ical on a 5 year-old. The last series of vaccines
clude the occurrence of severe side effects after will be administered. What is the preferred site
a previous dose as well as signs of encephalop- for injection by the nurse?
athy within 7 days of the immunization.
A) vastus intermedius
B) gluteus maximus
Question43 C) vastus lateralis
The mother of a 2 year-old hospitalized child asks D) dorsogluteaI
the nurse>s advice about the child>s screaming
every time the mother gets ready to leave the Review Information: The correct answer is C:
hospital room. What is the best response by the vastus lateralis. Vastus lateralis, a large and well
nurse? developed muscle, is the preferred site, since it is
removed from major nerves and blood vessels.
A) «I think you or your partner needs to stay with
the child while in the hospital.»
B) «Oh, that behavior will stop in a few days.» Question46
C) «Keep in mind that for the age this is a normal A 7 month pregnant woman is admitted with com-
Collected by :DeepaRajesh [ 91 ]
rajesh.ks21@gmail.com
Kuwait
plaints of painless vaginal bleeding over several Review Information: The correct answer is C:
hours. The nurse should prepare the client for an reassess the child>>s temperature. A child>>s
immediate temperature may have rapid fluctuations. The
nurse should listen to and show respect for what
A) Non stress test parents say. Parental caretakers are often quite
B) Abdominal ultrasound sensitive to variations in their children>>s condi-
C) Pelvic exam tion that may not be immediately evident to oth-
D) X-ray of abdomen ers.

Review Information: The correct answer is B:


Abdominal ultrasound. The standard for diagno- Question49
sis of placenta previa, which is suggested in the The nurse is caring for a client who was success-
client>>s history of painless bleeding, is abdomi- fully resuscitated from a pulseless dysrhythmia.
nal ultrasound. Which of the following assessments is critical for
the nurse to include in the plan of care?

Question47 A) hourly urine output


A nurse entering the room of a postpartum moth- B) white blood count
er observes the baby lying at the edge of the C) blood glucose every 4 hours
bed while the woman sits in a chair. The mother D) temperature every 2 hours
states «This is not my baby, and I do not want it.»
After repositioning the child safely, the nurse>s Review Information: The correct answer is A:
best response is hourly urine output. Clients who have had an epi-
sode of decreased glomerular perfusion are at
A) «This is a common occurrence after birth, but risk for pre-renal failure. This is caused by any
you will come to accept the baby.» abnormal decline in kidney perfusion that reduc-
B) «Many women have postpartum blues and es glomerular perfusion. Pre-renal failure occurs
need some time to love the baby.» when the effective arterial blood volume falls.
C) «What a beautiful baby! Her eyes are just like Examples of this phenomena include a drop in
yours.» circulating blood volume as in a cardiac arrest
D) «You seem upset; tell me what the pregnancy state or in low cardiac perfusion states such as
and birth were like for you.» congestive heart failure associated with a cardi-
omyopathy. Close observation of hourly urinary
Review Information: The correct answer is D: output is necessary for early detection of this
«You seem upset; tell me what the pregnancy condition.
and birth were like for you.». A non-judgmental,
open ended response facilitates dialogue be-
tween the client and nurse. Question50
A client is admitted to the rehabilitation unit fol-
lowing a cerebral vascular accident (CVA) and
Question48 mild dysphagia. The most appropriate interven-
The nurse notes that a 2 year-old child recovering tion for this client is to
from a tonsillectomy has an temperature of 98.2
degrees Fahrenheit at 8:00 AM. At 10:00 AM the A) position client in upright position while eating
child>s parent reports that the child «feels very B) place client on a clear liquid diet
warm» to touch. The first action by the nurse C) tilt head back to facilitate swallowing reflex
should be to D) offer finger foods such as crackers or pret-
zels
A) reassure the parent that this is normal
B) offer the child cold oral fluids Review Information: The correct answer is A:
C) reassess the child>s temperature position client in upright position while eating. An
D) administer the prescribed acetaminophen upright position facilitates proper chewing and
swallowing.
Collected by :DeepaRajesh [ 92 ]
rajesh.ks21@gmail.com
Kuwait
Question53
The nurse instructs the client taking dexametha-
Question51 sone (Decadron) to take it with food or milk. The
A 72 year-old client with osteomyelitis requires a physiological basis for this instruction is that the
6 week course of intravenous antibiotics. In plan- medication
ning for home care, what is the most important A) retards pepsin production
action by the nurse? B) stimulates hydrochloric acid production
C) slows stomach emptying time
A) Investigating the client>s insurance coverage D) decreases production of hydrochloric acid
for home IV antibiotic therapy
B) Determining if there are adequate hand wash- Review Information: The correct answer is
ing facilities in the home B: stimulates hydrochloric acid production. Dec-
C) Assessing the client>s ability to participate in adron increases the production of hydrochloric
self care and/or the reliability of a caregiver acid, which may cause gastrointestinal ulcers.
D) Selecting the appropriate venous access de-
vice
Question54
Review Information: The correct answer is A client receiving chlorpromazine HCL (Thora-
C: Assessing the client>>s ability to participate zine) is in psychiatric home care. During a home
in self care and/or the reliability of a caregiver. visit the nurse observes the client smacking her
The cognitive ability of the client as well as the lips alternately with grinding her teeth. The nurse
availability and reliability of a caregiver must be recognizes this assessment finding as what?
assessed to determine if home care is a feasible A) Dystonia
option. B) Akathisia
C) Brady dyskinesia
D) Tardive dyskinesia
Question52
A nurse administers the influenza vaccine to a Review Information: The correct answer is D:
client in a clinic. Within 15 minutes after the im- Tardive dyskinesia. Signs of tardive dyskinesia
munization was given, the client complains of include smacking lips, grinding of teeth and «fly
itchy and watery eyes, increased anxiety, and catching» tongue movements. These findings
difficulty breathing. The nurse expects that the are often described as Parkinsonian.
first action in the sequence of care for this client
will be to
Question55
A) Maintain the airway Which of the following findings contraindicate the
B) Administer epinephrine 1:1000 as ordered use of haloperidol (Haldol) and warrant withhold-
C) Monitor for hypotension with shock ing the dose?
D) Administer diphenhydramine as ordered A) Drowsiness, lethargy, and inactivity
B) Dry mouth, nasal congestion, and blurred vi-
Review Information: The correct answer is B: sion
Administer epinephrine 1:1000 as ordered. All C) Rash, blood dyscrasias, severe depression
the answers are correct given the circumstances, D) Hyperglycemia, weight gain, and edema
but the priority is to administer the epinephrine,
then maintain the airway. In the early stages of Review Information: The correct answer is
anaphylaxis, when the patient has not lost con- C: Rash, blood dyscrasias, severe depression.
sciousness and is normotensive, administering Rash and blood dyscrasias are side effects of
the epinephrine is first, and applying the oxygen, anti-psychotic drugs. A history of severe depres-
and watching for hypotension and shock, are lat- sion is a contraindication to the use of neurolep-
er responses. The prevention of a severe crisis tics.
is maintained by using diphenhydramine.

Collected by :DeepaRajesh [ 93 ]
rajesh.ks21@gmail.com
Kuwait
Question56 digitalis toxicity is a low potassium level. Clients
The nurse is reinforcing teaching to a 24 year-old must be taught that it is important to have ad-
woman receiving acyclovir (Zovirax) for a Herpes equate potassium intake especially if taking diu-
Simplex Virus type 2 infection. Which of these in- retics that enhance the loss of potassium while
structions should the nurse give the client? they are taking digitalis.

A) Complete the entire course of the medication


for an effective cure Question59
B) Begin treatment with acyclovir at the onset of A 42 year-old male client refuses to take pro-
symptoms of recurrence pranolol hydrochloride (Inderal) as prescribed.
C) Stop treatment if she thinks she may be preg- Which client statement from the assessment
nant to prevent birth defects data is likely to explain his noncompliance?
D) Continue to take prophylactic doses for at
least 5 years after the diagnosis A) «I have problems with diarrhea.»
B) «I have difficulty falling asleep.»
Review Information: The correct answer is C) «I have diminished sexual function.»
B: Begin treatment with acyclovir at the onset D) «I often feel jittery.»
of symptoms of recurrence. When the client is
aware of early symptoms, such as pain, itching Review Information: The correct answer is C:
or tingling, treatment is very effective. Medica- «I have diminished sexual function.». Inderal, a
tions for herpes simplex do not cure the disease; beta-blocking agent used in hypertension, pro-
they simply decrease the level of symptoms. hibits the release of epinephrine into the cells;
this may result in hypotension which results in
decreased libido and impotence.
Question57
A 14 month-old child ingested half a bottle of
aspirin tablets. Which of the following would the Question60
nurse expect to see in the child? The nurse caring for a 9 year-old child with a
fractured femur is told that a medication error
A) Hypothermia occurred. The child received twice the ordered
B) Edema dose of morphine an hour ago. Which nursing
C) Dyspnea diagnosis is a priority at this time?
D) Epistaxis
A) Risk for fluid volume deficit related to mor-
Review Information: The correct answer is phine overdose
D: Epistaxis. A large dose of aspirin inhibits pro- B) Decreased gastrointestinal mobility related to
thrombin formation and lowers platelet levels. mucosal irritation
With an overdose, clotting time is prolonged. C) Ineffective breathing patterns related to cen-
tral nervous system depression
D) Altered nutrition related to inability to control
Question58 nausea and vomiting
An 80 year-old client on digitalis (Lanoxin) re-
ports nausea, vomiting, abdominal cramps and Review Information: The correct answer is C:
halo vision. Which of the following laboratory re- Ineffective breathing patterns related to central
sults should the nurse analyze first? nervous system depression. Respiratory depres-
sion is a life-threatening risk in this overdose.
A) Potassium levels
B) Blood pH
C) Magnesium levels Question61
D) Blood urea nitrogen Lactulose (Chronulac) has been prescribed for a
client with advanced liver disease. Which of the
Review Information: The correct answer is A: following assessments would the nurse use to
Potassium levels. The most common cause of evaluate the effectiveness of this treatment?
Collected by :DeepaRajesh [ 94 ]
rajesh.ks21@gmail.com
Kuwait
A) An increase in appetite
B) A decrease in fluid retention Question64
C) A decrease in lethargy A young adult seeks treatment in an outpatient
D) A reduction in jaundice mental health center. The client tells the nurse
he is a government official being followed by
Review Information: The correct answer is spies. On further questioning, he reveals that
C: A decrease in lethargy. Lactulose produces his warnings must be heeded to prevent nuclear
an acid environment in the bowel and traps am- war. What is the most therapeutic approach by
monia in the gut; the laxative effect then aids in the nurse?
removing the ammonia from the body. This de-
creases the effects of hepatic encephalopathy, A) Listen quietly without comment
including lethargy and confusion. B) Ask for further information on the spies
C) Confront the client’s delusion
D) Contact the government agency
Question62
The nurse is teaching a class on HIV prevention. Review Information: The correct answer is A:
Which of the following should be emphasized as Listen quietly without comment. The client>>s
increasing risk? comments demonstrate grandiose ideas. The
most therapeutic response is to listen but avoid
A) Donating blood being incorporated into the client’s delusional
B) Using public bathrooms system.
C) Unprotected sex
D) Touching a person with AIDS Question65
The nurse is assessing a 17 year-old female cli-
Review Information: The correct answer is C: ent with bulimia. Which of the following labora-
Unprotected sex. Because HIV is spread through tory reports would the nurse anticipate?
exposure to bodily fluids, unprotected intercourse
and shared drug paraphernalia remain the high- A) Increased serum glucose
est risks for infection. B) Decreased albumin
C) Decreased potassium
D) Increased sodium retention
Question63
While interviewing a new admission, the nurse Review Information: The correct answer is C:
notices that the client is shifting positions, wring- Decreased potassium. In bulimia, loss of elec-
ing her hands, and avoiding eye contact. It is im- trolytes can occur in addition to other findings of
portant for the nurse to starvation and dehydration.
A) ask the client what she is feeling
B) assess the client for auditory hallucinations
C) recognize the behavior as a side effect of Question66
medication A client, recovering from alcoholism, asks the
D) re-focus the discussion on a less anxiety pro- nurse, «What can I do when I start recognizing
voking topic relapse triggers within myself?» How might the
nurse best respond?
Review Information: The correct answer is A:
ask the client what she is feeling. The initial step A) «When you have the impulse to stop in a bar,
in anxiety intervention is observing, identifying, contact a sober friend and talk with him.»
and assessing anxiety. The nurse should seek B) «Go to an AA meeting when you feel the urge
client validation of the accuracy of nursing as- to drink.»
sessments and avoid drawing conclusions based C) «It is important to exercise daily and get in-
on limited data. In the situation above, the client volved in activities that will cause you not to think
may simply need to use the restroom but be re- about drug use.»
luctant to communicate her need! D) «Let’s talk about possible options you have
Collected by :DeepaRajesh [ 95 ]
rajesh.ks21@gmail.com
Kuwait
when you recognize relapse triggers in your- client learns both tolerance and skills for relation-
self.» ships.

Review Information: The correct answer is D:


«Let’s talk about possible options you have when Question69
you recognize relapse triggers in yourself.». This An important goal in the development of a thera-
option encourages the process of self evalua- peutic inpatient milieu is to
tion and problem solving, while avoiding telling
the client what to do. Encouraging the client to A) provide a businesslike atmosphere where cli-
brainstorm about response options validates the ents can work on individual goals
nurse’s belief in the client’s personal competen- B) provide a group forum in which clients decide
cy and reinforces a coping strategy that will be on unit rules, regulations, and policies
needed when the nurse may not be available to C) provide a testing ground for new patterns of
offer solutions. behavior while the client takes responsibility for
his or her own actions
Question67 D) discourage expressions of anger because
Therapeutic nurse-client interaction occurs when they can be disruptive to other clients
the nurse
A) assists the client to clarify the meaning of what Review Information: The correct answer is C:
the client has said provide a testing ground for new patterns of be-
B) interprets the client’s covert communication havior while the client takes responsibility for his
C) praises the client for appropriate feelings and or her own actions. A therapeutic milieu is pur-
behavior poseful and planned to provide safety and a test-
D) advises the client on ways to resolve prob- ing ground for new patterns of behavior.
lems

Review Information: The correct answer is A: Question70


assists the client to clarify the meaning of what A client with paranoid delusions stares at the
the client has said. Clarification is a facilitating/ nurse over a period of several days. The client
therapeutic communication strategy. Interpreta- suddenly walks up to the nurse and shouts «You
tion, changing the focus/subject, giving approv- think you’re so perfect and pure and good.» An
al, and advising are non-therapeutic/barriers to appropriate response for the nurse is
communication.
A) «Is that why you’ve been staring at me?»
B) «You seem to be in a really bad mood.»
Question68 C) «Perfect? I don’t quite understand.»
Which nursing intervention will be most effective D) «You seem angry right now.»
in helping a withdrawn client to develop relation-
ship skills? Review Information: The correct answer is D:
A) Offer the client frequent opportunities to inter- «You seem angry right now.». The nurse recog-
act with 1 person nizes the underlying emotion with a matter of fact
B) Provide the client with frequent opportunities attitude, but avoids telling the clients how they
to interact with other clients feel.
C) Assist the client to analyze the meaning of the
withdrawn behavior
D) Discuss with the client the focus that other Question71
clients have similar problems A client who is a former actress enters the day
room wearing a sheer nightgown, high heels, nu-
Review Information: The correct answer is A: merous bracelets, bright red lipstick and heavily
Offer the client frequent opportunities to interact rouged cheeks. Which nursing action is the best
with 1 person. The withdrawn client is uncomfort- in response to the client’s attire?
able in social interaction. The nurse-client rela-
tionship is a corrective relationship in which the A) Gently remind her that she is no longer on
Collected by :DeepaRajesh [ 96 ]
rajesh.ks21@gmail.com
Kuwait
stage
B) Directly assist client to her room for appropri- Question74
ate apparel In a psychiatric setting, the nurse limits touch
C) Quietly point out to her the dress of other cli- or contact used with clients to handshaking be-
ents on the unit cause
D) Tactfully explain appropriate clothing for the
hospital A) some clients misconstrue hugs as an invita-
tion to sexual advances
Review Information: The correct answer is B: B) handshaking keeps the gesture on a profes-
Directly assist client to her room for appropriate sional level
apparel. It assists the client to maintain self-es- C) refusal to touch a client denotes lack of con-
teem while modifying behavior. cern
D) inappropriate touch often results in charges of
assault and battery
Question72
When teaching suicide prevention to the parents Review Information: The correct answer is A:
of a 15 year-old who recently attempted suicide, some clients misconstrue hugs as an invitation
the nurse describes the following behavioral cue to sexual advances. Touch denotes positive feel-
as indicating a need for intervention. ings for another person. The client may interpret
hugging and holding hands as sexual advanc-
A) Angry outbursts at significant others es.
B) Fear of being left alone
C) Giving away valued personal items
D) Experiencing the loss of a boyfriend Question75
A client with anorexia is hospitalized on a medi-
Review Information: The correct answer is C: cal unit due to electrolyte imbalance and cardiac
Giving away valued personal items. Eighty per- dysrhythmias. Additional assessment findings
cent of all potential suicide victims give some type that the nurse would expect to observe are
of indication that self-destructiveness should be
addressed. These clues might lead one to sus- A) brittle hair, lanugo, amenorrhea
pect that a client is having suicidal thoughts or is B) diarrhea, nausea, vomiting, dental erosion
developing a plan. C) hyperthermia, tachycardia, increased meta-
bolic rate
D) excessive anxiety about symptoms
Question73
Which statement made by a client indicates to Review Information: The correct answer is A:
the nurse that the client may have a thought dis- brittle hair, lanugo, amenorrhea. Physical find-
order? ings associated with anorexia also include re-
A) «I>m so angry about this. Wait until my part- duced metabolic rate and lower vital signs.
ner hears about this.»
B) «I>m a little confused. What time is it?»
C) «I can>t find my <mesmer> shoes. Have you Free NCLEX-RN Sample Test Ques-
seen them?» tions For Nursing Review (Pharmacol-
D) «I>m fine. It>s my daughter who has the prob-
ogy Set 2)
lem.»
Jul31,
Review Information: The correct answer is C:
A nurse is assigned to perform well-child assess-
«I can>>t find my <>mesmer>> shoes. Have you
ments at a day care center. A staff member in-
seen them?». A neologism is a new word self in-
terrupts the examinations to ask for assistance.
vented by a person and not readily understood
They find a crying 3 year-old child on the floor
by another. Using neologisms is often associ-
with mouth wide open and gums bleeding. Two
ated with a thought disorder.
unlabeled open bottles lie nearby. The nurse’s
first action should be
Collected by :DeepaRajesh [ 97 ]
rajesh.ks21@gmail.com
Kuwait
A) call the poison control center, then 911 The nurse practicing in a long term care facility
B) administer syrup of Ipecac to induce vomitingrecognizes that elderly clients are at greater risk
C) give the child milk to coat her stomach for drug toxicity than younger adults because of
D) ask the staff about the contents of the bottles
which of the following physiological changes of
advancing age?
A) Drugs are absorbed more readily from the GI
Review Information: The correct answer is D: tract
ask the staff about the contents of the bottles B) Elders have less body water and more fat
The nurse needs to assess what the child in- C) The elderly have more rapid hepatic metabo-
gested before determining the next action. Once lism
the substance is identified, the poison control D) Older people are often malnourished and
center and emergency response team should be anemic
called.

Question2 Review Information: The correct answer is B:


A client with atrial fibrillation is receiving digoxin Elders have less body water and more fat
(Lanoxin). Which of these assessments is most Because elderly persons have decreased lean
important for the nurse to perform? body tissue/water in which to distribute medica-
A) Monitor blood pressure every 4 hours tions, more drug remains in the circulatory system
B) Measure apical pulse prior to administration with potential for drug toxicity. Increased body fat
C) Maintain accurate intake and output records results in greater amounts of fat-soluble drugs
D) Record an EKG strip after administration being absorbed, leaving less in circulation, thus
increasing the duration of action of the drug

Review Information: The correct answer is B: Question5


Measure apical pulse prior to administration The nurse is assessing a client who is on long
Digitoxin decreases conduction velocity through term glucocorticoid therapy. Which of the follow-
the AV node and prolongs the refractory period. If ing findings would the nurse expect?
the apical heart rate is less than 60 beats/minute, A) Buffalo hump
withhold the drug. The apical pulse should be B) Increased muscle mass
taken with a stethoscope so that there will be no C) Peripheral edema
mistake about what the heart rate actually is. D) Jaundice

Question3
The nurse is administering an intravenous vesi- Review Information: The correct answer is A:
cant chemotherapeutic agent to a client. Which Buffalo hump
assessment would require the nurse’s immedi- With high doses of glucocorticoid, iatrogenic
ate action? Cushing’’s syndrome develops. The exaggerat-
A) Stomatitis lesion in the mouth ed physiological action causes abnormal fat dis-
B) Severe nausea and vomiting tribution which results in a moon-shaped face,
C) Complaints of pain at site of infusion a intrascapular pad on the neck (buffalo hump)
D) A rash on the client’s extremities and truncal obesity with slender limbs.

Review Information: The correct answer is C: Question6


Complaints of pain at site of infusion The health care provider has written “Morphine
A vesicant is a chemotherapeutic agent capa- sulfate 2 mgs IV every 3-4 hours prn for pain” on
ble of causing blistering of tissues and possible the chart of a child weighing 22 lb. (10 kg). What
tissue necrosis if there is extravasation. These is the nurse’s initial action?
agents are irritants which cause pain along the A) Check with the pharmacist
vein wall, with or without inflammation. B) Hold the medication and contact the provider
C) Administer the prescribed dose as ordered
Question4 D) Give the dose every 6-8 hours
Collected by :DeepaRajesh [ 98 ]
rajesh.ks21@gmail.com
Kuwait
The drug Decadron increases glycogenesis.
This may lead to hyperglycemia. Therefore the
Review Information: The correct answer is B: blood sugar level and acetone production must
Hold the medication and contact the provider be monitored.
The usual pediatric dose of morphine is 0.1 mg/
kg every 3 to 4 hours. At 10 kg, this child typically Question10
should receive 1.0 mg every 3 to 4 hours. The nurse is caring for a client with schizophrenia
who has been treated with quetiapine (Seroquel)
Question7 for 1 month. Today the client is increasingly agi-
A client is ordered atropine to be administered tated and complains of muscle stiffness. Which
preoperatively. Which physiological effect should of these findings should be reported to the health
the nurse monitor for? care provider?
A) Elevate blood pressure A) Elevated temperature and sweating.
B) Drying up of secretions B) Decreased pulse and blood pressure.
C) Reduce heart rate C) Mental confusion and general weakness.
D) Enhance sedation D) Muscle spasms and seizures.

Review Information: The correct answer is B: Review Information: The correct answer is A:
Drying up of secretions Elevated temperature and sweating.
Atropine dries secretions which may get in the Neuroleptic malignant syndrome (NMS) is a rare
way during the operative procedure. disorder that can occur as a side effect of antipsy-
chotic medications. It is characterized by muscu-
Question8 lar rigidity, tachycardia, hyperthermia, sweating,
A client is receiving digitalis. The nurse should altered consciousness, autonomic dysfunction,
instruct the client to report which of the following and increase in CPK. This is a life-threatening
side effects? complication.
A) Nausea, vomiting, fatigue
B) Rash, dyspnea, edema Question11
C) Polyuria, thirst, dry skin A child presents to the Emergency Department
D) Hunger, dizziness, diaphoresis with documented acetaminophen poisoning. In
order to provide counseling and education for
the parents, which principle must the nurse un-
Review Information: The correct answer is A: derstand?
Nausea, vomiting, fatigue A) The problem occurs in stages with recovery
Side effects of digitalis toxicity include fatigue, within 12-24 hours
nausea, vomiting, anorexia, and bradycardia. B) Hepatic problems may occur and may be life-
Digitalis inhibits the sodium potassium ATPase, threatening
which makes more calcium available for contrac- C) Full and rapid recovery can be expected in
tile proteins, resulting in increased cardiac out- most children
put. D) This poisoning is usually fatal, as no antidote
is available
Question9
A client is receiving dexamethasone (Decadron)
therapy. What should the nurse plan to monitor Review Information: The correct answer is B:
in this client? Hepatic problems may occur and may be life-
A) Urine output every 4 hours threatening
B) Blood glucose levels every 12 hours Clinical manifestations associated with acetami-
C) Neurological signs every 2 hours nophen poisoning occurs in 4 stages. The third
D) Oxygen saturation every 8 hours stage is hepatic involvement which may last up
to 7 days and be permanent. Clients who do not
Review Information: The correct answer is B: die in the hepatic stage gradually recover.
Blood glucose levels every 12 hours
Collected by :DeepaRajesh [ 99 ]
rajesh.ks21@gmail.com
Kuwait
Question12 Review Information: The correct answer is C:
A client has been receiving dexamethasone Mental status changes
(Decadron) for control of cerebral edema. Which Use of serotonergic agents may result in Se-
of the following assessments would indicate that rotonin Syndrome with confusion, nausea, pal-
the treatment is effective? pitations, increased muscle tone with twitching
A) A positive Babinski’s reflex muscles, and agitation. Serotonin syndrome is
B) Increased response to motor stimuli most often reported in patients taking 2 or more
C) A widening pulse pressure medications that increase CNS serotonin levels
D) Temperature of 37 degrees Celsius by different mechanisms. The most common
drug combinations associated with serotonin
syndrome involve the MAOIs, SSRIs, and the tri-
cyclic antidepressants.
Review Information: The correct answer is B:
Increased response to motor stimuli Question15
Decadron is a corticosteroid that acts on the cell A client with bi-polar disorder is taking lithium
membrane to decrease inflammatory responses (Lithane). What should the nurse emphasize
as well as stabilize the blood-brain barrier. Once when teaching about this medication?
Decadron reaches a therapeutic level, there A) Take the medication before meals
should be a decrease in symptomology with im- B) Maintain adequate daily salt intake
provement in motor skills. C) Reduce fluid intake to minimize diuresis
D) Use antacids to prevent heartburn
Question13
The provider has ordered transdermal nitroglyc-
erin patches for a client. Which of these instruc- Review Information: The correct answer is B:
tions should be included when teaching a client Maintain adequate daily salt intake
about how to use the patches? Salt intake affects fluid volume, which can affect
A) Remove the patch when swimming or bath- lithium (Lithane) levels; therefore, maintaining
ing adequate salt intake is advised.
B) Apply the patch to any non-hairy area of the
body Question16
C) Apply a second patch with chest pain A client with anemia has a new prescription for
D) Remove the patch if ankle edema occurs ferrous sulfate. In teaching the client about diet
and iron supplements, the nurse should empha-
size that absorption of iron is enhanced if taken
Review Information: The correct answer is with which substance?
B: Apply the patch to any non-hairy area of the A) Acetaminophen
body B) Orange juice
The patch application sites should be rotated. C) Low fat milk
D) An antacid
Question14
A newly admitted client has a diagnosis of depres- Review Information: The correct answer is B:
sion. She complains of “twitching muscles” and Orange juice
a “racing heart”, and states she stopped taking Ascorbic acid enhances the absorption of iron.
Zoloft a few days ago because it was not helping
her depression. Instead, she began to take her Question17
partner’s Parnate. The nurse should immediately A client with an aplastic sickle cell crisis is receiv-
assess for which of these adverse reactions? ing a blood transfusion and begins to complain of
A) Pulmonary edema “feeling hot.” Almost immediately, the client be-
B) Atrial fibrillation gins to wheeze. What is the nurse’s first action?
C) Mental status changes A) Stop the blood infusion
D) Muscle weakness B) Notify the health care provider
C) Take/record vital signs
D) Send blood samples to lab
Collected by :DeepaRajesh [ 100 ]
rajesh.ks21@gmail.com
Kuwait
symptoms resulting from complications due to
Review Information: The correct answer is A: aspirin ingestion.
Stop the blood infusion
If a reaction of any type is suspected during ad- Question20
ministration of blood products, stop the infusion The nurse is caring for a client who is receiving
immediately, keep the line open with saline, no- procainamide (Pronestyl) intravenously. It is im-
tify the health care provider, monitor vital signs portant for the nurse to monitor which of the fol-
and other changes, and then send a blood sam- lowing parameters?
ple to the lab. A) Hourly urinary output
B) Serum potassium levels
Question18 * C) Continuous EKG readings
A client confides in the RN that a friend has told D) Neurological signs
her the medication she takes for depression,
Wellbutrin, was taken off the market because
it caused seizures. What is an appropriate re- Review Information: The correct answer is C:
sponse by the nurse? Continuous EKG readings
A) “Ask your friend about the source of this infor- Procainamide (Pronestyl) is used to suppress
mation.” cardiac arrhythmias. When administered intra-
B) “Omit the next doses until you talk with the venously, it must be accompanied by continuous
doctor.” cardiac monitoring by ECG.
C) “There were problems, but the recommended
dose is changed.” Question21
D) “Your health care provider knows the best The nurse is providing education for a client with
drug for your condition.” newly diagnosed tuberculosis. Which statement
should be included in the information that is giv-
en to the client?
Review Information: The correct answer is C: A) “Isolate yourself from others until you are fin-
“There were problems, but the recommended ished taking your medication.”
dose is changed.” B) “Follow up with your primary care provider in
Wellbutrin was introduced in the U.S. in 1985 3 months.”
and then withdrawn because of the occurrence C) “Continue to take your medications even when
of seizures in some patients taking the drug. The you are feeling fine.”
drug was reintroduced in 1989 with specific rec- D) “Continue to get yearly tuberculin skin tests.”
ommendations regarding dose ranges to limit
the occurrence of seizures. The risk of seizure
appears to be strongly associated with dose. Review Information: The correct answer is C:
“Continue to take your medications even when
Question19 you are feeling fine.”
When providing discharge teaching to a client The most important piece of information the tu-
with asthma, the nurse will warn against the use berculosis client needs is to understand the im-
of which of the following over-the-counter medi- portance of medication compliance, even if no
cations? longer experiencing symptoms. Clients are most
A) Cortisone ointments for skin rashes infectious early in the course of therapy. The
B) Aspirin products for pain relief numbers of acid-fast bacilli are greatly reduced
C) Cough medications containing guaifenesin as early as 2 weeks after therapy begins.
D) Histamine blockers for gastric distress
Question22
The nurse is applying silver sulfadiazine (Sil-
Review Information: The correct answer is B: vadene) to a child with severe burns to arms
Aspirin products for pain relief and legs. Which side effect should the nurse be
Aspirin is known to induce asthma attacks. As- monitoring for?
pirin can also cause nasal polyps and rhinitis. A) Skin discoloration
Warn individuals with asthma about signs and B) Hardened eschar
Collected by :DeepaRajesh [ 101 ]
rajesh.ks21@gmail.com
Kuwait
C) Increased neutrophils Philadelphia: Saunders.
D) Urine sulfa crystals
Question25
A nurse is caring for a client who is receiving
Review Information: The correct answer is D: methyldopa hydrochloride (Aldomet) intrave-
Urine sulfa crystals nously. Which of the following assessment find-
Silver sulfadiazine is a broad spectrum anti- ings would indicate to the nurse that the client
microbial, especially effective against pseu- may be having an adverse reaction to the medi-
domonas. When applied to extensive areas, cation?
however, it may cause a transient neutropenia, A) Headache
as well as renal function changes with sulfa crys- B) Mood changes
tals production and kernicterus. C) Hyperkalemia
D) Palpitations
Question23
The nurse is monitoring a client receiving a
thrombolytic agent, alteplase (Activase tissue Review Information: The correct answer is B:
plasminogen activator), for treatment of a myo- Mood changes
cardial infarction. What outcome indicates the The nurse should assess the client for alterations
client is receiving adequate therapy within the in mental status such as mood changes. These
first hours of treatment? symptoms should be reported promptly.
A) Absence of a dysrhythmia (or arrhythmia) Deglin, J.D. and Vallerand, A.H. (2001). Davis’
B) Blood pressure reduction drug guide for nurses. (7th edition). Philadelphia:
C) Cardiac enzymes are within normal limits F.A. Davis Company.
D) Return of ST segment to baseline on ECG Wilson, B.A., Shannon, M.T., and Stang, C.L.
(2004). Nurse’s drug guide. Upper Saddle River,
New Jersey: Pearson Prentice Hall.
Review Information: The correct answer is D:
Return of ST segment to baseline on ECG Question26
Improved perfusion should result from this medi- The nurse is teaching a child and the family about
cation, along with the reduction of ST segment the medication phenytoin (Dilantin) prescribed
elevation. for seizure control. Which of the following side
effects is most likely to occur?
Question24 A) Vertigo
The provider has ordered daily high doses of B) Drowsiness
aspirin for a client with rheumatoid arthritis. The C) Gingival hyperplasia
nurse instructs the client to discontinue the medi- D) Vomiting
cation and contact the provider if which of the
following symptoms occur? Review Information: The correct answer is C:
A) Infection of the gums Gingival hyperplasia
B) Diarrhea for more than one day Swollen and tender gums occur often with use of
C) Numbness in the lower extremities phenytoin. Good oral hygiene and regular visits
D) Ringing in the ears to the dentist should be emphasized.

Question27
Review Information: The correct answer is D: The use of atropine for treatment of symptomatic
Ringing in the ears bradycardia is contraindicated for a client with
Aspirin stimulates the central nervous system which of the following conditions?
which may result in ringing in the ears. A) Urinary incontinence
Deglin, J.D. and Vallerand, A.H. (2001). Davis’ B) Glaucoma
drug guide for nurses. (7th edition). Philadelphia: C) Increased intracranial pressure
F.A. Davis Company. D) Right sided heart failure
Key, J.L. and Hayes, E.R. (2003). Pharmacol-
ogy, a nursing process approach. (4th edition).
Collected by :DeepaRajesh [ 102 ]
rajesh.ks21@gmail.com
Kuwait
Review Information: The correct answer is B: Review Information: The correct answer is B:
Glaucoma Blood pressure
Atropine is contraindicated in clients with angle- Diltiazem (Cardizem) is a calcium channel block-
closure glaucoma because it can cause pupillary er that causes systemic vasodilation resulting in
dilation with an increase in aqueous humor, lead- decreased blood pressure.
ing to a resultant increase in optic pressure.
Question31
Question28 The nurse is instructing a client with moderate
A pregnant woman is hospitalized for treatment persistent asthma on the proper method for us-
of pregnancy induced hypertension (PIH) in the ing MDIs (multi-dose inhalers). Which medica-
third trimester. She is receiving magnesium sul- tion should be administered first?
fate intravenously. The nurse understands that A) Steroid
this medication is used mainly for what pur- B) Anticholinergic
pose? C) Mast cell stabilizer
A) Maintain normal blood pressure D) Beta agonist
B) Prevent convulsive seizures
C) Decrease the respiratory rate
D) Increase uterine blood flow Review Information: The correct answer is D:
Beta agonist
The beta-agonist drugs help to relieve bronchos-
Review Information: The correct answer is B: pasm by relaxing the smooth muscle of the air-
Prevent convulsive seizures way. These drugs should be taken first so that
Magnesium sulfate is a central nervous system other medications can reach the lungs.
depressant. While it has many systemic effects,
it is used in the client with pregnancy induced Question32
hypertension (PIH) to prevent seizures. A post-operative client has a prescription for
acetaminophen with codeine. What should the
nurse recognizes as a primary effect of this com-
Question29 bination?
The nurse is teaching a group of women in a com- A) Enhanced pain relief
munity clinic about prevention of osteoporosis. B) Minimized side effects
Which of the following over-the-counter medica- C) Prevention of drug tolerance
tions should the nurse recognize as having the D) Increased onset of action
most elemental calcium per tablet?
A) Calcium chloride
B) Calcium citrate Review Information: The correct answer is A:
C) Calcium gluconate Enhanced pain relief
D) Calcium carbonate Combination of analgesics with different mecha-
nisms of action can afford greater pain relief.

Review Information: The correct answer is D:


Calcium carbonate Question33
Calcium carbonate contains 400mg of elemental A client is receiving erythromycin 500mg IV eve-
calcium in 1 gram of calcium carbonate. ry 6 hours to treat a pneumonia. Which of the
following is the most common side effect of the
Question30 medication?
The nurse is administering diltiazem (Cardizem) A) Blurred vision
to a client. Prior to administration, it is important B) Nausea and vomiting
for the nurse to assess which parameter? C) Severe headache
A) Temperature D) Insomnia
B) Blood pressure
C) Vision
D) Bowel sounds Review Information: The correct answer is B:
Collected by :DeepaRajesh [ 103 ]
rajesh.ks21@gmail.com
Kuwait
Nausea and vomiting C) Lung sounds
Nausea is a common side-effect of erythromycin D) Skin turgor
in both oral and intravenous forms.

Question34 Review Information: The correct answer is B:


The health care provider orders an IV aminophyl- Weight
line infusion at 30 mg/hr. The pharmacy sends a Check the client’’s weight because dosage is
1,000 ml bag of D5W containing 500 mg of ami- calculated on the basis of weight.
nophylline. In order to administer 30 mg per hour,
the RN will set the infusion rate at: Question37
A) 20 ml per hour In providing care for a client with pain from a sick-
B) 30 ml per hour le cell crisis, which one of the following medica-
C) 50 ml per hour tion orders for pain control should be questioned
D) 60 ml per hour by the nurse?
A) Demerol
B) Morphine
Review Information: The correct answer is D: C) Methadone
60 ml per hour D) Codeine
Using the ratio method to calculate infusion rate:
mg to be given (30) : ml to be infused (X) :: mg Review Information: The correct answer is A:
available (500) : ml of solution (1,000). Solve for Demerol
X by cross-multiplying: 30 x 1,000 = 500 x X (or Meperidine is not recommended in clients with
cancel), 30,000 = 500 X, X = 30,000/500, X = sickle cell disease. Normeperidine, a metabolite
60ml per hour. of meperidine, is a central nervous system stim-
ulant that produces anxiety, tremors, myoclonus,
Question35 and generalized seizures when it accumulates
The nurse is assessing a 7 year-old after several with repetitive dosing. Clients with sickle cell dis-
days of treatment for a documented strep throat. ease are particularly at risk for normeperidine-
Which of the following statements suggests that induced seizures.
further teaching is needed?
A) “Sometimes I take my medicine with fruit Question38
juice.” A 5 year-old has been rushed to the emergency
B) “My mother makes me take my medicine right room several hours after acetaminophen poison-
after school.” ing. Which laboratory result should receive atten-
C) “Sometimes I take the pills in the morning and tion by the nurse?
other times at night.” A) Sedimentation rate
D) “I am feeling much better than I did last B) Profile 2
week.” C) Bilirubin
D) Neutrophils

Review Information: The correct answer is C:


“Sometimes I take the pills in the morning and Review Information: The correct answer is C:
other times at night.” Bilirubin
Inconsistency in taking the prescribed medica- Bilirubin, along with liver enzymes ALT and AST,
tion indicates more teaching is needed. may rise in the second stage (1-3 days) after a
significant overdose, indicating cellular necrosis
Question36 and liver dysfunction. A prolonged prothrombin
The nurse is caring for a 10 year-old client who time may also be found.
will be placed on heparin therapy. Which assess-
ment is critical for the nurse to make before initi- Question39
ating therapy An elderly client is on an anticholinergic metered
A) Vital signs dose inhaler (MDI) for chronic obstructive pulmo-
B) Weight nary disease. The nurse would suggest a spacer
Collected by :DeepaRajesh [ 104 ]
rajesh.ks21@gmail.com
Kuwait
to
A) enhance the administration of the medication
B) increase client compliance Review Information: The correct answer is B:
C) improve aerosol delivery in clients who are Completing the full course of medications
not able to coordinate the MDI In order for antibiotic therapy to be effective in
D) prevent exacerbation of COPD eradicating an infection, the client must compete
the entire course of prescribed therapy. When
findings subside, stopping the medication early
Review Information: The correct answer is C: may lead to recurrence or subsequent drug re-
improve aerosol delivery in clients who are not sistance.
able to coordinate the MDI
Spacers improve the medication delivery in cli-
ents who are unable to coordinate the move- Question2
ments of administering a dose with an MDI. A 72 year-old client is admitted for possible de-
hydration. The nurse knows that older adults are
particularly at risk for dehydration because they
Question40 have
The nurse is teaching a parent how to adminis- A) an increased need for extravascular fluid
ter oral iron supplements to a 2 year-old child. B) a decreased sensation of thirst
Which of the following interventions should be C) an increase in diaphoresis
included in the teaching? D) higher metabolic demands
A) Stop the medication if the stools become tarry
green
B) Give the medicine with orange juice and Review Information: The correct answer is B:
through a straw a decreased sensation of thirst
C) Add the medicine to a bottle of formula The elderly have a reduction in thirst sensation
D) Administer the iron with your child’s meals causing them to consume less fluid. Other risk
factors may include fear of incontinence, inability
to drink fluids independently and lack of motiva-
Review Information: The correct answer is B: tion.
Give the medicine with orange juice and through
a straw
Absorption of iron is facilitated in an environment Question3
rich in Vitamin C. Since liquid iron preparation A male client is admitted with a spinal cord injury
will stain teeth, a straw is preferred. at level C4. The client asks the nurse how the
injury is going to affect his sexual function. The
0 comments nurse would respond
A) “Normal sexual function is not possible.”
Labels: free nclex-rn sample review questions, B) “Sexual functioning will not be impaired at
nclex-rn practice test questions, nursing review all.”
C) “Erections will be possible.”
Free NCLEX-RN Sample Test Ques- D) “Ejaculation will be normal.”
tions For Nursing Review (Pharmacol-
ogy Set 1)

Question1
Review Information: The correct answer is C:
A client has an order for antibiotic therapy after
“Erections will be possible.”
hospital treatment of a staph infection. Which of
Because they are a reflex reaction, erections can
the following should the nurse emphasize?
be stimulated by stroking the genitalia.
A) Scheduling follow-up blood cultures
B) Completing the full course of medications
C) Visiting the provider in a few weeks
D) Monitoring for signs of recurrent infection Question4
Collected by :DeepaRajesh [ 105 ]
rajesh.ks21@gmail.com
Kuwait
An 82 year-old client complains of chronic con- turnover.
stipation. To improve bowel function, the nurse
should first suggest Question7
A) Increasing fiber intake to 20-30 grams daily You are caring for a client with deep vein throm-
B) Daily use of laxatives bosis who is on Heparin IV. The latest APTT is 50
C) Avoidance of binding foods such as cheese seconds. If the laboratory normal range is 16-24
and chocolate seconds, you would anticipate
D) Monitoring a balance between activity and A) maintaining the current heparin dose
rest B) increasing the heparin as it does not appear
therapeutic.
Review Information: The correct answer is A: C) giving protamine sulfate as an antidote.
Increasing fiber intake to 20-30 grams daily D) repeating the blood test 1 hour after giving
The incorporation of high fiber into the diet is an heparin.
effective way to promote bowel elimination in the
elderly.
Review Information: The correct answer is A:
Question5 maintaining the current heparin dose
A 4 year-old child is admitted with burns on his The range for a therapeutic APTT is 1.5-2 times
legs and lower abdomen. When assessing the the control. Therefore the client is receiving a
child’s hydration status, which of the following in- therapeutic dose of Heparin.
dicates a less than adequate fluid replacement?
A) Decreasing hematocrit and increasing urine
volume Question8
B) Rising hematocrit and decreasing urine vol- A client is admitted with a diagnosis of nodal
ume bigeminy. The nurse knows that the atrioven-
C) Falling hematocrit and decreasing urine vol- tricular (AV) node has an intrinsic rate of
ume A) 60-100 beats/minute
D) Stable hematocrit and increasing urine vol- B) 10-30 beats/minute
ume C) 40-70 beats/minute
D) 20-50 beats/minute

Review Information: The correct answer is B:


Rising hematocrit and decreasing urine volume Review Information: The correct answer is C:
A rising hematocrit indicates a decreased total 40-70 beats/minute
blood volume, a finding consistent with dehydra- The intrinsic rate of the AV node is within the
tion. range of 40-70 beats per minute.

Question6 Question9
A client receiving chemotherapy has developed A client is to receive 3 doses of potassium chlo-
sores in his mouth. He asks the nurse why this ride 10 mEq in 100cc normal saline to infuse
happened. What is the nurse’s best response? over 30 minutes each. Which of the following is a
A) “It is a sign that the medication is working.” priority assessment to perform before giving this
B) “You need to have better oral hygiene.” medication?
C) “The cells in the mouth are sensitive to the A) Oral fluid intake
chemotherapy.” B) Bowel sounds
D) “This always happens with chemotherapy.” C) Grip strength
D) Urine output

Review Information: The correct answer is


C: “The cells in the mouth are sensitive to the Review Information: The correct answer is D:
chemotherapy.” Urine output
The epithelial cells in the mouth are very sensi- Potassium chloride should only be administered
tive to chemotherapy due to their high rate of cell after adequate urine output (>20cc/hour for 2
Collected by :DeepaRajesh [ 106 ]
rajesh.ks21@gmail.com
Kuwait
consecutive hours) has been established. Im-
paired ability to excrete potassium via the kid-
neys can result in hyperkalemia. Review Information: The correct answer is C:
Deeply for 3-4 seconds
The client should be instructed to breath in the
Question10 medication for 3-4 seconds in order to receive
The unlicensed assistive personnel (UAP) re- the correct dosage of medication.
ports to the nurse that a client with cirrhosis who
had a paracentesis yesterday has become more
lethargic and has musty smelling breath. A criti- Question13
cal assessment for increasing encephalopathy The nurse is caring for clients over the age of 70.
is The nurse knows that due to age-related chang-
A) monitor the client’s clotting status es, the elderly clients tolerate diets that are
B) assess upper abdomen for bruits A) high protein
C) assess for flap-like tremors of the hands B) high carbohydrates
D) measure abdominal girth changes C) low fat
D) high calories

Review Information: The correct answer is C:


assess for flap-like tremors of the hands Review Information: The correct answer is C:
A client with cirrhosis of the liver who develops low fat
subtle changes in mental status and has a musty Due to age related changes, the diet of the eld-
odor to the breath is at risk for developing more erly should include a lower quantity and higher
advanced signs of encephalopathy. quality of food. Fewer carbohydrates and fats
are required in their diets.

Question11
A client is scheduled for an intravenous pyelo- Question14
gram (IVP). After the contrast material is inject- A woman with a 28 week pregnancy is on the
ed, which of the following client reactions should way to the emergency department by ambulance
be reported immediately? with a tentative diagnosis of abruptio placenta.
A) Feeling warm Which should the nurse do first when the woman
B) Face flushing arrives?
C) Salty taste A) administer oxygen by mask at 100%
D) Hives B) start a second IV with an 18 gauge cannula
C) check fetal heart rate every 15 minutes
D) insert urethral catheter with hourly urine out-
Review Information: The correct answer is D: puts
Hives
This is a sign of anaphylaxis and should be re-
ported immediately. The other reactions are con- Review Information: The correct answer is A:
sidered normal and the client should be informed administer oxygen by mask at 100%
that they may occur. Administering oxygen in this situation would in-
. crease the circulating oxygen in the mother’s cir-
culation to the fetus’s circulation. This action will
Question12 minimize complications.
A client is prescribed an inhaler. How should the
nurse instruct the client to breathe in the medica-
tion? Question15
A) As quickly as possible A client in respiratory distress is admitted with
B) As slowly as possible arterial blood gas results of: PH 7.30; PO2 58,
C) Deeply for 3-4 seconds PCO2 34; and HCO3 19. The nurse determines
D) Until hearing whistling by the spacer that the client is in
Collected by :DeepaRajesh [ 107 ]
rajesh.ks21@gmail.com
Kuwait
A) metabolic acidosis should the nurse base the response on?
B) metabolic alkalosis A) Testicular cancer has a cure rate of 90% with
C) respiratory acidosis early diagnosis
D) respiratory alkalosis B) Testicular cancer has a cure rate of 50% with
early diagnosis
C) Intensive chemotherapy is the treatment of
Review Information: The correct answer is A: choice
metabolic acidosis D) Testicular cancer is usually fatal
These lab values indicate metabolic acidosis:
the PH is low, PCO2 is normal, and bicarbonate
level is low. Review Information: The correct answer is
A: Testicular cancer has a cure rate of 90% with
Question16 early diagnosis
A client is diagnosed with gastroesophageal re- With aggressive treatment and early detection/
flux disease (GERD). The nurse’s instruction to diagnosis the cure rate is 90%.
the client regarding diet should be to
A) avoid all raw fruits and vegetables Question19
B) increase intake of milk products A client newly diagnosed with Type I Diabetes
C) decrease intake of fatty foods Mellitus asks the purpose of the test measur-
D) focus on 3 average size meals a day ing glycosylated hemoglobin. The nurse should
explain that the purpose of this test is to deter-
mine:
Review Information: The correct answer is C: A) The presence of anemia often associated with
decrease intake of fatty foods Diabetes
GERD may be aggravated by a fatty diet. A diet B) The oxygen carrying capacity of the client’s
low in fat would decrease the symptoms of GERD. red cells
Other agents which should also be decreased or C) The average blood glucose for the past 2-3
avoided are: cigarette smoking, caffeine, alco- months
hol, chocolate, and meperidine (Demerol). D) The client’s risk for cardiac complications

Question17 Review Information: The correct answer is


After surgery, a client with a nasogastric tube C: The average blood glucose for the past 2-3
complains of nausea. What action would the months
nurse take? By testing the portion of the hemoglobin that ab-
A) Call the health care provider sorbs glucose, it is possible to determine the av-
B) Administer an antiemetic erage blood glucose over the life span of the red
C) Put the bed in Fowler’s position cell, 120 days.
D) Check the patency of the tube

Question20
Review Information: The correct answer is D: A client is admitted for a possible pacemaker in-
Check the patency of the tube sertion. What is the intrinsic rate of the heart’s
An indication that the nasogastric tube is ob- own pacemaker?
structed is a client’s complaint of nausea. Na- A) 30-50 beats/minute
sogastric tubes may become obstructed with B) 60-100 beats/minute
mucus or sediment. C) 20-60 beats/minute
D) 90-100 beats/minute

Question18
A client with testicular cancer has had an orchiec- Review Information: The correct answer is B:
tomy. Prior to discharge the client expresses his 60-100 beats/minute
fears related to his prognosis. Which principle This is the intrinsic rate of the SA node.
Collected by :DeepaRajesh [ 108 ]
rajesh.ks21@gmail.com
Kuwait
Question21 nated and the client’s depth and rate of respi-
The nurse discusses nutrition with a pregnant rations will decrease. Therefore the first action
woman who is iron deficient and follows a veg- should be to lower the oxygen rate.
etarian diet. The selection of which foods indi-
cates the woman has learned sources of iron?
A) Cereal and dried fruits Question24
B) Whole grains and yellow vegetables The client with goiter is treated with potassium
C) Leafy green vegetables and oranges iodide preoperatively. What should the nurse
D) Fish and dairy products recognize as the purpose of this medication?
A) Reduce vascularity of the thyroid
B) Correct chronic hyperthyroidism
Review Information: The correct answer is A: C) Destroy the thyroid gland function
Cereal and dried fruits D) Balance enzymes and electrolytes
Both of these foods would be a good source of
iron. Review Information: The correct answer is A:
Reduce vascularity of the thyroid
Potassium iodide solution, or Lugol’’s solution
Question22 may be used preoperatively to reduce the size
Prior to administering Alteplase (TPA) to a client and vascularity of the thyroid gland.
admitted for a cerebral vascular accident (CVA),
it is critical that the nurse assess:
A) Neuro signs Question25
B) Mental status One hour before the first treatment is scheduled,
C) Blood pressure the client becomes anxious and states he does
D) PT/PTT not wish to go through with electroconvulsive
therapy. Which response by the nurse is most
appropriate?
Review Information: The correct answer is D: A) “I’ll go with you and will be there with you dur-
PT/PTT ing the treatment.”
TPA is a potent thrombolytic enzyme. Because B) “You’ll be asleep and won’t remember any-
bleeding is the most common side effect, it is thing.”
most essential to evaluate clotting studies in- C) “You have the right to change your mind. You
cluding PT, PTT, APTT, platelets, and hematocrit seem anxious. Can we talk about it?”
before beginning therapy. D) “I’ll call the health care provider to notify them
of your decision.”

Question23
The nurse enters the room of a client diagnosed Review Information: The correct answer is
with COPD. The client’s skin is pink, and respi- C: “You have the right to change your mind. You
rations are 8 per minute. The client’s oxygen is seem anxious. Can we talk about it?”
running at 6 liters per minute. What should be This response indicates acknowledgment of the
the nurse’s first action? client’s rights and the opportunity for the client
A) Call the health care provider to clarify and ventilate concerns. After this, if the
B) Put the client in Fowler’s position client continues to refuse, the provider should be
C) Lower the oxygen rate notified.
D) Take the vital signs

Question26
Review Information: The correct answer is C: A nurse who has been named in a lawsuit can
Lower the oxygen rate use which of these factors for the best protection
In client’s diagnosed with COPD, the drive to in a court of law?
breathe is hypoxia. If oxygen is delivered at too A) Clinical specialty certification in the associ-
high of a concentration, this drive will be elimi- ated area of practice
Collected by :DeepaRajesh [ 109 ]
rajesh.ks21@gmail.com
Kuwait
B) Documentation on the specific client record C) Avoid the use of salt substitutes
with a focus on the nursing process D) Take the medication with meals
C) Yearly evaluations and proficiency reports
prepared by nurse’s manager
D) Verification of provider’s orders for the plan of Review Information: The correct answer is C:
care with identification of outcomes Avoid the use of salt substitutes
Captopril can cause an accumulation of potas-
sium or hyperkalemia. Clients should avoid the
Review Information: The correct answer is B: use of salt substitutes, which are generally po-
Documentation on the specific client record with tassium-based.
a focus on the nursing process
Documentation is the key to protect nurses when
a lawsuit is filed. The thorough documentation Question29
should include all steps of the nursing process – A client has bilateral knee pain from osteoarthri-
assessment, analysis, plan, intervention, evalu- tis. In addition to taking the prescribed non-ster-
ation. In addition, it should include pertinent data oidal anti-inflammatory drug (NSAID), the nurse
such as times, dosages and sites of actions, should instruct the client to
assessment data, the nurse’s response to a A) start a regular exercise program
change in the client’s condition, specific actions B) rest the knees as much as possible to de-
taken, if and when the notification occurred to crease inflammation
the provider or other health care team members, C) avoid foods high in citric acid
and what was prescribed along with the client’s D) keep the legs elevated when sitting
outcomes.

Review Information: The correct answer is A:


Question27 start a regular exercise program
The nurse is caring for clients over the age of 70. A regular exercise program is beneficial in treat-
The nurse is aware that when giving medications ing osteoarthritis. It can restore self-esteem and
to older clients, it is best to improve physical functioning.
A) start low, go slow
B) avoid stopping a medication entirely
C) avoid drugs with side effects that impact cog- Question30
nition An arterial blood gases test (ABG) is ordered
D) review the drug regimen yearly for a confused client. The respiratory therapist
draws the blood and then asks the nurse to apply
pressure to the area so the therapist can take the
Review Information: The correct answer is A: specimen to the lab. How long should the nurse
start low, go slow apply pressure to the area?
Due to physiological changes in the elderly, as A) 3 minutes
well as conditions such as dehydration, hyper- B) 5 minutes
thermia, immobility and liver disease, the effec- C) 8 minutes
tive metabolism of drugs may decrease. As a D) 10 minutes
result, drugs can accumulate to toxic levels and
cause serious adverse reactions.
Review Information: The correct answer is B:
5 minutes
Question28 It is necessary to apply pressure to the area for
You are caring for a hypertensive client with a 5 minutes to prevent bleeding and the formation
new order for captopril (Capoten). Which infor- of hematomas.
mation should the nurse include in client teach-
ing?
A) Avoid green leafy vegetables Question31
B) Restrict fluids to 1000cc/day Which of these clients should the charge nurse
Collected by :DeepaRajesh [ 110 ]
rajesh.ks21@gmail.com
Kuwait
assign to the registered nurse (RN)? ordered oxygen. The nurse knows that the major
A) A 56 year-old with atrial fibrillation receiving reason that oxygen is administered in this situa-
digoxin tion is to
B) A 60 year-old client with COPD on oxygen at A) saturate the red blood cells
2 L/min B) relieve dyspnea
C) A 24 year-old post-op client with type 1 diabe- C) decrease cyanosis
tes in the process of discharge D) increase oxygen level in the myocardium
D) An 80 year-old client recovering 24 hours post
right hip replacement
Review Information: The correct answer is D:
Review Information: The correct answer is C: increase oxygen level in the myocardium
A 24 year-old post-op client with type 1 diabetes Anoxia of the myocardium occurs in myocardial
in the process of discharge infarction. Oxygen administration will help re-
Discharge teaching must be done by an RN. lieve dyspnea and cyanosis associated with the
Practical nurses (PNs) or unlicensed assistive condition but the major purpose is to increase
personnel (UAPs) can reinforce education after the oxygen concentration in the damaged myo-
the RN does the initial teaching. cardial tissue.
.
Question32 Question35
A hypertensive client is started on atenolol (Ten- The nurse is teaching a client with chronic re-
ormin). The nurse instructs the client to immedi- nal failure (CRF) about medications. The client
ately report which of these findings? questions the purpose of aluminum hydroxide
A) Rapid breathing (Amphojel) in her medication regimen. What is
B) Slow, bounding pulse the best explanation for the nurse to give the cli-
C) Jaundiced sclera ent about the therapeutic effects of this medica-
D) Weight gain tion?
A) It decreases serum phosphate
Review Information: The correct answer is B: B) It will reduce serum calcium
Slow, bounding pulse C) Amphojel increases urine output
Atenolol (Tenormin) is a beta-blocker that can D) The drug is taken to control gastric acid se-
cause side effects including bradycardia and hy- cretion
potension.

Question33 Review Information: The correct answer is A:


An 80 year-old client is admitted with a diagnosis It decreases serum phosphate
of malnutrition. In addition to physical assess- Aluminum binds phosphates that tend to accu-
ments, which of the following lab tests should be mulate in the patient with chronic renal failure
closely monitored? due to decreased filtration capacity of the kidney.
A) Urine protein Antacids such as Amphojel are commonly used
B) Urine creatinine to accomplish this.
C) Serum calcium
D) Serum albumin
Question36
Review Information: The correct answer is D: A 66 year-old client is admitted for mitral valve
Serum albumin replacement surgery. The client has a history
Serum albumin is a valuable indicator of protein of mitral valve regurgitation and mitral stenosis
deficiency and, later, nutritional status in adults. since her teenage years. During the admission
A normal reading for an elder’s serum albumin is assessment, the nurse should ask the client if as
between 3.0-5.0 g/dl. a child she had
A) measles
Question34 B) rheumatic fever
Upon admission to an intensive care unit, a client C) hay fever
diagnosed with an acute myocardial infarction is D) encephalitis
Collected by :DeepaRajesh [ 111 ]
rajesh.ks21@gmail.com
Kuwait
Review Information: The correct answer is B: A) Assess the pulse rate q 4 hours
rheumatic fever B) Monitor her level of consciousness q shift
Clients that present with mitral stenosis often C) Test her stools for occult blood
have a history of rheumatic fever or bacterial en- D) Discuss fiber in the diet to prevent constipa-
docarditis. tion

Question37 Review Information: The correct answer is C:


During nursing rounds which of these assess- Test her stools for occult blood
ments would require immediate corrective action Both Prednisone and ASA can lead to GI bleed-
and further instruction to the practical nurse (PN) ing, therefore monitoring for occult blood would
about proper care? be appropriate.
A) The weights of the skin traction of a client are
hanging about 2 inches from the floor Question40
B) A client with a hip prosthesis 1 day post op- A client with testicular cancer is scheduled for a
eratively is lying in bed with internal rotation and right orchiectomy. The nurse knows that an or-
adduction of the affected leg chiectomy is the
C) The nurse observes that the PN moves the A) surgical removal of the entire scrotum
extremity of a client with an external fixation de- B) surgical removal of a testicle
vice by picking up the frame C) dissection of related lymph nodes
D) A client with skeletal traction states “The other D) partial surgical removal of the penis
nurse said that the clear, yellow and crusty drain-
age around the pin site is a good sign”
Review Information: The correct answer is B:
Review Information: The correct answer is B: surgical removal of a testicle
A client with a hip prosthesis 1 day post opera- The affected testicle is surgically removed along
tively is lying in bed with internal rotation and ad- with its tunica and spermatic cord.
duction of the affected leg
This position should be prevented in order to 0 comments
prevent dislodgment of the hip prosthesis, es-
pecially in the first 48 to 72 hours post-op. The Labels: free nclex-rn sample review questions,
other assessments are not of concern. nclex-rn practice test questions, nursing review

Question38 Free NCLEX-RN Sample Test Ques-


A client diagnosed with gouty arthritis is admit- tions For Nursing Review (Part 5)
ted with severe pain and edema in the right foot.
When the nurse develops a plan of care, which Question1
intervention should be included? A client complains of some discomfort after a
A) high protein diet below the knee amputation. Which action by the
B) salicylates nurse is most appropriate initially?
C) hot compresses to affected joints A) Conduct guided imagery or distraction
D) intake of at least 3000cc/day B) Ensure that the stump is elevated the first day
post-op
C) Wrap the stump snugly in an elastic bandage
Review Information: The correct answer is D: D) Administer opioid narcotics as ordered
intake of at least 3000cc/day
Fluid intake should be increased to prevent pre- Review Information: The correct answer is B:
cipitation of urate in the kidneys. Ensure that the stump is elevated the first day
post-op
Question39 This priority intervention prevents pressure
A 55 year-old woman is taking Prednisone and caused by pooling of blood, thus minimizing the
aspirin (ASA) as part of her treatment for rheu- pain. Without this measure, a firm elastic band-
matoid arthritis. Which of the following would be age, opioid narcotics, or guided imagery will
an appropriate intervention for the nurse? have little effect. Opioid narcotics are given for
Collected by :DeepaRajesh [ 112 ]
rajesh.ks21@gmail.com
Kuwait
severe pain. ing
D) Avoid touching the neonate with cold hands
Question2
A 78 year-old client with pneumonia has a pro- Review Information: The correct answer is B:
ductive cough, but is confused. Safety protective Monitor the neonate’s temperature
devices (restraints) have been ordered for this When using a warming device the neonate’s
client. How can the nurse prevent aspiration? temperature should be continuously monitored
A) Suction the client frequently while restrained for undesired elevations. The use of heat lamps
B) Secure all 4 restraints to 1 side of bed is not safe as there is no way to regulate their
C) Obtain a sitter for the client while restrained temperature. Warming medications and fluids is
D) Request an order for a cough suppressant not indicated. While touching with cold hands can
startle the infant it does not pose a safety risk.
Review Information: The correct answer is C:
Obtain a sitter for the client while restrained Question5
The plan to use safety devices (restraints) should Which oxygen delivery system would the nurse
be rethought. Restraints are used to protect the apply that would provide the highest concentra-
client from harm caused by removing tubes or tions of oxygen to the client?
getting out of bed. In the event that this restricted A) Venturi mask
movement could cause more harm, such as as- B) Partial rebreather mask
piration, then a sitter should be requested. These C) Non-rebreather mask
are to be provided by the facility in the event the D) Simple face mask
family cannot do so. This client needs to cough
and be watched rather than restricted. Suction- Review Information: The correct answer is C:
ing will not prevent aspiration in this situation. Non-rebreather mask
Cough suppressants should be avoided for this The non-rebreather mask has a one-way valve
client. that prevents exhales air from entering the res-
ervoir bag and one or more valves covering the
Question3 air holes on the face mask itself to prevent inha-
A couple trying to conceive asks the nurse when lation of room air but to allow exhalation of air.
ovulation occurs. The woman reports a regular When a tight seal is achieved around the mask
32 day cycle. Which response by the nurse is up to 100% of the oxygen is available.
correct?
A) Days 7-10 Question6
B) Days 10-13 At a senior citizens meeting a nurse talks with a
C) Days 14-16 client who has Type 1 diabetes mellitus. Which
D) Days 17-19 statement by the client during the conversation
is most predictive of a potential for impaired skin
Review Information: The correct answer is D: integrity?
Days 17-19 A) “I give my insulin to myself in my thighs.”
Ovulation occurs 14 days prior to menses. Con- B) “Sometimes when I put my shoes on I don’t
sidering that the woman’’s cycle is 32 days, sub- know where my toes are.”
tracting 14 from 32 suggests ovulation is at about C) “Here are my up and down glucose readings
the 18th day. that I wrote on my calendar.”
D) “If I bathe more than once a week my skin
Question4 feels too dry.”
A newborn is having difficulty maintaining a tem-
perature above 98 degrees Fahrenheit and has Review Information: The correct answer is
been placed in an incubator. Which action is a B: “Sometimes when I put my shoes on I don’’t
nursing priority? know where my toes are.”
A) Protect the eyes of the neonate from the heat Peripheral neuropathy can lead to lack of sensa-
lamp tion in the lower extremities. Clients who do not
B) Monitor the neonate’s temperature feel pressure and/or pain are at high risk for skin
C) Warm all medications and liquids before giv- impairment.
Collected by :DeepaRajesh [ 113 ]
rajesh.ks21@gmail.com
Kuwait
Question7 60 year-old with herpes simplex
A client returns from surgery after an open reduc- Clients who have herpes simplex infections must
tion of a femur fracture. There is a small blood- have contact precautions in addition to standard
stain on the cast. Four hours later, the nurse ob- precautions because of the associated, potential-
serves that the stain has doubled in size. What is ly weeping, skin lesions. Contact precautions are
the best action for the nurse to take? used for clients who are infected by microorgan-
A) Call the health care provider isms that are transmitted by direct contact with
B) Access the site by cutting a window in the the client, including hand or skin-to-skin contact.
cast
C) Simply record the findings in the nurse’s notes Question10
only Which of the following situations is most likely to
D) Outline the spot with a pencil and note the produce sepsis in the neonate?
time and date on the cast A) Maternal diabetes
B) Prolonged rupture of membranes
C) Cesarean delivery
Review Information: The correct answer is D: D) Precipitous vaginal birth
Outline the spot with a pencil and note the time
and date on the cast Review Information: The correct answer is B:
This is a good way to assess the amount of bleed- Prolonged rupture of membranes
ing over a period of time. The bleeding does not Premature rupture of the membranes (PROM) is
appear to be excessive and some bleeding is a leading cause of newborn sepsis. After 12-24
expected with this type of surgery. The bleed- hours of leaking fluid, measures are taken to re-
ing should also be documented in the nurse’s duce the risk to mother and the fetus/newborn.
notes.
Question8 Question11
The nurse is caring for a 1 year-old child who The nurse is teaching a parent about side effects
has 6 teeth. What is the best way for the nurse to of routine immunizations. Which of the following
give mouth care to this child? must be reported immediately?
A) Using a moist soft brush or cloth to clean teeth A) Irritability
and gums B) Slight edema at site
B) Swabbing teeth and gums with flavored C) Local tenderness
mouthwash D) Seizure activity
C) Offering a bottle of water for the child to drink
D) Brushing with toothpaste and flossing each Review Information: The correct answer is D:
tooth Seizure activity
Other reactions that should be reported include
Review Information: The correct answer is A: crying for >3 hours, temperature over 104.8 de-
Using a moist soft brush or cloth to clean teeth grees Fahrenheit following DPT immunization,
and gums and tender, swollen, reddened areas.
The nurse should use a soft cloth or soft brush to
do mouth care so that the child can adjust to the
routine of cleaning the mouth and teeth.
Question12
The nurse is at the community center speaking
with retired people about glaucoma. Which com-
Question9 ment by one of the retirees would the nurse sup-
In addition to standard precautions, a nurse port to reinforce correct information?
should implement contact precautions for which A) “I usually avoid driving at night since lights
client? sometimes seem to make things blur.”
A) 60 year-old with herpes simplex B) “I take half of the usual dose for my sinuses to
B) 6 year-old with mononucleosis maintain my blood pressure.”
C) 45 year-old with pneumonia C) “I have to sit at the side of the pool with the
D) 3 year-old with scarlet fever grandchildren since I can’t swim with this eye
problem.”
Review Information: The correct answer is A: D) “I take extra fiber and drink lots of water to
Collected by :DeepaRajesh [ 114 ]
rajesh.ks21@gmail.com
Kuwait
avoid getting constipated.” ing old ones will ensure that the tracheostomy
is secured during the entire cleaning procedure.
Review Information: The correct answer is D: The obturator is useful to keep the airway open
“I take extra fiber and drink lots of water to avoid only after the tracheostomy outer tube is coughed
getting constipated.” out. A second nurse is not needed. Changing the
Any activity that involves straining should be position may not prevent a dislodged tracheos-
avoided in clients with glaucoma. Such activities tomy.
would increase intraocular pressure.
Question15
Question13 A 4 year-old hospitalized child begins to have a
A newborn has hyperbilirubinemia and is un- seizure while playing with hard plastic toys in the
dergoing phototherapy with a fiberoptic blanket. hallway. Of the following nursing actions, which
Which safety measure is most important during one should the nurse do first?
this process? A) Place the child in the nearest bed
A) Regulate the neonate’s temperature using a B) Administer IV medication to slow down the
radiant heater seizure
B) Withhold feedings while under the photother- C) Place a padded tongue blade in the child’s
apy mouth
C) Provide water feedings at least every 2 hours D) Remove the child’s toys from the immediate
D) Protect the eyes of neonate from the photo- area
therapy lights
Review Information: The correct answer is
D: Remove the child’’s toys from the immediate
Review Information: The correct answer is C: area
Provide water feedings at least every 2 hours Nursing care for a child having a seizure in-
Protecting the eyes of the neonates is very im- cludes, maintaining airway patency, ensuring
portant to prevent damage when under the ultra- safety, administering medications, and providing
violet lights, but since the blanket is used, extra emotional support. Since the seizure has already
protection of the eyes is unnecessary. It is rec- started, nothing should be forced into the child’’s
ommended that the neonate remain under the mouth and the child should not be moved. Of the
lights for extended periods. The neonate’s skin is choices given, the first priority would be to pro-
exposed to the light and the temperature is moni- vide a safe environment.
tored, but a heater may not be necessary. There
is no reason to withhold feedings. Frequent wa- Question16
ter or feedings are given to help with the excre- The nurse is teaching home care to the parents
tion of the bilirubin in the stool. of a child with acute spasmodic croup. The most
. important aspects of this care is/are
Question14 A) sedation as needed to prevent exhaustion
A nurse is performing the routine daily cleaning B) antibiotic therapy for 10 to 14 days
of a tracheostomy. During the procedure, the cli- C) humidified air and increased oral fluids
ent coughs and displaces the tracheostomy tube. D) antihistamines to decrease allergic response
This negative outcome could have avoided by
A) placing an obturator at the client’s bedside Review Information: The correct answer is C:
B) having another nurse assist with the proce- humidified air and increased oral fluids
dure The most important aspects of home care for a
C) fastening clean tracheostomy ties before re- child with acute spasmodic croup are humidified
moving old ties air and increased oral fluids. Moisture soothes
D) placing the client in a flat, supine position inflamed membranes. Adequate systemic hydra-
tion aids is mucociliary clearance and keeps se-
Review Information: The correct answer is C: cretions thin, white, watery, and easily removed
fastening clean tracheostomy ties before remov- with minimal coughing.
ing old ties
Fastening clean tracheostomy ties before remov-
Collected by :DeepaRajesh [ 115 ]
rajesh.ks21@gmail.com
Kuwait
Question17
The nurse is assigned to care for a client who
has a leaking intracranial aneurysm. To minimize Question19
the risk of rebleeding, the nurse should plan to A newborn delivered at home without a birth at-
A) restrict visitors to immediate family tendant is admitted to the hospital for observa-
B) avoid arousal of the client except for family tion. The initial temperature is 95 degrees Fahr-
visits enheit (35 degrees Celsius) axillary. The nurse
C) keep client’s hips flexed at no less than 90 recognizes that cold stress may lead to what
degrees complication?
D) apply a warming blanket for temperatures of A) Lowered BMR
98 degrees Fahrenheit or less B) Reduced PaO2
C) Lethargy
Review Information: The correct answer is A: D) Metabolic alkalosis
restrict visitors to immediate family
Maintaining a quiet environment will assist in Review Information: The correct answer is B:
minimizing cerebral rebleeding. When family vis- Reduced PaO2
it, the client should not be disturbed. If the client Cold stress causes increased risk for respira-
is awake, topics of a general nature are better tory distress. The baby delivered in such circum-
choices for discussion than topics that result in stances needs careful monitoring. In this situa-
emotional or physiological stimulation. tion, the newborn must be warmed immediately
to increase its temperature to at least 97 degrees
Question18 Fahrenheit (36 degrees Celsius).
A client who is 12 hour post-op becomes con-
fused and says: “Giant sharks are swimming Question20
across the ceiling.” Which assessment is neces- Which contraindication should the nurse assess
sary to adequately identify the source of this cli- for prior to giving a child immunizations?
ent’s behavior? A) Mild cold symptoms
A) Cardiac rhythm strip B) Chronic asthma
B) Pupillary response C) Depressed immune system
C) Pulse oximetry D) Allergy to eggs
D) Peripheral glucose stick
Review Information: The correct answer is C:
Review Information: The correct answer is C: Depressed immune system
Pulse oximetry Children who have a depressed immune system
A sudden change in mental status in any post- related to HIV or chemotherapy should not be
op client should trigger a nursing intervention given routine immunizations.
directed toward respiratory evaluation. Pulse
oximetry would be the initial assessment. If avail- Question21
able, arterial blood gases would be better. Acute The nurse is caring for a client with a myocardial
respiratory failure is the sudden inability of the infarction. Which finding requires the nurse’s im-
respiratory system to maintain adequate gas mediate action?
exchange which may result in hypercapnia and/ A) Periorbital edema
or hypoxemia. Clinical findings of hypoxemia in- B) Dizzy spells
clude these finding which are listed in order of C) Lethargy
initial to later findings: restlessness, irritability, D) Shortness of breath
agitation, dyspnea, disorientation, confusion, de-
lirium, hallucinations, and loss of consciousness. Review Information: The correct answer is B:
While there may be other factors influencing the Dizzy spells
client’’s behavior, the first nursing action should Cardiac dysrhythmias may cause a transient
be directed toward maintaining oxygenation. drop in cardiac output and decreased blood flow
Once respiratory or oxygenation issues are ruled to the brain. Near syncope refers to lightheart-
out then significant changes in glucose would be edness, dizziness, temporary confusion. Such
evaluated. “spells” may indicate runs of ventricular tachy-
Collected by :DeepaRajesh [ 116 ]
rajesh.ks21@gmail.com
Kuwait
cardia or periods of asystole and should be re-
ported immediately. Question25
A 16 year-old boy is admitted for Ewing’s sar-
Question22 coma of the tibia. In discussing his care with the
Decentralized scheduling is used on a nursing parents, the nurse understands that the initial
unit. A chief advantage of this management strat- treatment most often includes
egy is that it: A) amputation just above the tumor
A) considers client and staff needs B) surgical excision of the mass
B) conserves time spent on planning C) bone marrow graft in the affected leg
C) frees the nurse manager to handle other pri- D) radiation and chemotherapy
orities
D) allows requests for special privileges Review Information: The correct answer is D:
radiation and chemotherapy
Review Information: The correct answer is A: The initial treatment of choice for Ewing’’s sar-
considers client and staff needs coma is a combination of radiation and chemo-
Decentralized staffing takes into consideration therapy.
specific client needs and staff interests and abili-
ties. Question26
Question23 A new nurse manager is responsible for inter-
Included in teaching the client with tuberculo- viewing applicants for a staff nurse position.
sis taking isoniazid (INH) about follow-up home Which interview strategy would be the best ap-
care, the nurse should emphasize that a labora- proach?
tory appointment for which of the following lab A) Vary the interview style for each candidate to
tests is critical? learn different techniques
A) Liver function B) Use simple questions requiring “yes” and “no”
B) Kidney function answers to gain definitive information
C) Blood sugar C) Obtain an interview guide from human re-
D) Cardiac enzymes sources for consistency in interviewing each
candidate
Review Information: The correct answer is A: D) Ask personal information of each applicant to
Liver function assure he/she can meet job demands
INH can cause hepatocellular injury and hepati-
tis. This side effect is age-related and can be de-
tected with regular assessment of liver enzymes, Review Information: The correct answer is C:
which are released into the blood from damaged Obtain an interview guide from human resources
liver cells. for consistency in interviewing each candidate
An interview guide used for each candidate ena-
Question24 bles the nurse manager to be more objective in
A woman in her third trimester complains of se- the decision making. The nurse should use re-
vere heartburn. What is appropriate teaching sources available in the agency before attempts
by the nurse to help the woman alleviate these to develop one from scratch. Certain personal
symptoms? questions are prohibited, and HR can identify
A) Drink small amounts of liquids frequently these for novice managers.
B) Eat the evening meal just before retiring
C) Take sodium bicarbonate after each meal Question27
D) Sleep with head propped on several pillows What is the best way that parents of pre-school-
ers can begin teaching their child about injury
Review Information: The correct answer is D: prevention?
Sleep with head propped on several pillows A) Set good examples themselves
Heartburn is a burning sensation caused by re- B) Protect their child from outside influences
gurgitation of gastric contents. It is best relieved C) Make sure their child understands all the
by sleeping position, eating small meals, and not safety rules
eating before bedtime. D) Discuss the consequences of not wearing
Collected by :DeepaRajesh [ 117 ]
rajesh.ks21@gmail.com
Kuwait
protective devices causes. During the process of taking client his-
tory, which of these items should the nurse iden-
tify as related to the client’s greatest risk factors
Review Information: The correct answer is A: for osteoporosis?
Set good examples themselves A) History of menopause at age 50
The preschool years are the time for parents to B) Taking high doses of steroids for arthritis for
begin emphasizing safety principles as well as many years
providing protection. Setting a good example C) Maintaining an inactive lifestyle for the past
themselves is crucial because of the imitative 10 years
behaviors of pre-schoolers; they are quick to no- D) Drinking 2 glasses of red wine each day for
tice discrepancies between what they see and the past 30 years
what they are told.

Question28 Review Information: The correct answer is


A nurse assessing the newborn of a mother with B: Taking high doses of steroids for arthritis for
diabetes understands that hypoglycemia is re- many years
lated to what pathophysiological process? The use of steroids, especially at high doses over
A) Disruption of fetal glucose supply time, increases the risk for osteoporosis. The
B) Pancreatic insufficiency other options also predispose to osteoporosis,
C) Maternal insulin dependency as do low bone mass, poor calcium absorption
and moderate to high alcohol ingestion. Long-
D) Reduced glycogen reserves
term steroid treatment is the most significant risk
factor, however.
Review Information: The correct answer is A:
Disruption of fetal glucose supply
After delivery, the high glucose levels which
crossed the placenta to the fetus are suddenly Question31
stopped. The newborn continues to secrete insu- The nurse is caring for a 2 year-old who is being
lin in anticipation of glucose. When oral feedings treated with chelation therapy, calcium disodium
begin, the newborn will adjust insulin production edetate, for lead poisoning. The nurse should be
within a day or two. alert for which of the following side effects?
A) Neurotoxicity
B) Hepatomegaly
C) Nephrotoxicity
Question29
D) Ototoxicity
The nurse is caring for a client with extracellular
fluid volume deficit. Which of the following as-
sessments would the nurse anticipate finding?
Review Information: The correct answer is C:
A) bounding pulse
Nephrotoxicity
B) rapid respirations
Nephrotoxicity is a common side effect of calci-
C) oliguria
um disodium edetate, in addition to lead poison-
D) neck veins are distended
ing in general.

Question32
Review Information: The correct answer is C:
The parents of a toddler ask the nurse how long
oliguria
their child will have to sit in a car seat while in the
Kidneys maintain fluid volume through adjust-
automobile. What is the nurse’s best response to
ments in urine volume.
the parents?
A) “Your child must use a care seat until he
weighs at least 40 pounds.”
Question30 B) “The child must be 5 years of age to use a
A 70 year-old woman is evaluated in the emer- regular seat belt.”
gency department for a wrist fracture of unknown C) “Your child must reach a height of 50 inches
Collected by :DeepaRajesh [ 118 ]
rajesh.ks21@gmail.com
Kuwait
to sit in a seat belt.” A) decrease the client’s discomfort
D) “The child can use a regular seat belt when B) reduce viscosity of secretions
he can sit still.” C) prevent client aspiration
D) remove a mucus plug

Review Information: The correct answer is A: Review Information: The correct answer is D:
“Your child must use a care seat until he weighs remove a mucus plug
at least 40 pounds.” While no longer recommended for routine suc-
Children should use car seats until they weigh tioning, saline may thin and loosen viscous se-
40 pounds. cretions that are very difficult to move, perhaps
making them easier to suction.

Question33
A client asks the nurse to explain the basic ideas Question36
of homeopathic medicine. The response that best The nurse is performing a gestational age as-
explains this approach is that such remedies sessment on a newborn delivered 2 hours ago.
A) destroy organisms causing disease When coming to a conclusion using the Bal-
B) maintain fluid balance lard scale, which of these factors may affect the
C) boost the immune system score?
D) increase bodily energy A) Birth weight
B) Racial differences
C) Fetal distress in labor
D) Birth trauma
Review Information: The correct answer is C:
boost the immune system
The practitioner treats with minute doses of plant,
mineral or animal substances which provide a Review Information: The correct answer is C:
gentle stimulus to the body’’s own defenses. Fetal distress in labor
The effects of earlier distress may alter the find-
ings of reflex responses as measured on the Bal-
Question34 lard tool. Other physical characteristics that esti-
A client with a fractured femur has been in Rus- mate gestational age, such as amount of lanugo,
sell’s traction for 24 hours. Which nursing action sole creases and ear cartilage are unaffected by
is associated with this therapy? the other factors.
A) Check the skin on the sacrum for breakdown
B) Inspect the pin site for signs of infection
C) Auscultate the lungs for atelectasis Question37
D) Perform a neurovascular check for circula- A nurse is caring for a client who had a closed
tion reduction of a fractured right wrist followed by
the application of a fiberglass cast 12 hours ago.
Which finding requires the nurse’s immediate at-
Review Information: The correct answer is D: tention?
Perform a neurovascular check for circulation A) Capillary refill of fingers on right hand is 3 sec-
While each of these is an important assessment, onds
the neurovascular integrity check is most associ- B) Skin warm to touch and normally colored
ated with this type of traction. Russell’s traction C) Client reports prickling sensation in the right
is Buck’s traction with a sling under the knee. hand
D) Slight swelling of fingers of right hand

Question35
When suctioning a client’s tracheostomy, the Review Information: The correct answer is
nurse should instill saline in order to C: Client reports prickling sensation in the right
Collected by :DeepaRajesh [ 119 ]
rajesh.ks21@gmail.com
Kuwait
hand
A prickling sensation is an indication of compart- Question40
ment syndrome and requires immediate action The nurse is teaching the mother of a 5 month-
by the nurse. The other findings are normal for a old about nutrition for her baby. Which state-
client in this situation. ment by the mother indicates the need for further
teaching?
A) “I’m going to try feeding my baby some rice
Question38 cereal.”
A client is admitted with the diagnosis of pulmo- B) “When he wakes at night for a bottle, I feed
nary embolism. While taking a history, the client him.”
tells the nurse he was admitted for the same thing C) “I dip his pacifier in honey so he’ll take it.”
twice before, the last time just 3 months ago. The D) “I keep formula in the refrigerator for 24
nurse would anticipate the provider ordering hours.”
A) pulmonary embolectomy
B) vena caval interruption
C) increasing the Coumadin therapy to an INR Review Information: The correct answer is C:
of 3-4 “I dip his pacifier in honey so he’’ll take it.”
D) thrombolytic therapy Honey has been associated with infant botu-
lism and should be avoided. Older children and
adults have digestive enzymes that kill the botu-
Review Information: The correct answer is B: lism spores.
vena caval interruption
Clients with contraindications to Heparin, re- 0 comments
current PE or those with complications related
to the medical therapy may require vena caval Labels: free nclex-rn sample review questions,
interruption by the placement of a filter device nclex-rn practice test questions, nursing review
in the inferior vena cava. A filter can be placed
transvenously to trap clots before they travel to Free NCLEX-RN Sample Test Ques-
the pulmonary circulation. tions For Nursing Review (Part 4)

Question1
Question39 The clinic nurse is counseling a substance-abus-
Which client is at highest risk for developing a ing post partum client on the risks of continued
pressure ulcer? cocaine use. In order to provide continuity of
A) 23 year-old in traction for fractured femur care, which nursing diagnosis is a priority?
B) 72 year-old with peripheral vascular disease, A) Social isolation
who is unable to walk without assistance B) Ineffective coping
C) 75 year-old with left sided paresthesia who is C) Altered parenting
incontinent of urine and stool D) Sexual dysfunction
D) 30 year-old who is comatose following a rup-
tured aneurysm
Review Information: The correct answer is C:
Altered parenting
Review Information: The correct answer is C: The cocaine abusing mother puts her newborn
75 year-old with left sided paresthesia who is in- and other children at risk for neglect and abuse.
continent of urine and stool Continuing to use drugs has the potential to im-
Risk factors for pressure ulcers include: immo- pact parenting behaviors. Social service referrals
bility, absence of sensation, decreased LOC, are indicated.
poor nutrition and hydration, skin moisture, in-
continence, increased age, decreased immune
response. This client has the greatest number of
Question2
risk factors.
The nurse is teaching about nonsteroidal anti-
inflammatory drugs (NSAIDs) to a group of ar-
Collected by :DeepaRajesh [ 120 ]
rajesh.ks21@gmail.com
Kuwait
thritic clients. To minimize the side effects, the A) 1 year of age
nurse should emphasize which of the following B) 2 years of age
actions? C) 3 years of age
A) Reporting joint stiffness in the morning D) 4 years of age
B) Taking the medication 1 hour before or 2 hours
after meals
C) Using alcohol in moderation unless driving Review Information: The correct answer is B:
D) Continuing to take aspirin for short term relief 2 years of age
A child should be at least 2 years of age to use
the radial pulse to assess heart rate.

Question5
A client is receiving Total Parenteral Nutrition
Review Information: The correct answer is B: (TPN) via a Hickman catheter. The catheter acci-
Taking the medication 1 hour before or 2 hours dentally becomes dislodged from the site. Which
after meals action by the nurse should take priority?
Taking the medication 1 hour before or 2 hours A) Check that the catheter tip is intact
after meals will result in a more rapid effect. B) Apply a pressure dressing to the site
C) Monitor respiratory status
D) Assess for mental status changes
Question3
The nurse is preparing to administer a tube feed-
ing to a postoperative client. To accurately as- Question6
sess for a gastrostomy tube placement, the pri- A pregnant client who is at 34 weeks gestation
ority is to is diagnosed with a pulmonary embolism (PE).
A) auscultate the abdomen while instilling 10 cc Which of these medications would the nurse an-
of air into the tube ticipate the provider ordering?
B) place the end of the tube in water to check for A) Oral Coumadin therapy
air bubbles B) Heparin 5000 units subcutaneously B.I.D.
C) retract the tube several inches to check for C) Heparin infusion to maintain the PTT at 1.5-
resistance 2.5 times the control value
D) measure the length of tubing from nose to epi- D) Heparin by subcutaneous injection to main-
gastrium tain the PTT at 1.5 times the control value

Review Information: The correct answer is A: Review Information: The correct answer is D:
auscultate the abdomen while instilling 10 cc of Heparin by subcutaneous injection to maintain
air into the tube the PTT at 1.5 times the control value
If a swoosh of air is heard over the abdominal Several studies have been conducted in preg-
cavity while instilling air into the gastric tube, this nant women where oral anticoagulation agents
indicates that it is accurately placed in the stom- are contraindicated. Warfarin (Coumadin) is
ach. The feeding can begin after further assess- known to cross the placenta and is therefore re-
ing the client for bowel sounds. ported to be teratogenic.

Question4 Question7
While assessing the vital signs in children, the The nurse is caring for a client with Hodgkin’s
nurse should know that the apical heart rate is disease who will be receiving radiation therapy.
preferred until the radial pulse can be accurately The nurse recognizes that, as a result of the ra-
assessed at about what age? diation therapy, the client is most likely to experi-
Collected by :DeepaRajesh [ 121 ]
rajesh.ks21@gmail.com
Kuwait
ence
A) high fever
B) nausea
C) face and neck edema Review Information: The correct answer is B:
D) night sweats The MMR vaccine should be given now, prior to
the transplant
MMR is a live virus vaccine, and should be given
Review Information: The correct answer is B: at this time. Post-transplant, immunosuppressive
nausea drugs will be given and the administration of the
Because the client with Hodgkin’’s disease is live vaccine at that time would be contraindicated
usually healthy when therapy begins, the nausea because of the compromised immune system.
is especially troubling.
.
Question10
Question8 The nurse is preparing to take a toddler’s blood
A client is brought to the emergency room fol- pressure for the first time. Which of the following
lowing a motor vehicle accident. When assess- actions should the nurse perform first?
ing the client one-half hour after admission, the A) Explain that the procedure will help him to get
nurse notes several physical changes. Which well
finding would require the nurse’s immediate at- B) Show a cartoon character with a blood pres-
tention? sure cuff
A) increased restlessness C) Explain that the blood pressure checks the
B) tachycardia heart pump
C) tracheal deviation D) Permit handling the equipment before putting
D) tachypnea the cuff in place

Review Information: The correct answer is D:


Permit handling the equipment before putting the
Review Information: The correct answer is C: cuff in place
tracheal deviation The best way to gain the toddler’’s cooperation is
The deviated trachea is a sign that a mediastinal to encourage handling the equipment. Detailed
shift has occurred. This is a medical emergency. explanations are not helpful.

Question9 Question11
An 18 month-old child is on peritoneal dialysis Which statement made by a nurse about the
in preparation for a renal transplant in the near goal of total quality management or continuous
future. When the nurse obtains the child’s health quality improvement in a health care setting is
history, the mother indicates that the child has correct?
not had the first measles, mumps, rubella (MMR) A) It is to observe reactive service and product
immunization. The nurse understands that which problem solving
of the following is true in regards to giving immu- B) Improvement of the processes in a proactive,
nizations to this child? preventive mode is paramount
A) Live vaccines are withheld in children with re- C) A chart audits to finds common errors in prac-
nal chronic illness tice and outcomes associated with goals
B) The MMR vaccine should be given now, prior D) A flow chart to organize daily tasks is critical
to the transplant to the initial stages
C) An inactivated form of the vaccine can be giv-
en at any time
D) The risk of vaccine side effects precludes giv-
ing the vaccine
Review Information: The correct answer is
Collected by :DeepaRajesh [ 122 ]
rajesh.ks21@gmail.com
Kuwait
B: Improvement of the processes in a proactive, ease (COPD) and a history of coronary artery
preventive mode is paramount disease is receiving aminophylline, 25mg/hour.
Total quality management and continuous qual- Which one of the following findings by the nurse
ity improvement have a major goal of identifying would require immediate intervention?
ways to do the right thing at the right time in the A) Decreased blood pressure and respirations
right way by proactive problem-solving. B) Flushing and headache
C) Restlessness and palpitations
D) Increased heart rate and blood pressure
Question12
Which of the following drugs should the nurse
anticipate administering to a client before they
are to receive electroconvulsive therapy?
A) Benzodiazepines Review Information: The correct answer is C:
B) Chlorpromazine (Thorazine) Restlessness and palpitations
C) Succinylcholine (Anectine) Side effects of Aminophylline include restless-
D) Thiopental sodium (Pentothal Sodium) ness and palpitations.

Question15
A client has gastroesophageal reflux. Which rec-
Review Information: The correct answer is C: ommendation made by the nurse would be most
Succinylcholine (Anectine) helpful to the client?
Succinylcholine is given intravenously to pro- A) Avoid liquids unless a thickening agent is
mote skeletal muscle relaxation. used
B) Sit upright for at least 1 hour after eating
C) Maintain a diet of soft foods and cooked veg-
Question13 etables
Which approach is a priority for the nurse who D) Avoid eating 2 hours before going to sleep
works with clients from many different cultures?
A) Speak at least 2 other languages of clients in
the neighborhood
B) Learn about the cultures of clients who are
most often encountered Review Information: The correct answer is D:
C) Have a list of persons for referral when inter- Avoid eating 2 hours before going to sleep
action with these clients occur Eating before sleeping enhances the regurgita-
D) Recognize personal attitudes about cultural tion of stomach contents, which have increased
differences and real or expected biases acidity, into the esophagus. An upright posture
should be maintained for about 2 hours after eat-
ing to allow for the stomach emptying. Options A
and C are interventions for clients with swallow-
ing difficulties.
Review Information: The correct answer is D: .
Recognize personal attitudes about cultural dif-
ferences and real or expected biases Question16
The nurse must discover personal attitudes, prej- A client with a panic disorder has a new prescrip-
udices and biases specific to different cultures. tion for Xanax (alprazolam). In teaching the client
Awareness of these will prevent negative conse- about the drug’s actions and side effects, which
quences for interactions with clients and families of the following should the nurse emphasize?
across cultures. A) Short-term relief can be expected
B) The medication acts as a stimulant
C) Dosage will be increased as tolerated
Question14 D) Initial side effects often continue
A client with chronic obstructive pulmonary dis-
Collected by :DeepaRajesh [ 123 ]
rajesh.ks21@gmail.com
Kuwait
Review Information: The correct answer is D:
Review Information: The correct answer is A: ask the client to talk about concerns regarding
Short-term relief can be expected “hot” treatments
Xanax is a short-acting benzodiazepine useful in The “hot-cold” system is found among Mexican-
controlling panic symptoms quickly. Americans, Puerto Ricans, and other Hispan-
ic-Latinos. Most foods, beverages, herbs, and
medicines are categorized as hot or cold, which
Question17 are symbolic designations and do not necessar-
A client being discharged from the cardiac step- ily indicate temperature or spiciness. Care and
down unit following a myocardial infarction (MI), treatment regimens can be negotiated with cli-
is given a prescription for a beta-blocking drug. A ents within this framework.
nursing student asks the charge nurse why this
drug would be used by a client who is not hyper-
tensive. What is an appropriate response by the Question19
charge nurse? A 72 year-old client is scheduled to have a cardi-
A) “Most people develop hypertension following oversion. A nurse reviews the client’s medication
an MI.” administration record. The nurse should notify the
B) “A beta-Blocker will prevent orthostatic hypo- health care provider if the client received which
tension.” medication during the preceding 24 hours?
C) “This drug will decrease the workload on his A) Digoxin (Lanoxin)
heart.” B) Diltiazem (Cardizem)
D) “Beta-blockers increase the strength of heart C) Nitroglycerine ointment
contractions.” D) Metoprolol (Toprol XL)

Review Information: The correct answer is Review Information: The correct answer is A:
C: “This drug will decrease the workload on his Digoxin (Lanoxin)
heart.” Digoxin increases ventricular irritability and in-
One action of beta-blockers is to decrease sys- creases the risk of ventricular fibrillation follow-
temic vascular resistance by dilating arterioles. ing cardioversion. The other medications do not
This is useful for the client with coronary artery increase ventricular irritability.
disease, and will reduce the risk of another MI or
sudden death.
Question20
Which of these clients, all of whom have the find-
Question18 ings of a board-like abdomen, would the nurse
A 35-year-old client of Puerto Rican-American suggest that the provider examine first?
descent is diagnosed with ovarian cancer. The A) An elderly client who stated, “My awful pain
client states, “I refuse both radiation and chemo- in my right side suddenly stopped about 3 hours
therapy because they are ‘hot.’” The next action ago.”
for the nurse to take is to B) A pregnant woman of 8 weeks newly diag-
A) document the situation in the notes nosed with an ectopic pregnancy
B) report the situation to the health care provid- C) A middle-aged client admitted with diverticu-
er litis who has taken only clear liquids for the past
C) talk with the client’s family about the situation week
D) ask the client to talk about concerns regarding D) A teenager with a history of falling off a bicycle
“hot” treatments without hitting the handle bars

Collected by :DeepaRajesh [ 124 ]


rajesh.ks21@gmail.com
Kuwait
Review Information: The correct answer is A: Review Information: The correct answer is D:
An elderly client who stated, “My awful pain in Administering two anti-tuberculosis drugs
my right side suddenly stopped about 3 hours Resistance of the tubercle bacilli often occurs to
ago.” a single antimicrobial agent. Therefore, therapy
This client has the highest risk for hypovolemic with multiple drugs over a long period of time
and septic shock since the appendix has most helps to ensure eradication of the organism.
likely ruptured, based on the history of the pain
suddenly stopping over three hours ago. Elderly
clients have less functional reserve for the body Question23
to cope with shock and infection over long peri- The nurse is assessing a comatose client receiv-
ods. The others are at risk for shock also, how- ing gastric tube feedings. Which of the following
ever given that they fall in younger age groups, assessments requires an immediate response
they would more likely be able to tolerate an im- from the nurse?
balance in circulation. A common complication of A) Decreased breath sounds in right lower lobe
falling off a bicycle is hitting the handle bars in B) Aspiration of a residual of 100cc of formula
the upper abdomen often on the left, resulting in C) Decrease in bowel sounds
a ruptured spleen. D) Urine output of 250 cc in past 8 hours

Question21
The nurse is teaching parents of a 7 month-old
about adding table foods. Which of the following Review Information: The correct answer is A:
is an appropriate finger food? Decreased breath sounds in right lower lobe
A) Hot dog pieces The most common problem associated with en-
B) Sliced bananas teral feedings is atelectasis. Maintain client at 30
C) Whole grapes degrees of head elevation during feedings and
D) Popcorn monitor for signs of aspiration. Check for tube
placement prior to each feeding or every 4 to 8
hours if the client is receiving continuous feed-
ing.

Review Information: The correct answer is B:


Sliced bananas Question24
Finger foods should be bite-size pieces of soft A client is prescribed warfarin sodium (Couma-
food such as bananas. Hot dogs and grapes can din) to be continued at home. Which focus is
accidentally be swallowed whole and can occlude critical to be included in the nurse’s discharge
the airway. Popcorn is too difficult to chew at this instruction?
age and can irritate the airway if swallowed. A) Maintain a consistent intake of green leafy
foods
B) Report any nose or gum bleeds
Question22 C) Take Tylenol for minor pains
To prevent drug resistance from developing, the D) Use a soft toothbrush
nurse is aware that which of the following is a
characteristic of the typical treatment plan to
eliminate the tuberculosis bacilli?
A) An anti-inflammatory agent
B) High doses of B complex vitamins Review Information: The correct answer is B:
C) Aminoglycoside antibiotics Report any nose or gum bleeds
D) Administering two anti-tuberculosis drugs The client should notify the health care provider
if blood is noted in stools or urine, or any other
signs of bleeding occur.

Collected by :DeepaRajesh [ 125 ]


rajesh.ks21@gmail.com
Kuwait
Question25
When teaching a client about the side effects of
fluoxetine (Prozac), which of the following will
the nurse include?
A) Tachycardia blurred vision, hypotension, ano- Review Information: The correct answer is D:
rexia Autonomy
B) Orthostatic hypotension, vertigo, reactions to Individuals must be free to make independent
tyramine-rich foods decisions about participation in research without
C) Diarrhea, dry mouth, weight loss, reduced li- coercion from others.
bido
D) Photosensitivity, seizures, edema, hypergly-
cemia Question28
The nurse is talking with the family of an 18
months-old newly diagnosed with retinoblasto-
Review Information: The correct answer is C: ma. A priority in communicating with the parents
Diarrhea, dry mouth, weight loss, reduced libido is
Commonly reported side effects for fluoxetine A) Discuss the need for genetic counseling
(Prozac) are diarrhea, dry mouth, weight loss B) Inform them that combined therapy is seldom
and reduced libido. effective
C) Prepare for the child’s permanent disfigure-
ment
Question26 D) Suggest that total blindness may follow sur-
A newborn weighed 7 pounds 2 ounces at birth. gery
The nurse assesses the newborn at home 2
days later and finds the weight to be 6 pounds 7
ounces. What should the nurse tell the parents Review Information: The correct answer is A:
about this weight loss? Discuss the need for genetic counseling
A) The newborn needs additional assessments The hereditary aspects of this disease are well
B) The mother should breast feed more often documented. While the parents focus on the
C) A change to formula is indicated needs of this child, they should be aware that
D) The loss is within normal limits the risk is high for future offspring.

Question29
The nurse is planning care for an 8 year-old child.
Review Information: The correct answer is D: Which of the following should be included in the
The loss is within normal limits plan of care?
A newborn is expected to lose 5-10% of the birth A) Encourage child to engage in activities in the
weight in the first few days post-partum because playroom
of changes in elimination and feeding. B) Promote independence in activities of daily
living
C) Talk with the child and allow him to express
Question27 his opinions
The nurse manager informs the nursing staff at D) Provide frequent reassurance and cuddling
morning report that the clinical nurse specialist
will be conducting a research study on staff at-
titudes toward client care. All staff are invited to
participate in the study if they wish. This affirms
the ethical principle of Review Information: The correct answer is
A) Anonymity A: Encourage child to engage in activities in the
B) Beneficence playroom
C) Justice According to Erikson, the school age child is in
D) Autonomy the stage of industry versus inferiority. To help
Collected by :DeepaRajesh [ 126 ]
rajesh.ks21@gmail.com
Kuwait
them achieve industry, the nurse should encour- A client taking isoniazid (INH) for tuberculosis
age them to carry out tasks and activities in their asks the nurse about side effects of the medica-
room or in the playroom. tion. The client should be instructed to immedi-
ately report which of these?
A) Double vision and visual halos
Question30 B) Extremity tingling and numbness
The nurse is assigned to care for 4 clients. Which C) Confusion and lightheadedness
of the following should be assessed immediately D) Sensitivity of sunlight
after hearing the report?
A) The client with asthma who is now ready for
discharge Review Information: The correct answer is B:
B) The client with a peptic ulcer who has been Extremity tingling and numbness
vomiting all night Peripheral neuropathy is the most common side
C) The client with chronic renal failure returning effect of INH and should be reported to the pro-
from dialysis vider. It can be reversed.
D) The client with pancreatitis who was admitted
yesterday
Question33
Which of these questions is priority when as-
sessing a client with hypertension?
A) “What over-the-counter medications do you
Review Information: The correct answer is B: take?”
The client with a peptic ulcer who has been vom- B) “Describe your usual exercise and activity
iting all night patterns.”
A perforated peptic ulcer could cause nausea, C) “Tell me about your usual diet.”
vomiting and abdominal distention, and may be D) “Describe your family’s cardiovascular his-
a life threatening situation. The client should be tory.”
assessed immediately and findings reported to
the provider.
.

Question31 Review Information: The correct answer is


During a routine check-up, an insulin-depend- A: “What over-the-counter medications do you
ent diabetic has his glycosylated hemoglobin take?”
checked. The results indicate a level of 11%. Over-the-counter medications, especially those
Based on this result, what teaching should the that contain cold preparations can increase the
nurse emphasize? blood pressure to the point of hypertension.
A) Rotation of injection sties
B) Insulin mixing and preparation
C) Daily blood sugar monitoring Question34
D) Regular high protein diet The nurse is performing an assessment of the
motor function in a client with a head injury. The
best technique is
A) touching the trapezius muscle or arm firmly
B) pinching any body part
Review Information: The correct answer is C: C) shaking a limb vigorously
Daily blood sugar monitoring D) rubbing the sternum
Normal hemoglobin A1C (glycosylated hemo-
globin) level is 7 to 9%. Elevation indicates el-
evated glucose levels over time.

Review Information: The correct answer is D:


Question32 rubbing the sternum
Collected by :DeepaRajesh [ 127 ]
rajesh.ks21@gmail.com
Kuwait
The purpose is to assess the non-responsive cli-
ent’s reaction to a painful stimulus after less nox-
ious methods have been tried. Question37
Which of these clients would the nurse monitor
for the complication of C. difficile diarrhea?
Question35 A) An adolescent taking medications for acne
A nurse admits a client transferred from the emer- B) An elderly client living in a retirement center
gency room (ER). The client, diagnosed with a taking prednisone
myocardial infarction, is complaining of subster- C) A young adult at home taking a prescribed
nal chest pain, diaphoresis and nausea. The first aminoglycoside
action by the nurse should be to D) A hospitalized middle aged client receiving
A) order an EKG clindamycin
B) administer morphine sulfate
C) start an IV
D) measure vital signs

Review Information: The correct answer is D:


A hospitalized middle aged client receiving clin-
damycin
Review Information: The correct answer is B: Hospitalized patients, especially those receiving
administer morphine sulfate antibiotic therapy, are primary targets for C. diffi-
Decreasing the clients pain is the most important cile. Of clients receiving antibiotics, 5-38% expe-
priority at this time. As long as pain is present rience antibiotic-associated diarrhea; C. difficile
there is danger in extending the infarcted area. causes 15 to 20% of the cases. Several antibiot-
Morphine will decrease the oxygen demands of ic agents have been associated with C. difficile.
the heart and act as a mild diuretic as well. It is Broad-spectrum agents, such as clindamycin,
probable that an EKG and IV insertion were per- ampicillin, amoxicillin, and cephalosporins, are
formed in the ER. the most frequent sources of C. difficile. Also,
C. difficile infection has been caused by the ad-
ministration of agents containing beta-lactamase
Question36 inhibitors (i.e., clavulanic acid, sulbactam, tazo-
The nurse admits a 2 year-old child who has had bactam) and intravenous agents that achieve
a seizure. Which of the following statement by substantial colonic intraluminal concentrations
the child’s parent would be important in deter- (i.e., ceftriaxone, nafcillin, oxacillin). Fluoroqui-
mining the etiology of the seizure? nolones, aminoglycosides, vancomycin, and tri-
A) “He has been taking long naps for a week.” methoprim are seldom associated with C. difficile
B) “He has had an ear infection for the past 2 infection or pseudomembranous colitis.
days.”
C) “He has been eating more red meat lately.”
D) “He seems to be going to the bathroom more Question38
frequently.” The nurse is performing an assessment on a cli-
ent who is cachectic and has developed an ente-
rocutaneous fistula following surgery to relieve a
small bowel obstruction. The client’s total protein
level is reported as 4.5 g/dl. Which of the follow-
Review Information: The correct answer is ing would the nurse anticipate?
B: “He has had an ear infection for the past 2 A) Additional potassium will be given IV
days.” B) Blood for coagulation studies will be drawn
Contributing factors to seizures in children in- C) Total parenteral nutrition (TPN) will be start-
clude those such as age (more common in first ed
2 years), infections (late infancy and early child- D) Serum lipase levels will be evaluated
hood), fatigue, not eating properly and excessive
fluid intake or fluid retention.
Collected by :DeepaRajesh [ 128 ]
rajesh.ks21@gmail.com
Kuwait
infant: 20-60 mg/dl or 1.1-3.3 mmol/L, Neonate:
30-60 mg/dl or 1.7-3.3 mmol/L, Infant: 40-90 mg/
Review Information: The correct answer is C: dl or 2.2-5.0 mmol/L. Critical values are: Infant:
Total parenteral nutrition (TPN) will be started <40 mg/dl and in a Newborn: <30 and >300 mg/
The client is not absorbing nutrients adequately dl. Because of the increased birth weight which
as evidenced by the cachexia and low protein can be associated with diabetes mellitus, repeat-
levels. (A normal total serum protein level is 6.0- ed blood sugars will be drawn
8.0 g/dl.) TPN will promote a positive nitrogen
balance in this client who is unable to digest and 0 comments
absorb nutrients adequately.
Labels: free nclex-rn sample review questions,
nclex-rn practice test questions, nursing review
Question39
During a situation of pain management, which Free NCLEX-RN Sample Test Ques-
statement is a priority to consider for the ethical tions For Nursing Review (Part 3)
guidelines of the nurse?
A) The client’s self-report is the most important Question1
consideration A client diagnosed with chronic depression is
B) Cultural sensitivity is fundamental to pain maintained on tranylcypromine (Parnate). An im-
management portant nursing intervention is to teach the client
C) Clients have the right to have their pain re- to avoid which of the following foods?
lieved A) Wine, beer, cheese, liver and chocolate
D) Nurses should not prejudge a client’s pain us- B) Wine, citrus fruits, yogurt and broccoli
ing their own values C) Beer, cheese, beef and carrots
D) Wine, apples, sour cream and beef steak

Review Information: The correct answer is A:


Review Information: The correct answer is Wine, beer, cheese, liver and chocolate
A: The client’’s self-report is the most important These foods are tyramine rich and ingestion of
consideration these foods while taking monoamine oxidase in-
Pain is a complex phenomenon that is perceived hibitors (MAOIs) can precipitate a life-threaten-
differently by each individual. Pain is whatever ing hypertensive crisis.
the client says it is. The other statements are cor-
rect but not the most important considerations.
Question2
The nurse is working in a high risk antepartum
clinic. A 40 year-old woman in the first trimes-
Question40 ter gives a thorough health history. Which infor-
As a part of a 9 pound full-term newborn’s as- mation should receive priority attention by the
sessment, the nurse performs a dextro-stick at 1 nurse?
hour post birth. The serum glucose reading is 45 A) Her father and brother are insulin dependent
mg/dl. What action by the nurse is appropriate at diabetics
this time? B) She has taken 800 mcg of folic acid daily for
A) Give oral glucose water the past year
B) Notify the pediatrician C) Her husband was treated for tuberculosis as
C) Repeat the test in 2 hours a child
D) Check the pulse oximetry reading D) She reports recent use of over-the counter si-
nus remedies
Review Information: The correct answer is C:
Repeat the test in 2 hours Review Information: The correct answer is D:
This blood sugar is within the normal range for a She reports recent use of over-the counter sinus
full-term newborn. Normal values are: Premature remedies
Collected by :DeepaRajesh [ 129 ]
rajesh.ks21@gmail.com
Kuwait
Over-the-counter drugs are a possible danger in Blood alcohol levels are generally obtained to
early pregnancy. A report by the client that she determine the level of intoxication. The amount
has taken medications should be followed up im- of alcohol consumed determines how much
mediately. medication the client needs for detoxification and
treatment. Reports of alcohol consumption are
Question3 notoriously inaccurate.
What must be the priority consideration for nurs-
es when communicating with children? Question6
A) Present environment Which clinical finding would the nurse expect to
B) Physical condition assess first in a newborn with spastic cerebral
C) Nonverbal cues palsy?
D) Developmental level A) cognitive impairment
B) hypotonic muscular activity
Review Information: The correct answer is D: C) seizures
Developmental level D) criss-crossing leg movement
While each of the factors affect communication,
the nurse recognizes that developmental differ-
ences have implications for processing and un- Review Information: The correct answer is D:
derstanding information. Consequently, a child’s criss-crossing leg movement
developmental level must be considered when Cerebral palsy is a neuromuscular impairment
selecting communication approaches. resulting in muscular and reflexive hypertonicity
and the criss-crossing, or scissoring leg move-
Question4 ments.
The nurse is assessing a client’s home in prep-
aration for discharge. Which of the following Question7
should be given priority consideration? Which medication is more helpful in treating
A) Family understanding of client needs bulimia than anorexia?
B) Financial status A) Amphetamines
C) Location of bathrooms B) Sedatives
D) Proximity to emergency services C) Anticholinergics
D) Narcotics
Review Information: The correct answer is A:
Family understanding of client needs Review Information: The correct answer is C:
Functional communication patterns between Anticholinergics
family members are fundamental to meeting the In contrast to anorexics, individuals with bulimia
needs of the client and family. are troubled by their behavioral characteristics
and become depressed. The person feels com-
pelled to binge, purge and fast. Feeling helpless
to stop the behavior, feelings of self-disgust oc-
Question5 cur.
As a general guide for emergency management
of acute alcohol intoxication, it is important for Question8
the nurse initially to obtain data regarding which The nurse is assessing a woman in early labor.
of the following? While positioning for a vaginal exam, she com-
A) What and how much the client drinks, accord- plains of dizziness and nausea and appears
ing to family and friends pale. Her blood pressure has dropped slightly.
B) The blood alcohol level of the client What should be the initial nursing action?
C) The blood pressure level of the client A) Call the health care provider
D) The blood glucose level of the client B) Encourage deep breathing
C) Elevate the foot of the bed
D) Turn her to her left side
Review Information: The correct answer is B:
The blood alcohol level of the client Review Information: The correct answer is D:
Collected by :DeepaRajesh [ 130 ]
rajesh.ks21@gmail.com
Kuwait
Turn her to her left side ing lab data is available: PaO2 95, PaCO2 30,
The weight of the uterus can put pressure on the pH 7.5, K 3.2 mEq/l. Which should be the nurse’s
vena cava and aorta when a pregnant woman first action?
is flat on her back causing supine hypotension. A) Monitor respiratory rate
Action is needed to relieve the pressure on the B) Monitor intake and output every hour
vena cava and aorta. Turning the woman to the C) Assist the client to breathe into a paper bag
side reduces this pressure and relieves postural D) Prepare to administer oxygen by mask
hypotension.

Question9 Review Information: The correct answer is C:


A client has been started on a long term corticos- Assist the client to breathe into a paper bag
teroid therapy. Which of the following comments Side effects of aspirin toxicity include hyperventi-
by the client indicate the need for further teach- lation, which can result in respiratory alkalosis in
ing? the initial stages. Breathing into a paper bag will
A) “I will keep a weekly weight record.” prevent further reduction in PaCO2.
B) “I will take medication with food.”
C) “I will stop taking the medication for 1 week Question12
every month.” After assessing a 70 year-old male client’s labo-
D) “I will eat foods high in potassium.” ratory results during a routine clinic visit, which
one of the following findings would indicate an
area in which teaching is needed:
Review Information: The correct answer is C: A) Serum albumin 2.5 g/dl
“I will stop taking the medication for 1week every B) LDL Cholesterol 140 mg/dl
month.” C) Serum glucose 90 mg/dl
Emphatically warn against discontinuing steroid D) RBC 5.0 million/mm3
dosage abruptly because that may produce a fa-
tal adrenal crisis.
Review Information: The correct answer is A:
Question10 Serum albumin 2.5 g/dl
A male client calls for a nurse because of chest Serum albumin level is low (normal 3.0 – 5.0 g/dl
pain. Which statement by the client would require in elders), indicating nutritional counseling to in-
the most immediate action by the nurse? crease dietary protein is needed. Socioeconomic
A) “When I take in a deep breath, it stabs like a factors may need to be addressed to help the
knife.” client comply with the recommendation.
B) “The pain came on after dinner. That soup
seemed very spicy.” Question13
C) “When I turn in bed to reach the remote for the When teaching a client with a new prescription
TV, my chest hurts.” for lithium (Lithane) for treatment of a bi-polar
D) “I feel pressure in the middle of my chest, like disorder which of these should the nurse empha-
an elephant is sitting on my chest.” size?
A) Maintaining a salt restricted diet
B) Reporting vomiting or diarrhea
Review Information: The correct answer is D: C) Taking other medication as usual
“I feel pressure in the middle of my chest, like an D) Substituting generic form if desired
elephant is sitting on my chest.”
This is a classic description of chest pain in men
caused by myocardial ischemia. Women experi- Review Information: The correct answer is B:
ence vague feelings of fatigue and back and jaw Reporting vomiting or diarrhea
pain. If dehydration results from vomiting, diarrhea
or excessive perspiration, tolerance to the drug
Question11 may be altered and symptoms may return.
A nurse is caring for a client who has just been
admitted with an overdose of aspirin. The follow-
Collected by :DeepaRajesh [ 131 ]
rajesh.ks21@gmail.com
Kuwait
Question14
A client is discharged on warfarin sulfate (Cou- Question17
madin). Which statement by the client indicated A client is admitted to the hospital with a diag-
a need for further teaching? nosis of deep vein thrombosis. During the ini-
A) “I know I must avoid crowds.” tial assessment, the client complains of sudden
B) “I will keep all laboratory appointments.” shortness of breath. The SaO2 is 87. The priority
C) “I plan to use an electric razor for shaving.” nursing assessment at this time is
D) “I will report any bruises for bleeding.” A) bowel sounds
B) heart rate
C) peripheral pulses
D) lung sounds
Review Information: The correct answer is A:
“I know I must avoid crowds.”
There are no specific reasons for the client on Review Information: The correct answer is D:
Coumadin to avoid crowds. General instructions lung sounds
for any cardiac surgical client include limiting ex- Lung sounds are critical assessments at this
posure to infection. point. The nurse should be alert to crackles or a
pleural friction rub, highly suggestive of a pulmo-
Question15 nary embolism.
A client is taking tranylcypromine (Parnate) and
has received dietary instruction. Which of the fol- Question18
lowing food selections would be contraindicated The nurse is administering lidocaine (Xylocaine)
for this client? to a client with a myocardial infarction. Which of
A) Fresh juice, carrots, vanilla pudding the following assessment findings requires the
B) Apple juice, ham salad, fresh pineapple nurse’s immediate action?
C) Hamburger, fries, strawberry shake A) Central venous pressure reading of 11
D) Red wine, fava beans, aged cheese B) Respiratory rate of 22
C) Pulse rate of 48 BPM
D) Blood pressure of 144/92
Review Information: The correct answer is D:
Red wine, fava beans, aged cheese
Red wine and cheese contain tyramine (as do Review Information: The correct answer is C:
chicken liver and ripe bananas) and so are con- Pulse rate of 48 BPM
traindicated when taking MAOIs. Fava beans One of the side effects of lidocaine is bradycardia,
contain other vasopressors that can interact with heart block, cardiovascular collapse and cardiac
MAOIs also causing malignant hypertension. arrest (this drug should never be administered
without continuous EKG monitoring).
Question16
A client is admitted with severe injuries from an Question19
auto accident. The client’s vital signs are BP The nurse is teaching a group of college students
120/50, pulse rate 110, and respiratory rate of about breast self-examination. A woman asks for
28. The initial nursing intervention would be to the best time to perform the monthly exam. What
A) begin intravenous therapy is the best reply by the nurse?
B) initiate continuous blood pressure monitoring A) “The first of every month, because it is easiest
C) administer oxygen therapy to remember”
D) institute cardiac monitoring B) “Right after the period, when your breasts are
less tender”
Review Information: The correct answer is C: C) “Do the exam at the same time every month”
administer oxygen therapy D) “Ovulation, or mid-cycle is the best time to de-
Early findings of shock reveal hypoxia with rapid tect changes”
heart rate and rapid respirations, and oxygen is
the most critical initial intervention. The other in-
terventions are secondary to oxygen therapy. Review Information: The correct answer is B:
Collected by :DeepaRajesh [ 132 ]
rajesh.ks21@gmail.com
Kuwait
“Right after the period, when your breasts are Review Information: The correct answer is B:
less tender” Sodium
The best time for a breast self exam (BSE) is a Clients taking lithium need to maintain an ade-
week after a menstrual cycle, when the breasts quate intake of sodium. Serum lithium concen-
are no longer swollen and tender due to hormone trations may increase in the presence of condi-
elevation. tions that cause sodium loss.

Question20 Question23
The nurse is caring for a post-operative client A client is receiving lithium carbonate 600 mg
who develops a wound evisceration. The first T.I.D. to treat bipolar disorder. Which of these in-
nursing intervention should be to dicate early signs of toxicity?
A) medicate the client for pain A) Ataxia and course hand tremors
B) call the provider B) Vomiting, diarrhea and lethargy
C) cover the wound with sterile saline dressing C) Pruritus, rash and photosensitivity
D) place the bed in a flat position D) Electrolyte imbalance and cardiac arrhyth-
mias

Review Information: The correct answer is C: Review Information: The correct answer is B:
cover the wound with sterile saline dressing Vomiting, diarrhea and lethargy
When evisceration occurs, the wound should first These are early signs of lithium toxicity.
be quickly covered by sterile dressings soaked in
sterile saline. This prevents tissue damage until Question24
a repair can be effected. The nurse can best ensure the safety of a client
suffering from dementia who wanders from the
room by which action?
Question21 A) Repeatedly remind the client of the time and
The spouse of a client with Alzheimer’s disease location
expresses concern about the burden of caregiv- B) Explain the risks of walking with no purpose
ing. Which of the following actions by the nurse C) Use protective devices to keep the client in
should be a priority? the bed or chair in the room
A) Link the caregiver with a support group D) Attach a wander-guard sensor band to the cli-
B) Ask friends to visit regularly ent’s wrist
C) Schedule a home visit each week
D) Request anti-anxiety prescriptions
Review Information: The correct answer is D:
Review Information: The correct answer is A: Attach a wander-guard sensor band to the cli-
Link the caregiver with a support group ent’’s wrist
Assisting caregivers to locate and join support This type of identification band easily tracks the
groups is most helpful. Families share feelings client’’s movements and ensures safety while the
and learn about services such as respite care. client wanders on the unit. Restriction of activity
Health education is also available through local is inappropriate for any client unless they are po-
and national Alzheimer’’s chapters. tentially harmful to themselves or others.

Question22 Question25
Clients taking lithium must be particularly sure to The nurse is teaching a client about the difference
maintain adequate intake of which of these ele- between tardive dyskinesia (TD) and neuroleptic
ments? malignant syndrome (NMS). Which statement is
A) Potassium true with regards to tardive dyskinesia?
B) Sodium A) TD develops within hours or years of contin-
C) Chloride ued antipsychotic drug use in people under 20
D) Calcium and over 30
B) It can occur in clients taking antipsychotic
drugs longer than 2 years
Collected by :DeepaRajesh [ 133 ]
rajesh.ks21@gmail.com
Kuwait
C) Tardive dyskinesia occurs within minutes of medical emergency.
the first dose of antipsychotic drugs and is re-
versible Question28
D) TD can easily be treated with anticholinergic The nurse is caring for a 2 month-old infant with
drugs a congenital heart defect. Which of the following
is a priority nursing action?
A) Provide small feedings every 3 hours
Review Information: The correct answer is B: B) Maintain intravenous fluids
It can occur in clients taking antipsychotic drugs C) Add strained cereal to the diet
longer than 2 years D) Change to reduced calorie formula
Tardive dyskinesia is a extrapyramidal side ef-
fect that appears after prolonged treatment with Review Information: The correct answer is A:
antipsychotic medication. Early symptoms of tar- Provide small feedings every 3 hours
dive dyskinesia are fasciculations of the tongue Infants with congenital heart defects are at in-
or constant smacking of the lips. creased risk for developing congestive heart fail-
ure. Infants with congestive heart failure have an
Question26 increased metabolic rate and require additional
The nurse is aware that the effect of antihyper- calories to grow. At the same time, however, rest
tensive drug therapy may be affected by a 75 and conservation of energy for eating is impor-
year-old client’s tant. Feedings should be smaller and every 3
A) poor nutritional status hours rather than the usual 4 hour schedule.
B) decreased gastrointestinal motility
C) increased splanchnic blood flow Question29
D) altered peripheral resistance The nurse is caring for a client receiving intrave-
nous nitroglycerin for acute angina. What is the
most important assessment during treatment?
Review Information: The correct answer is B: A) Heart rate
decreased gastrointestinal motility B) Neurologic status
Together with shrinkage of the gastric mucosa, C) Urine output
and changes in the levels of hydrochloric acid, D) Blood pressure
this will decrease absorption of medications and
interfere with their actions.
Review Information: The correct answer is D:
Question27 Blood pressure
In response to a call for assistance by a client in The vasodilatation that occurs as a result of this
labor, the nurse notes that a loop on the umbili- medication can cause profound hypotension.
cal cord protrudes from the vagina. What is the The client’’s blood pressure must be evaluated
priority nursing action? every 15 minutes until stable and then every 30
A) call the health care provider minutes to every hour.
B) check fetal heart beat
C) put the client in knee-chest position Question30
D) turn the client to the side A client telephones the clinic to ask about a home
pregnancy test she used this morning. The nurse
understands that the presence of which hormone
Review Information: The correct answer is C: strongly suggests a woman is pregnant?
put the client in knee-chest position A) Estrogen
Immediate action is needed to relieve pressure B) HCG
on the cord, which puts the fetus at risk due to C) Alpha-fetoprotein
hypoxia. The Trendelenburg position accom- D) Progesterone
plishes this. The exposed cord is covered with
saline soaked gauze, not reinserted. The fe- Review Information: The correct answer is B:
tal heart rate also should be checked, and the HCG
provider called. A prolapsed umbilical cord is a Human chorionic gonadotropin (HCG) is the
Collected by :DeepaRajesh [ 134 ]
rajesh.ks21@gmail.com
Kuwait
biologic marker on which pregnancy tests are the bathroom. The nurse notices a large amount
based. Reliability is about 98%, but the test does of clear fluid on the bed linens. The nurse knows
not conclusively confirm pregnancy. that fetal monitoring must now assess for what
complication?
Question31 A) Early decelerations
A client, admitted to the unit because of severe B) Late accelerations
depression and suicidal threats, is placed on sui- C) Variable decelerations
cidal precautions. The nurse should be aware D) Periodic accelerations
that the danger of the client committing suicide
is greatest
A) during the night shift when staffing is limited Review Information: The correct answer is C:
B) when the client’s mood improves with an in- Variable decelerations
crease in energy level When the membranes rupture, there is increased
C) at the time of the client’s greatest despair risk initially of cord prolapse. Fetal heart rate pat-
D) after a visit from the client’s estranged part- terns may show variable decelerations, which
ner require immediate nursing action to promote gas
exchange.

Question34
Review Information: The correct answer is The nurse is assessing a client with chronic ob-
B: when the client’s mood improves with an in- structive pulmonary disease receiving oxygen for
crease in energy level low PaO2 levels. Which assessment is a nursing
Suicide potential is often increased when there priority?
is an improvement in mood and energy level. At A) Evaluating SaO2 levels frequently
this time ambivalence is often decreased and a B) Observing skin color changes
decision is made to commit suicide. C) Assessing for clubbing fingers
D) Identifying tactile fremitus
Question32
After 4 electroconvulsive treatments over 2 Review Information: The correct answer is A:
weeks, a client is very upset and states “I am so Evaluating SaO2 levels frequently
confused. I lose my money. I just can’t remem- The best method to evaluate a client’’s oxygena-
ber telephone numbers.” The most therapeutic tion is to evaluate the SaO2. This is just as effec-
response for the nurse to make is tive as an arterial blood gas reading to evaluate
A) “You were seriously ill and needed the treat- oxygenation status, and is less traumatic and
ments.” expensive.
B) “Don’t get upset. The confusion will clear up
in a day or two.” Question35
C) “It is to be expected since most clients have The visiting nurse makes a postpartum visit to
the same results.” a married female client. Upon arrival, the nurse
D) “I can hear your concern and that your confu- observes that the client has a black eye and nu-
sion is upsetting to you.” merous bruises on her arms and legs. The initial
nursing intervention would be to
A) call the police to report indications of domestic
Review Information: The correct answer is D: violence
“I can hear your concern and that your confusion B) confront the husband about abusing his wife
is upsetting to you.” C) leave the home because of the unsafe envi-
Communicating caring and empathy with the ac- ronment
knowledgement of feelings is the initial response. D) interview the client alone to determine the ori-
Afterwards, teaching about the expected short gin of the injuries
term effects would be discussed.

Question33 Review Information: The correct answer is D:


A woman in labor calls the nurse to assist her in interview the client alone to determine the origin
Collected by :DeepaRajesh [ 135 ]
rajesh.ks21@gmail.com
Kuwait
of the injuries Serum potassium
It would be wrong to assume domestic violence Potassium is lost in diabetic ketoacidosis during
without further assessment. Separate the sus- rehydration and insulin administration. Review of
pected victim from the partner until battering has this lab finding suggests the nurse has knowl-
been ruled out. edge of this problem.

Question39
Question36 A male client is preparing for discharge follow-
When teaching a client about an oral hypoglyc- ing an acute myocardial infarction. He asks the
emic medication, the nurse should place primary nurse about his sexual activity once he is home.
emphasis on What would be the nurse’s initial response?
A) recognizing findings of toxicity A) Give him written material from the American
B) taking the medication at specified times Heart Association about sexual activity with heart
C) increasing the dosage based on blood glu- disease
cose B) Answer his questions accurately in a private
D) distinguishing hypoglycemia from hyperglyc- environment
emia C) Schedule a private, uninterrupted teaching
session with both the client and his wife
Review Information: The correct answer is B: D) Assess the client’s knowledge about his health
taking the medication at specified times problems
A regular interval between doses should be
maintained since oral hypoglycemics stimulate
the islets of Langerhans to produce insulin. Review Information: The correct answer is D:
Assess the client’’s knowledge about his health
Question37 problems
Initial postoperative nursing care for an infant The nursing process is continuous and cyclical in
who has had a pyloromyotomy would initially in- nature. When a client expresses a specific con-
clude cern, the nurse performs a focused assessment
A) bland diet appropriate for age to gather additional data prior to planning and
B) intravenous fluids for 3-4 days implementing nursing interventions.
C) NPO then glucose and electrolyte solutions
D) formula or breast milk as tolerated Question40
The client asks the nurse how the health care
provider could tell she was pregnant “just by
Review Information: The correct answer is C: looking inside.” What is the best explanation by
NPO then glucose and electrolyte solutions the nurse?
Post-operatively, the initial feedings are clear liq- A) Bluish coloration of the cervix and vaginal
uids in small quantities to provide calories and walls
electrolytes. B) Pronounced softening of the cervix
C) Clot of very thick mucous that obstructs the
Question38 cervical canal
A client is treated in the emergency room for D) Slight rotation of the uterus to the right
diabetic ketoacidosis and a glucose level of
650mg.D/L. In assessing the client, the nurse’s
review of which of the following tests suggests Review Information: The correct answer is A:
an understanding of this health problem? Bluish coloration of the cervix and vaginal walls
A) Serum calcium Chadwick’’s sign is a bluish-purple coloration of
B) Serum magnesium the cervix and vaginal walls, occurring at 4 weeks
C) Serum creatinine of pregnancy, that is caused by vasocongestion.
D) Serum potassium
0 comments

Review Information: The correct answer is D: Labels: free nclex-rn sample review questions,
Collected by :DeepaRajesh [ 136 ]
rajesh.ks21@gmail.com
Kuwait
nclex-rn practice test questions, nursing review
Question4
Free NCLEX-RN Sample Test Ques- A 12 year-old child is admitted with a broken arm
tions For Nursing Review (Part 2) and is told surgery is required. The nurse finds
him crying and unwilling to talk. What is the most
Question1 appropriate response by the nurse?
The feeling of trust can best be established by A) Give him privacy
the nurse during the process of the development B) Tell him he will get through the surgery with
of a nurse-client relationship by which of these no problem
characteristics? C) Try to distract him
A) Reliability and kindness D) Make arrangements for his friends to visit
B) Demeanor and sincerity
C) Honesty and consistency
D) Sympathy and appreciativeness Review Information: The correct answer is A:
Give him privacy
Review Information: The correct answer is C: A 12 year-old child needs the opportunity to ex-
Honesty and consistency press his emotions privately.
Characteristics of a trusting relationship include
respect, honesty, consistency, faith and caring. Question5
In discharge teaching, the nurse should empha-
Question2 size that which of these is a common side effect
A nurse has administered several blood trans- of clozapine (Clozaril) therapy?
fusions over 3 days to a 12 year-old client with A) Dry mouth
Thalassemia. What lab value should the nurse B) Rhinitis
monitor closely during this therapy? C) Dry skin
A) Hemoglobin D) Extreme salivation
B) Red Blood Cell Indices
C) Platelet count Review Information: The correct answer is D:
D) Neutrophil percent Extreme salivation
A significant number of clients receiving Clozap-
Review Information: The correct answer is A: ine (Clozaril) therapy experience extreme saliva-
Hemoglobin tion.
Hemoglobin should be in a therapeutic range of
approximately 10 g/dl (100gL). “This level is low Question6
enough to foster the patient’’s own erythropoiesis A client has had a positive reaction to purified
without enlarging the spleen.” protein derivative (PPD). The client asks the
nurse what this means. The nurse should indi-
Question3 cate that the client has
The nurse is providing care to a newly a hospital- A) active tuberculosis
ized adolescent. What is the major threat experi- B) been exposed to mycobacterium tuberculo-
enced by the hospitalized adolescent? sis
A) Pain management C) never had tuberculosis
B) Restricted physical activity D) never been infected with mycobacterium tu-
C) Altered body image berculosis
D) Separation from family
Review Information: The correct answer is B:
Review Information: The correct answer is C: been exposed to mycobacterium tuberculosis
Altered body image The PPD skin test is used to determine the pres-
The hospitalized adolescent may see each of ence of tuberculosis antibodies and a positive re-
these as a threat, but the major threat that they sult indicates that the person has been exposed
feel when hospitalized is the fear of altered body to mycobacterium tuberculosis. Additional tests
image, because of the emphasis on physical ap- are needed to determine if active tuberculosis is
pearance during this developmental stage. present.
Collected by :DeepaRajesh [ 137 ]
rajesh.ks21@gmail.com
Kuwait
B) “Seek medical attention for serious injuries.”
C) “Report exposure to this illness.”
Question7 D) “Avoid use of aspirin for viral infections.”
A client is receiving and IV antibiotic infusion and
is scheduled to have blood drawn at 1:00 pm
for a “peak” antibiotic level measurement. The Review Information: The correct answer is D:
nurse notes that the IV infusion is running behind “Avoid use of aspirin for viral infections.”
schedule and will not be competed by 1:00. The The link between aspirin use and Reye’’s Syn-
nurse should: drome has not been confirmed, but evidence
A) Notify the client’s health care provider suggests that the risk is sufficiently grave to in-
B) Stop the infusion at 1:00 pm clude the warning on aspirin products.
C) Reschedule the laboratory test
D) Increase the infusion rate Question10
A post-operative client is admitted to the post-an-
esthesia recovery room (PACU). The anesthet-
Review Information: The correct answer is C: ist reports that malignant hyperthermia occurred
Reschedule the laboratory test during surgery. The nurse recognizes that this
If the antibiotic infusion will not be completed at complication is related to what factor?
the time the peak blood level is due to be drawn, A) Allergy to general anesthesia
the nurse should ask that the blood sampling B) Pre-existing bacterial infection
time be adjusted C) A genetic predisposition
D) Selected surgical procedures
Question8
The nurse is caring for a client with a new order Review Information: The correct answer is C:
for bupropion (Wellbutrin) for treatment of de- A genetic predisposition
pression. The order reads “Wellbutrin 175 mg. Malignant hyperthermia is a rare, potentially fatal
BID x 4 days.” What is the appropriate action? adverse reaction to inhaled anesthetics. There is
A) Give the medication as ordered a genetic predisposition to this disorder.
B) Questionthis medication dose
C) Observe the client for mood swings
D) Monitor neuro signs frequently Question11
A 9 year-old is taken to the emergency room with
right lower quadrant pain and vomiting. When
Review Information: The correct an- preparing the child for an emergency appen-
dectomy, what must the nurse expect to be the
swer is B: Questionthis medication
child’s greatest fear?
dose A) Change in body image
Bupropion (Wellbutrin) should be started at B) An unfamiliar environment
100mg BID for three days then increased to C) Perceived loss of control
150mg BID. When used for depression, it may D) Guilt over being hospitalized
take up to four weeks for results. Common side
effects are dry mouth, headache, and agitation. Review Information: The correct answer is C:
Doses should be administered in equally spaced Perceived loss of control
time increments throughout the day to minimize For school age children, major fears are loss of
the risk of seizures. control and separation from friends/peers.

Question9 Question12
The clinic nurse is discussing health promotion A client is to begin taking Fosamax. The nurse
with a group of parents. A mother is concerned must emphasize which of these instructions to
about Reye’s Syndrome, and asks about pre- the client when taking this medication? “Take
vention. Which of these demonstrates appropri- Fosamax
ate teaching? A) on an empty stomach.”
A) “Immunize your child against this disease.” B) after meals.”
Collected by :DeepaRajesh [ 138 ]
rajesh.ks21@gmail.com
Kuwait
C) with calcium.” ent admitted after a severe motor vehicle crash
D) with milk 2 hours after meals.” is in acidosis?
A) Hemoglobin 15 gm/dl
B) Chloride 100 mEq/L
Review Information: The correct answer is A: C) Sodium 130 mEq/L
on an empty stomach.” D) Carbon dioxide 20 mEq/L
Fosamax should be taken first thing in the morn-
ing with 6-8 ounces of plain water at least 30
minutes before other medication or food. Food Review Information: The correct answer is D:
and fluids (other than water) greatly decrease Carbon dioxide 20 mEq/L
the absorption of Fosamax. The client must also Serum carbon dioxide is an indicator of acid-base
be instructed to remain in the upright position for status. This finding would indicate acidosis.
30 minutes following the dose to facilitate pas-
sage into the stomach and minimize irritation of Question16
the esophagus. The nurse has just received report on a group
of clients and plans to delegate care of several
Question13 of the clients to a practical nurse (PN). The first
An older adult client is to receive and antibiotic, thing the RN should do before the delegation of
gentamicin. What diagnostic finding indicates care is
the client may have difficult excreting the medi- A) Provide a time-frame for the completion of the
cation? client care
A) High gastric pH B) Assure the PN that the RN will be available for
B) High serum creatinine assistance
C) Low serum albumin C) Ask about prior experience with similar cli-
D) Low serum blood urea nitrogen ents
D) Review the specific procedures unique to the
assignment
Review Information: The correct answer is B:
High serum creatinine Review Information: The correct answer is C:
An elevated serum creatinine indicates reduced Ask about prior experience with similar clients.
renal function. Reduced renal function will delay The first step in delegation is to determine the
the excretion of many mediations. qualifications of the person to whom one is dele-
gating. By asking about the PN’’s prior experience
Question14 with similar clients/tasks, the RN can determine
A nurse is assigned to care for a comatose dia- whether the PN has the requisite experience to
betic on IV insulin therapy. Which task would be care for the assigned clients.
most appropriate to delegate to an unlicensed
assistive personnel (UAP)?
A) Check the client’s level of consciousness Question17
B) Obtain the regular blood glucose readings The mother of a 4 month-old infant asks the
C) Determine if special skin care is needed nurse about the dangers of sunburn while they
D) Answer questions from the client’s spouse are on vacation at the beach. Which of the fol-
about the plan of care lowing is the best advice about sun protection for
this child?
A) “Use a sunscreen with a minimum sun protec-
Review Information: The correct answer is B: tive factor of 15.”
Obtain the regular blood glucose readings B) “Applications of sunscreen should be repeat-
The UAP can safely obtain blood glucose read- ed every few hours.”
ings, which are routine tasks. C) “An infant should be protected by the maxi-
mum strength sunscreen.”
Question15 D) “Sunscreens are not recommended in chil-
Which of the following laboratory results would dren younger than 6 months.”
suggest to the emergency room nurse that a cli-
Collected by :DeepaRajesh [ 139 ]
rajesh.ks21@gmail.com
Kuwait
B) “Unless you had previous problems, every 2
Review Information: The correct answer is D: years is best.”
“Sunscreens are not recommended in children C) “Once a woman reaches 50, she should have
younger than 6 months.” a mammogram yearly.”
Infants under 6 months of age should be kept out D) “Yearly mammograms are advised for all
of the sun or shielded from it. Even on a cloudy women over 35.”
day, the infant can be sunburned while near wa-
ter. A hat and light protective clothing should be
worn. Review Information: The correct answer is C:
“Once a woman reaches 50, she should have a
Question18 mammogram yearly.”
The nurse administers cimetidine (Tagamet) to a The American Cancer Society recommends a
79 year-old male with a gastric ulcer. Which pa- screening mammogram by age 40, every 1 - 2
rameter may be affected by this drug, and should years for women 40-49, and every year from age
be closely monitored by the nurse? 50. If there are family or personal health risks,
A) Blood pressure other assessments may be recommended.
B) Liver function
C) Mental status Question21
D) Hemoglobin The nurse is planning care for a client who is tak-
ing cyclosporin (Neoral). What would be an ap-
Review Information: The correct answer is C: propriate nursing diagnosis for this client?
Mental status A) Alteration in body image
The elderly are at risk for developing confusion B) High risk for infection
when taking cimetidine, a drug that interacts with C) Altered growth and development
many other medications. D) Impaired physical mobility

Question19
The nurse assesses the use of coping mecha- Review Information: The correct answer is B:
nisms by an adolescent 1 week after the client High risk for infection
had a motor vehicle accident resulting in multiple Cyclosporin (Neoral) inhibits normal immune re-
serious injuries. Which of these characteristics sponses. Clients receiving cyclosporin are at risk
are most likely to be displayed? for infection.
A) Ambivalence, dependence, demanding
B) Denial, projection, regression Question22
C) Intellectualization, rationalization, repression A client on telemetry begins having premature
D) Identification, assimilation, withdrawal ventricular beats (PVBs) at 12 per minute. In re-
viewing the most recent laboratory results, which
Review Information: The correct answer is B: would require immediate action by the nurse?
Denial, projection, regression A) Calcium 9 mg/dl
Helplessness and hopelessness may contribute B) Magnesium 2.5 mg/dl
to regressive, dependent behavior which often C) Potassium 2.5 mEq/L
occurs at any age with hospitalization. Deny- D) PTT 70 seconds
ing or minimizing the seriousness of the illness
is used to avoid facing the worst situation. Re-
call that denial is the initial step in the process of Review Information: The correct answer is C:
working through any loss. Potassium 2.5 mEq/L
The patient is at risk for ventricular dysrhythmias
Question20 when the potassium level is low.
A 52 year-old post menopausal woman asks the Daniels, R. (2003).
nurse how frequently she should have a mam-
mogram. What is the nurse’s best response? Question23
A) “Your doctor will advise you about your The nurse is caring for a client who is 4 days
risks.” post-op for a transverse colostomy. The client is
Collected by :DeepaRajesh [ 140 ]
rajesh.ks21@gmail.com
Kuwait
ready for discharge and asks the nurse to empty Anticholinergics
his colostomy pouch. What is the best response An anticholinergic medication will decrease gas-
by the nurse? tric emptying and the pressure on the lower es-
A) “You should be emptying the pouch yourself.” ophageal sphincter.
B) “Let me demonstrate to you how to empty the
pouch.” Question26
C) “What have you learned about emptying your A client is receiving a nitroglycerin infusion for
pouch?” unstable angina. What assessment would be a
D) “Show me what you have learned about emp- priority when monitoring the effects of this medi-
tying your pouch.” cation?
A) Blood pressure
B) Cardiac enzymes
Review Information: The correct answer is D: C) ECG analysis
“Show me what you have learned about empty- D) Respiratory rate
ing your pouch.”
Most adult learners obtain skills by participating
in the activities. Anxiety about discharge can be Review Information: The correct answer is A:
causing the client to forget that they have mas- Blood pressure
tered the skill of emptying the pouch. The client Since an effect of this drug is vasodilation, the
should show the nurse how the pouch is emp- client must be monitored for hypotension.
tied.
Question27
Question24 The nurse is caring for a 10 year-old child who
A 3 year-old child has tympanostomy tubes in has just been diagnosed with diabetes insipidus.
place. The child’s parent asks the nurse if he can The parents ask about the treatment prescribed,
swim in the family pool. The best response from vasopressin. A What is priority in teaching the
the nurse is child and family about this drug?
A) “Your child should not swim at all while the A) The child should carry a nasal spray for emer-
tubes are in place.” gency use
B) “Your child may swim in your own pool but not B) The family must observe the child for dehy-
in a lake or ocean.” dration
C) “Your child may swim if he wears ear plugs.” C) Parents should administer the daily intramus-
D) “Your child may swim anywhere.” cular injections
D) The client needs to take daily injections in the
short-term
Review Information: The correct answer is C:
“Your child may swim if he wears ear plugs.”
Review Information: The correct answer is A:
Water should not enter the ears. Children should The child should carry a nasal spray for emer-
use ear plugs when bathing or swimming and gency use
should not put their heads under the water. Diabetes insipidus results from reduced secre-
tion of the antidiuretic hormone, vasopressin.
Question25 The child will need to administer daily injections
The nurse is caring for a client with asthma who of vasopressin, and should have the nasal spray
has developed gastroesophageal reflux disease form of the medication readily available. A medi-
(GERD). Which of the following medications pre- cal alert tag should be worn.
scribed for the client may aggravate GERD?
A) Anticholinergics Question28
B) Corticosteroids A client diagnosed with cirrhosis is started on
C) Histamine blocker lactulose (Cephulac). The main purpose of the
D) Antibiotics drug for this client is to
A) add dietary fiber
Review Information: The correct answer is A: B) reduce ammonia levels
Collected by :DeepaRajesh [ 141 ]
rajesh.ks21@gmail.com
Kuwait
C) stimulate peristalsis
D) control portal hypertension Question31
A client has many delusions. As the nurse helps
the client prepare for breakfast the client com-
Review Information: The correct answer is B: ments “Don’t waste good food on me. I’m dying
reduce ammonia levels from this disease I have.” The appropriate re-
Lactulose blocks the absorption of ammonia from sponse would be
the GI tract and secondarily stimulates bowel A) “You need some nutritious food to help you
elimination. regain your weight.”
B) “None of the laboratory reports show that you
Question29 have any physical disease.”
The nurse is explaining the effects of cocaine C) “Try to eat a little bit, breakfast is the most
abuse to a pregnant client. Which of the follow- important meal of the day.”
ing must the nurse understand as a basis for D) “I know you believe that you have an incur-
teaching? able disease.”
A) Cocaine use can cause fetal growth retarda-
tion
B) The drug has been linked to neural tube de- Review Information: The correct answer is D:
fects “I know you believe that you have an incurable
C) Newborn withdrawal generally occurs imme- disease.”
diately after birth This response does not challenge the client’s
D) Breast feeding promotes positive parenting delusional system and thus forms an alliance by
behaviors providing reassurance of desire to help the cli-
ent.

Review Information: The correct answer is A: Question32


Cocaine use can cause fetal growth retardation A client with paranoid thoughts refuses to eat be-
Cocaine is vasoconstrictive, and this effect in cause of the belief that the food is poisoned. The
the placental vessels causes fetal hypoxia and appropriate statement at this time for the nurse
diminished growth. Other risks of continued co- to say is
caine use during pregnancy include preterm la- A) “Here, I will pour a little of the juice in a medi-
bor, congenital abnormalities, altered brain de- cine cup to drink it to show you that it is OK.”
velopment and subsequent behavioral problems B) “The food has been prepared in our kitchen
in the infant. and is not poisoned.”
C) “Let’s see if your partner could bring food from
Question30 home.”
A client has just been diagnosed with breast D) “If you don’t eat, I will have to suggest for you
cancer. The nurse enters the room and the cli- to be tube fed.”
ent tells the nurse that she is stupid. What is the
most therapeutic response by the nurse?
A) Explore what is going on with the client Review Information: The correct answer is C:
B) Accept the client’s statement without com- “Let’’s see if your partner could bring food from
ment home.”
C) Tell the client that the comment is inappropri- Reassurance is ineffective when a client is ac-
ate tively delusional. This option avoids both arguing
D) Leave the client’s room with the client and agreeing with the delusional
premise. Option D offers a logical response to
a primarily affective concern. When the client’s
Review Information: The correct answer is A: condition has improved, gentle negation of the
Explore what is going on with the client delusional premise can be employed.
Exploring feelings with the verbally aggressive
client helps to put angry feelings into words and Question33
then to engage in problem solving. A client with tuberculosis is started on Rifampin.
Collected by :DeepaRajesh [ 142 ]
rajesh.ks21@gmail.com
Kuwait
Which one of the following statements by the D) Combines safely with antihypertensives
nurse would be appropriate to include in teach-
ing? “You may notice:
A) an orange-red color to your urine.” Review Information: The correct answer is A:
B) your appetite may increase for the first Promotes sodium and chloride excretion
week.” Spironolactone promotes sodium and chloride
C) it is common to experience occasional sleep excretion while sparing potassium and decreas-
disturbances.” ing aldosterone levels. It had no effect on am-
D) if you take the medication with food, you may monia levels.
have nausea.”
Question36
A client was admitted to the psychiatric unit for
Review Information: The correct answer is A: severe depression. After several days, the cli-
an orange-red color to your urine.” ent continues to withdraw from the other clients.
Discoloration of the urine and other body fluids Which of these statements by the nurse would
may occur. It is a harmless response to the drug, be the most appropriate to promote interaction
but the patient needs to be aware it may hap- with other clients?
pen. A) “Your team here thinks it’s good for you to
spend time with others.”
Question34 B) “It is important for you to participate in group
A client tells the RN she has decided to stop tak- activities.”
ing sertraline (Zoloft) because she doesn’t like C) “Come with me so you can paint a picture to
the nightmares, sex dreams, and obsessions help you feel better.”
she’s experiencing since starting on the medi- D) “Come play Chinese Checkers with Gloria
cation. What is an appropriate response by the and me.”
nurse?
A) “It is unsafe to abruptly stop taking any pre- Review Information: The correct answer is D:
scribed medication.” “Come play Chinese Checkers with Gloria and
B) “Side effects and benefits should be discussed me.”
with your health care provider.” This gradually engages the client in interactions
C) “This medication should be continued despite with others in small groups rather than large
unpleasant symptoms.” groups. In addition, focusing on an activity is less
D) “Many medications have potential side ef- anxiety-provoking than unstructured discussion.
fects.” The statement is an example of a positive be-
havioral expectation.

Review Information: The correct answer is Question37


A: “It is unsafe to abruptly stop taking any pre- The nurse is teaching a school-aged child and
scribed medication.” family about the use of inhalers prescribed for
Abrupt withdrawal may occasionally cause sero- asthma. What is the best way to evaluate effec-
tonin syndrome, consisting of lethargy, nausea, tiveness of the treatments?
headache, fever, sweating and chills. A slow with-
drawal may be prescribed with sertraline to avoid A) Rely on child’s self-report
dizziness, nausea, vomiting, and diarrhea. B) Use a peak-flow meter
C) Note skin color changes
Question35 D) Monitor pulse rate
A client is admitted to the hospital with findings of
liver failure with ascites. The health care provider
orders spironolactone (Aldactone). What is the Review Information: The correct answer is B:
pharmacological effect of this medication? Use a peak-flow meter
A) Promotes sodium and chloride excretion The peak flowmeter, if used correctly, shows ef-
B) Increases aldosterone levels fectiveness of inhalants.
C) Depletes potassium reserves
Collected by :DeepaRajesh [ 143 ]
rajesh.ks21@gmail.com
Kuwait
Question38 Conduct a diet history to determine her normal
The nurse is teaching a client about the toxicity eating routines.
of digoxin. Which one of the following statements
made by the client to the nurse indicates more Assessment is always the first step in planning
teaching is needed? teaching for any client. A thorough and accurate
A) “I may experience a loss of appetite.” history is essential for gathering the needed in-
B) “I can expect occasional double vision.” formation.
C) “Nausea and vomiting may last a few days.”
D) “I must report a bounding pulse of 62 imme- 0 comments
diately.”
Labels: free nclex-rn sample review questions,
nclex-rn practice test questions, nursing review
Review Information: The correct answer is D:
“I must report a bounding pulse of 62 immedi- Free NCLEX-RN Sample Test Ques-
ately.” tions For Nursing Review (Part 1)
Slow heart rate is related to increased cardiac
output and an intended effect of digoxin. The ide- These are sample nursing review questions and
al heart rate is above 60 BPM with digoxin. The not actual test questions made for educational
client needs further teaching. and practice test purposes only. 75 questions
have been posted here with answer keys.

Question39 Question1
Which of the following assessments by the nurse A client has been hospitalized after an automo-
would indicate that the client is having a possible bile accident. A full leg cast was applied in the
adverse response to the isoniazid (INH)? emergency room. The most important reason for
A) Severe headache the nurse to elevate the casted leg is to
B) Appearance of jaundice A) Promote the client’s comfort
C) Tachycardia B) Reduce the drying time
D) Decreased hearing C) Decrease irritation to the skin
D) Improve venous return
Review Information: The correct answer is B:
Appearance of jaundice Review Information: The correct answer is D:
Clients receiving INH therapy are at risk for de- Improve venous return. Elevating the leg both
veloping drug induced hepatitis. The appearance improves venous return and reduces swelling.
of jaundice may indicate that the client has liver Client comfort will be improved as well.
damage.

Question40 Question2
The nurse is beginning nutritional counseling/ The nurse is reviewing with a client how to col-
teaching with a pregnant woman. What is the ini- lect a clean catch urine specimen. What is the
tial step in this interaction? appropriate sequence to teach the client?

A) Teach her how to meet the needs of self and A) Clean the meatus, begin voiding, then catch
her family urine stream
B) Explain the changes in diet necessary for B) Void a little, clean the meatus, then collect
pregnant women specimen
C) Questionher understanding and use C) Clean the meatus, then urinate into container
of the food pyramid D) Void continuously and catch some of the
D) Conduct a diet history to determine her nor- urine
mal eating routines
Review Information: The correct answer is
A: Clean the meatus, begin voiding, then catch
Review Information: The correct answer is D: urine stream. A clean catch urine is difficult to
Collected by :DeepaRajesh [ 144 ]
rajesh.ks21@gmail.com
Kuwait
obtain and requires clear directions. Instructing consciousness. Any score less than 13 indicates
the client to carefully clean the meatus, then void a neurological impairment. Using the term coma-
naturally with a steady stream prevents surface tose provides too much room for interpretation
bacteria from contaminating the urine specimen. and is not very precise.
As starting and stopping flow can be difficult,
once the client begins voiding it’’s best to just slip
the container into the stream. Other responses Question5
do not reflect correct technique. When caring for a client receiving warfarin so-
dium (Coumadin), which lab test would the nurse
monitor to determine therapeutic response to the
Question3 drug?
Following change-of-shift report on an orthoped- A) Bleeding time
ic unit, which client should the nurse see first? B) Coagulation time
A) 16 year-old who had an open reduction of a C) Prothrombin time
fractured wrist 10 hours ago D) Partial thromboplastin time
B) 20 year-old in skeletal traction for 2 weeks
since a motor cycle accident Review Information: The correct answer is
C) 72 year-old recovering from surgery after a C: Prothrombin time. Coumadin is ordered dai-
hip replacement 2 hours ago ly, based on the client’’s prothrombin time (PT).
D) 75 year-old who is in skin traction prior to This test evaluates the adequacy of the extrinsic
planned hip pinning surgery. system and common pathway in the clotting cas-
cade; Coumadin affects the Vitamin K depend-
Review Information: The correct answer is C: ent clotting factors.
72 year-old recovering from surgery after a hip
replacement 2 hours ago. Look for the client who
has the most imminent risks and acute vulnerabil- Question6
ity. The client who returned from surgery 2 hours A client with moderate persistent asthma is ad-
ago is at risk for life threatening hemorrhage and mitted for a minor surgical procedure. On ad-
should be seen first. The 16 year-old should be mission the peak flow meter is measured at 480
seen next because it is still the first post-op day. liters/minute. Post-operatively the client is com-
The 75 year-old is potentially vulnerable to age- plaining of chest tightness. The peak flow has
related physical and cognitive consequences in dropped to 200 liters/minute. What should the
skin traction should be seen next. The client who nurse do first?
can safely be seen last is the 20 year-old who is A) Notify both the surgeon and provider
2 weeks post-injury. B) Administer the prn dose of albuterol
C) Apply oxygen at 2 liters per nasal cannula
D) Repeat the peak flow reading in 30 minutes
Question4
A client with Guillain Barre is in a nonresponsive Review Information: The correct answer is B:
state, yet vital signs are stable and breathing is Administer the prn dose of albuterol. Peak flow
independent. What should the nurse document monitoring during exacerbations of asthma is
to most accurately describe the client’s condi- recommended for clients with moderate-to-se-
tion? vere persistent asthma to determine the severity
A) Comatose, breathing unlabored of the exacerbation and to guide the treatment. A
B) Glascow Coma Scale 8, respirations regular peak flow reading of less than 50% of the client’’s
C) Appears to be sleeping, vital signs stable baseline reading is a medical alert condition and
D) Glascow Coma Scale 13, no ventilator re- a short-acting beta-agonist must be taken imme-
quired diately.

Review Information: The correct answer is


B: Glascow Coma Scale 8, respirations regular. Question7
The Glascow Coma Scale provides a standard A client had 20 mg of Lasix (furosemide) PO at
reference for assessing or monitoring level of 10 AM. Which would be essential for the nurse to
Collected by :DeepaRajesh [ 145 ]
rajesh.ks21@gmail.com
Kuwait
include at the change of shift report? lobe. Acute asthma is characterized by expiratory
A) The client lost 2 pounds in 24 hours wheezes caused by obstruction of the airways.
B) The client’s potassium level is 4 mEq/liter. Wheezes are a high pitched musical sounds pro-
C) The client’s urine output was 1500 cc in 5 duced by air moving through narrowed airways.
hours Clients often associate wheezes with the feeling
D) The client is to receive another dose of Lasix of tightness in the chest. However, sudden ces-
at 10 PM sation of wheezing is an ominous or bad sign
that indicates an emergency -- the small airways
Review Information: The correct answer is C: are now collapsed.
The client’s urine output was 1500 cc in 5 hours.
Although all of these may be correct information
to include in report, the essential piece would be Question10
the urine output. During the initial home visit, a nurse is discussing
the care of a client newly diagnosed with Alzhe-
imer’s disease with family members. Which of
Question8 these interventions would be most helpful at this
A client has been tentatively diagnosed with time?
Graves’ disease (hyperthyroidism). Which of A) leave a book about relaxation techniques
these findings noted on the initial nursing assess- B) write out a daily exercise routine for them to
ment requires quick intervention by the nurse? assist the client to do
A) a report of 10 pounds weight loss in the last C) list actions to improve the client’s daily nutri-
month tional intake
B) a comment by the client “I just can’t sit still.” D) suggest communication strategies
C) the appearance of eyeballs that appear to
“pop” out of the client’s eye sockets Review Information: The correct answer is D:
D) a report of the sudden onset of irritability in suggest communication strategies. Alzheimer’’s
the past 2 weeks disease, a progressive chronic illness, greatly
challenges caregivers. The nurse can be of
Review Information: The correct answer is C: greatest assistance in helping the family to use
the appearance of eyeballs that appear to “pop” communication strategies to enhance their ability
out of the client’’s eye sockets. Exophthalmos or to relate to the client. By use of select verbal and
protruding eyeballs is a distinctive characteristic nonverbal communication strategies the family
of Graves’’ Disease. It can result in corneal abra- can best support the client’s strengths and cope
sions with severe eye pain or damage when the with any aberrant behavior.
eyelid is unable to blink down over the protruding
eyeball. Eye drops or ointment may be needed.
Question11
An 80 year-old client admitted with a diagnosis
Question9 of possible cerebral vascular accident has had a
The nurse has performed the initial assessments blood pressure from 160/100 to 180/110 over the
of 4 clients admitted with an acute episode of past 2 hours. The nurse has also noted increased
asthma. Which assessment finding would cause lethargy. Which assessment finding should the
the nurse to call the provider immediately? nurse report immediately to the provider?
A) prolonged inspiration with each breath A) Slurred speech
B) expiratory wheezes that are suddenly absent B) Incontinence
in 1 lobe C) Muscle weakness
C) expectoration of large amounts of purulent D) Rapid pulse
mucous
D) appearance of the use of abdominal muscles Review Information: The correct answer is A:
for breathing Slurred speech. Changes in speech patterns and
level of conscious can be indicators of continued
Review Information: The correct answer is B: intracranial bleeding or extension of the stroke.
expiratory wheezes that are suddenly absent in 1 Further diagnostic testing may be indicated.
Collected by :DeepaRajesh [ 146 ]
rajesh.ks21@gmail.com
Kuwait
client. The potassium and PaO2 levels are near
normal.
Question12
A school-aged child has had a long leg (hip to
ankle) synthetic cast applied 4 hours ago. Which Question14
statement from the parent indicates that teach- The nurse is preparing a client with a deep vein
ing has been inadequate? thrombosis (DVT) for a Venous Doppler evalua-
A) “I will keep the cast uncovered for the next tion. Which of the following would be necessary
day to prevent burning of the skin.” for preparing the client for this test?
B) “I can apply an ice pack over the area to re- A) Client should be NPO after midnight
lieve itching inside the cast.” B) Client should receive a sedative medication
C) “The cast should be propped on at least 2 pil- prior to the test
lows when my child is lying down.” C) Discontinue anti-coagulant therapy prior to
D) “I think I remember that my child should not the test
stand until after 72 hours.” D) No special preparation is necessary

Review Information: The correct answer is D: Review Information: The correct answer is
“I think I remember that my child should not stand D: No special preparation is necessary. This is
until after 72 hours.”. Synthetic casts will typical- a non-invasive procedure and does not require
ly set up in 30 minutes and dry in a few hours. preparation other than client education.
Thus, the client may stand within the initial 24
hours. With plaster casts, the set up and drying
time, especially in a long leg cast which is thicker Question15
than an arm cast, can take up to 72 hours. Both A client is admitted with infective endocarditis
types of casts give off a lot of heat when drying (IE). Which finding would alert the nurse to a
and it is preferable to keep the cast uncovered complication of this condition?
for the first 24 hours. Clients may complain of a A) dyspnea
chill from the wet cast and therefore can simply B) heart murmur
be covered lightly with a sheet or blanket. Apply- C) macular rash
ing ice is a safe method of relieving the itching. D) hemorrhage

Review Information: The correct answer is


Question13 B: heart murmur. Large, soft, rapidly developing
Which blood serum finding in a client with dia- vegetations attach to the heart valves. They have
betic ketoacidosis alerts the nurse that immedi- a tendency to break off, causing emboli and leav-
ate action is required? ing ulcerations on the valve leaflets. These em-
A) pH below 7.3 boli produce findings of cardiac murmur, fever,
B) Potassium of 5.0 anorexia, malaise and neurologic sequelae of
C) HCT of 60 emboli. Furthermore, the vegetations may travel
D) Pa O2 of 79% to various organs such as spleen, kidney, coro-
nary artery, brain and lungs, and obstruct blood
Review Information: The correct answer is C: flow.
HCT of 60. This high hematocrit is indicative of
severe dehydration which requires priority atten-
tion in diabetic ketoacidosis. Without sufficient Question16
hydration, all systems of the body are at risk The nurse explains an autograft to a client sched-
for hypoxia from a lack of or sluggish circula- uled for excision of a skin tumor. The nurse knows
tion. In the absence of insulin, which facilitates the client understands the procedure when the
the transport of glucose into the cell, the body client says, “I will receive tissue from
breaks down fats and proteins to supply energy A) a tissue bank.”
ketones, a by-product of fat metabolism. These B) a pig.”
accumulate causing metabolic acidosis (pH < C) my thigh.”
7.3), which would be the second concern for this D) synthetic skin.”
Collected by :DeepaRajesh [ 147 ]
rajesh.ks21@gmail.com
Kuwait
B) The overview cardiac rehabilitation
Review Information: The correct answer is C: C) Medication and diet guideline
my thigh.”. Autografts are done with tissue trans- D) Activity and rest guidelines
planted from the client’’s own skin.
Review Information: The correct answer is A:
Daily needs and concerns. At 2 days post-MI, the
Question17 client’s education should be focused on the im-
A client is admitted to the emergency room fol- mediate needs and concerns for the day.
lowing an acute asthma attack. Which of the fol-
lowing assessments would be expected by the
nurse? Question20
A 3 year-old child is brought to the clinic by his
A) Diffuse expiratory wheezing grandmother to be seen for “scratching his bot-
B) Loose, productive cough tom and wetting the bed at night.” Based on these
C) No relief from inhalant complaints, the nurse would initially assess for
D) Fever and chills which problem?
A) allergies
Review Information: The correct answer is A: B) scabies
Diffuse expiratory wheezing. In asthma, the air- C) regression
ways are narrowed, creating difficulty getting air D) pinworms
in. A wheezing sound results.
Review Information: The correct answer is
D: pinworms. Signs of pinworm infection include
Question18 intense perianal itching, poor sleep patterns,
A client has been admitted with a fractured femur general irritability, restlessness, bed-wetting,
and has been placed in skeletal traction. Which distractibility and short attention span. Scabies
of the following nursing interventions should re- is an itchy skin condition caused by a tiny, eight-
ceive priority? legged burrowing mite called Sarcoptes scabiei .
A) Maintaining proper body alignment The presence of the mite leads to intense itching
B) Frequent neurovascular assessments of the in the area of its burrows.
affected leg
C) Inspection of pin sites for evidence of drain-
age or inflammation Question21
D) Applying an over-bed trapeze to assist the cli- The nurse is caring for a newborn with tra-
ent with movement in bed cheoesophageal fistula. Which nursing diagno-
sis is a priority?
Review Information: The correct answer is A) Risk for dehydration
B: Frequent neurovascular assessments of the B) Ineffective airway clearance
affected leg. The most important activity for the C) Altered nutrition
nurse is to assess neurovascular status. Com- D) Risk for injury
partment syndrome is a serious complication of
fractures. Prompt recognition of this neurovascu- Review Information: The correct answer is B:
lar problem and early intervention may prevent Ineffective airway clearance. The most common
permanent limb damage. form of TEF is one in which the proximal esopha-
geal segment terminates in a blind pouch and
the distal segment is connected to the trachea
Question19 or primary bronchus by a short fistula at or near
The nurse is assigned to care for a client who the bifurcation. Thus, a priority is maintaining an
had a myocardial infarction (MI) 2 days ago. The open airway, preventing aspiration. Other nurs-
client has many questions about this condition. ing diagnoses are then addressed.
What area is a priority for the nurse to discuss at
this time?
A) Daily needs and concerns Question22
Collected by :DeepaRajesh [ 148 ]
rajesh.ks21@gmail.com
Kuwait
The nurse is developing a meal plan that would D) Place NPO for 24 hours, then rehydrate with
provide the maximum possible amount of iron for milk and water
a child with anemia. Which dinner menu would
be best? Review Information: The correct answer is B:
A) Fish sticks, french fries, banana, cookies, Continue with the regular diet and include oral
milk rehydration fluids. Current recommendations for
B) Ground beef patty, lima beans, wheat roll, rai- mild to moderate diarrhea are to maintain a nor-
sins, milk mal diet with fluids to rehydrate.
C) Chicken nuggets, macaroni, peas, canta-
loupe, milk
D) Peanut butter and jelly sandwich, apple slic- Question25
es, milk The nurse is teaching parents about the appro-
priate diet for a 4 month-old infant with gastro-
Review Information: The correct answer is B: enteritis and mild dehydration. In addition to oral
Ground beef patty, lima beans, wheat roll, raisins, rehydration fluids, the diet should include
milk. Iron rich foods include red meat, fish, egg
yolks, green leafy vegetables, legumes, whole A) formula or breast milk
grains, and dried fruits such as raisins. This din- B) broth and tea
ner is the best choice: It is high in iron and is ap- C) rice cereal and apple juice
propriate for a toddler. D) gelatin and ginger ale

Review Information: The correct answer is A:


Question23 formula or breast milk. The usual diet for a young
The nurse admitting a 5 month-old who vomited infant should be followed.
9 times in the past 6 hours should observe for
signs of which overall imbalance?
A) Metabolic acidosis Question26
B) Metabolic alkalosis A child is injured on the school playground and
C) Some increase in the serum hemoglobin appears to have a fractured leg. The first action
D) A little decrease in the serum potassium the school nurse should take is

Review Information: The correct answer is B: A) call for emergency transport to the hospital
Metabolic alkalosis. Vomiting causes loss of acid B) immobilize the limb and joints above and be-
from the stomach. Prolonged vomiting can re- low the injury
sult in excess loss of acid and lead to metabolic C) assess the child and the extent of the injury
alkalosis. Findings include irritability, increased D) apply cold compresses to the injured area
activity, hyperactive reflexes, muscle twitching
and elevated pulse. Options C and D are correct Review Information: The correct answer is
answers but not the best answers since they are C: assess the child and the extent of the injury.
too general. When applying the nursing process, assessment
is the first step in providing care. The “5 Ps” of
vascular impairment can be used as a guide
Question24 (pain, pulse, pallor, paresthesia, paralysis).
A two year-old child is brought to the provider’s
office with a chief complaint of mild diarrhea for
two days. Nutritional counseling by the nurse Question27
should include which statement? The mother of a 3 month-old infant tells the nurse
A) Place the child on clear liquids and gelatin for that she wants to change from formula to whole
24 hours milk and add cereal and meats to the diet. What
B) Continue with the regular diet and include oral should be emphasized as the nurse teaches
rehydration fluids about infant nutrition?
C) Give bananas, apples, rice and toast as toler-
ated A) Solid foods should be introduced at 3-4
Collected by :DeepaRajesh [ 149 ]
rajesh.ks21@gmail.com
Kuwait
months and symptomatic treatment. The priority of care
B) Whole milk is difficult for a young infant to di- is pain relief. In a 12 year-old child, client control-
gest led analgesia promotes maximum comfort.
C) Fluoridated tap water should be used to dilute
milk
D) Supplemental apple juice can be used be- Question30
tween feedings The nurse is performing a physical assessment
on a toddler. Which of the following actions
Review Information: The correct answer is B: should be the first?
Whole milk is difficult for a young infant to digest.
Cow’’s milk is not given to infants younger than A) Perform traumatic procedures
1 year because the tough, hard curd is difficult to B) Use minimal physical contact
digest. In addition, it contains little iron and cre- C) Proceed from head to toe
ates a high renal solute load. D) Explain the exam in detail

Question28 Review Information: The correct answer is B:


The nurse is preparing a handout on infant feed- Use minimal physical contact. The nurse should
ing to be distributed to families visiting the clinic. approach the toddler slowly and use minimal
Which notation should be included in the teach- physical contact initially so as to gain the tod-
ing materials? dler’’s cooperation. Be flexible in the sequence
of the exam, and give only brief simple explana-
A) Solid foods are introduced one at a time be- tions just prior to the action.
ginning with cereal
B) Finely ground meat should be started early to
provide iron Question31
C) Egg white is added early to increase protein What finding signifies that children have attained
intake the stage of concrete operations (Piaget)?
D) Solid foods should be mixed with formula in
a bottle A) Explores the environment with the use of sight
and movement
Review Information: The correct answer is A: B) Thinks in mental images or word pictures
Solid foods are introduced one at a time begin- C) Makes the moral judgment that “stealing is
ning with cereal. Solid foods should be added wrong”
one at a time between 4-6 months. If the infant is D) Reasons that homework is time-consuming
able to tolerate the food, another may be added yet necessary
in a week. Iron fortified cereal is the recommend-
ed first food. Review Information: The correct answer is
C: Makes the moral judgment that “stealing is
wrong”. The stage of concrete operations is de-
Question29 picted by logical thinking and moral judgments.
The nurse planning care for a 12 year-old child
with sickle cell disease in a vaso-occlusive crisis
of the elbow should include which one of the fol- Question32
lowing as a priority? The mother of a child with a neural tube defect
asks the nurse what she can do to decrease the
A) Limit fluids chances of having another baby with a neural
B) Client controlled analgesia tube defect. What is the best response by the
C) Cold compresses to elbow nurse?
D) Passive range of motion exercise
A) “Folic acid should be taken before and after
Review Information: The correct answer is B: conception.”
Client controlled analgesia. Management of a B) “Multivitamin supplements are recommended
sickle cell crisis is directed towards supportive during pregnancy.”
Collected by :DeepaRajesh [ 150 ]
rajesh.ks21@gmail.com
Kuwait
C) “A well balanced diet promotes normal fetal old child. Which statement by the parent indi-
development.” cates understanding of appropriate precautions
D) “Increased dietary iron improves the health of to take with the children?
mother and fetus.”
A) “I strap the infant car seat on the front seat to
Review Information: The correct answer is face backwards.”
A: “Folic acid should be taken before and after B) “I place my infant in the middle of the living
conception.”. The American Academy of Pedi- room floor on a blanket to play with my four year-
atrics recommends that all childbearing women old while I make supper in the kitchen.”
increase folic acid from dietary sources and/or C) “My sleeping baby lies so cute in the crib with
supplements. There is evidence that increased the little buttocks stuck up in the air while the four
amounts of folic acid prevents neural tube de- year-old naps on the sofa.”
fects. D) “I have the four year-old hold and help feed
the four month-old a bottle in the kitchen while I
make supper.”
Question33
The provider orders Lanoxin (digoxin) 0.125 mg Review Information: The correct answer is
PO and furosemide 40 mg every day. Which of D: “I have the four year-old hold and help feed
these foods would the nurse reinforce for the cli- the four month-old a bottle in the kitchen while I
ent to eat at least daily? make supper.”. The infant seat is to be placed on
the rear seat. Small children and infants are not
A) Spaghetti to be left unsupervised. Infants are
B) Watermelon
C) Chicken
D) Tomatoes Question36
The nurse admits a 7 year-old to the emergency
Review Information: The correct answer is B: room after a leg injury. The x-rays show a femur
Watermelon. Watermelon is high in potassium fracture near the epiphysis. The parents ask what
and will replace potassium lost by the diuretic. will be the outcome of this injury. The appropriate
The other foods are not high in potassium. response by the nurse should be which of these
statements?

Question34 A) “The injury is expected to heal quickly be-


While teaching the family of a child who will take cause of thin periosteum.”
phenytoin (Dilantin) regularly for seizure control, B) “In some instances the result is a retarded
it is most important for the nurse to teach them bone growth.”
about which of the following actions? C) “Bone growth is stimulated in the affected
leg.”
A) Maintain good oral hygiene and dental care D) “This type of injury shows more rapid union
B) Omit medication if the child is seizure free than that of younger children.”
C) Administer acetaminophen to promote sleep
D) Serve a diet that is high in iron Review Information: The correct answer is B:
“In some instances the result is a retarded bone
Review Information: The correct answer is growth.”. An epiphyseal (growth) plate fracture
A: Maintain good oral hygiene and dental care. in a 7 year-old often results in retarded bone
Swollen and tender gums occur often with use of growth. The leg often will be different in length
phenytoin. Good oral hygiene and regular visits than the uninjured leg.
to the dentist should be emphasized.

Question37
Question35 The parents of a 4 year-old hospitalized child tell
The nurse is offering safety instructions to a par- the nurse, “We are leaving now and will be back
ent with a four month-old infant and a four year- at 6 PM.” A few hours later the child asks the
Collected by :DeepaRajesh [ 151 ]
rajesh.ks21@gmail.com
Kuwait
nurse when the parents will come again. What is several minutes will diminish the lead contami-
the best response by the nurse? nation.

A) “They will be back right after supper.”


B) “In about 2 hours, you will see them.” Question40
C) “After you play awhile, they will be here.” Which of the following manifestations observed
D) “When the clock hands are on 6 and 12.” by the school nurse confirms the presence of pe-
diculosis capitis in students?
Review Information: The correct answer is A:
“They will be back right after supper.”. Time is A) Scratching the head more than usual
not completely understood by a 4 year-old. Pre- B) Flakes evident on a student’s shoulders
schoolers interpret time with their own frame of C) Oval pattern occipital hair loss
reference. Thus, it is best to explain time in rela- D) Whitish oval specks sticking to the hair
tionship to a known, common event.
Review Information: The correct answer is D:
Whitish oval specks sticking to the hair. Diagno-
Question38 sis of pediculosis capitis is made by observation
The nurse is giving instructions to the parents of the white eggs (nits) firmly attached to the
of a child with cystic fibrosis. The nurse would hair shafts. Treatment can include application of
emphasize that pancreatic enzymes should be a medicated shampoo with lindane for children
taken over 2 years of age, and meticulous combing
A) once each day and removal of all nits.
B) 3 times daily after meals
C) with each meal or snack
D) each time carbohydrates are eaten Question41
When interviewing the parents of a child with
Review Information: The correct answer is C: asthma, it is most important to assess the child’s
with each meal or snack. Pancreatic enzymes environment for what factor?
should be taken with each meal and every snack
to allow for digestion of all foods that are eaten. A) Household pets
B) New furniture
C) Lead based paint
Question39 D) Plants such as cactus
A nurse is providing a parenting class to individu-
als living in a community of older homes. In dis- Review Information: The correct answer is A:
cussing formula preparation, which of the follow- Household pets. Animal dander is a very com-
ing is most important to prevent lead poisoning? mon allergen affecting persons with asthma.
Other triggers may include pollens, carpeting
A) Use ready-to-feed commercial infant formula and household dust.
B) Boil the tap water for 10 minutes prior to pre-
paring the formula
C) Let tap water run for 2 minutes before adding Question42
to concentrate The mother of a 2 month-old baby calls the nurse
D) Buy bottled water labeled “lead free” to mix 2 days after the first DTaP, IPV, Hepatitis B and
the formula HIB immunizations. She reports that the baby
feels very warm, cries inconsolably for as long as
Review Information: The correct answer is C: 3 hours, and has had several shaking spells. In
Let tap water run for 2 minutes before adding to addition to referring her to the emergency room,
concentrate. Use of lead-contaminated water to the nurse should document the reaction on the
prepare formula is a major source of poisoning baby’s record and expect which immunization to
in infants. Drinking water may be contaminated be most associated with the findings the infant is
by lead from old lead pipes or lead solder used displaying?
in sealing water pipes. Letting tap water run for
Collected by :DeepaRajesh [ 152 ]
rajesh.ks21@gmail.com
Kuwait
A) DTaP port persons. To communicate in a therapeutic
B) Hepatitis B manner, the nurse’’s goal is to help the couple
C) Polio begin the grief process by suggesting they talk
D) H. Influenza to each other, seek family, friends and support
groups to listen to their feelings.
Review Information: The correct answer is A:
DTaP. The majority of reactions occur with the
administration of the DTaP vaccination. Contra- Question45
dictions to giving repeat DTaP immunizations in- The nurse is performing a pre-kindergarten phys-
clude the occurrence of severe side effects after ical on a 5 year-old. The last series of vaccines
a previous dose as well as signs of encephalop- will be administered. What is the preferred site
athy within 7 days of the immunization. for injection by the nurse?

A) vastus intermedius
Question43 B) gluteus maximus
The mother of a 2 year-old hospitalized child asks C) vastus lateralis
the nurse’s advice about the child’s screaming D) dorsogluteaI
every time the mother gets ready to leave the
hospital room. What is the best response by the Review Information: The correct answer is C:
nurse? vastus lateralis. Vastus lateralis, a large and well
developed muscle, is the preferred site, since it is
A) “I think you or your partner needs to stay with removed from major nerves and blood vessels.
the child while in the hospital.”
B) “Oh, that behavior will stop in a few days.”
C) “Keep in mind that for the age this is a normal Question46
response to being in the hospital.” A 7 month pregnant woman is admitted with com-
D) “You might want to “sneak out” of the room plaints of painless vaginal bleeding over several
once the child falls asleep.” hours. The nurse should prepare the client for an
immediate
Review Information: The correct answer is C:
“Keep in mind that for the age this is a normal A) Non stress test
response to being in the hospital.”. The protest B) Abdominal ultrasound
phase of separation anxiety is a normal response C) Pelvic exam
for a child this age. In toddlers, ages 1 to 3, sepa- D) X-ray of abdomen
ration anxiety is at its peak
Review Information: The correct answer is B:
Abdominal ultrasound. The standard for diagno-
Question44 sis of placenta previa, which is suggested in the
A couple experienced the loss of a 7 month-old client’’s history of painless bleeding, is abdomi-
fetus. In planning for discharge, what should the nal ultrasound.
nurse emphasize?

A) To discuss feelings with each other and use Question47


support persons A nurse entering the room of a postpartum moth-
B) To focus on the other healthy children and er observes the baby lying at the edge of the
move through the loss bed while the woman sits in a chair. The mother
C) To seek causes for the fetal death and come states “This is not my baby, and I do not want it.”
to some safe conclusion After repositioning the child safely, the nurse’s
D) To plan for another pregnancy within 2 years best response is
and maintain physical health
A) “This is a common occurrence after birth, but
Review Information: The correct answer is A: you will come to accept the baby.”
To discuss feelings with each other and use sup- B) “Many women have postpartum blues and
Collected by :DeepaRajesh [ 153 ]
rajesh.ks21@gmail.com
Kuwait
need some time to love the baby.” when the effective arterial blood volume falls.
C) “What a beautiful baby! Her eyes are just like Examples of this phenomena include a drop in
yours.” circulating blood volume as in a cardiac arrest
D) “You seem upset; tell me what the pregnancy state or in low cardiac perfusion states such as
and birth were like for you.” congestive heart failure associated with a cardi-
omyopathy. Close observation of hourly urinary
Review Information: The correct answer is D: output is necessary for early detection of this
“You seem upset; tell me what the pregnancy and condition.
birth were like for you.”. A non-judgmental, open
ended response facilitates dialogue between the
client and nurse. Question50
A client is admitted to the rehabilitation unit fol-
lowing a cerebral vascular accident (CVA) and
Question48 mild dysphagia. The most appropriate interven-
The nurse notes that a 2 year-old child recov- tion for this client is to
ering from a tonsillectomy has an temperature
of 98.2 degrees Fahrenheit at 8:00 AM. At 10:00 A) position client in upright position while eating
AM the child’s parent reports that the child “feels B) place client on a clear liquid diet
very warm” to touch. The first action by the nurse C) tilt head back to facilitate swallowing reflex
should be to D) offer finger foods such as crackers or pret-
zels
A) reassure the parent that this is normal
B) offer the child cold oral fluids Review Information: The correct answer is A:
C) reassess the child’s temperature position client in upright position while eating. An
D) administer the prescribed acetaminophen upright position facilitates proper chewing and
swallowing.
Review Information: The correct answer is
C: reassess the child’’s temperature. A child’’s
temperature may have rapid fluctuations. The Question51
nurse should listen to and show respect for what A 72 year-old client with osteomyelitis requires a
parents say. Parental caretakers are often quite 6 week course of intravenous antibiotics. In plan-
sensitive to variations in their children’’s condi- ning for home care, what is the most important
tion that may not be immediately evident to oth- action by the nurse?
ers.
A) Investigating the client’s insurance coverage
for home IV antibiotic therapy
Question49 B) Determining if there are adequate hand wash-
The nurse is caring for a client who was success- ing facilities in the home
fully resuscitated from a pulseless dysrhythmia. C) Assessing the client’s ability to participate in
Which of the following assessments is critical for self care and/or the reliability of a caregiver
the nurse to include in the plan of care? D) Selecting the appropriate venous access de-
vice
A) hourly urine output
B) white blood count Review Information: The correct answer is C:
C) blood glucose every 4 hours Assessing the client’’s ability to participate in self
D) temperature every 2 hours care and/or the reliability of a caregiver. The cog-
nitive ability of the client as well as the availability
Review Information: The correct answer is A: and reliability of a caregiver must be assessed to
hourly urine output. Clients who have had an epi- determine if home care is a feasible option.
sode of decreased glomerular perfusion are at
risk for pre-renal failure. This is caused by any
abnormal decline in kidney perfusion that reduc- Question52
es glomerular perfusion. Pre-renal failure occurs A nurse administers the influenza vaccine to a
Collected by :DeepaRajesh [ 154 ]
rajesh.ks21@gmail.com
Kuwait
client in a clinic. Within 15 minutes after the im- Tardive dyskinesia. Signs of tardive dyskinesia
munization was given, the client complains of include smacking lips, grinding of teeth and “fly
itchy and watery eyes, increased anxiety, and catching” tongue movements. These findings are
difficulty breathing. The nurse expects that the often described as Parkinsonian.
first action in the sequence of care for this client
will be to
Question55
A) Maintain the airway Which of the following findings contraindicate the
B) Administer epinephrine 1:1000 as ordered use of haloperidol (Haldol) and warrant withhold-
C) Monitor for hypotension with shock ing the dose?
D) Administer diphenhydramine as ordered A) Drowsiness, lethargy, and inactivity
B) Dry mouth, nasal congestion, and blurred vi-
Review Information: The correct answer is B: sion
Administer epinephrine 1:1000 as ordered. All C) Rash, blood dyscrasias, severe depression
the answers are correct given the circumstances, D) Hyperglycemia, weight gain, and edema
but the priority is to administer the epinephrine,
then maintain the airway. In the early stages of Review Information: The correct answer is
anaphylaxis, when the patient has not lost con- C: Rash, blood dyscrasias, severe depression.
sciousness and is normotensive, administering Rash and blood dyscrasias are side effects of
the epinephrine is first, and applying the oxygen, anti-psychotic drugs. A history of severe depres-
and watching for hypotension and shock, are lat- sion is a contraindication to the use of neurolep-
er responses. The prevention of a severe crisis tics.
is maintained by using diphenhydramine.

Question56
Question53 The nurse is reinforcing teaching to a 24 year-old
The nurse instructs the client taking dexametha- woman receiving acyclovir (Zovirax) for a Herpes
sone (Decadron) to take it with food or milk. The Simplex Virus type 2 infection. Which of these in-
physiological basis for this instruction is that the structions should the nurse give the client?
medication
A) retards pepsin production A) Complete the entire course of the medication
B) stimulates hydrochloric acid production for an effective cure
C) slows stomach emptying time B) Begin treatment with acyclovir at the onset of
D) decreases production of hydrochloric acid symptoms of recurrence
C) Stop treatment if she thinks she may be preg-
Review Information: The correct answer is nant to prevent birth defects
B: stimulates hydrochloric acid production. Dec- D) Continue to take prophylactic doses for at
adron increases the production of hydrochloric least 5 years after the diagnosis
acid, which may cause gastrointestinal ulcers.
Review Information: The correct answer is
B: Begin treatment with acyclovir at the onset
Question54 of symptoms of recurrence. When the client is
A client receiving chlorpromazine HCL (Thora- aware of early symptoms, such as pain, itching
zine) is in psychiatric home care. During a home or tingling, treatment is very effective. Medica-
visit the nurse observes the client smacking her tions for herpes simplex do not cure the disease;
lips alternately with grinding her teeth. The nurse they simply decrease the level of symptoms.
recognizes this assessment finding as what?
A) Dystonia
B) Akathisia Question57
C) Brady dyskinesia A 14 month-old child ingested half a bottle of
D) Tardive dyskinesia aspirin tablets. Which of the following would the
nurse expect to see in the child?
Review Information: The correct answer is D:
Collected by :DeepaRajesh [ 155 ]
rajesh.ks21@gmail.com
Kuwait
A) Hypothermia occurred. The child received twice the ordered
B) Edema dose of morphine an hour ago. Which nursing
C) Dyspnea diagnosis is a priority at this time?
D) Epistaxis
A) Risk for fluid volume deficit related to mor-
Review Information: The correct answer is phine overdose
D: Epistaxis. A large dose of aspirin inhibits pro- B) Decreased gastrointestinal mobility related to
thrombin formation and lowers platelet levels. mucosal irritation
With an overdose, clotting time is prolonged. C) Ineffective breathing patterns related to cen-
tral nervous system depression
D) Altered nutrition related to inability to control
Question58 nausea and vomiting
An 80 year-old client on digitalis (Lanoxin) re-
ports nausea, vomiting, abdominal cramps and Review Information: The correct answer is C:
halo vision. Which of the following laboratory re- Ineffective breathing patterns related to central
sults should the nurse analyze first? nervous system depression. Respiratory depres-
sion is a life-threatening risk in this overdose.
A) Potassium levels
B) Blood pH
C) Magnesium levels Question61
D) Blood urea nitrogen Lactulose (Chronulac) has been prescribed for a
client with advanced liver disease. Which of the
Review Information: The correct answer is A: following assessments would the nurse use to
Potassium levels. The most common cause of evaluate the effectiveness of this treatment?
digitalis toxicity is a low potassium level. Clients
must be taught that it is important to have ad- A) An increase in appetite
equate potassium intake especially if taking diu- B) A decrease in fluid retention
retics that enhance the loss of potassium while C) A decrease in lethargy
they are taking digitalis. D) A reduction in jaundice

Review Information: The correct answer is


Question59 C: A decrease in lethargy. Lactulose produces
A 42 year-old male client refuses to take pro- an acid environment in the bowel and traps am-
pranolol hydrochloride (Inderal) as prescribed. monia in the gut; the laxative effect then aids in
Which client statement from the assessment removing the ammonia from the body. This de-
data is likely to explain his noncompliance? creases the effects of hepatic encephalopathy,
including lethargy and confusion.
A) “I have problems with diarrhea.”
B) “I have difficulty falling asleep.”
C) “I have diminished sexual function.” Question62
D) “I often feel jittery.” The nurse is teaching a class on HIV prevention.
Which of the following should be emphasized as
Review Information: The correct answer is C: increasing risk?
“I have diminished sexual function.”. Inderal, a
beta-blocking agent used in hypertension, pro- A) Donating blood
hibits the release of epinephrine into the cells; B) Using public bathrooms
this may result in hypotension which results in C) Unprotected sex
decreased libido and impotence. D) Touching a person with AIDS

Review Information: The correct answer is C:


Question60 Unprotected sex. Because HIV is spread through
The nurse caring for a 9 year-old child with a exposure to bodily fluids, unprotected intercourse
fractured femur is told that a medication error and shared drug paraphernalia remain the high-
Collected by :DeepaRajesh [ 156 ]
rajesh.ks21@gmail.com
Kuwait
est risks for infection. B) Decreased albumin
C) Decreased potassium
D) Increased sodium retention
Question63
While interviewing a new admission, the nurse Review Information: The correct answer is C:
notices that the client is shifting positions, wring- Decreased potassium. In bulimia, loss of elec-
ing her hands, and avoiding eye contact. It is im- trolytes can occur in addition to other findings of
portant for the nurse to starvation and dehydration.
A) ask the client what she is feeling
B) assess the client for auditory hallucinations
C) recognize the behavior as a side effect of Question66
medication A client, recovering from alcoholism, asks the
D) re-focus the discussion on a less anxiety pro- nurse, “What can I do when I start recognizing
voking topic relapse triggers within myself?” How might the
nurse best respond?
Review Information: The correct answer is A:
ask the client what she is feeling. The initial step A) “When you have the impulse to stop in a bar,
in anxiety intervention is observing, identifying, contact a sober friend and talk with him.”
and assessing anxiety. The nurse should seek B) “Go to an AA meeting when you feel the urge
client validation of the accuracy of nursing as- to drink.”
sessments and avoid drawing conclusions based C) “It is important to exercise daily and get in-
on limited data. In the situation above, the client volved in activities that will cause you not to think
may simply need to use the restroom but be re- about drug use.”
luctant to communicate her need! D) “Let’s talk about possible options you have
when you recognize relapse triggers in your-
self.”
Question64
A young adult seeks treatment in an outpatient Review Information: The correct answer is D:
mental health center. The client tells the nurse “Let’s talk about possible options you have when
he is a government official being followed by you recognize relapse triggers in yourself.”. This
spies. On further questioning, he reveals that option encourages the process of self evalua-
his warnings must be heeded to prevent nuclear tion and problem solving, while avoiding telling
war. What is the most therapeutic approach by the client what to do. Encouraging the client to
the nurse? brainstorm about response options validates the
nurse’s belief in the client’s personal competen-
A) Listen quietly without comment cy and reinforces a coping strategy that will be
B) Ask for further information on the spies needed when the nurse may not be available to
C) Confront the client’s delusion offer solutions.
D) Contact the government agency
Question67
Review Information: The correct answer is Therapeutic nurse-client interaction occurs when
A: Listen quietly without comment. The client’’s the nurse
comments demonstrate grandiose ideas. The A) assists the client to clarify the meaning of what
most therapeutic response is to listen but avoid the client has said
being incorporated into the client’s delusional B) interprets the client’s covert communication
system. C) praises the client for appropriate feelings and
behavior
Question65 D) advises the client on ways to resolve prob-
The nurse is assessing a 17 year-old female cli- lems
ent with bulimia. Which of the following labora-
tory reports would the nurse anticipate? Review Information: The correct answer is A:
assists the client to clarify the meaning of what
A) Increased serum glucose the client has said. Clarification is a facilitating/
Collected by :DeepaRajesh [ 157 ]
rajesh.ks21@gmail.com
Kuwait
therapeutic communication strategy. Interpreta- suddenly walks up to the nurse and shouts “You
tion, changing the focus/subject, giving approv- think you’re so perfect and pure and good.” An
al, and advising are non-therapeutic/barriers to appropriate response for the nurse is
communication.
A) “Is that why you’ve been staring at me?”
B) “You seem to be in a really bad mood.”
Question68 C) “Perfect? I don’t quite understand.”
Which nursing intervention will be most effective D) “You seem angry right now.”
in helping a withdrawn client to develop relation-
ship skills? Review Information: The correct answer is D:
A) Offer the client frequent opportunities to inter- “You seem angry right now.”. The nurse recog-
act with 1 person nizes the underlying emotion with a matter of fact
B) Provide the client with frequent opportunities attitude, but avoids telling the clients how they
to interact with other clients feel.
C) Assist the client to analyze the meaning of the
withdrawn behavior
D) Discuss with the client the focus that other Question71
clients have similar problems A client who is a former actress enters the day
room wearing a sheer nightgown, high heels, nu-
Review Information: The correct answer is A: merous bracelets, bright red lipstick and heavily
Offer the client frequent opportunities to interact rouged cheeks. Which nursing action is the best
with 1 person. The withdrawn client is uncomfort- in response to the client’s attire?
able in social interaction. The nurse-client rela-
tionship is a corrective relationship in which the A) Gently remind her that she is no longer on
client learns both tolerance and skills for relation- stage
ships. B) Directly assist client to her room for appropri-
ate apparel
C) Quietly point out to her the dress of other cli-
Question69 ents on the unit
An important goal in the development of a thera- D) Tactfully explain appropriate clothing for the
peutic inpatient milieu is to hospital

A) provide a businesslike atmosphere where cli- Review Information: The correct answer is B:
ents can work on individual goals Directly assist client to her room for appropriate
B) provide a group forum in which clients decide apparel. It assists the client to maintain self-es-
on unit rules, regulations, and policies teem while modifying behavior.
C) provide a testing ground for new patterns of
behavior while the client takes responsibility for
his or her own actions Question72
D) discourage expressions of anger because When teaching suicide prevention to the parents
they can be disruptive to other clients of a 15 year-old who recently attempted suicide,
the nurse describes the following behavioral cue
Review Information: The correct answer is C: as indicating a need for intervention.
provide a testing ground for new patterns of be-
havior while the client takes responsibility for his A) Angry outbursts at significant others
or her own actions. A therapeutic milieu is pur- B) Fear of being left alone
poseful and planned to provide safety and a test- C) Giving away valued personal items
ing ground for new patterns of behavior. D) Experiencing the loss of a boyfriend

Review Information: The correct answer is C:


Question70 Giving away valued personal items. Eighty per-
A client with paranoid delusions stares at the cent of all potential suicide victims give some type
nurse over a period of several days. The client of indication that self-destructiveness should be
Collected by :DeepaRajesh [ 158 ]
rajesh.ks21@gmail.com
Kuwait
addressed. These clues might lead one to sus- A) brittle hair, lanugo, amenorrhea
pect that a client is having suicidal thoughts or is B) diarrhea, nausea, vomiting, dental erosion
developing a plan. C) hyperthermia, tachycardia, increased meta-
bolic rate
D) excessive anxiety about symptoms
Question73
Which statement made by a client indicates to Review Information: The correct answer is A:
the nurse that the client may have a thought dis- brittle hair, lanugo, amenorrhea. Physical find-
order? ings associated with anorexia also include re-
A) “I’m so angry about this. Wait until my partner duced metabolic rate and lower vital signs.
hears about this.”
B) “I’m a little confused. What time is it?” Free NCLEX-RN Sample Test Ques-
C) “I can’t find my ‘mesmer’ shoes. Have you tions For Nursing Review (Pharmacol-
seen them?” ogy Set 2)
D) “I’m fine. It’s my daughter who has the prob- Jul31,
lem.”
A nurse is assigned to perform well-child assess-
Review Information: The correct answer is C: ments at a day care center. A staff member in-
“I can’’t find my ‘’mesmer’’ shoes. Have you seen terrupts the examinations to ask for assistance.
them?”. A neologism is a new word self invented They find a crying 3 year-old child on the floor
by a person and not readily understood by an- with mouth wide open and gums bleeding. Two
other. Using neologisms is often associated with unlabeled open bottles lie nearby. The nurse’s
a thought disorder. first action should be
A) call the poison control center, then 911
B) administer syrup of Ipecac to induce vomiting
Question74 C) give the child milk to coat her stomach
In a psychiatric setting, the nurse limits touch D) ask the staff about the contents of the bottles
or contact used with clients to handshaking be-
cause
Review Information: The correct answer is D:
A) some clients misconstrue hugs as an invita- ask the staff about the contents of the bottles
tion to sexual advances The nurse needs to assess what the child in-
B) handshaking keeps the gesture on a profes- gested before determining the next action. Once
sional level the substance is identified, the poison control
C) refusal to touch a client denotes lack of con- center and emergency response team should be
cern called.
D) inappropriate touch often results in charges of
assault and battery Question2
A client with atrial fibrillation is receiving digoxin
Review Information: The correct answer is A: (Lanoxin). Which of these assessments is most
some clients misconstrue hugs as an invitation important for the nurse to perform?
to sexual advances. Touch denotes positive feel- A) Monitor blood pressure every 4 hours
ings for another person. The client may interpret B) Measure apical pulse prior to administration
hugging and holding hands as sexual advanc- C) Maintain accurate intake and output records
es. D) Record an EKG strip after administration

Question75 Review Information: The correct answer is B:


A client with anorexia is hospitalized on a medi- Measure apical pulse prior to administration
cal unit due to electrolyte imbalance and cardiac Digitoxin decreases conduction velocity through
dysrhythmias. Additional assessment findings the AV node and prolongs the refractory period. If
that the nurse would expect to observe are the apical heart rate is less than 60 beats/minute,
withhold the drug. The apical pulse should be
Collected by :DeepaRajesh [ 159 ]
rajesh.ks21@gmail.com
Kuwait
taken with a stethoscope so that there will be no C) Peripheral edema
mistake about what the heart rate actually is. D) Jaundice

Question3
The nurse is administering an intravenous vesi- Review Information: The correct answer is A:
cant chemotherapeutic agent to a client. Which Buffalo hump
assessment would require the nurse’s immedi- With high doses of glucocorticoid, iatrogenic
ate action? Cushing>>s syndrome develops. The exagger-
A) Stomatitis lesion in the mouth ated physiological action causes abnormal fat
B) Severe nausea and vomiting distribution which results in a moon-shaped face,
C) Complaints of pain at site of infusion a intrascapular pad on the neck (buffalo hump)
D) A rash on the client’s extremities and truncal obesity with slender limbs.

Review Information: The correct answer is C: Question6


Complaints of pain at site of infusion The health care provider has written «Morphine
A vesicant is a chemotherapeutic agent capa- sulfate 2 mgs IV every 3-4 hours prn for pain» on
ble of causing blistering of tissues and possible the chart of a child weighing 22 lb. (10 kg). What
tissue necrosis if there is extravasation. These is the nurse>s initial action?
agents are irritants which cause pain along the A) Check with the pharmacist
vein wall, with or without inflammation. B) Hold the medication and contact the provider
C) Administer the prescribed dose as ordered
Question4 D) Give the dose every 6-8 hours
The nurse practicing in a long term care facility
recognizes that elderly clients are at greater risk
for drug toxicity than younger adults because of Review Information: The correct answer is B:
which of the following physiological changes of Hold the medication and contact the provider
advancing age? The usual pediatric dose of morphine is 0.1 mg/
A) Drugs are absorbed more readily from the GI kg every 3 to 4 hours. At 10 kg, this child typically
tract should receive 1.0 mg every 3 to 4 hours.
B) Elders have less body water and more fat
C) The elderly have more rapid hepatic metabo- Question7
lism A client is ordered atropine to be administered
D) Older people are often malnourished and preoperatively. Which physiological effect should
anemic the nurse monitor for?
A) Elevate blood pressure
B) Drying up of secretions
Review Information: The correct answer is B: C) Reduce heart rate
Elders have less body water and more fat D) Enhance sedation
Because elderly persons have decreased lean
body tissue/water in which to distribute medica-
tions, more drug remains in the circulatory system Review Information: The correct answer is B:
with potential for drug toxicity. Increased body fat Drying up of secretions
results in greater amounts of fat-soluble drugs Atropine dries secretions which may get in the
being absorbed, leaving less in circulation, thus way during the operative procedure.
increasing the duration of action of the drug
Question8
Question5 A client is receiving digitalis. The nurse should
The nurse is assessing a client who is on long instruct the client to report which of the following
term glucocorticoid therapy. Which of the follow- side effects?
ing findings would the nurse expect? A) Nausea, vomiting, fatigue
A) Buffalo hump B) Rash, dyspnea, edema
B) Increased muscle mass C) Polyuria, thirst, dry skin
Collected by :DeepaRajesh [ 160 ]
rajesh.ks21@gmail.com
Kuwait
D) Hunger, dizziness, diaphoresis with documented acetaminophen poisoning. In
order to provide counseling and education for
the parents, which principle must the nurse un-
Review Information: The correct answer is A: derstand?
Nausea, vomiting, fatigue A) The problem occurs in stages with recovery
Side effects of digitalis toxicity include fatigue, within 12-24 hours
nausea, vomiting, anorexia, and bradycardia. B) Hepatic problems may occur and may be life-
Digitalis inhibits the sodium potassium ATPase, threatening
which makes more calcium available for contrac- C) Full and rapid recovery can be expected in
tile proteins, resulting in increased cardiac out- most children
put. D) This poisoning is usually fatal, as no antidote
is available
Question9
A client is receiving dexamethasone (Decadron)
therapy. What should the nurse plan to monitor Review Information: The correct answer is B:
in this client? Hepatic problems may occur and may be life-
A) Urine output every 4 hours threatening
B) Blood glucose levels every 12 hours Clinical manifestations associated with acetami-
C) Neurological signs every 2 hours nophen poisoning occurs in 4 stages. The third
D) Oxygen saturation every 8 hours stage is hepatic involvement which may last up
to 7 days and be permanent. Clients who do not
Review Information: The correct answer is B: die in the hepatic stage gradually recover.
Blood glucose levels every 12 hours
The drug Decadron increases glycogenesis. Question12
This may lead to hyperglycemia. Therefore the A client has been receiving dexamethasone
blood sugar level and acetone production must (Decadron) for control of cerebral edema. Which
be monitored. of the following assessments would indicate that
the treatment is effective?
Question10 A) A positive Babinski>s reflex
The nurse is caring for a client with schizophrenia B) Increased response to motor stimuli
who has been treated with quetiapine (Seroquel) C) A widening pulse pressure
for 1 month. Today the client is increasingly agi- D) Temperature of 37 degrees Celsius
tated and complains of muscle stiffness. Which
of these findings should be reported to the health
care provider?
A) Elevated temperature and sweating. Review Information: The correct answer is B:
B) Decreased pulse and blood pressure. Increased response to motor stimuli
C) Mental confusion and general weakness. Decadron is a corticosteroid that acts on the cell
D) Muscle spasms and seizures. membrane to decrease inflammatory responses
as well as stabilize the blood-brain barrier. Once
Decadron reaches a therapeutic level, there
Review Information: The correct answer is A: should be a decrease in symptomology with im-
Elevated temperature and sweating. provement in motor skills.
Neuroleptic malignant syndrome (NMS) is a rare
disorder that can occur as a side effect of antipsy- Question13
chotic medications. It is characterized by muscu- The provider has ordered transdermal nitroglyc-
lar rigidity, tachycardia, hyperthermia, sweating, erin patches for a client. Which of these instruc-
altered consciousness, autonomic dysfunction, tions should be included when teaching a client
and increase in CPK. This is a life-threatening about how to use the patches?
complication. A) Remove the patch when swimming or bath-
ing
Question11 B) Apply the patch to any non-hairy area of the
A child presents to the Emergency Department body
Collected by :DeepaRajesh [ 161 ]
rajesh.ks21@gmail.com
Kuwait
C) Apply a second patch with chest pain Question16
D) Remove the patch if ankle edema occurs A client with anemia has a new prescription for
ferrous sulfate. In teaching the client about diet
and iron supplements, the nurse should empha-
Review Information: The correct answer is size that absorption of iron is enhanced if taken
B: Apply the patch to any non-hairy area of the with which substance?
body A) Acetaminophen
The patch application sites should be rotated. B) Orange juice
C) Low fat milk
Question14 D) An antacid
A newly admitted client has a diagnosis of de-
pression. She complains of “twitching muscles” Review Information: The correct answer is B:
and a “racing heart”, and states she stopped Orange juice
taking Zoloft a few days ago because it was not Ascorbic acid enhances the absorption of iron.
helping her depression. Instead, she began to
take her partner>s Parnate. The nurse should Question17
immediately assess for which of these adverse A client with an aplastic sickle cell crisis is receiv-
reactions? ing a blood transfusion and begins to complain
A) Pulmonary edema of «feeling hot.» Almost immediately, the client
B) Atrial fibrillation begins to wheeze. What is the nurse>s first ac-
C) Mental status changes tion?
D) Muscle weakness A) Stop the blood infusion
B) Notify the health care provider
C) Take/record vital signs
Review Information: The correct answer is C: D) Send blood samples to lab
Mental status changes
Use of serotonergic agents may result in Se- Review Information: The correct answer is A:
rotonin Syndrome with confusion, nausea, pal- Stop the blood infusion
pitations, increased muscle tone with twitching If a reaction of any type is suspected during ad-
muscles, and agitation. Serotonin syndrome is ministration of blood products, stop the infusion
most often reported in patients taking 2 or more immediately, keep the line open with saline, no-
medications that increase CNS serotonin levels tify the health care provider, monitor vital signs
by different mechanisms. The most common and other changes, and then send a blood sam-
drug combinations associated with serotonin ple to the lab.
syndrome involve the MAOIs, SSRIs, and the tri-
cyclic antidepressants. Question18
A client confides in the RN that a friend has told
Question15 her the medication she takes for depression,
A client with bi-polar disorder is taking lithium Wellbutrin, was taken off the market because
(Lithane). What should the nurse emphasize it caused seizures. What is an appropriate re-
when teaching about this medication? sponse by the nurse?
A) Take the medication before meals A) «Ask your friend about the source of this in-
B) Maintain adequate daily salt intake formation.»
C) Reduce fluid intake to minimize diuresis B) «Omit the next doses until you talk with the
D) Use antacids to prevent heartburn doctor.»
C) «There were problems, but the recommended
dose is changed.»
Review Information: The correct answer is B: D) «Your health care provider knows the best
Maintain adequate daily salt intake drug for your condition.»
Salt intake affects fluid volume, which can affect
lithium (Lithane) levels; therefore, maintaining
adequate salt intake is advised. Review Information: The correct answer is C:
«There were problems, but the recommended
Collected by :DeepaRajesh [ 162 ]
rajesh.ks21@gmail.com
Kuwait
dose is changed.» B) «Follow up with your primary care provider in
Wellbutrin was introduced in the U.S. in 1985 3 months.»
and then withdrawn because of the occurrence C) «Continue to take your medications even
of seizures in some patients taking the drug. The when you are feeling fine.»
drug was reintroduced in 1989 with specific rec- D) «Continue to get yearly tuberculin skin
ommendations regarding dose ranges to limit tests.»
the occurrence of seizures. The risk of seizure
appears to be strongly associated with dose.
Review Information: The correct answer is C:
Question19 «Continue to take your medications even when
When providing discharge teaching to a client you are feeling fine.»
with asthma, the nurse will warn against the use The most important piece of information the tu-
of which of the following over-the-counter medi- berculosis client needs is to understand the im-
cations? portance of medication compliance, even if no
A) Cortisone ointments for skin rashes longer experiencing symptoms. Clients are most
B) Aspirin products for pain relief infectious early in the course of therapy. The
C) Cough medications containing guaifenesin numbers of acid-fast bacilli are greatly reduced
D) Histamine blockers for gastric distress as early as 2 weeks after therapy begins.

Question22
Review Information: The correct answer is B: The nurse is applying silver sulfadiazine (Sil-
Aspirin products for pain relief vadene) to a child with severe burns to arms
Aspirin is known to induce asthma attacks. As- and legs. Which side effect should the nurse be
pirin can also cause nasal polyps and rhinitis. monitoring for?
Warn individuals with asthma about signs and A) Skin discoloration
symptoms resulting from complications due to B) Hardened eschar
aspirin ingestion. C) Increased neutrophils
D) Urine sulfa crystals
Question20
The nurse is caring for a client who is receiving
procainamide (Pronestyl) intravenously. It is im- Review Information: The correct answer is D:
portant for the nurse to monitor which of the fol- Urine sulfa crystals
lowing parameters? Silver sulfadiazine is a broad spectrum anti-
A) Hourly urinary output microbial, especially effective against pseu-
B) Serum potassium levels domonas. When applied to extensive areas,
* C) Continuous EKG readings however, it may cause a transient neutropenia,
D) Neurological signs as well as renal function changes with sulfa crys-
tals production and kernicterus.

Review Information: The correct answer is C: Question23


Continuous EKG readings The nurse is monitoring a client receiving a
Procainamide (Pronestyl) is used to suppress thrombolytic agent, alteplase (Activase tissue
cardiac arrhythmias. When administered intra- plasminogen activator), for treatment of a myo-
venously, it must be accompanied by continuous cardial infarction. What outcome indicates the
cardiac monitoring by ECG. client is receiving adequate therapy within the
first hours of treatment?
Question21 A) Absence of a dysrhythmia (or arrhythmia)
The nurse is providing education for a client with B) Blood pressure reduction
newly diagnosed tuberculosis. Which statement C) Cardiac enzymes are within normal limits
should be included in the information that is giv- D) Return of ST segment to baseline on ECG
en to the client?
A) «Isolate yourself from others until you are fin-
ished taking your medication.» Review Information: The correct answer is D:
Collected by :DeepaRajesh [ 163 ]
rajesh.ks21@gmail.com
Kuwait
Return of ST segment to baseline on ECG Question26
Improved perfusion should result from this medi- The nurse is teaching a child and the family about
cation, along with the reduction of ST segment the medication phenytoin (Dilantin) prescribed
elevation. for seizure control. Which of the following side
effects is most likely to occur?
Question24 A) Vertigo
The provider has ordered daily high doses of B) Drowsiness
aspirin for a client with rheumatoid arthritis. The C) Gingival hyperplasia
nurse instructs the client to discontinue the medi- D) Vomiting
cation and contact the provider if which of the
following symptoms occur?
A) Infection of the gums
B) Diarrhea for more than one day Review Information: The correct answer is C:
C) Numbness in the lower extremities Gingival hyperplasia
D) Ringing in the ears Swollen and tender gums occur often with use of
phenytoin. Good oral hygiene and regular visits
to the dentist should be emphasized.
Review Information: The correct answer is D:
Ringing in the ears Question27
Aspirin stimulates the central nervous system The use of atropine for treatment of symptomatic
which may result in ringing in the ears. bradycardia is contraindicated for a client with
Deglin, J.D. and Vallerand, A.H. (2001). Davis’ which of the following conditions?
drug guide for nurses. (7th edition). Philadelphia: A) Urinary incontinence
F.A. Davis Company. B) Glaucoma
Key, J.L. and Hayes, E.R. (2003). Pharmacol- C) Increased intracranial pressure
ogy, a nursing process approach. (4th edition). D) Right sided heart failure
Philadelphia: Saunders.

Question25 Review Information: The correct answer is B:


A nurse is caring for a client who is receiving Glaucoma
methyldopa hydrochloride (Aldomet) intrave- Atropine is contraindicated in clients with angle-
nously. Which of the following assessment find- closure glaucoma because it can cause pupillary
ings would indicate to the nurse that the client dilation with an increase in aqueous humor, lead-
may be having an adverse reaction to the medi- ing to a resultant increase in optic pressure.
cation?
A) Headache Question28
B) Mood changes A pregnant woman is hospitalized for treatment
C) Hyperkalemia of pregnancy induced hypertension (PIH) in the
D) Palpitations third trimester. She is receiving magnesium sul-
fate intravenously. The nurse understands that
this medication is used mainly for what pur-
Review Information: The correct answer is B: pose?
Mood changes A) Maintain normal blood pressure
The nurse should assess the client for alterations B) Prevent convulsive seizures
in mental status such as mood changes. These C) Decrease the respiratory rate
symptoms should be reported promptly. D) Increase uterine blood flow
Deglin, J.D. and Vallerand, A.H. (2001). Davis’
drug guide for nurses. (7th edition). Philadelphia:
F.A. Davis Company. Review Information: The correct answer is B:
Wilson, B.A., Shannon, M.T., and Stang, C.L. Prevent convulsive seizures
(2004). Nurse’s drug guide. Upper Saddle River, Magnesium sulfate is a central nervous system
New Jersey: Pearson Prentice Hall. depressant. While it has many systemic effects,
it is used in the client with pregnancy induced
Collected by :DeepaRajesh [ 164 ]
rajesh.ks21@gmail.com
Kuwait
hypertension (PIH) to prevent seizures.
Question32
A post-operative client has a prescription for
Question29 acetaminophen with codeine. What should the
The nurse is teaching a group of women in a com- nurse recognizes as a primary effect of this com-
munity clinic about prevention of osteoporosis. bination?
Which of the following over-the-counter medica- A) Enhanced pain relief
tions should the nurse recognize as having the B) Minimized side effects
most elemental calcium per tablet? C) Prevention of drug tolerance
A) Calcium chloride D) Increased onset of action
B) Calcium citrate
C) Calcium gluconate
D) Calcium carbonate Review Information: The correct answer is A:
Enhanced pain relief
Combination of analgesics with different mecha-
Review Information: The correct answer is D: nisms of action can afford greater pain relief.
Calcium carbonate
Calcium carbonate contains 400mg of elemental
calcium in 1 gram of calcium carbonate. Question33
A client is receiving erythromycin 500mg IV eve-
Question30 ry 6 hours to treat a pneumonia. Which of the
The nurse is administering diltiazem (Cardizem) following is the most common side effect of the
to a client. Prior to administration, it is important medication?
for the nurse to assess which parameter? A) Blurred vision
A) Temperature B) Nausea and vomiting
B) Blood pressure C) Severe headache
C) Vision D) Insomnia
D) Bowel sounds

Review Information: The correct answer is B:


Review Information: The correct answer is B: Nausea and vomiting
Blood pressure Nausea is a common side-effect of erythromycin
Diltiazem (Cardizem) is a calcium channel block- in both oral and intravenous forms.
er that causes systemic vasodilation resulting in
decreased blood pressure. Question34
The health care provider orders an IV aminophyl-
Question31 line infusion at 30 mg/hr. The pharmacy sends a
The nurse is instructing a client with moderate 1,000 ml bag of D5W containing 500 mg of ami-
persistent asthma on the proper method for us- nophylline. In order to administer 30 mg per hour,
ing MDIs (multi-dose inhalers). Which medica- the RN will set the infusion rate at:
tion should be administered first? A) 20 ml per hour
A) Steroid B) 30 ml per hour
B) Anticholinergic C) 50 ml per hour
C) Mast cell stabilizer D) 60 ml per hour
D) Beta agonist

Review Information: The correct answer is D:


Review Information: The correct answer is D: 60 ml per hour
Beta agonist Using the ratio method to calculate infusion rate:
The beta-agonist drugs help to relieve bronchos- mg to be given (30) : ml to be infused (X) :: mg
pasm by relaxing the smooth muscle of the air- available (500) : ml of solution (1,000). Solve for
way. These drugs should be taken first so that X by cross-multiplying: 30 x 1,000 = 500 x X (or
other medications can reach the lungs. cancel), 30,000 = 500 X, X = 30,000/500, X =
Collected by :DeepaRajesh [ 165 ]
rajesh.ks21@gmail.com
Kuwait
60ml per hour. of meperidine, is a central nervous system stim-
ulant that produces anxiety, tremors, myoclonus,
Question35 and generalized seizures when it accumulates
The nurse is assessing a 7 year-old after several with repetitive dosing. Clients with sickle cell dis-
days of treatment for a documented strep throat. ease are particularly at risk for normeperidine-
Which of the following statements suggests that induced seizures.
further teaching is needed?
A) «Sometimes I take my medicine with fruit Question38
juice.» A 5 year-old has been rushed to the emergency
B) «My mother makes me take my medicine right room several hours after acetaminophen poison-
after school.» ing. Which laboratory result should receive atten-
C) «Sometimes I take the pills in the morning tion by the nurse?
and other times at night.» A) Sedimentation rate
D) «I am feeling much better than I did last B) Profile 2
week.» C) Bilirubin
D) Neutrophils

Review Information: The correct answer is C:


«Sometimes I take the pills in the morning and Review Information: The correct answer is C:
other times at night.» Bilirubin
Inconsistency in taking the prescribed medica- Bilirubin, along with liver enzymes ALT and AST,
tion indicates more teaching is needed. may rise in the second stage (1-3 days) after a
significant overdose, indicating cellular necrosis
Question36 and liver dysfunction. A prolonged prothrombin
The nurse is caring for a 10 year-old client who time may also be found.
will be placed on heparin therapy. Which assess-
ment is critical for the nurse to make before initi- Question39
ating therapy An elderly client is on an anticholinergic metered
A) Vital signs dose inhaler (MDI) for chronic obstructive pulmo-
B) Weight nary disease. The nurse would suggest a spacer
C) Lung sounds to
D) Skin turgor A) enhance the administration of the medication
B) increase client compliance
C) improve aerosol delivery in clients who are
Review Information: The correct answer is B: not able to coordinate the MDI
Weight D) prevent exacerbation of COPD
Check the client>>s weight because dosage is
calculated on the basis of weight.
Review Information: The correct answer is C:
Question37 improve aerosol delivery in clients who are not
In providing care for a client with pain from a sick- able to coordinate the MDI
le cell crisis, which one of the following medica- Spacers improve the medication delivery in cli-
tion orders for pain control should be questioned ents who are unable to coordinate the move-
by the nurse? ments of administering a dose with an MDI.
A) Demerol
B) Morphine
C) Methadone Question40
D) Codeine The nurse is teaching a parent how to adminis-
ter oral iron supplements to a 2 year-old child.
Review Information: The correct answer is A: Which of the following interventions should be
Demerol included in the teaching?
Meperidine is not recommended in clients with A) Stop the medication if the stools become tarry
sickle cell disease. Normeperidine, a metabolite green
Collected by :DeepaRajesh [ 166 ]
rajesh.ks21@gmail.com
Kuwait
B) Give the medicine with orange juice and Review Information: The correct answer is B:
through a straw a decreased sensation of thirst
C) Add the medicine to a bottle of formula The elderly have a reduction in thirst sensation
D) Administer the iron with your child>s meals causing them to consume less fluid. Other risk
factors may include fear of incontinence, inability
to drink fluids independently and lack of motiva-
Review Information: The correct answer is B: tion.
Give the medicine with orange juice and through
a straw
Absorption of iron is facilitated in an environment Question3
rich in Vitamin C. Since liquid iron preparation A male client is admitted with a spinal cord injury
will stain teeth, a straw is preferred. at level C4. The client asks the nurse how the
injury is going to affect his sexual function. The
0 comments nurse would respond
A) «Normal sexual function is not possible.»
Labels: free nclex-rn sample review questions, B) «Sexual functioning will not be impaired at
nclex-rn practice test questions, nursing review all.»
C) «Erections will be possible.»
Free NCLEX-RN Sample Test Ques- D) «Ejaculation will be normal.»
tions For Nursing Review (Pharmacol-
ogy Set 1)

Question1
Review Information: The correct answer is C:
A client has an order for antibiotic therapy after
«Erections will be possible.»
hospital treatment of a staph infection. Which of
Because they are a reflex reaction, erections can
the following should the nurse emphasize?
be stimulated by stroking the genitalia.
A) Scheduling follow-up blood cultures
B) Completing the full course of medications
C) Visiting the provider in a few weeks
D) Monitoring for signs of recurrent infection Question4
An 82 year-old client complains of chronic con-
stipation. To improve bowel function, the nurse
Review Information: The correct answer is B: should first suggest
Completing the full course of medications A) Increasing fiber intake to 20-30 grams daily
In order for antibiotic therapy to be effective in B) Daily use of laxatives
eradicating an infection, the client must compete C) Avoidance of binding foods such as cheese
the entire course of prescribed therapy. When and chocolate
findings subside, stopping the medication early D) Monitoring a balance between activity and
may lead to recurrence or subsequent drug re- rest
sistance.

Review Information: The correct answer is A:


Increasing fiber intake to 20-30 grams daily
Question2
The incorporation of high fiber into the diet is an
A 72 year-old client is admitted for possible de-
effective way to promote bowel elimination in the
hydration. The nurse knows that older adults are
elderly.
particularly at risk for dehydration because they
have
A) an increased need for extravascular fluid
B) a decreased sensation of thirst Question5
C) an increase in diaphoresis A 4 year-old child is admitted with burns on his
D) higher metabolic demands legs and lower abdomen. When assessing the
child’s hydration status, which of the following in-
dicates a less than adequate fluid replacement?
Collected by :DeepaRajesh [ 167 ]
rajesh.ks21@gmail.com
Kuwait
A) Decreasing hematocrit and increasing urine The range for a therapeutic APTT is 1.5-2 times
volume the control. Therefore the client is receiving a
B) Rising hematocrit and decreasing urine vol- therapeutic dose of Heparin.
ume
C) Falling hematocrit and decreasing urine vol-
ume Question8
D) Stable hematocrit and increasing urine vol- A client is admitted with a diagnosis of nodal
ume bigeminy. The nurse knows that the atrioven-
tricular (AV) node has an intrinsic rate of
A) 60-100 beats/minute
Review Information: The correct answer is B: B) 10-30 beats/minute
Rising hematocrit and decreasing urine volume C) 40-70 beats/minute
A rising hematocrit indicates a decreased total D) 20-50 beats/minute
blood volume, a finding consistent with dehydra-
tion.
Review Information: The correct answer is C:
40-70 beats/minute
Question6 The intrinsic rate of the AV node is within the
A client receiving chemotherapy has developed range of 40-70 beats per minute.
sores in his mouth. He asks the nurse why this
happened. What is the nurse’s best response?
A) «It is a sign that the medication is working.» Question9
B) «You need to have better oral hygiene.» A client is to receive 3 doses of potassium chlo-
C) «The cells in the mouth are sensitive to the ride 10 mEq in 100cc normal saline to infuse
chemotherapy.» over 30 minutes each. Which of the following is a
D) «This always happens with chemotherapy.» priority assessment to perform before giving this
medication?
A) Oral fluid intake
B) Bowel sounds
C) Grip strength
Review Information: The correct answer is D) Urine output
C: «The cells in the mouth are sensitive to the
chemotherapy.»
The epithelial cells in the mouth are very sensi- Review Information: The correct answer is D:
tive to chemotherapy due to their high rate of cell Urine output
turnover. Potassium chloride should only be administered
after adequate urine output (>20cc/hour for 2
consecutive hours) has been established. Im-
Question7 paired ability to excrete potassium via the kid-
You are caring for a client with deep vein throm- neys can result in hyperkalemia.
bosis who is on Heparin IV. The latest APTT is 50
seconds. If the laboratory normal range is 16-24
seconds, you would anticipate Question10
A) maintaining the current heparin dose The unlicensed assistive personnel (UAP) re-
B) increasing the heparin as it does not appear ports to the nurse that a client with cirrhosis who
therapeutic. had a paracentesis yesterday has become more
C) giving protamine sulfate as an antidote. lethargic and has musty smelling breath. A criti-
D) repeating the blood test 1 hour after giving cal assessment for increasing encephalopathy
heparin. is
A) monitor the client>s clotting status
B) assess upper abdomen for bruits
Review Information: The correct answer is A: C) assess for flap-like tremors of the hands
maintaining the current heparin dose D) measure abdominal girth changes
Collected by :DeepaRajesh [ 168 ]
rajesh.ks21@gmail.com
Kuwait
D) high calories

Review Information: The correct answer is C:


assess for flap-like tremors of the hands Review Information: The correct answer is C:
A client with cirrhosis of the liver who develops low fat
subtle changes in mental status and has a musty Due to age related changes, the diet of the eld-
odor to the breath is at risk for developing more erly should include a lower quantity and higher
advanced signs of encephalopathy. quality of food. Fewer carbohydrates and fats
are required in their diets.

Question11
A client is scheduled for an intravenous pyelo- Question14
gram (IVP). After the contrast material is inject- A woman with a 28 week pregnancy is on the
ed, which of the following client reactions should way to the emergency department by ambulance
be reported immediately? with a tentative diagnosis of abruptio placenta.
A) Feeling warm Which should the nurse do first when the woman
B) Face flushing arrives?
C) Salty taste A) administer oxygen by mask at 100%
D) Hives B) start a second IV with an 18 gauge cannula
C) check fetal heart rate every 15 minutes
D) insert urethral catheter with hourly urine out-
Review Information: The correct answer is D: puts
Hives
This is a sign of anaphylaxis and should be re-
ported immediately. The other reactions are con- Review Information: The correct answer is A:
sidered normal and the client should be informed administer oxygen by mask at 100%
that they may occur. Administering oxygen in this situation would in-
. crease the circulating oxygen in the mother’s cir-
culation to the fetus’s circulation. This action will
Question12 minimize complications.
A client is prescribed an inhaler. How should the
nurse instruct the client to breathe in the medica-
tion? Question15
A) As quickly as possible A client in respiratory distress is admitted with
B) As slowly as possible arterial blood gas results of: PH 7.30; PO2 58,
C) Deeply for 3-4 seconds PCO2 34; and HCO3 19. The nurse determines
D) Until hearing whistling by the spacer that the client is in
A) metabolic acidosis
B) metabolic alkalosis
Review Information: The correct answer is C: C) respiratory acidosis
Deeply for 3-4 seconds D) respiratory alkalosis
The client should be instructed to breath in the
medication for 3-4 seconds in order to receive
the correct dosage of medication.

Review Information: The correct answer is A:


Question13 metabolic acidosis
The nurse is caring for clients over the age of 70. These lab values indicate metabolic acidosis:
The nurse knows that due to age-related chang- the PH is low, PCO2 is normal, and bicarbonate
es, the elderly clients tolerate diets that are level is low.
A) high protein
B) high carbohydrates
C) low fat Question16
Collected by :DeepaRajesh [ 169 ]
rajesh.ks21@gmail.com
Kuwait
A client is diagnosed with gastroesophageal re- With aggressive treatment and early detection/
flux disease (GERD). The nurse>s instruction to diagnosis the cure rate is 90%.
the client regarding diet should be to
A) avoid all raw fruits and vegetables
B) increase intake of milk products Question19
C) decrease intake of fatty foods A client newly diagnosed with Type I Diabetes
D) focus on 3 average size meals a day Mellitus asks the purpose of the test measur-
ing glycosylated hemoglobin. The nurse should
explain that the purpose of this test is to deter-
Review Information: The correct answer is C: mine:
decrease intake of fatty foods A) The presence of anemia often associated with
GERD may be aggravated by a fatty diet. A diet Diabetes
low in fat would decrease the symptoms of GERD. B) The oxygen carrying capacity of the client>s
Other agents which should also be decreased or red cells
avoided are: cigarette smoking, caffeine, alco- C) The average blood glucose for the past 2-3
hol, chocolate, and meperidine (Demerol). months
D) The client>s risk for cardiac complications

Question17
After surgery, a client with a nasogastric tube
complains of nausea. What action would the
nurse take? Review Information: The correct answer is
A) Call the health care provider C: The average blood glucose for the past 2-3
B) Administer an antiemetic months
C) Put the bed in Fowler’s position By testing the portion of the hemoglobin that ab-
D) Check the patency of the tube sorbs glucose, it is possible to determine the av-
erage blood glucose over the life span of the red
cell, 120 days.
Review Information: The correct answer is D:
Check the patency of the tube
An indication that the nasogastric tube is ob- Question20
structed is a client’s complaint of nausea. Na- A client is admitted for a possible pacemaker in-
sogastric tubes may become obstructed with sertion. What is the intrinsic rate of the heart>s
mucus or sediment. own pacemaker?
A) 30-50 beats/minute
B) 60-100 beats/minute
Question18 C) 20-60 beats/minute
A client with testicular cancer has had an orchiec- D) 90-100 beats/minute
tomy. Prior to discharge the client expresses his
fears related to his prognosis. Which principle
should the nurse base the response on? Review Information: The correct answer is B:
A) Testicular cancer has a cure rate of 90% with 60-100 beats/minute
early diagnosis This is the intrinsic rate of the SA node.
B) Testicular cancer has a cure rate of 50% with
early diagnosis
C) Intensive chemotherapy is the treatment of Question21
choice The nurse discusses nutrition with a pregnant
D) Testicular cancer is usually fatal woman who is iron deficient and follows a veg-
etarian diet. The selection of which foods indi-
cates the woman has learned sources of iron?
Review Information: The correct answer is A) Cereal and dried fruits
A: Testicular cancer has a cure rate of 90% with B) Whole grains and yellow vegetables
early diagnosis C) Leafy green vegetables and oranges
Collected by :DeepaRajesh [ 170 ]
rajesh.ks21@gmail.com
Kuwait
D) Fish and dairy products recognize as the purpose of this medication?
A) Reduce vascularity of the thyroid
B) Correct chronic hyperthyroidism
Review Information: The correct answer is A: C) Destroy the thyroid gland function
Cereal and dried fruits D) Balance enzymes and electrolytes
Both of these foods would be a good source of
iron.

Question22 Review Information: The correct answer is A:


Prior to administering Alteplase (TPA) to a client Reduce vascularity of the thyroid
admitted for a cerebral vascular accident (CVA), Potassium iodide solution, or Lugol>>s solution
it is critical that the nurse assess: may be used preoperatively to reduce the size
A) Neuro signs and vascularity of the thyroid gland.
B) Mental status
C) Blood pressure
D) PT/PTT Question25
One hour before the first treatment is scheduled,
the client becomes anxious and states he does
Review Information: The correct answer is D: not wish to go through with electroconvulsive
PT/PTT therapy. Which response by the nurse is most
TPA is a potent thrombolytic enzyme. Because appropriate?
bleeding is the most common side effect, it is A) «I’ll go with you and will be there with you dur-
most essential to evaluate clotting studies in- ing the treatment.»
cluding PT, PTT, APTT, platelets, and hematocrit B) «You’ll be asleep and won’t remember any-
before beginning therapy. thing.»
C) «You have the right to change your mind. You
seem anxious. Can we talk about it?»
Question23 D) «I’ll call the health care provider to notify them
The nurse enters the room of a client diagnosed of your decision.»
with COPD. The client’s skin is pink, and respi-
rations are 8 per minute. The client’s oxygen is
running at 6 liters per minute. What should be Review Information: The correct answer is C:
the nurse’s first action? «You have the right to change your mind. You
A) Call the health care provider seem anxious. Can we talk about it?»
B) Put the client in Fowler’s position This response indicates acknowledgment of the
C) Lower the oxygen rate client’s rights and the opportunity for the client
D) Take the vital signs to clarify and ventilate concerns. After this, if the
client continues to refuse, the provider should be
notified.
Review Information: The correct answer is C:
Lower the oxygen rate
In client’s diagnosed with COPD, the drive to Question26
breathe is hypoxia. If oxygen is delivered at too A nurse who has been named in a lawsuit can
high of a concentration, this drive will be elimi- use which of these factors for the best protection
nated and the client’s depth and rate of respi- in a court of law?
rations will decrease. Therefore the first action A) Clinical specialty certification in the associ-
should be to lower the oxygen rate. ated area of practice
B) Documentation on the specific client record
with a focus on the nursing process
Question24 C) Yearly evaluations and proficiency reports
The client with goiter is treated with potassium prepared by nurse’s manager
iodide preoperatively. What should the nurse D) Verification of provider>s orders for the plan
Collected by :DeepaRajesh [ 171 ]
rajesh.ks21@gmail.com
Kuwait
of care with identification of outcomes Avoid the use of salt substitutes
Captopril can cause an accumulation of potas-
sium or hyperkalemia. Clients should avoid the
Review Information: The correct answer is B: use of salt substitutes, which are generally po-
Documentation on the specific client record with tassium-based.
a focus on the nursing process
Documentation is the key to protect nurses when
a lawsuit is filed. The thorough documentation Question29
should include all steps of the nursing process – A client has bilateral knee pain from osteoarthri-
assessment, analysis, plan, intervention, evalu- tis. In addition to taking the prescribed non-ster-
ation. In addition, it should include pertinent data oidal anti-inflammatory drug (NSAID), the nurse
such as times, dosages and sites of actions, should instruct the client to
assessment data, the nurse’s response to a A) start a regular exercise program
change in the client’s condition, specific actions B) rest the knees as much as possible to de-
taken, if and when the notification occurred to crease inflammation
the provider or other health care team members, C) avoid foods high in citric acid
and what was prescribed along with the client’s D) keep the legs elevated when sitting
outcomes.

Review Information: The correct answer is A:


Question27 start a regular exercise program
The nurse is caring for clients over the age of 70. A regular exercise program is beneficial in treat-
The nurse is aware that when giving medications ing osteoarthritis. It can restore self-esteem and
to older clients, it is best to improve physical functioning.
A) start low, go slow
B) avoid stopping a medication entirely
C) avoid drugs with side effects that impact cog- Question30
nition An arterial blood gases test (ABG) is ordered
D) review the drug regimen yearly for a confused client. The respiratory therapist
draws the blood and then asks the nurse to apply
pressure to the area so the therapist can take the
Review Information: The correct answer is A: specimen to the lab. How long should the nurse
start low, go slow apply pressure to the area?
Due to physiological changes in the elderly, as A) 3 minutes
well as conditions such as dehydration, hyper- B) 5 minutes
thermia, immobility and liver disease, the effec- C) 8 minutes
tive metabolism of drugs may decrease. As a D) 10 minutes
result, drugs can accumulate to toxic levels and
cause serious adverse reactions.
Review Information: The correct answer is B:
5 minutes
Question28 It is necessary to apply pressure to the area for
You are caring for a hypertensive client with a 5 minutes to prevent bleeding and the formation
new order for captopril (Capoten). Which infor- of hematomas.
mation should the nurse include in client teach-
ing?
A) Avoid green leafy vegetables Question31
B) Restrict fluids to 1000cc/day Which of these clients should the charge nurse
C) Avoid the use of salt substitutes assign to the registered nurse (RN)?
D) Take the medication with meals A) A 56 year-old with atrial fibrillation receiving
digoxin
B) A 60 year-old client with COPD on oxygen at
Review Information: The correct answer is C: 2 L/min
Collected by :DeepaRajesh [ 172 ]
rajesh.ks21@gmail.com
Kuwait
C) A 24 year-old post-op client with type 1 diabe- Question34
tes in the process of discharge Upon admission to an intensive care unit, a client
D) An 80 year-old client recovering 24 hours post diagnosed with an acute myocardial infarction is
right hip replacement ordered oxygen. The nurse knows that the major
reason that oxygen is administered in this situa-
tion is to
A) saturate the red blood cells
B) relieve dyspnea
Review Information: The correct answer is C: C) decrease cyanosis
A 24 year-old post-op client with type 1 diabetes D) increase oxygen level in the myocardium
in the process of discharge
Discharge teaching must be done by an RN.
Practical nurses (PNs) or unlicensed assistive Review Information: The correct answer is D:
personnel (UAPs) can reinforce education after increase oxygen level in the myocardium
the RN does the initial teaching. Anoxia of the myocardium occurs in myocardial
infarction. Oxygen administration will help re-
lieve dyspnea and cyanosis associated with the
Question32 condition but the major purpose is to increase
A hypertensive client is started on atenolol (Ten- the oxygen concentration in the damaged myo-
ormin). The nurse instructs the client to immedi- cardial tissue.
ately report which of these findings? .
A) Rapid breathing
B) Slow, bounding pulse Question35
C) Jaundiced sclera The nurse is teaching a client with chronic re-
D) Weight gain nal failure (CRF) about medications. The client
questions the purpose of aluminum hydroxide
(Amphojel) in her medication regimen. What is
Review Information: The correct answer is B: the best explanation for the nurse to give the cli-
Slow, bounding pulse ent about the therapeutic effects of this medica-
Atenolol (Tenormin) is a beta-blocker that can tion?
cause side effects including bradycardia and hy- A) It decreases serum phosphate
potension. B) It will reduce serum calcium
C) Amphojel increases urine output
D) The drug is taken to control gastric acid se-
Question33 cretion
An 80 year-old client is admitted with a diagnosis
of malnutrition. In addition to physical assess-
ments, which of the following lab tests should be Review Information: The correct answer is A:
closely monitored? It decreases serum phosphate
A) Urine protein Aluminum binds phosphates that tend to accu-
B) Urine creatinine mulate in the patient with chronic renal failure
C) Serum calcium due to decreased filtration capacity of the kidney.
D) Serum albumin Antacids such as Amphojel are commonly used
to accomplish this.

Review Information: The correct answer is D:


Serum albumin Question36
Serum albumin is a valuable indicator of protein A 66 year-old client is admitted for mitral valve
deficiency and, later, nutritional status in adults. replacement surgery. The client has a history
A normal reading for an elder’s serum albumin is of mitral valve regurgitation and mitral stenosis
between 3.0-5.0 g/dl. since her teenage years. During the admission
assessment, the nurse should ask the client if as
a child she had
Collected by :DeepaRajesh [ 173 ]
rajesh.ks21@gmail.com
Kuwait
A) measles
B) rheumatic fever Review Information: The correct answer is D:
C) hay fever intake of at least 3000cc/day
D) encephalitis Fluid intake should be increased to prevent pre-
cipitation of urate in the kidneys.

Question39
Review Information: The correct answer is B: A 55 year-old woman is taking Prednisone and
rheumatic fever aspirin (ASA) as part of her treatment for rheu-
Clients that present with mitral stenosis often matoid arthritis. Which of the following would be
have a history of rheumatic fever or bacterial en- an appropriate intervention for the nurse?
docarditis. A) Assess the pulse rate q 4 hours
B) Monitor her level of consciousness q shift
C) Test her stools for occult blood
Question37 D) Discuss fiber in the diet to prevent constipa-
During nursing rounds which of these assess- tion
ments would require immediate corrective action
and further instruction to the practical nurse (PN)
about proper care? Review Information: The correct answer is C:
A) The weights of the skin traction of a client are Test her stools for occult blood
hanging about 2 inches from the floor Both Prednisone and ASA can lead to GI bleed-
B) A client with a hip prosthesis 1 day post op- ing, therefore monitoring for occult blood would
eratively is lying in bed with internal rotation and be appropriate.
adduction of the affected leg
C) The nurse observes that the PN moves the
extremity of a client with an external fixation de- Question40
vice by picking up the frame A client with testicular cancer is scheduled for a
D) A client with skeletal traction states «The other right orchiectomy. The nurse knows that an or-
nurse said that the clear, yellow and crusty drain- chiectomy is the
age around the pin site is a good sign» A) surgical removal of the entire scrotum
B) surgical removal of a testicle
C) dissection of related lymph nodes
Review Information: The correct answer is B: D) partial surgical removal of the penis
A client with a hip prosthesis 1 day post opera-
tively is lying in bed with internal rotation and ad-
duction of the affected leg Review Information: The correct answer is B:
This position should be prevented in order to surgical removal of a testicle
prevent dislodgment of the hip prosthesis, es- The affected testicle is surgically removed along
pecially in the first 48 to 72 hours post-op. The with its tunica and spermatic cord.
other assessments are not of concern.
0 comments

Question38 Labels: free nclex-rn sample review questions,


A client diagnosed with gouty arthritis is admit- nclex-rn practice test questions, nursing review
ted with severe pain and edema in the right foot.
When the nurse develops a plan of care, which Free NCLEX-RN Sample Test Ques-
intervention should be included? tions For Nursing Review (Part 5)
A) high protein diet
B) salicylates
Question1
C) hot compresses to affected joints
A client complains of some discomfort after a
D) intake of at least 3000cc/day
below the knee amputation. Which action by the
nurse is most appropriate initially?
Collected by :DeepaRajesh [ 174 ]
rajesh.ks21@gmail.com
Kuwait
A) Conduct guided imagery or distraction B) Days 10-13
B) Ensure that the stump is elevated the first day C) Days 14-16
post-op D) Days 17-19
C) Wrap the stump snugly in an elastic bandage
D) Administer opioid narcotics as ordered

Review Information: The correct answer is D:


Days 17-19
Review Information: The correct answer is B: Ovulation occurs 14 days prior to menses. Con-
Ensure that the stump is elevated the first day sidering that the woman>>s cycle is 32 days,
post-op subtracting 14 from 32 suggests ovulation is at
This priority intervention prevents pressure about the 18th day.
caused by pooling of blood, thus minimizing the
pain. Without this measure, a firm elastic band-
age, opioid narcotics, or guided imagery will Question4
have little effect. Opioid narcotics are given for A newborn is having difficulty maintaining a tem-
severe pain. perature above 98 degrees Fahrenheit and has
been placed in an incubator. Which action is a
nursing priority?
Question2 A) Protect the eyes of the neonate from the heat
A 78 year-old client with pneumonia has a pro- lamp
ductive cough, but is confused. Safety protective B) Monitor the neonate’s temperature
devices (restraints) have been ordered for this C) Warm all medications and liquids before giv-
client. How can the nurse prevent aspiration? ing
A) Suction the client frequently while restrained D) Avoid touching the neonate with cold hands
B) Secure all 4 restraints to 1 side of bed
C) Obtain a sitter for the client while restrained
D) Request an order for a cough suppressant

Review Information: The correct answer is B:


Review Information: The correct answer is C: Monitor the neonate’s temperature
Obtain a sitter for the client while restrained When using a warming device the neonate’s
The plan to use safety devices (restraints) should temperature should be continuously monitored
be rethought. Restraints are used to protect the for undesired elevations. The use of heat lamps
client from harm caused by removing tubes or is not safe as there is no way to regulate their
getting out of bed. In the event that this restricted temperature. Warming medications and fluids is
movement could cause more harm, such as as- not indicated. While touching with cold hands can
piration, then a sitter should be requested. These startle the infant it does not pose a safety risk.
are to be provided by the facility in the event the
family cannot do so. This client needs to cough
and be watched rather than restricted. Suction- Question5
ing will not prevent aspiration in this situation. Which oxygen delivery system would the nurse
Cough suppressants should be avoided for this apply that would provide the highest concentra-
client. tions of oxygen to the client?
A) Venturi mask
B) Partial rebreather mask
Question3 C) Non-rebreather mask
A couple trying to conceive asks the nurse when D) Simple face mask
ovulation occurs. The woman reports a regular
32 day cycle. Which response by the nurse is
correct?
A) Days 7-10
Collected by :DeepaRajesh [ 175 ]
rajesh.ks21@gmail.com
Kuwait
Review Information: The correct answer is C:
Non-rebreather mask Review Information: The correct answer is D:
The non-rebreather mask has a one-way valve Outline the spot with a pencil and note the time
that prevents exhales air from entering the res- and date on the cast
ervoir bag and one or more valves covering the This is a good way to assess the amount of bleed-
air holes on the face mask itself to prevent inha- ing over a period of time. The bleeding does not
lation of room air but to allow exhalation of air. appear to be excessive and some bleeding is
When a tight seal is achieved around the mask expected with this type of surgery. The bleed-
up to 100% of the oxygen is available. ing should also be documented in the nurse’s
notes.

Question6
At a senior citizens meeting a nurse talks with a Question8
client who has Type 1 diabetes mellitus. Which The nurse is caring for a 1 year-old child who
statement by the client during the conversation has 6 teeth. What is the best way for the nurse to
is most predictive of a potential for impaired skin give mouth care to this child?
integrity? A) Using a moist soft brush or cloth to clean teeth
A) «I give my insulin to myself in my thighs.» and gums
B) «Sometimes when I put my shoes on I don>t B) Swabbing teeth and gums with flavored
know where my toes are.» mouthwash
C) «Here are my up and down glucose readings C) Offering a bottle of water for the child to drink
that I wrote on my calendar.» D) Brushing with toothpaste and flossing each
D) «If I bathe more than once a week my skin tooth
feels too dry.»

Review Information: The correct answer is A:


Using a moist soft brush or cloth to clean teeth
and gums
Review Information: The correct answer is B: The nurse should use a soft cloth or soft brush to
«Sometimes when I put my shoes on I don>>t do mouth care so that the child can adjust to the
know where my toes are.» routine of cleaning the mouth and teeth.
Peripheral neuropathy can lead to lack of sensa-
tion in the lower extremities. Clients who do not
feel pressure and/or pain are at high risk for skin Question9
impairment. In addition to standard precautions, a nurse
should implement contact precautions for which
client?
Question7 A) 60 year-old with herpes simplex
A client returns from surgery after an open reduc- B) 6 year-old with mononucleosis
tion of a femur fracture. There is a small blood- C) 45 year-old with pneumonia
stain on the cast. Four hours later, the nurse ob- D) 3 year-old with scarlet fever
serves that the stain has doubled in size. What is
the best action for the nurse to take?
A) Call the health care provider
B) Access the site by cutting a window in the
cast Review Information: The correct answer is A:
C) Simply record the findings in the nurse>s 60 year-old with herpes simplex
notes only Clients who have herpes simplex infections must
D) Outline the spot with a pencil and note the have contact precautions in addition to standard
time and date on the cast precautions because of the associated, potential-
ly weeping, skin lesions. Contact precautions are
Collected by :DeepaRajesh [ 176 ]
rajesh.ks21@gmail.com
Kuwait
used for clients who are infected by microorgan- B) «I take half of the usual dose for my sinuses
isms that are transmitted by direct contact with to maintain my blood pressure.»
the client, including hand or skin-to-skin contact. C) «I have to sit at the side of the pool with the
grandchildren since I can>t swim with this eye
problem.»
Question10 D) «I take extra fiber and drink lots of water to
Which of the following situations is most likely to avoid getting constipated.»
produce sepsis in the neonate?
A) Maternal diabetes
B) Prolonged rupture of membranes
C) Cesarean delivery
D) Precipitous vaginal birth Review Information: The correct answer is D:
«I take extra fiber and drink lots of water to avoid
getting constipated.»
Any activity that involves straining should be
avoided in clients with glaucoma. Such activities
Review Information: The correct answer is B: would increase intraocular pressure.
Prolonged rupture of membranes
Premature rupture of the membranes (PROM) is
a leading cause of newborn sepsis. After 12-24 Question13
hours of leaking fluid, measures are taken to re- A newborn has hyperbilirubinemia and is un-
duce the risk to mother and the fetus/newborn. dergoing phototherapy with a fiberoptic blanket.
Which safety measure is most important during
this process?
A) Regulate the neonate’s temperature using a
radiant heater
Question11 B) Withhold feedings while under the photother-
The nurse is teaching a parent about side effects apy
of routine immunizations. Which of the following C) Provide water feedings at least every 2 hours
must be reported immediately? D) Protect the eyes of neonate from the photo-
A) Irritability therapy lights
B) Slight edema at site
C) Local tenderness
D) Seizure activity Review Information: The correct answer is C:
Provide water feedings at least every 2 hours
Protecting the eyes of the neonates is very im-
portant to prevent damage when under the ultra-
violet lights, but since the blanket is used, extra
Review Information: The correct answer is D: protection of the eyes is unnecessary. It is rec-
Seizure activity ommended that the neonate remain under the
Other reactions that should be reported include lights for extended periods. The neonate’s skin is
crying for >3 hours, temperature over 104.8 de- exposed to the light and the temperature is moni-
grees Fahrenheit following DPT immunization, tored, but a heater may not be necessary. There
and tender, swollen, reddened areas. is no reason to withhold feedings. Frequent wa-
ter or feedings are given to help with the excre-
tion of the bilirubin in the stool.
Question12 .
The nurse is at the community center speaking
with retired people about glaucoma. Which com- Question14
ment by one of the retirees would the nurse sup- A nurse is performing the routine daily cleaning
port to reinforce correct information? of a tracheostomy. During the procedure, the cli-
A) «I usually avoid driving at night since lights ent coughs and displaces the tracheostomy tube.
sometimes seem to make things blur.» This negative outcome could have avoided by
Collected by :DeepaRajesh [ 177 ]
rajesh.ks21@gmail.com
Kuwait
A) placing an obturator at the client’s bedside The nurse is teaching home care to the parents
B) having another nurse assist with the proce- of a child with acute spasmodic croup. The most
dure important aspects of this care is/are
C) fastening clean tracheostomy ties before re- A) sedation as needed to prevent exhaustion
moving old ties B) antibiotic therapy for 10 to 14 days
D) placing the client in a flat, supine position C) humidified air and increased oral fluids
D) antihistamines to decrease allergic response

Review Information: The correct answer is C:


fastening clean tracheostomy ties before remov- Review Information: The correct answer is C:
ing old ties humidified air and increased oral fluids
Fastening clean tracheostomy ties before remov- The most important aspects of home care for a
ing old ones will ensure that the tracheostomy child with acute spasmodic croup are humidified
is secured during the entire cleaning procedure. air and increased oral fluids. Moisture soothes
The obturator is useful to keep the airway open inflamed membranes. Adequate systemic hydra-
only after the tracheostomy outer tube is coughed tion aids is mucociliary clearance and keeps se-
out. A second nurse is not needed. Changing the cretions thin, white, watery, and easily removed
position may not prevent a dislodged tracheos- with minimal coughing.
tomy.

Question17
Question15 The nurse is assigned to care for a client who
A 4 year-old hospitalized child begins to have a has a leaking intracranial aneurysm. To minimize
seizure while playing with hard plastic toys in the the risk of rebleeding, the nurse should plan to
hallway. Of the following nursing actions, which A) restrict visitors to immediate family
one should the nurse do first? B) avoid arousal of the client except for family
A) Place the child in the nearest bed visits
B) Administer IV medication to slow down the C) keep client>s hips flexed at no less than 90
seizure degrees
C) Place a padded tongue blade in the child>s D) apply a warming blanket for temperatures of
mouth 98 degrees Fahrenheit or less
D) Remove the child>s toys from the immediate
area

Review Information: The correct answer is A:


restrict visitors to immediate family
Review Information: The correct answer is D: Maintaining a quiet environment will assist in
Remove the child>>s toys from the immediate minimizing cerebral rebleeding. When family vis-
area it, the client should not be disturbed. If the client
Nursing care for a child having a seizure includes, is awake, topics of a general nature are better
maintaining airway patency, ensuring safety, ad- choices for discussion than topics that result in
ministering medications, and providing emotion- emotional or physiological stimulation.
al support. Since the seizure has already started,
nothing should be forced into the child>>s mouth
and the child should not be moved. Of the choic- Question18
es given, the first priority would be to provide a A client who is 12 hour post-op becomes con-
safe environment. fused and says: “Giant sharks are swimming
across the ceiling.” Which assessment is nec-
essary to adequately identify the source of this
Question16 client>s behavior?
Collected by :DeepaRajesh [ 178 ]
rajesh.ks21@gmail.com
Kuwait
A) Cardiac rhythm strip Fahrenheit (36 degrees Celsius).
B) Pupillary response
C) Pulse oximetry
D) Peripheral glucose stick Question20
Which contraindication should the nurse assess
for prior to giving a child immunizations?
A) Mild cold symptoms
B) Chronic asthma
Review Information: The correct answer is C: C) Depressed immune system
Pulse oximetry D) Allergy to eggs
A sudden change in mental status in any post-
op client should trigger a nursing intervention
directed toward respiratory evaluation. Pulse
oximetry would be the initial assessment. If avail-
able, arterial blood gases would be better. Acute Review Information: The correct answer is C:
respiratory failure is the sudden inability of the Depressed immune system
respiratory system to maintain adequate gas Children who have a depressed immune system
exchange which may result in hypercapnia and/ related to HIV or chemotherapy should not be
or hypoxemia. Clinical findings of hypoxemia in- given routine immunizations.
clude these finding which are listed in order of
initial to later findings: restlessness, irritability,
agitation, dyspnea, disorientation, confusion, de- Question21
lirium, hallucinations, and loss of consciousness. The nurse is caring for a client with a myocardial
While there may be other factors influencing the infarction. Which finding requires the nurse>s
client>>s behavior, the first nursing action should immediate action?
be directed toward maintaining oxygenation. A) Periorbital edema
Once respiratory or oxygenation issues are ruled B) Dizzy spells
out then significant changes in glucose would be C) Lethargy
evaluated. D) Shortness of breath

Question19
A newborn delivered at home without a birth at-
tendant is admitted to the hospital for observa- Review Information: The correct answer is B:
tion. The initial temperature is 95 degrees Fahr- Dizzy spells
enheit (35 degrees Celsius) axillary. The nurse Cardiac dysrhythmias may cause a transient
recognizes that cold stress may lead to what drop in cardiac output and decreased blood flow
complication? to the brain. Near syncope refers to lightheart-
A) Lowered BMR edness, dizziness, temporary confusion. Such
B) Reduced PaO2 «spells» may indicate runs of ventricular tachy-
C) Lethargy cardia or periods of asystole and should be re-
D) Metabolic alkalosis ported immediately.

Question22
Decentralized scheduling is used on a nursing
Review Information: The correct answer is B: unit. A chief advantage of this management strat-
Reduced PaO2 egy is that it:
Cold stress causes increased risk for respira- A) considers client and staff needs
tory distress. The baby delivered in such circum- B) conserves time spent on planning
stances needs careful monitoring. In this situa- C) frees the nurse manager to handle other pri-
tion, the newborn must be warmed immediately orities
to increase its temperature to at least 97 degrees D) allows requests for special privileges
Collected by :DeepaRajesh [ 179 ]
rajesh.ks21@gmail.com
Kuwait
Question25
A 16 year-old boy is admitted for Ewing>s sar-
Review Information: The correct answer is A: coma of the tibia. In discussing his care with the
considers client and staff needs parents, the nurse understands that the initial
Decentralized staffing takes into consideration treatment most often includes
specific client needs and staff interests and abili- A) amputation just above the tumor
ties. B) surgical excision of the mass
C) bone marrow graft in the affected leg
D) radiation and chemotherapy
Question23
Included in teaching the client with tuberculo-
sis taking isoniazid (INH) about follow-up home
care, the nurse should emphasize that a labora-
tory appointment for which of the following lab Review Information: The correct answer is D:
tests is critical? radiation and chemotherapy
A) Liver function The initial treatment of choice for Ewing>>s sar-
B) Kidney function coma is a combination of radiation and chemo-
C) Blood sugar therapy.
D) Cardiac enzymes

Question26
A new nurse manager is responsible for inter-
viewing applicants for a staff nurse position.
Review Information: The correct answer is A: Which interview strategy would be the best ap-
Liver function proach?
INH can cause hepatocellular injury and hepati- A) Vary the interview style for each candidate to
tis. This side effect is age-related and can be de- learn different techniques
tected with regular assessment of liver enzymes, B) Use simple questions requiring «yes» and
which are released into the blood from damaged «no» answers to gain definitive information
liver cells. C) Obtain an interview guide from human re-
sources for consistency in interviewing each
candidate
Question24 D) Ask personal information of each applicant to
A woman in her third trimester complains of se- assure he/she can meet job demands
vere heartburn. What is appropriate teaching
by the nurse to help the woman alleviate these
symptoms? Review Information: The correct answer is C:
A) Drink small amounts of liquids frequently Obtain an interview guide from human resources
B) Eat the evening meal just before retiring for consistency in interviewing each candidate
C) Take sodium bicarbonate after each meal An interview guide used for each candidate ena-
D) Sleep with head propped on several pillows bles the nurse manager to be more objective in
the decision making. The nurse should use re-
sources available in the agency before attempts
to develop one from scratch. Certain personal
questions are prohibited, and HR can identify
Review Information: The correct answer is D: these for novice managers.
Sleep with head propped on several pillows
Heartburn is a burning sensation caused by re-
gurgitation of gastric contents. It is best relieved Question27
by sleeping position, eating small meals, and not What is the best way that parents of pre-school-
eating before bedtime. ers can begin teaching their child about injury
Collected by :DeepaRajesh [ 180 ]
rajesh.ks21@gmail.com
Kuwait
prevention? oliguria
A) Set good examples themselves Kidneys maintain fluid volume through adjust-
B) Protect their child from outside influences ments in urine volume.
C) Make sure their child understands all the
safety rules
D) Discuss the consequences of not wearing Question30
protective devices A 70 year-old woman is evaluated in the emer-
gency department for a wrist fracture of unknown
causes. During the process of taking client his-
Review Information: The correct answer is A: tory, which of these items should the nurse iden-
Set good examples themselves tify as related to the client’s greatest risk factors
The preschool years are the time for parents to for osteoporosis?
begin emphasizing safety principles as well as A) History of menopause at age 50
providing protection. Setting a good example B) Taking high doses of steroids for arthritis for
themselves is crucial because of the imitative many years
behaviors of pre-schoolers; they are quick to no- C) Maintaining an inactive lifestyle for the past
tice discrepancies between what they see and 10 years
what they are told. D) Drinking 2 glasses of red wine each day for
the past 30 years

Question28
A nurse assessing the newborn of a mother with Review Information: The correct answer is
diabetes understands that hypoglycemia is re- B: Taking high doses of steroids for arthritis for
lated to what pathophysiological process? many years
A) Disruption of fetal glucose supply The use of steroids, especially at high doses over
B) Pancreatic insufficiency time, increases the risk for osteoporosis. The
C) Maternal insulin dependency other options also predispose to osteoporosis,
D) Reduced glycogen reserves as do low bone mass, poor calcium absorption
and moderate to high alcohol ingestion. Long-
term steroid treatment is the most significant risk
Review Information: The correct answer is A: factor, however.
Disruption of fetal glucose supply
After delivery, the high glucose levels which
crossed the placenta to the fetus are suddenly Question31
stopped. The newborn continues to secrete insu- The nurse is caring for a 2 year-old who is being
lin in anticipation of glucose. When oral feedings treated with chelation therapy, calcium disodium
begin, the newborn will adjust insulin production edetate, for lead poisoning. The nurse should be
within a day or two. alert for which of the following side effects?
A) Neurotoxicity
B) Hepatomegaly
Question29 C) Nephrotoxicity
The nurse is caring for a client with extracellular D) Ototoxicity
fluid volume deficit. Which of the following as-
sessments would the nurse anticipate finding?
A) bounding pulse Review Information: The correct answer is C:
B) rapid respirations Nephrotoxicity
C) oliguria Nephrotoxicity is a common side effect of calci-
D) neck veins are distended um disodium edetate, in addition to lead poison-
ing in general.

Question32
Review Information: The correct answer is C: The parents of a toddler ask the nurse how long
Collected by :DeepaRajesh [ 181 ]
rajesh.ks21@gmail.com
Kuwait
their child will have to sit in a car seat while in the Perform a neurovascular check for circulation
automobile. What is the nurse’s best response to While each of these is an important assessment,
the parents? the neurovascular integrity check is most associ-
A) «Your child must use a care seat until he ated with this type of traction. Russell’s traction
weighs at least 40 pounds.» is Buck’s traction with a sling under the knee.
B) «The child must be 5 years of age to use a
regular seat belt.»
C) «Your child must reach a height of 50 inches Question35
to sit in a seat belt.» When suctioning a client>s tracheostomy, the
D) «The child can use a regular seat belt when nurse should instill saline in order to
he can sit still.» A) decrease the client>s discomfort
B) reduce viscosity of secretions
C) prevent client aspiration
D) remove a mucus plug

Review Information: The correct answer is A:


«Your child must use a care seat until he weighs Review Information: The correct answer is D:
at least 40 pounds.» remove a mucus plug
Children should use car seats until they weigh While no longer recommended for routine suc-
40 pounds. tioning, saline may thin and loosen viscous se-
cretions that are very difficult to move, perhaps
making them easier to suction.
Question33
A client asks the nurse to explain the basic ideas
of homeopathic medicine. The response that best Question36
explains this approach is that such remedies The nurse is performing a gestational age as-
A) destroy organisms causing disease sessment on a newborn delivered 2 hours ago.
B) maintain fluid balance When coming to a conclusion using the Bal-
C) boost the immune system lard scale, which of these factors may affect the
D) increase bodily energy score?
A) Birth weight
B) Racial differences
C) Fetal distress in labor
D) Birth trauma
Review Information: The correct answer is C:
boost the immune system
The practitioner treats with minute doses of plant,
mineral or animal substances which provide a
gentle stimulus to the body>>s own defenses. Review Information: The correct answer is C:
Fetal distress in labor
The effects of earlier distress may alter the find-
Question34 ings of reflex responses as measured on the Bal-
A client with a fractured femur has been in Rus- lard tool. Other physical characteristics that esti-
sell’s traction for 24 hours. Which nursing action mate gestational age, such as amount of lanugo,
is associated with this therapy? sole creases and ear cartilage are unaffected by
A) Check the skin on the sacrum for breakdown the other factors.
B) Inspect the pin site for signs of infection
C) Auscultate the lungs for atelectasis
D) Perform a neurovascular check for circula- Question37
tion A nurse is caring for a client who had a closed
reduction of a fractured right wrist followed by
the application of a fiberglass cast 12 hours ago.
Review Information: The correct answer is D: Which finding requires the nurse’s immediate at-
Collected by :DeepaRajesh [ 182 ]
rajesh.ks21@gmail.com
Kuwait
tention?
A) Capillary refill of fingers on right hand is 3 sec- Review Information: The correct answer is C:
onds 75 year-old with left sided paresthesia who is in-
B) Skin warm to touch and normally colored continent of urine and stool
C) Client reports prickling sensation in the right Risk factors for pressure ulcers include: immo-
hand bility, absence of sensation, decreased LOC,
D) Slight swelling of fingers of right hand poor nutrition and hydration, skin moisture, in-
continence, increased age, decreased immune
response. This client has the greatest number of
Review Information: The correct answer is risk factors.
C: Client reports prickling sensation in the right
hand
A prickling sensation is an indication of compart- Question40
ment syndrome and requires immediate action The nurse is teaching the mother of a 5 month-
by the nurse. The other findings are normal for a old about nutrition for her baby. Which state-
client in this situation. ment by the mother indicates the need for further
teaching?
A) «I>m going to try feeding my baby some rice
Question38 cereal.»
A client is admitted with the diagnosis of pulmo- B) «When he wakes at night for a bottle, I feed
nary embolism. While taking a history, the client him.»
tells the nurse he was admitted for the same thing C) «I dip his pacifier in honey so he>ll take it.»
twice before, the last time just 3 months ago. The D) «I keep formula in the refrigerator for 24
nurse would anticipate the provider ordering hours.»
A) pulmonary embolectomy
B) vena caval interruption
C) increasing the Coumadin therapy to an INR Review Information: The correct answer is C:
of 3-4 «I dip his pacifier in honey so he>>ll take it.»
D) thrombolytic therapy Honey has been associated with infant botu-
lism and should be avoided. Older children and
adults have digestive enzymes that kill the botu-
Review Information: The correct answer is B: lism spores.
vena caval interruption
Clients with contraindications to Heparin, re- 0 comments
current PE or those with complications related
to the medical therapy may require vena caval Labels: free nclex-rn sample review questions,
interruption by the placement of a filter device nclex-rn practice test questions, nursing review
in the inferior vena cava. A filter can be placed
transvenously to trap clots before they travel to Free NCLEX-RN Sample Test Ques-
the pulmonary circulation. tions For Nursing Review (Part 4)

Question1
Question39 The clinic nurse is counseling a substance-abus-
Which client is at highest risk for developing a ing post partum client on the risks of continued
pressure ulcer? cocaine use. In order to provide continuity of
A) 23 year-old in traction for fractured femur care, which nursing diagnosis is a priority?
B) 72 year-old with peripheral vascular disease, A) Social isolation
who is unable to walk without assistance B) Ineffective coping
C) 75 year-old with left sided paresthesia who is C) Altered parenting
incontinent of urine and stool D) Sexual dysfunction
D) 30 year-old who is comatose following a rup-
tured aneurysm
Review Information: The correct answer is C:
Collected by :DeepaRajesh [ 183 ]
rajesh.ks21@gmail.com
Kuwait
Altered parenting cavity while instilling air into the gastric tube, this
The cocaine abusing mother puts her newborn indicates that it is accurately placed in the stom-
and other children at risk for neglect and abuse. ach. The feeding can begin after further assess-
Continuing to use drugs has the potential to im- ing the client for bowel sounds.
pact parenting behaviors. Social service referrals
are indicated.
Question4
While assessing the vital signs in children, the
Question2 nurse should know that the apical heart rate is
The nurse is teaching about nonsteroidal anti- preferred until the radial pulse can be accurately
inflammatory drugs (NSAIDs) to a group of ar- assessed at about what age?
thritic clients. To minimize the side effects, the A) 1 year of age
nurse should emphasize which of the following B) 2 years of age
actions? C) 3 years of age
A) Reporting joint stiffness in the morning D) 4 years of age
B) Taking the medication 1 hour before or 2 hours
after meals
C) Using alcohol in moderation unless driving Review Information: The correct answer is B:
D) Continuing to take aspirin for short term relief 2 years of age
A child should be at least 2 years of age to use
the radial pulse to assess heart rate.

Question5
A client is receiving Total Parenteral Nutrition
Review Information: The correct answer is B: (TPN) via a Hickman catheter. The catheter acci-
Taking the medication 1 hour before or 2 hours dentally becomes dislodged from the site. Which
after meals action by the nurse should take priority?
Taking the medication 1 hour before or 2 hours A) Check that the catheter tip is intact
after meals will result in a more rapid effect. B) Apply a pressure dressing to the site
C) Monitor respiratory status
D) Assess for mental status changes
Question3
The nurse is preparing to administer a tube feed-
ing to a postoperative client. To accurately as- Question6
sess for a gastrostomy tube placement, the pri- A pregnant client who is at 34 weeks gestation
ority is to is diagnosed with a pulmonary embolism (PE).
A) auscultate the abdomen while instilling 10 cc Which of these medications would the nurse an-
of air into the tube ticipate the provider ordering?
B) place the end of the tube in water to check for A) Oral Coumadin therapy
air bubbles B) Heparin 5000 units subcutaneously B.I.D.
C) retract the tube several inches to check for C) Heparin infusion to maintain the PTT at 1.5-
resistance 2.5 times the control value
D) measure the length of tubing from nose to epi- D) Heparin by subcutaneous injection to main-
gastrium tain the PTT at 1.5 times the control value

Review Information: The correct answer is A: Review Information: The correct answer is D:
auscultate the abdomen while instilling 10 cc of Heparin by subcutaneous injection to maintain
air into the tube the PTT at 1.5 times the control value
If a swoosh of air is heard over the abdominal Several studies have been conducted in preg-
Collected by :DeepaRajesh [ 184 ]
rajesh.ks21@gmail.com
Kuwait
nant women where oral anticoagulation agents of the following is true in regards to giving immu-
are contraindicated. Warfarin (Coumadin) is nizations to this child?
known to cross the placenta and is therefore re- A) Live vaccines are withheld in children with re-
ported to be teratogenic. nal chronic illness
B) The MMR vaccine should be given now, prior
to the transplant
Question7 C) An inactivated form of the vaccine can be giv-
The nurse is caring for a client with Hodgkin>s en at any time
disease who will be receiving radiation therapy. D) The risk of vaccine side effects precludes giv-
The nurse recognizes that, as a result of the ra- ing the vaccine
diation therapy, the client is most likely to experi-
ence
A) high fever
B) nausea
C) face and neck edema Review Information: The correct answer is B:
D) night sweats The MMR vaccine should be given now, prior to
the transplant
MMR is a live virus vaccine, and should be given
Review Information: The correct answer is B: at this time. Post-transplant, immunosuppressive
nausea drugs will be given and the administration of the
Because the client with Hodgkin>>s disease is live vaccine at that time would be contraindicated
usually healthy when therapy begins, the nausea because of the compromised immune system.
is especially troubling.
.
Question10
Question8 The nurse is preparing to take a toddler>s blood
A client is brought to the emergency room fol- pressure for the first time. Which of the following
lowing a motor vehicle accident. When assess- actions should the nurse perform first?
ing the client one-half hour after admission, the A) Explain that the procedure will help him to get
nurse notes several physical changes. Which well
finding would require the nurse>s immediate at- B) Show a cartoon character with a blood pres-
tention? sure cuff
A) increased restlessness C) Explain that the blood pressure checks the
B) tachycardia heart pump
C) tracheal deviation D) Permit handling the equipment before putting
D) tachypnea the cuff in place

Review Information: The correct answer is D:


Permit handling the equipment before putting the
Review Information: The correct answer is C: cuff in place
tracheal deviation The best way to gain the toddler>>s cooperation
The deviated trachea is a sign that a mediastinal is to encourage handling the equipment. Detailed
shift has occurred. This is a medical emergency. explanations are not helpful.

Question9 Question11
An 18 month-old child is on peritoneal dialysis in Which statement made by a nurse about the
preparation for a renal transplant in the near fu- goal of total quality management or continuous
ture. When the nurse obtains the child>s health quality improvement in a health care setting is
history, the mother indicates that the child has correct?
not had the first measles, mumps, rubella (MMR) A) It is to observe reactive service and product
immunization. The nurse understands that which problem solving
Collected by :DeepaRajesh [ 185 ]
rajesh.ks21@gmail.com
Kuwait
B) Improvement of the processes in a proactive, Recognize personal attitudes about cultural dif-
preventive mode is paramount ferences and real or expected biases
C) A chart audits to finds common errors in prac- The nurse must discover personal attitudes, prej-
tice and outcomes associated with goals udices and biases specific to different cultures.
D) A flow chart to organize daily tasks is critical Awareness of these will prevent negative conse-
to the initial stages quences for interactions with clients and families
across cultures.

Question14
Review Information: The correct answer is A client with chronic obstructive pulmonary dis-
B: Improvement of the processes in a proactive, ease (COPD) and a history of coronary artery
preventive mode is paramount disease is receiving aminophylline, 25mg/hour.
Total quality management and continuous qual- Which one of the following findings by the nurse
ity improvement have a major goal of identifying would require immediate intervention?
ways to do the right thing at the right time in the A) Decreased blood pressure and respirations
right way by proactive problem-solving. B) Flushing and headache
C) Restlessness and palpitations
D) Increased heart rate and blood pressure
Question12
Which of the following drugs should the nurse
anticipate administering to a client before they
are to receive electroconvulsive therapy?
A) Benzodiazepines Review Information: The correct answer is C:
B) Chlorpromazine (Thorazine) Restlessness and palpitations
C) Succinylcholine (Anectine) Side effects of Aminophylline include restless-
D) Thiopental sodium (Pentothal Sodium) ness and palpitations.

Question15
A client has gastroesophageal reflux. Which rec-
Review Information: The correct answer is C: ommendation made by the nurse would be most
Succinylcholine (Anectine) helpful to the client?
Succinylcholine is given intravenously to pro- A) Avoid liquids unless a thickening agent is
mote skeletal muscle relaxation. used
B) Sit upright for at least 1 hour after eating
C) Maintain a diet of soft foods and cooked veg-
Question13 etables
Which approach is a priority for the nurse who D) Avoid eating 2 hours before going to sleep
works with clients from many different cultures?
A) Speak at least 2 other languages of clients in
the neighborhood
B) Learn about the cultures of clients who are
most often encountered Review Information: The correct answer is D:
C) Have a list of persons for referral when inter- Avoid eating 2 hours before going to sleep
action with these clients occur Eating before sleeping enhances the regurgita-
D) Recognize personal attitudes about cultural tion of stomach contents, which have increased
differences and real or expected biases acidity, into the esophagus. An upright posture
should be maintained for about 2 hours after eat-
ing to allow for the stomach emptying. Options A
and C are interventions for clients with swallow-
ing difficulties.
Review Information: The correct answer is D: .
Collected by :DeepaRajesh [ 186 ]
rajesh.ks21@gmail.com
Kuwait
client states, “I refuse both radiation and chemo-
Question16 therapy because they are <hot.>” The next ac-
A client with a panic disorder has a new prescrip- tion for the nurse to take is to
tion for Xanax (alprazolam). In teaching the client A) document the situation in the notes
about the drug>s actions and side effects, which B) report the situation to the health care provid-
of the following should the nurse emphasize? er
A) Short-term relief can be expected C) talk with the client>s family about the situa-
B) The medication acts as a stimulant tion
C) Dosage will be increased as tolerated D) ask the client to talk about concerns regarding
D) Initial side effects often continue «hot» treatments

Review Information: The correct answer is A: Review Information: The correct answer is D:
Short-term relief can be expected ask the client to talk about concerns regarding
Xanax is a short-acting benzodiazepine useful in «hot» treatments
controlling panic symptoms quickly. The «hot-cold» system is found among Mexican-
Americans, Puerto Ricans, and other Hispanic-
Latinos. Most foods, beverages, herbs, and
Question17 medicines are categorized as hot or cold, which
A client being discharged from the cardiac step- are symbolic designations and do not necessar-
down unit following a myocardial infarction (MI), ily indicate temperature or spiciness. Care and
is given a prescription for a beta-blocking drug. A treatment regimens can be negotiated with cli-
nursing student asks the charge nurse why this ents within this framework.
drug would be used by a client who is not hyper-
tensive. What is an appropriate response by the
charge nurse? Question19
A) «Most people develop hypertension following A 72 year-old client is scheduled to have a cardi-
an MI.» oversion. A nurse reviews the client’s medication
B) «A beta-Blocker will prevent orthostatic hypo- administration record. The nurse should notify the
tension.» health care provider if the client received which
C) «This drug will decrease the workload on his medication during the preceding 24 hours?
heart.» A) Digoxin (Lanoxin)
D) «Beta-blockers increase the strength of heart B) Diltiazem (Cardizem)
contractions.» C) Nitroglycerine ointment
D) Metoprolol (Toprol XL)

Review Information: The correct answer is


C: «This drug will decrease the workload on his Review Information: The correct answer is A:
heart.» Digoxin (Lanoxin)
One action of beta-blockers is to decrease sys- Digoxin increases ventricular irritability and in-
temic vascular resistance by dilating arterioles. creases the risk of ventricular fibrillation follow-
This is useful for the client with coronary artery ing cardioversion. The other medications do not
disease, and will reduce the risk of another MI or increase ventricular irritability.
sudden death.

Question20
Question18 Which of these clients, all of whom have the find-
A 35-year-old client of Puerto Rican-American ings of a board-like abdomen, would the nurse
descent is diagnosed with ovarian cancer. The suggest that the provider examine first?
Collected by :DeepaRajesh [ 187 ]
rajesh.ks21@gmail.com
Kuwait
A) An elderly client who stated, «My awful pain To prevent drug resistance from developing, the
in my right side suddenly stopped about 3 hours nurse is aware that which of the following is a
ago.» characteristic of the typical treatment plan to
B) A pregnant woman of 8 weeks newly diag- eliminate the tuberculosis bacilli?
nosed with an ectopic pregnancy A) An anti-inflammatory agent
C) A middle-aged client admitted with diverticu- B) High doses of B complex vitamins
litis who has taken only clear liquids for the past C) Aminoglycoside antibiotics
week D) Administering two anti-tuberculosis drugs
D) A teenager with a history of falling off a bicycle
without hitting the handle bars

Review Information: The correct answer is A: Review Information: The correct answer is D:
An elderly client who stated, «My awful pain in Administering two anti-tuberculosis drugs
my right side suddenly stopped about 3 hours Resistance of the tubercle bacilli often occurs to
ago.» a single antimicrobial agent. Therefore, therapy
This client has the highest risk for hypovolemic with multiple drugs over a long period of time
and septic shock since the appendix has most helps to ensure eradication of the organism.
likely ruptured, based on the history of the pain
suddenly stopping over three hours ago. Elderly
clients have less functional reserve for the body Question23
to cope with shock and infection over long peri- The nurse is assessing a comatose client receiv-
ods. The others are at risk for shock also, how- ing gastric tube feedings. Which of the following
ever given that they fall in younger age groups, assessments requires an immediate response
they would more likely be able to tolerate an im- from the nurse?
balance in circulation. A common complication of A) Decreased breath sounds in right lower lobe
falling off a bicycle is hitting the handle bars in B) Aspiration of a residual of 100cc of formula
the upper abdomen often on the left, resulting in C) Decrease in bowel sounds
a ruptured spleen. D) Urine output of 250 cc in past 8 hours

Question21
The nurse is teaching parents of a 7 month-old
about adding table foods. Which of the following Review Information: The correct answer is A:
is an appropriate finger food? Decreased breath sounds in right lower lobe
A) Hot dog pieces The most common problem associated with en-
B) Sliced bananas teral feedings is atelectasis. Maintain client at 30
C) Whole grapes degrees of head elevation during feedings and
D) Popcorn monitor for signs of aspiration. Check for tube
placement prior to each feeding or every 4 to 8
hours if the client is receiving continuous feed-
ing.

Review Information: The correct answer is B:


Sliced bananas Question24
Finger foods should be bite-size pieces of soft A client is prescribed warfarin sodium (Couma-
food such as bananas. Hot dogs and grapes can din) to be continued at home. Which focus is
accidentally be swallowed whole and can occlude critical to be included in the nurse’s discharge
the airway. Popcorn is too difficult to chew at this instruction?
age and can irritate the airway if swallowed. A) Maintain a consistent intake of green leafy
foods
B) Report any nose or gum bleeds
Question22 C) Take Tylenol for minor pains
Collected by :DeepaRajesh [ 188 ]
rajesh.ks21@gmail.com
Kuwait
D) Use a soft toothbrush
Question27
The nurse manager informs the nursing staff at
morning report that the clinical nurse specialist
will be conducting a research study on staff at-
Review Information: The correct answer is B: titudes toward client care. All staff are invited to
Report any nose or gum bleeds participate in the study if they wish. This affirms
The client should notify the health care provider the ethical principle of
if blood is noted in stools or urine, or any other A) Anonymity
signs of bleeding occur. B) Beneficence
C) Justice
D) Autonomy
Question25
When teaching a client about the side effects of
fluoxetine (Prozac), which of the following will
the nurse include?
A) Tachycardia blurred vision, hypotension, ano- Review Information: The correct answer is D:
rexia Autonomy
B) Orthostatic hypotension, vertigo, reactions to Individuals must be free to make independent
tyramine-rich foods decisions about participation in research without
C) Diarrhea, dry mouth, weight loss, reduced li- coercion from others.
bido
D) Photosensitivity, seizures, edema, hypergly-
cemia Question28
The nurse is talking with the family of an 18
months-old newly diagnosed with retinoblasto-
Review Information: The correct answer is C: ma. A priority in communicating with the parents
Diarrhea, dry mouth, weight loss, reduced libido is
Commonly reported side effects for fluoxetine A) Discuss the need for genetic counseling
(Prozac) are diarrhea, dry mouth, weight loss B) Inform them that combined therapy is seldom
and reduced libido. effective
C) Prepare for the child>s permanent disfigure-
ment
Question26 D) Suggest that total blindness may follow sur-
A newborn weighed 7 pounds 2 ounces at birth. gery
The nurse assesses the newborn at home 2
days later and finds the weight to be 6 pounds 7
ounces. What should the nurse tell the parents Review Information: The correct answer is A:
about this weight loss? Discuss the need for genetic counseling
A) The newborn needs additional assessments The hereditary aspects of this disease are well
B) The mother should breast feed more often documented. While the parents focus on the
C) A change to formula is indicated needs of this child, they should be aware that
D) The loss is within normal limits the risk is high for future offspring.

Question29
The nurse is planning care for an 8 year-old child.
Review Information: The correct answer is D: Which of the following should be included in the
The loss is within normal limits plan of care?
A newborn is expected to lose 5-10% of the birth A) Encourage child to engage in activities in the
weight in the first few days post-partum because playroom
of changes in elimination and feeding. B) Promote independence in activities of daily
living
Collected by :DeepaRajesh [ 189 ]
rajesh.ks21@gmail.com
Kuwait
C) Talk with the child and allow him to express
his opinions
D) Provide frequent reassurance and cuddling

Review Information: The correct answer is C:


Daily blood sugar monitoring
Normal hemoglobin A1C (glycosylated hemo-
Review Information: The correct answer is globin) level is 7 to 9%. Elevation indicates el-
A: Encourage child to engage in activities in the evated glucose levels over time.
playroom
According to Erikson, the school age child is in
the stage of industry versus inferiority. To help Question32
them achieve industry, the nurse should encour- A client taking isoniazid (INH) for tuberculosis
age them to carry out tasks and activities in their asks the nurse about side effects of the medica-
room or in the playroom. tion. The client should be instructed to immedi-
ately report which of these?
A) Double vision and visual halos
Question30 B) Extremity tingling and numbness
The nurse is assigned to care for 4 clients. Which C) Confusion and lightheadedness
of the following should be assessed immediately D) Sensitivity of sunlight
after hearing the report?
A) The client with asthma who is now ready for
discharge Review Information: The correct answer is B:
B) The client with a peptic ulcer who has been Extremity tingling and numbness
vomiting all night Peripheral neuropathy is the most common side
C) The client with chronic renal failure returning effect of INH and should be reported to the pro-
from dialysis vider. It can be reversed.
D) The client with pancreatitis who was admitted
yesterday
Question33
Which of these questions is priority when as-
sessing a client with hypertension?
A) «What over-the-counter medications do you
Review Information: The correct answer is B: take?»
The client with a peptic ulcer who has been vom- B) «Describe your usual exercise and activity
iting all night patterns.»
A perforated peptic ulcer could cause nausea, C) «Tell me about your usual diet.»
vomiting and abdominal distention, and may be D) «Describe your family>s cardiovascular his-
a life threatening situation. The client should be tory.»
assessed immediately and findings reported to
the provider.
.

Question31 Review Information: The correct answer is


During a routine check-up, an insulin-depend- A: «What over-the-counter medications do you
ent diabetic has his glycosylated hemoglobin take?»
checked. The results indicate a level of 11%. Over-the-counter medications, especially those
Based on this result, what teaching should the that contain cold preparations can increase the
nurse emphasize? blood pressure to the point of hypertension.
A) Rotation of injection sties
B) Insulin mixing and preparation
C) Daily blood sugar monitoring Question34
D) Regular high protein diet The nurse is performing an assessment of the
Collected by :DeepaRajesh [ 190 ]
rajesh.ks21@gmail.com
Kuwait
motor function in a client with a head injury. The
best technique is
A) touching the trapezius muscle or arm firmly
B) pinching any body part
C) shaking a limb vigorously Review Information: The correct answer is
D) rubbing the sternum B: «He has had an ear infection for the past 2
days.»
Contributing factors to seizures in children in-
clude those such as age (more common in first
2 years), infections (late infancy and early child-
Review Information: The correct answer is D: hood), fatigue, not eating properly and excessive
rubbing the sternum fluid intake or fluid retention.
The purpose is to assess the non-responsive cli-
ent’s reaction to a painful stimulus after less nox-
ious methods have been tried. Question37
Which of these clients would the nurse monitor
for the complication of C. difficile diarrhea?
Question35 A) An adolescent taking medications for acne
A nurse admits a client transferred from the emer- B) An elderly client living in a retirement center
gency room (ER). The client, diagnosed with a taking prednisone
myocardial infarction, is complaining of subster- C) A young adult at home taking a prescribed
nal chest pain, diaphoresis and nausea. The first aminoglycoside
action by the nurse should be to D) A hospitalized middle aged client receiving
A) order an EKG clindamycin
B) administer morphine sulfate
C) start an IV
D) measure vital signs

Review Information: The correct answer is D:


A hospitalized middle aged client receiving clin-
damycin
Review Information: The correct answer is B: Hospitalized patients, especially those receiving
administer morphine sulfate antibiotic therapy, are primary targets for C. diffi-
Decreasing the clients pain is the most important cile. Of clients receiving antibiotics, 5-38% expe-
priority at this time. As long as pain is present rience antibiotic-associated diarrhea; C. difficile
there is danger in extending the infarcted area. causes 15 to 20% of the cases. Several antibiot-
Morphine will decrease the oxygen demands of ic agents have been associated with C. difficile.
the heart and act as a mild diuretic as well. It is Broad-spectrum agents, such as clindamycin,
probable that an EKG and IV insertion were per- ampicillin, amoxicillin, and cephalosporins, are
formed in the ER. the most frequent sources of C. difficile. Also,
C. difficile infection has been caused by the ad-
ministration of agents containing beta-lactamase
Question36 inhibitors (i.e., clavulanic acid, sulbactam, tazo-
The nurse admits a 2 year-old child who has had bactam) and intravenous agents that achieve
a seizure. Which of the following statement by substantial colonic intraluminal concentrations
the child>s parent would be important in deter- (i.e., ceftriaxone, nafcillin, oxacillin). Fluoroqui-
mining the etiology of the seizure? nolones, aminoglycosides, vancomycin, and tri-
A) «He has been taking long naps for a week.» methoprim are seldom associated with C. difficile
B) «He has had an ear infection for the past 2 infection or pseudomembranous colitis.
days.»
C) «He has been eating more red meat lately.»
D) «He seems to be going to the bathroom more Question38
frequently.» The nurse is performing an assessment on a cli-
Collected by :DeepaRajesh [ 191 ]
rajesh.ks21@gmail.com
Kuwait
ent who is cachectic and has developed an en- mg/dl. What action by the nurse is appropriate at
terocutaneous fistula following surgery to relieve this time?
a small bowel obstruction. The client>s total pro- A) Give oral glucose water
tein level is reported as 4.5 g/dl. Which of the B) Notify the pediatrician
following would the nurse anticipate? C) Repeat the test in 2 hours
A) Additional potassium will be given IV D) Check the pulse oximetry reading
B) Blood for coagulation studies will be drawn
C) Total parenteral nutrition (TPN) will be start-
ed Review Information: The correct answer is C:
D) Serum lipase levels will be evaluated Repeat the test in 2 hours
This blood sugar is within the normal range for a
full-term newborn. Normal values are: Premature
infant: 20-60 mg/dl or 1.1-3.3 mmol/L, Neonate:
30-60 mg/dl or 1.7-3.3 mmol/L, Infant: 40-90 mg/
Review Information: The correct answer is C: dl or 2.2-5.0 mmol/L. Critical values are: Infant:
Total parenteral nutrition (TPN) will be started <40 mg/dl and in a Newborn: <30 and >300 mg/
The client is not absorbing nutrients adequately dl. Because of the increased birth weight which
as evidenced by the cachexia and low protein can be associated with diabetes mellitus, repeat-
levels. (A normal total serum protein level is 6.0- ed blood sugars will be drawn
8.0 g/dl.) TPN will promote a positive nitrogen
balance in this client who is unable to digest and 0 comments
absorb nutrients adequately.
Labels: free nclex-rn sample review questions,
nclex-rn practice test questions, nursing review
Question39
During a situation of pain management, which Free NCLEX-RN Sample Test Ques-
statement is a priority to consider for the ethical tions For Nursing Review (Part 3)
guidelines of the nurse?
A) The client>s self-report is the most important Question1
consideration A client diagnosed with chronic depression is
B) Cultural sensitivity is fundamental to pain maintained on tranylcypromine (Parnate). An im-
management portant nursing intervention is to teach the client
C) Clients have the right to have their pain re- to avoid which of the following foods?
lieved A) Wine, beer, cheese, liver and chocolate
D) Nurses should not prejudge a client>s pain B) Wine, citrus fruits, yogurt and broccoli
using their own values C) Beer, cheese, beef and carrots
D) Wine, apples, sour cream and beef steak

Review Information: The correct answer is A:


Review Information: The correct answer is A: Wine, beer, cheese, liver and chocolate
The client>>s self-report is the most important These foods are tyramine rich and ingestion of
consideration these foods while taking monoamine oxidase in-
Pain is a complex phenomenon that is perceived hibitors (MAOIs) can precipitate a life-threaten-
differently by each individual. Pain is whatever ing hypertensive crisis.
the client says it is. The other statements are cor-
rect but not the most important considerations.
Question2
The nurse is working in a high risk antepartum
clinic. A 40 year-old woman in the first trimes-
Question40 ter gives a thorough health history. Which infor-
As a part of a 9 pound full-term newborn>s as- mation should receive priority attention by the
sessment, the nurse performs a dextro-stick at 1 nurse?
hour post birth. The serum glucose reading is 45 A) Her father and brother are insulin dependent
Collected by :DeepaRajesh [ 192 ]
rajesh.ks21@gmail.com
Kuwait
diabetics of acute alcohol intoxication, it is important for
B) She has taken 800 mcg of folic acid daily for the nurse initially to obtain data regarding which
the past year of the following?
C) Her husband was treated for tuberculosis as A) What and how much the client drinks, accord-
a child ing to family and friends
D) She reports recent use of over-the counter si- B) The blood alcohol level of the client
nus remedies C) The blood pressure level of the client
D) The blood glucose level of the client

Review Information: The correct answer is D:


She reports recent use of over-the counter sinus Review Information: The correct answer is B:
remedies The blood alcohol level of the client
Over-the-counter drugs are a possible danger in Blood alcohol levels are generally obtained to
early pregnancy. A report by the client that she determine the level of intoxication. The amount
has taken medications should be followed up im- of alcohol consumed determines how much
mediately. medication the client needs for detoxification and
treatment. Reports of alcohol consumption are
Question3 notoriously inaccurate.
What must be the priority consideration for nurs-
es when communicating with children? Question6
A) Present environment Which clinical finding would the nurse expect to
B) Physical condition assess first in a newborn with spastic cerebral
C) Nonverbal cues palsy?
D) Developmental level A) cognitive impairment
B) hypotonic muscular activity
Review Information: The correct answer is D: C) seizures
Developmental level D) criss-crossing leg movement
While each of the factors affect communication,
the nurse recognizes that developmental differ-
ences have implications for processing and un- Review Information: The correct answer is D:
derstanding information. Consequently, a child’s criss-crossing leg movement
developmental level must be considered when Cerebral palsy is a neuromuscular impairment
selecting communication approaches. resulting in muscular and reflexive hypertonicity
and the criss-crossing, or scissoring leg move-
Question4 ments.
The nurse is assessing a client>s home in prep-
aration for discharge. Which of the following Question7
should be given priority consideration? Which medication is more helpful in treating
A) Family understanding of client needs bulimia than anorexia?
B) Financial status A) Amphetamines
C) Location of bathrooms B) Sedatives
D) Proximity to emergency services C) Anticholinergics
D) Narcotics
Review Information: The correct answer is A:
Family understanding of client needs Review Information: The correct answer is C:
Functional communication patterns between Anticholinergics
family members are fundamental to meeting the In contrast to anorexics, individuals with bulimia
needs of the client and family. are troubled by their behavioral characteristics
and become depressed. The person feels com-
pelled to binge, purge and fast. Feeling helpless
to stop the behavior, feelings of self-disgust oc-
Question5 cur.
As a general guide for emergency management
Collected by :DeepaRajesh [ 193 ]
rajesh.ks21@gmail.com
Kuwait
Question8
The nurse is assessing a woman in early labor. Review Information: The correct answer is D:
While positioning for a vaginal exam, she com- «I feel pressure in the middle of my chest, like an
plains of dizziness and nausea and appears elephant is sitting on my chest.»
pale. Her blood pressure has dropped slightly. This is a classic description of chest pain in men
What should be the initial nursing action? caused by myocardial ischemia. Women experi-
A) Call the health care provider ence vague feelings of fatigue and back and jaw
B) Encourage deep breathing pain.
C) Elevate the foot of the bed
D) Turn her to her left side Question11
A nurse is caring for a client who has just been
Review Information: The correct answer is D: admitted with an overdose of aspirin. The fol-
Turn her to her left side lowing lab data is available: PaO2 95, PaCO2
The weight of the uterus can put pressure on the 30, pH 7.5, K 3.2 mEq/l. Which should be the
vena cava and aorta when a pregnant woman nurse>s first action?
is flat on her back causing supine hypotension. A) Monitor respiratory rate
Action is needed to relieve the pressure on the B) Monitor intake and output every hour
vena cava and aorta. Turning the woman to the C) Assist the client to breathe into a paper bag
side reduces this pressure and relieves postural D) Prepare to administer oxygen by mask
hypotension.

Question9 Review Information: The correct answer is C:


A client has been started on a long term corticos- Assist the client to breathe into a paper bag
teroid therapy. Which of the following comments Side effects of aspirin toxicity include hyperventi-
by the client indicate the need for further teach- lation, which can result in respiratory alkalosis in
ing? the initial stages. Breathing into a paper bag will
A) «I will keep a weekly weight record.» prevent further reduction in PaCO2.
B) «I will take medication with food.»
C) «I will stop taking the medication for 1 week Question12
every month.» After assessing a 70 year-old male client>s labo-
D) «I will eat foods high in potassium.» ratory results during a routine clinic visit, which
one of the following findings would indicate an
area in which teaching is needed:
Review Information: The correct answer is C: A) Serum albumin 2.5 g/dl
«I will stop taking the medication for 1week every B) LDL Cholesterol 140 mg/dl
month.» C) Serum glucose 90 mg/dl
Emphatically warn against discontinuing steroid D) RBC 5.0 million/mm3
dosage abruptly because that may produce a fa-
tal adrenal crisis.
Review Information: The correct answer is A:
Question10 Serum albumin 2.5 g/dl
A male client calls for a nurse because of chest Serum albumin level is low (normal 3.0 – 5.0 g/dl
pain. Which statement by the client would require in elders), indicating nutritional counseling to in-
the most immediate action by the nurse? crease dietary protein is needed. Socioeconomic
A) «When I take in a deep breath, it stabs like a factors may need to be addressed to help the
knife.» client comply with the recommendation.
B) «The pain came on after dinner. That soup
seemed very spicy.» Question13
C) «When I turn in bed to reach the remote for When teaching a client with a new prescription
the TV, my chest hurts.» for lithium (Lithane) for treatment of a bi-polar
D) «I feel pressure in the middle of my chest, like disorder which of these should the nurse empha-
an elephant is sitting on my chest.» size?
A) Maintaining a salt restricted diet
Collected by :DeepaRajesh [ 194 ]
rajesh.ks21@gmail.com
Kuwait
B) Reporting vomiting or diarrhea 28. The initial nursing intervention would be to
C) Taking other medication as usual A) begin intravenous therapy
D) Substituting generic form if desired B) initiate continuous blood pressure monitoring
C) administer oxygen therapy
D) institute cardiac monitoring
Review Information: The correct answer is B:
Reporting vomiting or diarrhea Review Information: The correct answer is C:
If dehydration results from vomiting, diarrhea administer oxygen therapy
or excessive perspiration, tolerance to the drug Early findings of shock reveal hypoxia with rapid
may be altered and symptoms may return. heart rate and rapid respirations, and oxygen is
the most critical initial intervention. The other in-
terventions are secondary to oxygen therapy.
Question14
A client is discharged on warfarin sulfate (Cou- Question17
madin). Which statement by the client indicated A client is admitted to the hospital with a diag-
a need for further teaching? nosis of deep vein thrombosis. During the ini-
A) «I know I must avoid crowds.» tial assessment, the client complains of sudden
B) «I will keep all laboratory appointments.» shortness of breath. The SaO2 is 87. The priority
C) «I plan to use an electric razor for shaving.» nursing assessment at this time is
D) «I will report any bruises for bleeding.» A) bowel sounds
B) heart rate
C) peripheral pulses
D) lung sounds
Review Information: The correct answer is A:
«I know I must avoid crowds.»
There are no specific reasons for the client on Review Information: The correct answer is D:
Coumadin to avoid crowds. General instructions lung sounds
for any cardiac surgical client include limiting ex- Lung sounds are critical assessments at this
posure to infection. point. The nurse should be alert to crackles or a
pleural friction rub, highly suggestive of a pulmo-
Question15 nary embolism.
A client is taking tranylcypromine (Parnate) and
has received dietary instruction. Which of the fol- Question18
lowing food selections would be contraindicated The nurse is administering lidocaine (Xylocaine)
for this client? to a client with a myocardial infarction. Which of
A) Fresh juice, carrots, vanilla pudding the following assessment findings requires the
B) Apple juice, ham salad, fresh pineapple nurse>s immediate action?
C) Hamburger, fries, strawberry shake A) Central venous pressure reading of 11
D) Red wine, fava beans, aged cheese B) Respiratory rate of 22
C) Pulse rate of 48 BPM
D) Blood pressure of 144/92
Review Information: The correct answer is D:
Red wine, fava beans, aged cheese
Red wine and cheese contain tyramine (as do Review Information: The correct answer is C:
chicken liver and ripe bananas) and so are con- Pulse rate of 48 BPM
traindicated when taking MAOIs. Fava beans One of the side effects of lidocaine is bradycardia,
contain other vasopressors that can interact with heart block, cardiovascular collapse and cardiac
MAOIs also causing malignant hypertension. arrest (this drug should never be administered
without continuous EKG monitoring).
Question16
A client is admitted with severe injuries from an Question19
auto accident. The client>s vital signs are BP The nurse is teaching a group of college students
120/50, pulse rate 110, and respiratory rate of about breast self-examination. A woman asks for
Collected by :DeepaRajesh [ 195 ]
rajesh.ks21@gmail.com
Kuwait
the best time to perform the monthly exam. What Health education is also available through local
is the best reply by the nurse? and national Alzheimer>>s chapters.
A) «The first of every month, because it is easi-
est to remember» Question22
B) «Right after the period, when your breasts are Clients taking lithium must be particularly sure to
less tender» maintain adequate intake of which of these ele-
C) «Do the exam at the same time every ments?
month» A) Potassium
D) «Ovulation, or mid-cycle is the best time to B) Sodium
detect changes» C) Chloride
D) Calcium

Review Information: The correct answer is B:


«Right after the period, when your breasts are Review Information: The correct answer is B:
less tender» Sodium
The best time for a breast self exam (BSE) is a Clients taking lithium need to maintain an ade-
week after a menstrual cycle, when the breasts quate intake of sodium. Serum lithium concen-
are no longer swollen and tender due to hormone trations may increase in the presence of condi-
elevation. tions that cause sodium loss.

Question20 Question23
The nurse is caring for a post-operative client A client is receiving lithium carbonate 600 mg
who develops a wound evisceration. The first T.I.D. to treat bipolar disorder. Which of these in-
nursing intervention should be to dicate early signs of toxicity?
A) medicate the client for pain A) Ataxia and course hand tremors
B) call the provider B) Vomiting, diarrhea and lethargy
C) cover the wound with sterile saline dressing C) Pruritus, rash and photosensitivity
D) place the bed in a flat position D) Electrolyte imbalance and cardiac arrhyth-
mias

Review Information: The correct answer is C: Review Information: The correct answer is B:
cover the wound with sterile saline dressing Vomiting, diarrhea and lethargy
When evisceration occurs, the wound should first These are early signs of lithium toxicity.
be quickly covered by sterile dressings soaked in
sterile saline. This prevents tissue damage until Question24
a repair can be effected. The nurse can best ensure the safety of a client
suffering from dementia who wanders from the
room by which action?
Question21 A) Repeatedly remind the client of the time and
The spouse of a client with Alzheimer>s disease location
expresses concern about the burden of caregiv- B) Explain the risks of walking with no purpose
ing. Which of the following actions by the nurse C) Use protective devices to keep the client in
should be a priority? the bed or chair in the room
A) Link the caregiver with a support group D) Attach a wander-guard sensor band to the
B) Ask friends to visit regularly client>s wrist
C) Schedule a home visit each week
D) Request anti-anxiety prescriptions
Review Information: The correct answer is
Review Information: The correct answer is A: D: Attach a wander-guard sensor band to the
Link the caregiver with a support group client>>s wrist
Assisting caregivers to locate and join support This type of identification band easily tracks the
groups is most helpful. Families share feelings client>>s movements and ensures safety while
and learn about services such as respite care. the client wanders on the unit. Restriction of ac-
Collected by :DeepaRajesh [ 196 ]
rajesh.ks21@gmail.com
Kuwait
tivity is inappropriate for any client unless they C) put the client in knee-chest position
are potentially harmful to themselves or others. D) turn the client to the side

Question25
The nurse is teaching a client about the difference Review Information: The correct answer is C:
between tardive dyskinesia (TD) and neuroleptic put the client in knee-chest position
malignant syndrome (NMS). Which statement is Immediate action is needed to relieve pressure
true with regards to tardive dyskinesia? on the cord, which puts the fetus at risk due to
A) TD develops within hours or years of contin- hypoxia. The Trendelenburg position accom-
ued antipsychotic drug use in people under 20 plishes this. The exposed cord is covered with
and over 30 saline soaked gauze, not reinserted. The fe-
B) It can occur in clients taking antipsychotic tal heart rate also should be checked, and the
drugs longer than 2 years provider called. A prolapsed umbilical cord is a
C) Tardive dyskinesia occurs within minutes of medical emergency.
the first dose of antipsychotic drugs and is re-
versible Question28
D) TD can easily be treated with anticholinergic The nurse is caring for a 2 month-old infant with
drugs a congenital heart defect. Which of the following
is a priority nursing action?
A) Provide small feedings every 3 hours
Review Information: The correct answer is B: B) Maintain intravenous fluids
It can occur in clients taking antipsychotic drugs C) Add strained cereal to the diet
longer than 2 years D) Change to reduced calorie formula
Tardive dyskinesia is a extrapyramidal side ef-
fect that appears after prolonged treatment with Review Information: The correct answer is A:
antipsychotic medication. Early symptoms of tar- Provide small feedings every 3 hours
dive dyskinesia are fasciculations of the tongue Infants with congenital heart defects are at in-
or constant smacking of the lips. creased risk for developing congestive heart fail-
ure. Infants with congestive heart failure have an
Question26 increased metabolic rate and require additional
The nurse is aware that the effect of antihyper- calories to grow. At the same time, however, rest
tensive drug therapy may be affected by a 75 and conservation of energy for eating is impor-
year-old client>s tant. Feedings should be smaller and every 3
A) poor nutritional status hours rather than the usual 4 hour schedule.
B) decreased gastrointestinal motility
C) increased splanchnic blood flow Question29
D) altered peripheral resistance The nurse is caring for a client receiving intrave-
nous nitroglycerin for acute angina. What is the
most important assessment during treatment?
Review Information: The correct answer is B: A) Heart rate
decreased gastrointestinal motility B) Neurologic status
Together with shrinkage of the gastric mucosa, C) Urine output
and changes in the levels of hydrochloric acid, D) Blood pressure
this will decrease absorption of medications and
interfere with their actions.
Review Information: The correct answer is D:
Question27 Blood pressure
In response to a call for assistance by a client in The vasodilatation that occurs as a result of this
labor, the nurse notes that a loop on the umbili- medication can cause profound hypotension.
cal cord protrudes from the vagina. What is the The client>>s blood pressure must be evaluated
priority nursing action? every 15 minutes until stable and then every 30
A) call the health care provider minutes to every hour.
B) check fetal heart beat
Collected by :DeepaRajesh [ 197 ]
rajesh.ks21@gmail.com
Kuwait
Question30 sion is upsetting to you.»
A client telephones the clinic to ask about a home
pregnancy test she used this morning. The nurse
understands that the presence of which hormone Review Information: The correct answer is D:
strongly suggests a woman is pregnant? «I can hear your concern and that your confusion
A) Estrogen is upsetting to you.»
B) HCG Communicating caring and empathy with the ac-
C) Alpha-fetoprotein knowledgement of feelings is the initial response.
D) Progesterone Afterwards, teaching about the expected short
term effects would be discussed.
Review Information: The correct answer is B:
HCG Question33
Human chorionic gonadotropin (HCG) is the A woman in labor calls the nurse to assist her in
biologic marker on which pregnancy tests are the bathroom. The nurse notices a large amount
based. Reliability is about 98%, but the test does of clear fluid on the bed linens. The nurse knows
not conclusively confirm pregnancy. that fetal monitoring must now assess for what
complication?
Question31 A) Early decelerations
A client, admitted to the unit because of severe B) Late accelerations
depression and suicidal threats, is placed on sui- C) Variable decelerations
cidal precautions. The nurse should be aware D) Periodic accelerations
that the danger of the client committing suicide
is greatest
A) during the night shift when staffing is limited Review Information: The correct answer is C:
B) when the client’s mood improves with an in- Variable decelerations
crease in energy level When the membranes rupture, there is increased
C) at the time of the client>s greatest despair risk initially of cord prolapse. Fetal heart rate pat-
D) after a visit from the client>s estranged part- terns may show variable decelerations, which
ner require immediate nursing action to promote gas
exchange.

Question34
Review Information: The correct answer is The nurse is assessing a client with chronic ob-
B: when the client’s mood improves with an in- structive pulmonary disease receiving oxygen for
crease in energy level low PaO2 levels. Which assessment is a nursing
Suicide potential is often increased when there priority?
is an improvement in mood and energy level. At A) Evaluating SaO2 levels frequently
this time ambivalence is often decreased and a B) Observing skin color changes
decision is made to commit suicide. C) Assessing for clubbing fingers
D) Identifying tactile fremitus
Question32
After 4 electroconvulsive treatments over 2 Review Information: The correct answer is A:
weeks, a client is very upset and states “I am so Evaluating SaO2 levels frequently
confused. I lose my money. I just can’t remem- The best method to evaluate a client>>s oxygen-
ber telephone numbers.” The most therapeutic ation is to evaluate the SaO2. This is just as ef-
response for the nurse to make is fective as an arterial blood gas reading to evalu-
A) «You were seriously ill and needed the treat- ate oxygenation status, and is less traumatic and
ments.» expensive.
B) «Don>t get upset. The confusion will clear up
in a day or two.» Question35
C) «It is to be expected since most clients have The visiting nurse makes a postpartum visit to
the same results.» a married female client. Upon arrival, the nurse
D) «I can hear your concern and that your confu- observes that the client has a black eye and nu-
Collected by :DeepaRajesh [ 198 ]
rajesh.ks21@gmail.com
Kuwait
merous bruises on her arms and legs. The initial Question38
nursing intervention would be to A client is treated in the emergency room for
A) call the police to report indications of domestic diabetic ketoacidosis and a glucose level of
violence 650mg.D/L. In assessing the client, the nurse>s
B) confront the husband about abusing his wife review of which of the following tests suggests
C) leave the home because of the unsafe envi- an understanding of this health problem?
ronment A) Serum calcium
D) interview the client alone to determine the ori- B) Serum magnesium
gin of the injuries C) Serum creatinine
D) Serum potassium

Review Information: The correct answer is D:


interview the client alone to determine the origin Review Information: The correct answer is D:
of the injuries Serum potassium
It would be wrong to assume domestic violence Potassium is lost in diabetic ketoacidosis during
without further assessment. Separate the sus- rehydration and insulin administration. Review of
pected victim from the partner until battering has this lab finding suggests the nurse has knowl-
been ruled out. edge of this problem.

Question39
Question36 A male client is preparing for discharge follow-
When teaching a client about an oral hypoglyc- ing an acute myocardial infarction. He asks the
emic medication, the nurse should place primary nurse about his sexual activity once he is home.
emphasis on What would be the nurse>s initial response?
A) recognizing findings of toxicity A) Give him written material from the American
B) taking the medication at specified times Heart Association about sexual activity with heart
C) increasing the dosage based on blood glu- disease
cose B) Answer his questions accurately in a private
D) distinguishing hypoglycemia from hyperglyc- environment
emia C) Schedule a private, uninterrupted teaching
session with both the client and his wife
Review Information: The correct answer is B: D) Assess the client>s knowledge about his
taking the medication at specified times health problems
A regular interval between doses should be
maintained since oral hypoglycemics stimulate
the islets of Langerhans to produce insulin. Review Information: The correct answer is D:
Assess the client>>s knowledge about his health
Question37 problems
Initial postoperative nursing care for an infant The nursing process is continuous and cyclical in
who has had a pyloromyotomy would initially in- nature. When a client expresses a specific con-
clude cern, the nurse performs a focused assessment
A) bland diet appropriate for age to gather additional data prior to planning and
B) intravenous fluids for 3-4 days implementing nursing interventions.
C) NPO then glucose and electrolyte solutions
D) formula or breast milk as tolerated Question40
The client asks the nurse how the health care
provider could tell she was pregnant “just by
Review Information: The correct answer is C: looking inside.” What is the best explanation by
NPO then glucose and electrolyte solutions the nurse?
Post-operatively, the initial feedings are clear liq- A) Bluish coloration of the cervix and vaginal
uids in small quantities to provide calories and walls
electrolytes. B) Pronounced softening of the cervix
C) Clot of very thick mucous that obstructs the
Collected by :DeepaRajesh [ 199 ]
rajesh.ks21@gmail.com
Kuwait
cervical canal ized adolescent. What is the major threat experi-
D) Slight rotation of the uterus to the right enced by the hospitalized adolescent?
A) Pain management
B) Restricted physical activity
Review Information: The correct answer is A: C) Altered body image
Bluish coloration of the cervix and vaginal walls D) Separation from family
Chadwick>>s sign is a bluish-purple coloration
of the cervix and vaginal walls, occurring at 4 Review Information: The correct answer is C:
weeks of pregnancy, that is caused by vasocon- Altered body image
gestion. The hospitalized adolescent may see each of
these as a threat, but the major threat that they
0 comments feel when hospitalized is the fear of altered body
image, because of the emphasis on physical ap-
Labels: free nclex-rn sample review questions, pearance during this developmental stage.
nclex-rn practice test questions, nursing review
Question4
Free NCLEX-RN Sample Test Ques- A 12 year-old child is admitted with a broken arm
tions For Nursing Review (Part 2) and is told surgery is required. The nurse finds
him crying and unwilling to talk. What is the most
Question1 appropriate response by the nurse?
The feeling of trust can best be established by A) Give him privacy
the nurse during the process of the development B) Tell him he will get through the surgery with
of a nurse-client relationship by which of these no problem
characteristics? C) Try to distract him
A) Reliability and kindness D) Make arrangements for his friends to visit
B) Demeanor and sincerity
C) Honesty and consistency
D) Sympathy and appreciativeness Review Information: The correct answer is A:
Give him privacy
Review Information: The correct answer is C: A 12 year-old child needs the opportunity to ex-
Honesty and consistency press his emotions privately.
Characteristics of a trusting relationship include
respect, honesty, consistency, faith and caring. Question5
In discharge teaching, the nurse should empha-
Question2 size that which of these is a common side effect
A nurse has administered several blood trans- of clozapine (Clozaril) therapy?
fusions over 3 days to a 12 year-old client with A) Dry mouth
Thalassemia. What lab value should the nurse B) Rhinitis
monitor closely during this therapy? C) Dry skin
A) Hemoglobin D) Extreme salivation
B) Red Blood Cell Indices
C) Platelet count Review Information: The correct answer is D:
D) Neutrophil percent Extreme salivation
A significant number of clients receiving Clozap-
Review Information: The correct answer is A: ine (Clozaril) therapy experience extreme saliva-
Hemoglobin tion.
Hemoglobin should be in a therapeutic range of
approximately 10 g/dl (100gL). «This level is low Question6
enough to foster the patient>>s own erythropoi- A client has had a positive reaction to purified
esis without enlarging the spleen.» protein derivative (PPD). The client asks the
nurse what this means. The nurse should indi-
Question3 cate that the client has
The nurse is providing care to a newly a hospital- A) active tuberculosis
Collected by :DeepaRajesh [ 200 ]
rajesh.ks21@gmail.com
Kuwait
B) been exposed to mycobacterium tuberculo- 150mg BID. When used for depression, it may
sis take up to four weeks for results. Common side
C) never had tuberculosis effects are dry mouth, headache, and agitation.
D) never been infected with mycobacterium tu- Doses should be administered in equally spaced
berculosis time increments throughout the day to minimize
the risk of seizures.
Review Information: The correct answer is B:
been exposed to mycobacterium tuberculosis Question9
The PPD skin test is used to determine the pres- The clinic nurse is discussing health promotion
ence of tuberculosis antibodies and a positive re- with a group of parents. A mother is concerned
sult indicates that the person has been exposed about Reye>s Syndrome, and asks about pre-
to mycobacterium tuberculosis. Additional tests vention. Which of these demonstrates appropri-
are needed to determine if active tuberculosis is ate teaching?
present. A) «Immunize your child against this disease.»
B) «Seek medical attention for serious injuries.»
C) «Report exposure to this illness.»
Question7 D) «Avoid use of aspirin for viral infections.»
A client is receiving and IV antibiotic infusion and
is scheduled to have blood drawn at 1:00 pm
for a «peak» antibiotic level measurement. The Review Information: The correct answer is D:
nurse notes that the IV infusion is running behind «Avoid use of aspirin for viral infections.»
schedule and will not be competed by 1:00. The The link between aspirin use and Reye>>s Syn-
nurse should: drome has not been confirmed, but evidence
A) Notify the client>s health care provider suggests that the risk is sufficiently grave to in-
B) Stop the infusion at 1:00 pm clude the warning on aspirin products.
C) Reschedule the laboratory test
D) Increase the infusion rate Question10
A post-operative client is admitted to the post-an-
esthesia recovery room (PACU). The anesthet-
Review Information: The correct answer is C: ist reports that malignant hyperthermia occurred
Reschedule the laboratory test during surgery. The nurse recognizes that this
If the antibiotic infusion will not be completed at complication is related to what factor?
the time the peak blood level is due to be drawn, A) Allergy to general anesthesia
the nurse should ask that the blood sampling B) Pre-existing bacterial infection
time be adjusted C) A genetic predisposition
D) Selected surgical procedures
Question8
The nurse is caring for a client with a new order Review Information: The correct answer is C:
for bupropion (Wellbutrin) for treatment of de- A genetic predisposition
pression. The order reads “Wellbutrin 175 mg. Malignant hyperthermia is a rare, potentially fatal
BID x 4 days.” What is the appropriate action? adverse reaction to inhaled anesthetics. There is
A) Give the medication as ordered a genetic predisposition to this disorder.
B) Questionthis medication dose
C) Observe the client for mood swings
D) Monitor neuro signs frequently Question11
A 9 year-old is taken to the emergency room with
right lower quadrant pain and vomiting. When
Review Information: The correct an- preparing the child for an emergency appen-
swer is B: Questionthis medication dectomy, what must the nurse expect to be the
child>s greatest fear?
dose
A) Change in body image
Bupropion (Wellbutrin) should be started at
B) An unfamiliar environment
100mg BID for three days then increased to
C) Perceived loss of control
Collected by :DeepaRajesh [ 201 ]
rajesh.ks21@gmail.com
Kuwait
D) Guilt over being hospitalized B) Obtain the regular blood glucose readings
C) Determine if special skin care is needed
Review Information: The correct answer is C: D) Answer questions from the client>s spouse
Perceived loss of control about the plan of care
For school age children, major fears are loss of
control and separation from friends/peers.
Review Information: The correct answer is B:
Question12 Obtain the regular blood glucose readings
A client is to begin taking Fosamax. The nurse The UAP can safely obtain blood glucose read-
must emphasize which of these instructions to ings, which are routine tasks.
the client when taking this medication? «Take
Fosamax Question15
A) on an empty stomach.» Which of the following laboratory results would
B) after meals.» suggest to the emergency room nurse that a cli-
C) with calcium.» ent admitted after a severe motor vehicle crash
D) with milk 2 hours after meals.» is in acidosis?
A) Hemoglobin 15 gm/dl
B) Chloride 100 mEq/L
Review Information: The correct answer is A: C) Sodium 130 mEq/L
on an empty stomach.» D) Carbon dioxide 20 mEq/L
Fosamax should be taken first thing in the morn-
ing with 6-8 ounces of plain water at least 30
minutes before other medication or food. Food Review Information: The correct answer is D:
and fluids (other than water) greatly decrease Carbon dioxide 20 mEq/L
the absorption of Fosamax. The client must also Serum carbon dioxide is an indicator of acid-base
be instructed to remain in the upright position for status. This finding would indicate acidosis.
30 minutes following the dose to facilitate pas-
sage into the stomach and minimize irritation of Question16
the esophagus. The nurse has just received report on a group
of clients and plans to delegate care of several
Question13 of the clients to a practical nurse (PN). The first
An older adult client is to receive and antibiotic, thing the RN should do before the delegation of
gentamicin. What diagnostic finding indicates care is
the client may have difficult excreting the medi- A) Provide a time-frame for the completion of the
cation? client care
A) High gastric pH B) Assure the PN that the RN will be available for
B) High serum creatinine assistance
C) Low serum albumin C) Ask about prior experience with similar cli-
D) Low serum blood urea nitrogen ents
D) Review the specific procedures unique to the
assignment
Review Information: The correct answer is B:
High serum creatinine Review Information: The correct answer is C:
An elevated serum creatinine indicates reduced Ask about prior experience with similar clients.
renal function. Reduced renal function will delay The first step in delegation is to determine the
the excretion of many mediations. qualifications of the person to whom one is del-
egating. By asking about the PN>>s prior ex-
Question14 perience with similar clients/tasks, the RN can
A nurse is assigned to care for a comatose dia- determine whether the PN has the requisite ex-
betic on IV insulin therapy. Which task would be perience to care for the assigned clients.
most appropriate to delegate to an unlicensed
assistive personnel (UAP)?
A) Check the client>s level of consciousness Question17
Collected by :DeepaRajesh [ 202 ]
rajesh.ks21@gmail.com
Kuwait
The mother of a 4 month-old infant asks the Helplessness and hopelessness may contribute
nurse about the dangers of sunburn while they to regressive, dependent behavior which often
are on vacation at the beach. Which of the fol- occurs at any age with hospitalization. Deny-
lowing is the best advice about sun protection for ing or minimizing the seriousness of the illness
this child? is used to avoid facing the worst situation. Re-
A) «Use a sunscreen with a minimum sun pro- call that denial is the initial step in the process of
tective factor of 15.» working through any loss.
B) «Applications of sunscreen should be repeat-
ed every few hours.» Question20
C) «An infant should be protected by the maxi- A 52 year-old post menopausal woman asks the
mum strength sunscreen.» nurse how frequently she should have a mam-
D) «Sunscreens are not recommended in chil- mogram. What is the nurse>s best response?
dren younger than 6 months.» A) «Your doctor will advise you about your
risks.»
B) «Unless you had previous problems, every 2
Review Information: The correct answer is D: years is best.»
«Sunscreens are not recommended in children C) «Once a woman reaches 50, she should have
younger than 6 months.» a mammogram yearly.»
Infants under 6 months of age should be kept out D) «Yearly mammograms are advised for all
of the sun or shielded from it. Even on a cloudy women over 35.»
day, the infant can be sunburned while near wa-
ter. A hat and light protective clothing should be
worn. Review Information: The correct answer is C:
«Once a woman reaches 50, she should have a
Question18 mammogram yearly.»
The nurse administers cimetidine (Tagamet) to a The American Cancer Society recommends a
79 year-old male with a gastric ulcer. Which pa- screening mammogram by age 40, every 1 - 2
rameter may be affected by this drug, and should years for women 40-49, and every year from age
be closely monitored by the nurse? 50. If there are family or personal health risks,
A) Blood pressure other assessments may be recommended.
B) Liver function
C) Mental status Question21
D) Hemoglobin The nurse is planning care for a client who is tak-
ing cyclosporin (Neoral). What would be an ap-
Review Information: The correct answer is C: propriate nursing diagnosis for this client?
Mental status A) Alteration in body image
The elderly are at risk for developing confusion B) High risk for infection
when taking cimetidine, a drug that interacts with C) Altered growth and development
many other medications. D) Impaired physical mobility

Question19
The nurse assesses the use of coping mecha- Review Information: The correct answer is B:
nisms by an adolescent 1 week after the client High risk for infection
had a motor vehicle accident resulting in multiple Cyclosporin (Neoral) inhibits normal immune re-
serious injuries. Which of these characteristics sponses. Clients receiving cyclosporin are at risk
are most likely to be displayed? for infection.
A) Ambivalence, dependence, demanding
B) Denial, projection, regression Question22
C) Intellectualization, rationalization, repression A client on telemetry begins having premature
D) Identification, assimilation, withdrawal ventricular beats (PVBs) at 12 per minute. In re-
viewing the most recent laboratory results, which
Review Information: The correct answer is B: would require immediate action by the nurse?
Denial, projection, regression A) Calcium 9 mg/dl
Collected by :DeepaRajesh [ 203 ]
rajesh.ks21@gmail.com
Kuwait
B) Magnesium 2.5 mg/dl Water should not enter the ears. Children should
C) Potassium 2.5 mEq/L use ear plugs when bathing or swimming and
D) PTT 70 seconds should not put their heads under the water.

Question25
Review Information: The correct answer is C: The nurse is caring for a client with asthma who
Potassium 2.5 mEq/L has developed gastroesophageal reflux disease
The patient is at risk for ventricular dysrhythmias (GERD). Which of the following medications pre-
when the potassium level is low. scribed for the client may aggravate GERD?
Daniels, R. (2003). A) Anticholinergics
B) Corticosteroids
Question23 C) Histamine blocker
The nurse is caring for a client who is 4 days D) Antibiotics
post-op for a transverse colostomy. The client is
ready for discharge and asks the nurse to empty Review Information: The correct answer is A:
his colostomy pouch. What is the best response Anticholinergics
by the nurse? An anticholinergic medication will decrease gas-
A) «You should be emptying the pouch your- tric emptying and the pressure on the lower es-
self.» ophageal sphincter.
B) «Let me demonstrate to you how to empty the
pouch.» Question26
C) «What have you learned about emptying your A client is receiving a nitroglycerin infusion for
pouch?» unstable angina. What assessment would be a
D) «Show me what you have learned about emp- priority when monitoring the effects of this medi-
tying your pouch.» cation?
A) Blood pressure
B) Cardiac enzymes
Review Information: The correct answer is D: C) ECG analysis
«Show me what you have learned about empty- D) Respiratory rate
ing your pouch.»
Most adult learners obtain skills by participating
in the activities. Anxiety about discharge can be Review Information: The correct answer is A:
causing the client to forget that they have mas- Blood pressure
tered the skill of emptying the pouch. The client Since an effect of this drug is vasodilation, the
should show the nurse how the pouch is emp- client must be monitored for hypotension.
tied.
Question27
Question24 The nurse is caring for a 10 year-old child who
A 3 year-old child has tympanostomy tubes in has just been diagnosed with diabetes insipidus.
place. The child>s parent asks the nurse if he The parents ask about the treatment prescribed,
can swim in the family pool. The best response vasopressin. A What is priority in teaching the
from the nurse is child and family about this drug?
A) «Your child should not swim at all while the A) The child should carry a nasal spray for emer-
tubes are in place.» gency use
B) «Your child may swim in your own pool but not B) The family must observe the child for dehy-
in a lake or ocean.» dration
C) «Your child may swim if he wears ear plugs.» C) Parents should administer the daily intramus-
D) «Your child may swim anywhere.» cular injections
D) The client needs to take daily injections in the
short-term
Review Information: The correct answer is C:
«Your child may swim if he wears ear plugs.»
Review Information: The correct answer is A:
Collected by :DeepaRajesh [ 204 ]
rajesh.ks21@gmail.com
Kuwait
The child should carry a nasal spray for emer- ent tells the nurse that she is stupid. What is the
gency use most therapeutic response by the nurse?
Diabetes insipidus results from reduced secre- A) Explore what is going on with the client
tion of the antidiuretic hormone, vasopressin. B) Accept the client’s statement without com-
The child will need to administer daily injections ment
of vasopressin, and should have the nasal spray C) Tell the client that the comment is inappropri-
form of the medication readily available. A medi- ate
cal alert tag should be worn. D) Leave the client>s room

Question28
A client diagnosed with cirrhosis is started on Review Information: The correct answer is A:
lactulose (Cephulac). The main purpose of the Explore what is going on with the client
drug for this client is to Exploring feelings with the verbally aggressive
A) add dietary fiber client helps to put angry feelings into words and
B) reduce ammonia levels then to engage in problem solving.
C) stimulate peristalsis
D) control portal hypertension Question31
A client has many delusions. As the nurse helps
the client prepare for breakfast the client com-
Review Information: The correct answer is B: ments «Don’t waste good food on me. I’m dying
reduce ammonia levels from this disease I have.» The appropriate re-
Lactulose blocks the absorption of ammonia from sponse would be
the GI tract and secondarily stimulates bowel A) «You need some nutritious food to help you
elimination. regain your weight.»
B) «None of the laboratory reports show that you
Question29 have any physical disease.»
The nurse is explaining the effects of cocaine C) «Try to eat a little bit, breakfast is the most
abuse to a pregnant client. Which of the follow- important meal of the day.»
ing must the nurse understand as a basis for D) «I know you believe that you have an incur-
teaching? able disease.»
A) Cocaine use can cause fetal growth retarda-
tion
B) The drug has been linked to neural tube de- Review Information: The correct answer is D:
fects «I know you believe that you have an incurable
C) Newborn withdrawal generally occurs imme- disease.»
diately after birth This response does not challenge the client’s
D) Breast feeding promotes positive parenting delusional system and thus forms an alliance by
behaviors providing reassurance of desire to help the cli-
ent.

Review Information: The correct answer is A: Question32


Cocaine use can cause fetal growth retardation A client with paranoid thoughts refuses to eat be-
Cocaine is vasoconstrictive, and this effect incause of the belief that the food is poisoned. The
the placental vessels causes fetal hypoxia and appropriate statement at this time for the nurse
diminished growth. Other risks of continued co-to say is
caine use during pregnancy include preterm la- A) «Here, I will pour a little of the juice in a medi-
bor, congenital abnormalities, altered brain de-
cine cup to drink it to show you that it is OK.»
velopment and subsequent behavioral problems B) «The food has been prepared in our kitchen
in the infant. and is not poisoned.»
C) «Let>s see if your partner could bring food
Question30 from home.»
A client has just been diagnosed with breast D) «If you don>t eat, I will have to suggest for
cancer. The nurse enters the room and the cli- you to be tube fed.»
Collected by :DeepaRajesh [ 205 ]
rajesh.ks21@gmail.com
Kuwait
scribed medication.»
Abrupt withdrawal may occasionally cause sero-
Review Information: The correct answer is C: tonin syndrome, consisting of lethargy, nausea,
«Let>>s see if your partner could bring food from headache, fever, sweating and chills. A slow with-
home.» drawal may be prescribed with sertraline to avoid
Reassurance is ineffective when a client is ac- dizziness, nausea, vomiting, and diarrhea.
tively delusional. This option avoids both arguing
with the client and agreeing with the delusional Question35
premise. Option D offers a logical response to A client is admitted to the hospital with findings of
a primarily affective concern. When the client’s liver failure with ascites. The health care provider
condition has improved, gentle negation of the orders spironolactone (Aldactone). What is the
delusional premise can be employed. pharmacological effect of this medication?
A) Promotes sodium and chloride excretion
Question33 B) Increases aldosterone levels
A client with tuberculosis is started on Rifampin. C) Depletes potassium reserves
Which one of the following statements by the D) Combines safely with antihypertensives
nurse would be appropriate to include in teach-
ing? «You may notice:
A) an orange-red color to your urine.» Review Information: The correct answer is A:
B) your appetite may increase for the first Promotes sodium and chloride excretion
week.” Spironolactone promotes sodium and chloride
C) it is common to experience occasional sleep excretion while sparing potassium and decreas-
disturbances.» ing aldosterone levels. It had no effect on am-
D) if you take the medication with food, you may monia levels.
have nausea.»
Question36
A client was admitted to the psychiatric unit for
Review Information: The correct answer is A: severe depression. After several days, the cli-
an orange-red color to your urine.» ent continues to withdraw from the other clients.
Discoloration of the urine and other body fluids Which of these statements by the nurse would
may occur. It is a harmless response to the drug, be the most appropriate to promote interaction
but the patient needs to be aware it may hap- with other clients?
pen. A) «Your team here thinks it>s good for you to
spend time with others.»
Question34 B) «It is important for you to participate in group
A client tells the RN she has decided to stop tak- activities.»
ing sertraline (Zoloft) because she doesn’t like C) «Come with me so you can paint a picture to
the nightmares, sex dreams, and obsessions help you feel better.»
she’s experiencing since starting on the medi- D) «Come play Chinese Checkers with Gloria
cation. What is an appropriate response by the and me.»
nurse?
A) «It is unsafe to abruptly stop taking any pre- Review Information: The correct answer is D:
scribed medication.» «Come play Chinese Checkers with Gloria and
B) «Side effects and benefits should be dis- me.»
cussed with your health care provider.» This gradually engages the client in interactions
C) «This medication should be continued despite with others in small groups rather than large
unpleasant symptoms.» groups. In addition, focusing on an activity is less
D) «Many medications have potential side ef- anxiety-provoking than unstructured discussion.
fects.» The statement is an example of a positive be-
havioral expectation.

Review Information: The correct answer is Question37


A: «It is unsafe to abruptly stop taking any pre- The nurse is teaching a school-aged child and
Collected by :DeepaRajesh [ 206 ]
rajesh.ks21@gmail.com
Kuwait
family about the use of inhalers prescribed for The nurse is beginning nutritional counseling/
asthma. What is the best way to evaluate effec- teaching with a pregnant woman. What is the ini-
tiveness of the treatments? tial step in this interaction?

A) Rely on child>s self-report A) Teach her how to meet the needs of self and
B) Use a peak-flow meter her family
C) Note skin color changes B) Explain the changes in diet necessary for
D) Monitor pulse rate pregnant women
C) Questionher understanding and use
of the food pyramid
Review Information: The correct answer is B: D) Conduct a diet history to determine her nor-
Use a peak-flow meter mal eating routines
The peak flowmeter, if used correctly, shows ef-
fectiveness of inhalants.
Review Information: The correct answer is D:
Question38 Conduct a diet history to determine her normal
The nurse is teaching a client about the toxicity eating routines.
of digoxin. Which one of the following statements
made by the client to the nurse indicates more Assessment is always the first step in planning
teaching is needed? teaching for any client. A thorough and accurate
A) «I may experience a loss of appetite.» history is essential for gathering the needed in-
B) «I can expect occasional double vision.» formation.
C) «Nausea and vomiting may last a few days.»
D) «I must report a bounding pulse of 62 imme- 0 comments
diately.»
Labels: free nclex-rn sample review questions,
nclex-rn practice test questions, nursing review
Review Information: The correct answer is D:
«I must report a bounding pulse of 62 immedi- Free NCLEX-RN Sample Test Ques-
ately.»
tions For Nursing Review (Part 1)
Slow heart rate is related to increased cardiac
output and an intended effect of digoxin. The ide-
These are sample nursing review questions and
al heart rate is above 60 BPM with digoxin. The
not actual test questions made for educational
client needs further teaching.
and practice test purposes only. 75 questions
have been posted here with answer keys.

Question39
Question1
Which of the following assessments by the nurse
A client has been hospitalized after an automo-
would indicate that the client is having a possible
bile accident. A full leg cast was applied in the
adverse response to the isoniazid (INH)?
emergency room. The most important reason for
A) Severe headache
the nurse to elevate the casted leg is to
B) Appearance of jaundice
A) Promote the client>s comfort
C) Tachycardia
B) Reduce the drying time
D) Decreased hearing
C) Decrease irritation to the skin
D) Improve venous return
Review Information: The correct answer is B:
Appearance of jaundice
Review Information: The correct answer is D:
Clients receiving INH therapy are at risk for de-
Improve venous return. Elevating the leg both
veloping drug induced hepatitis. The appearance
improves venous return and reduces swelling.
of jaundice may indicate that the client has liver
Client comfort will be improved as well.
damage.

Question40
Collected by :DeepaRajesh [ 207 ]
rajesh.ks21@gmail.com
Kuwait
Question2 Question4
The nurse is reviewing with a client how to col- A client with Guillain Barre is in a nonresponsive
lect a clean catch urine specimen. What is the state, yet vital signs are stable and breathing is
appropriate sequence to teach the client? independent. What should the nurse document
to most accurately describe the client>s condi-
A) Clean the meatus, begin voiding, then catch tion?
urine stream A) Comatose, breathing unlabored
B) Void a little, clean the meatus, then collect B) Glascow Coma Scale 8, respirations regular
specimen C) Appears to be sleeping, vital signs stable
C) Clean the meatus, then urinate into container D) Glascow Coma Scale 13, no ventilator re-
D) Void continuously and catch some of the quired
urine
Review Information: The correct answer is
Review Information: The correct answer is B: Glascow Coma Scale 8, respirations regular.
A: Clean the meatus, begin voiding, then catch The Glascow Coma Scale provides a standard
urine stream. A clean catch urine is difficult to reference for assessing or monitoring level of
obtain and requires clear directions. Instructing consciousness. Any score less than 13 indicates
the client to carefully clean the meatus, then void a neurological impairment. Using the term coma-
naturally with a steady stream prevents surface tose provides too much room for interpretation
bacteria from contaminating the urine specimen. and is not very precise.
As starting and stopping flow can be difficult,
once the client begins voiding it>>s best to just
slip the container into the stream. Other respons- Question5
es do not reflect correct technique. When caring for a client receiving warfarin so-
dium (Coumadin), which lab test would the nurse
monitor to determine therapeutic response to the
Question3 drug?
Following change-of-shift report on an orthoped- A) Bleeding time
ic unit, which client should the nurse see first? B) Coagulation time
A) 16 year-old who had an open reduction of a C) Prothrombin time
fractured wrist 10 hours ago D) Partial thromboplastin time
B) 20 year-old in skeletal traction for 2 weeks
since a motor cycle accident Review Information: The correct answer is
C) 72 year-old recovering from surgery after a C: Prothrombin time. Coumadin is ordered daily,
hip replacement 2 hours ago based on the client>>s prothrombin time (PT).
D) 75 year-old who is in skin traction prior to This test evaluates the adequacy of the extrinsic
planned hip pinning surgery. system and common pathway in the clotting cas-
cade; Coumadin affects the Vitamin K depend-
Review Information: The correct answer is C: ent clotting factors.
72 year-old recovering from surgery after a hip
replacement 2 hours ago. Look for the client who
has the most imminent risks and acute vulnerabil- Question6
ity. The client who returned from surgery 2 hours A client with moderate persistent asthma is ad-
ago is at risk for life threatening hemorrhage and mitted for a minor surgical procedure. On ad-
should be seen first. The 16 year-old should be mission the peak flow meter is measured at 480
seen next because it is still the first post-op day. liters/minute. Post-operatively the client is com-
The 75 year-old is potentially vulnerable to age- plaining of chest tightness. The peak flow has
related physical and cognitive consequences in dropped to 200 liters/minute. What should the
skin traction should be seen next. The client who nurse do first?
can safely be seen last is the 20 year-old who is A) Notify both the surgeon and provider
2 weeks post-injury. B) Administer the prn dose of albuterol
C) Apply oxygen at 2 liters per nasal cannula
D) Repeat the peak flow reading in 30 minutes
Collected by :DeepaRajesh [ 208 ]
rajesh.ks21@gmail.com
Kuwait
needed.
Review Information: The correct answer is
B: Administer the prn dose of albuterol. Peak
flow monitoring during exacerbations of asthma Question9
is recommended for clients with moderate-to- The nurse has performed the initial assessments
severe persistent asthma to determine the se- of 4 clients admitted with an acute episode of
verity of the exacerbation and to guide the treat- asthma. Which assessment finding would cause
ment. A peak flow reading of less than 50% of the nurse to call the provider immediately?
the client>>s baseline reading is a medical alert A) prolonged inspiration with each breath
condition and a short-acting beta-agonist must B) expiratory wheezes that are suddenly absent
be taken immediately. in 1 lobe
C) expectoration of large amounts of purulent
mucous
Question7 D) appearance of the use of abdominal muscles
A client had 20 mg of Lasix (furosemide) PO at for breathing
10 AM. Which would be essential for the nurse to
include at the change of shift report? Review Information: The correct answer is B:
A) The client lost 2 pounds in 24 hours expiratory wheezes that are suddenly absent in 1
B) The client’s potassium level is 4 mEq/liter. lobe. Acute asthma is characterized by expiratory
C) The client’s urine output was 1500 cc in 5 wheezes caused by obstruction of the airways.
hours Wheezes are a high pitched musical sounds pro-
D) The client is to receive another dose of Lasix duced by air moving through narrowed airways.
at 10 PM Clients often associate wheezes with the feeling
of tightness in the chest. However, sudden ces-
Review Information: The correct answer is C: sation of wheezing is an ominous or bad sign
The client’s urine output was 1500 cc in 5 hours. that indicates an emergency -- the small airways
Although all of these may be correct information are now collapsed.
to include in report, the essential piece would be
the urine output.
Question10
During the initial home visit, a nurse is discuss-
Question8 ing the care of a client newly diagnosed with
A client has been tentatively diagnosed with Alzheimer>s disease with family members.
Graves> disease (hyperthyroidism). Which of Which of these interventions would be most
these findings noted on the initial nursing assess- helpful at this time?
ment requires quick intervention by the nurse? A) leave a book about relaxation techniques
A) a report of 10 pounds weight loss in the last B) write out a daily exercise routine for them to
month assist the client to do
B) a comment by the client «I just can>t sit C) list actions to improve the client>s daily nutri-
still.» tional intake
C) the appearance of eyeballs that appear to D) suggest communication strategies
«pop» out of the client>s eye sockets
D) a report of the sudden onset of irritability in Review Information: The correct answer is D:
the past 2 weeks suggest communication strategies. Alzheimer>>s
disease, a progressive chronic illness, greatly
Review Information: The correct answer is C: challenges caregivers. The nurse can be of
the appearance of eyeballs that appear to «pop» greatest assistance in helping the family to use
out of the client>>s eye sockets. Exophthalmos communication strategies to enhance their ability
or protruding eyeballs is a distinctive characteris- to relate to the client. By use of select verbal and
tic of Graves>> Disease. It can result in corneal nonverbal communication strategies the family
abrasions with severe eye pain or damage when can best support the client’s strengths and cope
the eyelid is unable to blink down over the pro- with any aberrant behavior.
truding eyeball. Eye drops or ointment may be
Collected by :DeepaRajesh [ 209 ]
rajesh.ks21@gmail.com
Kuwait
ate action is required?
Question11 A) pH below 7.3
An 80 year-old client admitted with a diagnosis B) Potassium of 5.0
of possible cerebral vascular accident has had a C) HCT of 60
blood pressure from 160/100 to 180/110 over the D) Pa O2 of 79%
past 2 hours. The nurse has also noted increased
lethargy. Which assessment finding should the Review Information: The correct answer is C:
nurse report immediately to the provider? HCT of 60. This high hematocrit is indicative of
A) Slurred speech severe dehydration which requires priority atten-
B) Incontinence tion in diabetic ketoacidosis. Without sufficient
C) Muscle weakness hydration, all systems of the body are at risk
D) Rapid pulse for hypoxia from a lack of or sluggish circula-
tion. In the absence of insulin, which facilitates
Review Information: The correct answer is A: the transport of glucose into the cell, the body
Slurred speech. Changes in speech patterns and breaks down fats and proteins to supply energy
level of conscious can be indicators of continued ketones, a by-product of fat metabolism. These
intracranial bleeding or extension of the stroke. accumulate causing metabolic acidosis (pH <
Further diagnostic testing may be indicated. 7.3), which would be the second concern for this
client. The potassium and PaO2 levels are near
normal.
Question12
A school-aged child has had a long leg (hip to
ankle) synthetic cast applied 4 hours ago. Which Question14
statement from the parent indicates that teach- The nurse is preparing a client with a deep vein
ing has been inadequate? thrombosis (DVT) for a Venous Doppler evalua-
A) «I will keep the cast uncovered for the next tion. Which of the following would be necessary
day to prevent burning of the skin.» for preparing the client for this test?
B) «I can apply an ice pack over the area to re- A) Client should be NPO after midnight
lieve itching inside the cast.» B) Client should receive a sedative medication
C) «The cast should be propped on at least 2 pil- prior to the test
lows when my child is lying down.» C) Discontinue anti-coagulant therapy prior to
D) «I think I remember that my child should not the test
stand until after 72 hours.» D) No special preparation is necessary

Review Information: The correct answer is D: Review Information: The correct answer is
«I think I remember that my child should not stand D: No special preparation is necessary. This is
until after 72 hours.». Synthetic casts will typi- a non-invasive procedure and does not require
cally set up in 30 minutes and dry in a few hours. preparation other than client education.
Thus, the client may stand within the initial 24
hours. With plaster casts, the set up and drying
time, especially in a long leg cast which is thicker Question15
than an arm cast, can take up to 72 hours. Both A client is admitted with infective endocarditis
types of casts give off a lot of heat when drying (IE). Which finding would alert the nurse to a
and it is preferable to keep the cast uncovered complication of this condition?
for the first 24 hours. Clients may complain of a A) dyspnea
chill from the wet cast and therefore can simply B) heart murmur
be covered lightly with a sheet or blanket. Apply- C) macular rash
ing ice is a safe method of relieving the itching. D) hemorrhage

Review Information: The correct answer is


Question13 B: heart murmur. Large, soft, rapidly developing
Which blood serum finding in a client with dia- vegetations attach to the heart valves. They have
betic ketoacidosis alerts the nurse that immedi- a tendency to break off, causing emboli and leav-
Collected by :DeepaRajesh [ 210 ]
rajesh.ks21@gmail.com
Kuwait
ing ulcerations on the valve leaflets. These em-
boli produce findings of cardiac murmur, fever, Review Information: The correct answer is
anorexia, malaise and neurologic sequelae of B: Frequent neurovascular assessments of the
emboli. Furthermore, the vegetations may travel affected leg. The most important activity for the
to various organs such as spleen, kidney, coro- nurse is to assess neurovascular status. Com-
nary artery, brain and lungs, and obstruct blood partment syndrome is a serious complication of
flow. fractures. Prompt recognition of this neurovascu-
lar problem and early intervention may prevent
permanent limb damage.
Question16
The nurse explains an autograft to a client sched-
uled for excision of a skin tumor. The nurse knows Question19
the client understands the procedure when the The nurse is assigned to care for a client who
client says, «I will receive tissue from had a myocardial infarction (MI) 2 days ago. The
A) a tissue bank.» client has many questions about this condition.
B) a pig.» What area is a priority for the nurse to discuss at
C) my thigh.» this time?
D) synthetic skin.» A) Daily needs and concerns
B) The overview cardiac rehabilitation
Review Information: The correct answer is C: C) Medication and diet guideline
my thigh.». Autografts are done with tissue trans- D) Activity and rest guidelines
planted from the client>>s own skin.
Review Information: The correct answer is A:
Daily needs and concerns. At 2 days post-MI, the
Question17 client’s education should be focused on the im-
A client is admitted to the emergency room fol- mediate needs and concerns for the day.
lowing an acute asthma attack. Which of the fol-
lowing assessments would be expected by the
nurse? Question20
A 3 year-old child is brought to the clinic by his
A) Diffuse expiratory wheezing grandmother to be seen for «scratching his bot-
B) Loose, productive cough tom and wetting the bed at night.» Based on
C) No relief from inhalant these complaints, the nurse would initially as-
D) Fever and chills sess for which problem?
A) allergies
Review Information: The correct answer is A: B) scabies
Diffuse expiratory wheezing. In asthma, the air- C) regression
ways are narrowed, creating difficulty getting air D) pinworms
in. A wheezing sound results.
Review Information: The correct answer is
D: pinworms. Signs of pinworm infection include
Question18 intense perianal itching, poor sleep patterns,
A client has been admitted with a fractured femur general irritability, restlessness, bed-wetting,
and has been placed in skeletal traction. Which distractibility and short attention span. Scabies
of the following nursing interventions should re- is an itchy skin condition caused by a tiny, eight-
ceive priority? legged burrowing mite called Sarcoptes scabiei .
A) Maintaining proper body alignment The presence of the mite leads to intense itching
B) Frequent neurovascular assessments of the in the area of its burrows.
affected leg
C) Inspection of pin sites for evidence of drain-
age or inflammation Question21
D) Applying an over-bed trapeze to assist the cli- The nurse is caring for a newborn with tra-
ent with movement in bed cheoesophageal fistula. Which nursing diagno-
Collected by :DeepaRajesh [ 211 ]
rajesh.ks21@gmail.com
Kuwait
sis is a priority? alkalosis. Findings include irritability, increased
A) Risk for dehydration activity, hyperactive reflexes, muscle twitching
B) Ineffective airway clearance and elevated pulse. Options C and D are correct
C) Altered nutrition answers but not the best answers since they are
D) Risk for injury too general.

Review Information: The correct answer is B:


Ineffective airway clearance. The most common Question24
form of TEF is one in which the proximal esopha- A two year-old child is brought to the provider>s
geal segment terminates in a blind pouch and office with a chief complaint of mild diarrhea for
the distal segment is connected to the trachea two days. Nutritional counseling by the nurse
or primary bronchus by a short fistula at or near should include which statement?
the bifurcation. Thus, a priority is maintaining an A) Place the child on clear liquids and gelatin for
open airway, preventing aspiration. Other nurs- 24 hours
ing diagnoses are then addressed. B) Continue with the regular diet and include oral
rehydration fluids
C) Give bananas, apples, rice and toast as toler-
Question22 ated
The nurse is developing a meal plan that would D) Place NPO for 24 hours, then rehydrate with
provide the maximum possible amount of iron for milk and water
a child with anemia. Which dinner menu would
be best? Review Information: The correct answer is B:
A) Fish sticks, french fries, banana, cookies, Continue with the regular diet and include oral
milk rehydration fluids. Current recommendations for
B) Ground beef patty, lima beans, wheat roll, rai- mild to moderate diarrhea are to maintain a nor-
sins, milk mal diet with fluids to rehydrate.
C) Chicken nuggets, macaroni, peas, canta-
loupe, milk
D) Peanut butter and jelly sandwich, apple slic- Question25
es, milk The nurse is teaching parents about the appro-
priate diet for a 4 month-old infant with gastro-
Review Information: The correct answer is B: enteritis and mild dehydration. In addition to oral
Ground beef patty, lima beans, wheat roll, raisins, rehydration fluids, the diet should include
milk. Iron rich foods include red meat, fish, egg
yolks, green leafy vegetables, legumes, whole A) formula or breast milk
grains, and dried fruits such as raisins. This din- B) broth and tea
ner is the best choice: It is high in iron and is ap- C) rice cereal and apple juice
propriate for a toddler. D) gelatin and ginger ale

Review Information: The correct answer is A:


Question23 formula or breast milk. The usual diet for a young
The nurse admitting a 5 month-old who vomited infant should be followed.
9 times in the past 6 hours should observe for
signs of which overall imbalance?
A) Metabolic acidosis Question26
B) Metabolic alkalosis A child is injured on the school playground and
C) Some increase in the serum hemoglobin appears to have a fractured leg. The first action
D) A little decrease in the serum potassium the school nurse should take is

Review Information: The correct answer is B: A) call for emergency transport to the hospital
Metabolic alkalosis. Vomiting causes loss of acid B) immobilize the limb and joints above and be-
from the stomach. Prolonged vomiting can re- low the injury
sult in excess loss of acid and lead to metabolic C) assess the child and the extent of the injury
Collected by :DeepaRajesh [ 212 ]
rajesh.ks21@gmail.com
Kuwait
D) apply cold compresses to the injured area in a week. Iron fortified cereal is the recommend-
ed first food.
Review Information: The correct answer is
C: assess the child and the extent of the injury.
When applying the nursing process, assessment Question29
is the first step in providing care. The «5 Ps» The nurse planning care for a 12 year-old child
of vascular impairment can be used as a guide with sickle cell disease in a vaso-occlusive crisis
(pain, pulse, pallor, paresthesia, paralysis). of the elbow should include which one of the fol-
lowing as a priority?

Question27 A) Limit fluids


The mother of a 3 month-old infant tells the nurse B) Client controlled analgesia
that she wants to change from formula to whole C) Cold compresses to elbow
milk and add cereal and meats to the diet. What D) Passive range of motion exercise
should be emphasized as the nurse teaches
about infant nutrition? Review Information: The correct answer is B:
Client controlled analgesia. Management of a
A) Solid foods should be introduced at 3-4 sickle cell crisis is directed towards supportive
months and symptomatic treatment. The priority of care
B) Whole milk is difficult for a young infant to di- is pain relief. In a 12 year-old child, client control-
gest led analgesia promotes maximum comfort.
C) Fluoridated tap water should be used to dilute
milk
D) Supplemental apple juice can be used be- Question30
tween feedings The nurse is performing a physical assessment
on a toddler. Which of the following actions
Review Information: The correct answer is B: should be the first?
Whole milk is difficult for a young infant to digest.
Cow>>s milk is not given to infants younger than A) Perform traumatic procedures
1 year because the tough, hard curd is difficult to B) Use minimal physical contact
digest. In addition, it contains little iron and cre- C) Proceed from head to toe
ates a high renal solute load. D) Explain the exam in detail

Question28 Review Information: The correct answer


The nurse is preparing a handout on infant feed- is B: Use minimal physical contact. The nurse
ing to be distributed to families visiting the clinic. should approach the toddler slowly and use min-
Which notation should be included in the teach- imal physical contact initially so as to gain the
ing materials? toddler>>s cooperation. Be flexible in the se-
quence of the exam, and give only brief simple
A) Solid foods are introduced one at a time be- explanations just prior to the action.
ginning with cereal
B) Finely ground meat should be started early to
provide iron Question31
C) Egg white is added early to increase protein What finding signifies that children have attained
intake the stage of concrete operations (Piaget)?
D) Solid foods should be mixed with formula in
a bottle A) Explores the environment with the use of sight
and movement
Review Information: The correct answer is A: B) Thinks in mental images or word pictures
Solid foods are introduced one at a time begin- C) Makes the moral judgment that «stealing is
ning with cereal. Solid foods should be added wrong»
one at a time between 4-6 months. If the infant is D) Reasons that homework is time-consuming
able to tolerate the food, another may be added yet necessary
Collected by :DeepaRajesh [ 213 ]
rajesh.ks21@gmail.com
Kuwait
it is most important for the nurse to teach them
Review Information: The correct answer is about which of the following actions?
C: Makes the moral judgment that «stealing is
wrong». The stage of concrete operations is de- A) Maintain good oral hygiene and dental care
picted by logical thinking and moral judgments. B) Omit medication if the child is seizure free
C) Administer acetaminophen to promote sleep
D) Serve a diet that is high in iron
Question32
The mother of a child with a neural tube defect Review Information: The correct answer is
asks the nurse what she can do to decrease the A: Maintain good oral hygiene and dental care.
chances of having another baby with a neural Swollen and tender gums occur often with use of
tube defect. What is the best response by the phenytoin. Good oral hygiene and regular visits
nurse? to the dentist should be emphasized.

A) «Folic acid should be taken before and after


conception.» Question35
B) «Multivitamin supplements are recommended The nurse is offering safety instructions to a par-
during pregnancy.» ent with a four month-old infant and a four year-
C) «A well balanced diet promotes normal fetal old child. Which statement by the parent indi-
development.» cates understanding of appropriate precautions
D) «Increased dietary iron improves the health of to take with the children?
mother and fetus.»
A) «I strap the infant car seat on the front seat to
Review Information: The correct answer is face backwards.»
A: «Folic acid should be taken before and after B) «I place my infant in the middle of the living
conception.». The American Academy of Pedi- room floor on a blanket to play with my four year-
atrics recommends that all childbearing women old while I make supper in the kitchen.»
increase folic acid from dietary sources and/or C) «My sleeping baby lies so cute in the crib with
supplements. There is evidence that increased the little buttocks stuck up in the air while the four
amounts of folic acid prevents neural tube de- year-old naps on the sofa.»
fects. D) «I have the four year-old hold and help feed
the four month-old a bottle in the kitchen while I
make supper.»
Question33
The provider orders Lanoxin (digoxin) 0.125 mg Review Information: The correct answer is
PO and furosemide 40 mg every day. Which of D: «I have the four year-old hold and help feed
these foods would the nurse reinforce for the cli- the four month-old a bottle in the kitchen while I
ent to eat at least daily? make supper.». The infant seat is to be placed
on the rear seat. Small children and infants are
A) Spaghetti not to be left unsupervised. Infants are
B) Watermelon
C) Chicken
D) Tomatoes Question36
The nurse admits a 7 year-old to the emergency
Review Information: The correct answer is B: room after a leg injury. The x-rays show a femur
Watermelon. Watermelon is high in potassium fracture near the epiphysis. The parents ask what
and will replace potassium lost by the diuretic. will be the outcome of this injury. The appropriate
The other foods are not high in potassium. response by the nurse should be which of these
statements?

Question34 A) «The injury is expected to heal quickly be-


While teaching the family of a child who will take cause of thin periosteum.»
phenytoin (Dilantin) regularly for seizure control, B) «In some instances the result is a retarded
Collected by :DeepaRajesh [ 214 ]
rajesh.ks21@gmail.com
Kuwait
bone growth.» cussing formula preparation, which of the follow-
C) «Bone growth is stimulated in the affected ing is most important to prevent lead poisoning?
leg.»
D) «This type of injury shows more rapid union A) Use ready-to-feed commercial infant formula
than that of younger children.» B) Boil the tap water for 10 minutes prior to pre-
paring the formula
Review Information: The correct answer is B: C) Let tap water run for 2 minutes before adding
«In some instances the result is a retarded bone to concentrate
growth.». An epiphyseal (growth) plate fracture D) Buy bottled water labeled «lead free» to mix
in a 7 year-old often results in retarded bone the formula
growth. The leg often will be different in length
than the uninjured leg. Review Information: The correct answer is C:
Let tap water run for 2 minutes before adding to
concentrate. Use of lead-contaminated water to
Question37 prepare formula is a major source of poisoning
The parents of a 4 year-old hospitalized child tell in infants. Drinking water may be contaminated
the nurse, “We are leaving now and will be back by lead from old lead pipes or lead solder used
at 6 PM.” A few hours later the child asks the in sealing water pipes. Letting tap water run for
nurse when the parents will come again. What is several minutes will diminish the lead contami-
the best response by the nurse? nation.

A) «They will be back right after supper.»


B) «In about 2 hours, you will see them.» Question40
C) «After you play awhile, they will be here.» Which of the following manifestations observed
D) «When the clock hands are on 6 and 12.» by the school nurse confirms the presence of pe-
diculosis capitis in students?
Review Information: The correct answer is A:
«They will be back right after supper.». Time is A) Scratching the head more than usual
not completely understood by a 4 year-old. Pre- B) Flakes evident on a student>s shoulders
schoolers interpret time with their own frame of C) Oval pattern occipital hair loss
reference. Thus, it is best to explain time in rela- D) Whitish oval specks sticking to the hair
tionship to a known, common event.
Review Information: The correct answer is D:
Whitish oval specks sticking to the hair. Diagno-
Question38 sis of pediculosis capitis is made by observation
The nurse is giving instructions to the parents of the white eggs (nits) firmly attached to the
of a child with cystic fibrosis. The nurse would hair shafts. Treatment can include application of
emphasize that pancreatic enzymes should be a medicated shampoo with lindane for children
taken over 2 years of age, and meticulous combing
A) once each day and removal of all nits.
B) 3 times daily after meals
C) with each meal or snack
D) each time carbohydrates are eaten Question41
When interviewing the parents of a child with
Review Information: The correct answer is C: asthma, it is most important to assess the child>s
with each meal or snack. Pancreatic enzymes environment for what factor?
should be taken with each meal and every snack
to allow for digestion of all foods that are eaten. A) Household pets
B) New furniture
C) Lead based paint
Question39 D) Plants such as cactus
A nurse is providing a parenting class to individu-
als living in a community of older homes. In dis- Review Information: The correct answer is A:
Collected by :DeepaRajesh [ 215 ]
rajesh.ks21@gmail.com
Kuwait
Household pets. Animal dander is a very com-
mon allergen affecting persons with asthma.
Other triggers may include pollens, carpeting Question44
and household dust. A couple experienced the loss of a 7 month-old
fetus. In planning for discharge, what should the
nurse emphasize?
Question42
The mother of a 2 month-old baby calls the nurse A) To discuss feelings with each other and use
2 days after the first DTaP, IPV, Hepatitis B and support persons
HIB immunizations. She reports that the baby B) To focus on the other healthy children and
feels very warm, cries inconsolably for as long as move through the loss
3 hours, and has had several shaking spells. In C) To seek causes for the fetal death and come
addition to referring her to the emergency room, to some safe conclusion
the nurse should document the reaction on the D) To plan for another pregnancy within 2 years
baby>s record and expect which immunization and maintain physical health
to be most associated with the findings the infant
is displaying? Review Information: The correct answer is A:
To discuss feelings with each other and use sup-
A) DTaP port persons. To communicate in a therapeutic
B) Hepatitis B manner, the nurse>>s goal is to help the couple
C) Polio begin the grief process by suggesting they talk
D) H. Influenza to each other, seek family, friends and support
groups to listen to their feelings.
Review Information: The correct answer is A:
DTaP. The majority of reactions occur with the
administration of the DTaP vaccination. Contra- Question45
dictions to giving repeat DTaP immunizations in- The nurse is performing a pre-kindergarten phys-
clude the occurrence of severe side effects after ical on a 5 year-old. The last series of vaccines
a previous dose as well as signs of encephalop- will be administered. What is the preferred site
athy within 7 days of the immunization. for injection by the nurse?

A) vastus intermedius
Question43 B) gluteus maximus
The mother of a 2 year-old hospitalized child asks C) vastus lateralis
the nurse>s advice about the child>s screaming D) dorsogluteaI
every time the mother gets ready to leave the
hospital room. What is the best response by the Review Information: The correct answer is C:
nurse? vastus lateralis. Vastus lateralis, a large and well
developed muscle, is the preferred site, since it is
A) «I think you or your partner needs to stay with removed from major nerves and blood vessels.
the child while in the hospital.»
B) «Oh, that behavior will stop in a few days.»
C) «Keep in mind that for the age this is a normal Question46
response to being in the hospital.» A 7 month pregnant woman is admitted with com-
D) «You might want to «sneak out» of the room plaints of painless vaginal bleeding over several
once the child falls asleep.» hours. The nurse should prepare the client for an
immediate
Review Information: The correct answer is C:
«Keep in mind that for the age this is a normal A) Non stress test
response to being in the hospital.». The protest B) Abdominal ultrasound
phase of separation anxiety is a normal response C) Pelvic exam
for a child this age. In toddlers, ages 1 to 3, sepa- D) X-ray of abdomen
ration anxiety is at its peak
Collected by :DeepaRajesh [ 216 ]
rajesh.ks21@gmail.com
Kuwait
Review Information: The correct answer is B:
Abdominal ultrasound. The standard for diagno- Question49
sis of placenta previa, which is suggested in the The nurse is caring for a client who was success-
client>>s history of painless bleeding, is abdomi- fully resuscitated from a pulseless dysrhythmia.
nal ultrasound. Which of the following assessments is critical for
the nurse to include in the plan of care?

Question47 A) hourly urine output


A nurse entering the room of a postpartum moth- B) white blood count
er observes the baby lying at the edge of the C) blood glucose every 4 hours
bed while the woman sits in a chair. The mother D) temperature every 2 hours
states «This is not my baby, and I do not want it.»
After repositioning the child safely, the nurse>s Review Information: The correct answer is A:
best response is hourly urine output. Clients who have had an epi-
sode of decreased glomerular perfusion are at
A) «This is a common occurrence after birth, but risk for pre-renal failure. This is caused by any
you will come to accept the baby.» abnormal decline in kidney perfusion that reduc-
B) «Many women have postpartum blues and es glomerular perfusion. Pre-renal failure occurs
need some time to love the baby.» when the effective arterial blood volume falls.
C) «What a beautiful baby! Her eyes are just like Examples of this phenomena include a drop in
yours.» circulating blood volume as in a cardiac arrest
D) «You seem upset; tell me what the pregnancy state or in low cardiac perfusion states such as
and birth were like for you.» congestive heart failure associated with a cardi-
omyopathy. Close observation of hourly urinary
Review Information: The correct answer is D: output is necessary for early detection of this
«You seem upset; tell me what the pregnancy condition.
and birth were like for you.». A non-judgmental,
open ended response facilitates dialogue be-
tween the client and nurse. Question50
A client is admitted to the rehabilitation unit fol-
lowing a cerebral vascular accident (CVA) and
Question48 mild dysphagia. The most appropriate interven-
The nurse notes that a 2 year-old child recovering tion for this client is to
from a tonsillectomy has an temperature of 98.2
degrees Fahrenheit at 8:00 AM. At 10:00 AM the A) position client in upright position while eating
child>s parent reports that the child «feels very B) place client on a clear liquid diet
warm» to touch. The first action by the nurse C) tilt head back to facilitate swallowing reflex
should be to D) offer finger foods such as crackers or pret-
zels
A) reassure the parent that this is normal
B) offer the child cold oral fluids Review Information: The correct answer is A:
C) reassess the child>s temperature position client in upright position while eating. An
D) administer the prescribed acetaminophen upright position facilitates proper chewing and
swallowing.
Review Information: The correct answer is C:
reassess the child>>s temperature. A child>>s
temperature may have rapid fluctuations. The Question51
nurse should listen to and show respect for what A 72 year-old client with osteomyelitis requires a
parents say. Parental caretakers are often quite 6 week course of intravenous antibiotics. In plan-
sensitive to variations in their children>>s condi- ning for home care, what is the most important
tion that may not be immediately evident to oth- action by the nurse?
ers.
A) Investigating the client>s insurance coverage
Collected by :DeepaRajesh [ 217 ]
rajesh.ks21@gmail.com
Kuwait
for home IV antibiotic therapy
B) Determining if there are adequate hand wash- Review Information: The correct answer is
ing facilities in the home B: stimulates hydrochloric acid production. Dec-
C) Assessing the client>s ability to participate in adron increases the production of hydrochloric
self care and/or the reliability of a caregiver acid, which may cause gastrointestinal ulcers.
D) Selecting the appropriate venous access de-
vice
Question54
Review Information: The correct answer is A client receiving chlorpromazine HCL (Thora-
C: Assessing the client>>s ability to participate zine) is in psychiatric home care. During a home
in self care and/or the reliability of a caregiver. visit the nurse observes the client smacking her
The cognitive ability of the client as well as the lips alternately with grinding her teeth. The nurse
availability and reliability of a caregiver must be recognizes this assessment finding as what?
assessed to determine if home care is a feasible A) Dystonia
option. B) Akathisia
C) Brady dyskinesia
D) Tardive dyskinesia
Question52
A nurse administers the influenza vaccine to a Review Information: The correct answer is D:
client in a clinic. Within 15 minutes after the im- Tardive dyskinesia. Signs of tardive dyskinesia
munization was given, the client complains of include smacking lips, grinding of teeth and «fly
itchy and watery eyes, increased anxiety, and catching» tongue movements. These findings
difficulty breathing. The nurse expects that the are often described as Parkinsonian.
first action in the sequence of care for this client
will be to
Question55
A) Maintain the airway Which of the following findings contraindicate the
B) Administer epinephrine 1:1000 as ordered use of haloperidol (Haldol) and warrant withhold-
C) Monitor for hypotension with shock ing the dose?
D) Administer diphenhydramine as ordered A) Drowsiness, lethargy, and inactivity
B) Dry mouth, nasal congestion, and blurred vi-
Review Information: The correct answer is B: sion
Administer epinephrine 1:1000 as ordered. All C) Rash, blood dyscrasias, severe depression
the answers are correct given the circumstances, D) Hyperglycemia, weight gain, and edema
but the priority is to administer the epinephrine,
then maintain the airway. In the early stages of Review Information: The correct answer is
anaphylaxis, when the patient has not lost con- C: Rash, blood dyscrasias, severe depression.
sciousness and is normotensive, administering Rash and blood dyscrasias are side effects of
the epinephrine is first, and applying the oxygen, anti-psychotic drugs. A history of severe depres-
and watching for hypotension and shock, are lat- sion is a contraindication to the use of neurolep-
er responses. The prevention of a severe crisis tics.
is maintained by using diphenhydramine.

Question56
Question53 The nurse is reinforcing teaching to a 24 year-old
The nurse instructs the client taking dexametha- woman receiving acyclovir (Zovirax) for a Herpes
sone (Decadron) to take it with food or milk. The Simplex Virus type 2 infection. Which of these in-
physiological basis for this instruction is that the structions should the nurse give the client?
medication
A) retards pepsin production A) Complete the entire course of the medication
B) stimulates hydrochloric acid production for an effective cure
C) slows stomach emptying time B) Begin treatment with acyclovir at the onset of
D) decreases production of hydrochloric acid symptoms of recurrence
Collected by :DeepaRajesh [ 218 ]
rajesh.ks21@gmail.com
Kuwait
C) Stop treatment if she thinks she may be preg- Which client statement from the assessment
nant to prevent birth defects data is likely to explain his noncompliance?
D) Continue to take prophylactic doses for at
least 5 years after the diagnosis A) «I have problems with diarrhea.»
B) «I have difficulty falling asleep.»
Review Information: The correct answer is C) «I have diminished sexual function.»
B: Begin treatment with acyclovir at the onset D) «I often feel jittery.»
of symptoms of recurrence. When the client is
aware of early symptoms, such as pain, itching Review Information: The correct answer is C:
or tingling, treatment is very effective. Medica- «I have diminished sexual function.». Inderal, a
tions for herpes simplex do not cure the disease; beta-blocking agent used in hypertension, pro-
they simply decrease the level of symptoms. hibits the release of epinephrine into the cells;
this may result in hypotension which results in
decreased libido and impotence.
Question57
A 14 month-old child ingested half a bottle of
aspirin tablets. Which of the following would the Question60
nurse expect to see in the child? The nurse caring for a 9 year-old child with a
fractured femur is told that a medication error
A) Hypothermia occurred. The child received twice the ordered
B) Edema dose of morphine an hour ago. Which nursing
C) Dyspnea diagnosis is a priority at this time?
D) Epistaxis
A) Risk for fluid volume deficit related to mor-
Review Information: The correct answer is phine overdose
D: Epistaxis. A large dose of aspirin inhibits pro- B) Decreased gastrointestinal mobility related to
thrombin formation and lowers platelet levels. mucosal irritation
With an overdose, clotting time is prolonged. C) Ineffective breathing patterns related to cen-
tral nervous system depression
D) Altered nutrition related to inability to control
Question58 nausea and vomiting
An 80 year-old client on digitalis (Lanoxin) re-
ports nausea, vomiting, abdominal cramps and Review Information: The correct answer is C:
halo vision. Which of the following laboratory re- Ineffective breathing patterns related to central
sults should the nurse analyze first? nervous system depression. Respiratory depres-
sion is a life-threatening risk in this overdose.
A) Potassium levels
B) Blood pH
C) Magnesium levels Question61
D) Blood urea nitrogen Lactulose (Chronulac) has been prescribed for a
client with advanced liver disease. Which of the
Review Information: The correct answer is A: following assessments would the nurse use to
Potassium levels. The most common cause of evaluate the effectiveness of this treatment?
digitalis toxicity is a low potassium level. Clients
must be taught that it is important to have ad- A) An increase in appetite
equate potassium intake especially if taking diu- B) A decrease in fluid retention
retics that enhance the loss of potassium while C) A decrease in lethargy
they are taking digitalis. D) A reduction in jaundice

Review Information: The correct answer is


Question59 C: A decrease in lethargy. Lactulose produces
A 42 year-old male client refuses to take pro- an acid environment in the bowel and traps am-
pranolol hydrochloride (Inderal) as prescribed. monia in the gut; the laxative effect then aids in
Collected by :DeepaRajesh [ 219 ]
rajesh.ks21@gmail.com
Kuwait
removing the ammonia from the body. This de-
creases the effects of hepatic encephalopathy, A) Listen quietly without comment
including lethargy and confusion. B) Ask for further information on the spies
C) Confront the client’s delusion
D) Contact the government agency
Question62
The nurse is teaching a class on HIV prevention. Review Information: The correct answer is A:
Which of the following should be emphasized as Listen quietly without comment. The client>>s
increasing risk? comments demonstrate grandiose ideas. The
most therapeutic response is to listen but avoid
A) Donating blood being incorporated into the client’s delusional
B) Using public bathrooms system.
C) Unprotected sex
D) Touching a person with AIDS Question65
The nurse is assessing a 17 year-old female cli-
Review Information: The correct answer is C: ent with bulimia. Which of the following labora-
Unprotected sex. Because HIV is spread through tory reports would the nurse anticipate?
exposure to bodily fluids, unprotected intercourse
and shared drug paraphernalia remain the high- A) Increased serum glucose
est risks for infection. B) Decreased albumin
C) Decreased potassium
D) Increased sodium retention
Question63
While interviewing a new admission, the nurse Review Information: The correct answer is C:
notices that the client is shifting positions, wring- Decreased potassium. In bulimia, loss of elec-
ing her hands, and avoiding eye contact. It is im- trolytes can occur in addition to other findings of
portant for the nurse to starvation and dehydration.
A) ask the client what she is feeling
B) assess the client for auditory hallucinations
C) recognize the behavior as a side effect of Question66
medication A client, recovering from alcoholism, asks the
D) re-focus the discussion on a less anxiety pro- nurse, «What can I do when I start recognizing
voking topic relapse triggers within myself?» How might the
nurse best respond?
Review Information: The correct answer is A:
ask the client what she is feeling. The initial step A) «When you have the impulse to stop in a bar,
in anxiety intervention is observing, identifying, contact a sober friend and talk with him.»
and assessing anxiety. The nurse should seek B) «Go to an AA meeting when you feel the urge
client validation of the accuracy of nursing as- to drink.»
sessments and avoid drawing conclusions based C) «It is important to exercise daily and get in-
on limited data. In the situation above, the client volved in activities that will cause you not to think
may simply need to use the restroom but be re- about drug use.»
luctant to communicate her need! D) «Let’s talk about possible options you have
when you recognize relapse triggers in your-
self.»
Question64
A young adult seeks treatment in an outpatient Review Information: The correct answer is D:
mental health center. The client tells the nurse «Let’s talk about possible options you have when
he is a government official being followed by you recognize relapse triggers in yourself.». This
spies. On further questioning, he reveals that option encourages the process of self evalua-
his warnings must be heeded to prevent nuclear tion and problem solving, while avoiding telling
war. What is the most therapeutic approach by the client what to do. Encouraging the client to
the nurse? brainstorm about response options validates the
Collected by :DeepaRajesh [ 220 ]
rajesh.ks21@gmail.com
Kuwait
nurse’s belief in the client’s personal competen- B) provide a group forum in which clients decide
cy and reinforces a coping strategy that will be on unit rules, regulations, and policies
needed when the nurse may not be available to C) provide a testing ground for new patterns of
offer solutions. behavior while the client takes responsibility for
his or her own actions
Question67 D) discourage expressions of anger because
Therapeutic nurse-client interaction occurs when they can be disruptive to other clients
the nurse
A) assists the client to clarify the meaning of what Review Information: The correct answer is C:
the client has said provide a testing ground for new patterns of be-
B) interprets the client’s covert communication havior while the client takes responsibility for his
C) praises the client for appropriate feelings and or her own actions. A therapeutic milieu is pur-
behavior poseful and planned to provide safety and a test-
D) advises the client on ways to resolve prob- ing ground for new patterns of behavior.
lems

Review Information: The correct answer is A: Question70


assists the client to clarify the meaning of what A client with paranoid delusions stares at the
the client has said. Clarification is a facilitating/ nurse over a period of several days. The client
therapeutic communication strategy. Interpreta- suddenly walks up to the nurse and shouts «You
tion, changing the focus/subject, giving approv- think you’re so perfect and pure and good.» An
al, and advising are non-therapeutic/barriers to appropriate response for the nurse is
communication.
A) «Is that why you’ve been staring at me?»
B) «You seem to be in a really bad mood.»
Question68 C) «Perfect? I don’t quite understand.»
Which nursing intervention will be most effective D) «You seem angry right now.»
in helping a withdrawn client to develop relation-
ship skills? Review Information: The correct answer is D:
A) Offer the client frequent opportunities to inter- «You seem angry right now.». The nurse recog-
act with 1 person nizes the underlying emotion with a matter of fact
B) Provide the client with frequent opportunities attitude, but avoids telling the clients how they
to interact with other clients feel.
C) Assist the client to analyze the meaning of the
withdrawn behavior
D) Discuss with the client the focus that other Question71
clients have similar problems A client who is a former actress enters the day
room wearing a sheer nightgown, high heels, nu-
Review Information: The correct answer is A: merous bracelets, bright red lipstick and heavily
Offer the client frequent opportunities to interact rouged cheeks. Which nursing action is the best
with 1 person. The withdrawn client is uncomfort- in response to the client’s attire?
able in social interaction. The nurse-client rela-
tionship is a corrective relationship in which the A) Gently remind her that she is no longer on
client learns both tolerance and skills for relation- stage
ships. B) Directly assist client to her room for appropri-
ate apparel
C) Quietly point out to her the dress of other cli-
Question69 ents on the unit
An important goal in the development of a thera- D) Tactfully explain appropriate clothing for the
peutic inpatient milieu is to hospital

A) provide a businesslike atmosphere where cli- Review Information: The correct answer is B:
ents can work on individual goals Directly assist client to her room for appropriate
Collected by :DeepaRajesh [ 221 ]
rajesh.ks21@gmail.com
Kuwait
apparel. It assists the client to maintain self-es- C) refusal to touch a client denotes lack of con-
teem while modifying behavior. cern
D) inappropriate touch often results in charges of
assault and battery
Question72
When teaching suicide prevention to the parents Review Information: The correct answer is A:
of a 15 year-old who recently attempted suicide, some clients misconstrue hugs as an invitation
the nurse describes the following behavioral cue to sexual advances. Touch denotes positive feel-
as indicating a need for intervention. ings for another person. The client may interpret
hugging and holding hands as sexual advanc-
A) Angry outbursts at significant others es.
B) Fear of being left alone
C) Giving away valued personal items
D) Experiencing the loss of a boyfriend Question75
A client with anorexia is hospitalized on a medi-
Review Information: The correct answer is C: cal unit due to electrolyte imbalance and cardiac
Giving away valued personal items. Eighty per- dysrhythmias. Additional assessment findings
cent of all potential suicide victims give some type that the nurse would expect to observe are
of indication that self-destructiveness should be
addressed. These clues might lead one to sus- A) brittle hair, lanugo, amenorrhea
pect that a client is having suicidal thoughts or is B) diarrhea, nausea, vomiting, dental erosion
developing a plan. C) hyperthermia, tachycardia, increased meta-
bolic rate
D) excessive anxiety about symptoms
Question73
Which statement made by a client indicates to Review Information: The correct answer is A:
the nurse that the client may have a thought dis- brittle hair, lanugo, amenorrhea. Physical find-
order? ings associated with anorexia also include re-
A) «I>m so angry about this. Wait until my part- duced metabolic rate and lower vital signs.
ner hears about this.»
B) «I>m a little confused. What time is it?»
C) «I can>t find my <mesmer> shoes. Have you
seen them?»
D) «I>m fine. It>s my daughter who has the prob-
lem.»

Review Information: The correct answer is C:


«I can>>t find my <>mesmer>> shoes. Have you
seen them?». A neologism is a new word self in-
vented by a person and not readily understood
by another. Using neologisms is often associ-
ated with a thought disorder.

Question74
In a psychiatric setting, the nurse limits touch
or contact used with clients to handshaking be-
cause

A) some clients misconstrue hugs as an invita-


tion to sexual advances
B) handshaking keeps the gesture on a profes-
sional level
Collected by :DeepaRajesh [ 222 ]
rajesh.ks21@gmail.com
Kuwait

You might also like